Problem Book Math in Khmer

Page 1

លឹម សុ វណ្ណវិចិត្រ

កម្រងលំហាត់គណិតវិទ្យា កត្មិរវិទ្យាល័យ ភាគ ២- ពីជគណ្ិរ វិភាគ

ផ្សាយល

ើកទ ី៤

មីនា ២០១០


blank


លឹម សុ វណ្ណវិចិត្រ

កម្រងលំហាត់គណិតវិទ្យា កត្មិរវិទ្យាល័យ ភាគ ២- ពីជគណ្ិរ វិភាគ

ផ្សាយលលើកទ ី៤ មីនា ២០១០


ភាគ១ - ទ្ទ ឹស្តីនៃចំៃួៃ ភាគ២ - ព ីជគណិត វភាគ ិ ផ្សាយលលើកទ ី១ : គណិតវទាអូ ិ ឡំព ិច -វស្មភាព ិ ២០០៨ ផ្សាយលលើកទ ី២ : កំរងលំ ហាត់គណិតវទា ិ -ព ីជគណិត វភាគ ិ កកក ដា២០០៩ ផ្សាយលលើកទ ី៣ : កំរងលំ ហាត់គណិតវទា ិ -ព ីជគណិត វភាគ ិ ស្ី ហា ២០០៩ ផ្សាយលលើកទ ី៤ : កទ្មងលំ ហាត់គណិតវទា ិ -ព ីជគណិត វភាគ ិ មីនា ២០១០

ដោយ

លឹ ម សុ វណ្ណវិចិត្រ

ទ្យំនាក់ ទ្យន ំ ង - វប ិ ុ សាយ

http://www.dahlina.com/

- អុ ីមមល

lsvichet@yahoo.com


សម្គាល់ 𝑥 ∈ 𝑎; 𝑏 មានន័យថា 𝑎 ≤ 𝑥 ≤ 𝑏 𝑥 ∈ 𝑎; 𝑏 មានន័យថា 𝑎 < 𝑥 < 𝑏 𝑥 ∈ 𝑎; 𝑏 មានន័យថា 𝑎 ≤ 𝑥 < 𝑏 𝑛 𝑚

𝑛!

= 𝐶𝑛𝑚 = 𝑚 ! 𝑛−𝑚

!

ℕ សំ ណុំចំនន ួ គត់ ធម្ម ជាតិ ដែលមានទំងលលខសូនយ

ℕ = {0; 1; 2; 3; … }

ℕ∗ សំ ណុំចំនន ួ គត់ ធម្ម ជាតិម្ិនសូនយ

ℕ∗ = {1; 2; 3; … }

ℤ សំ ណុំចំនន ួ គត់

ℤ = {… ; −3; −2; −1; 0; 1; 2; 3; … }

ℤ∗ សំ ណុំចំនន ួ គត់ ម្ិនសូនយ

ℤ∗ = {… ; −3; −2; −1; 1; 2; 3; … }

ិ មា ℤ+ សំ ណុំចំនន ួ គត់ វជ្ ជ នរ ឺសូនយ

ℤ+ = {0; 1; 2; 3; … }

ℝ សំ ណុំចំនន ួ ពិត

ិ មា ℝ+ សំ ណុំចំនន ួ ពិតម្ិនអវជ្ ជ ន

ℝ+ = 𝑥 𝑥 ≥ 0

សញ្ញបូក 𝑛

𝑥𝑖 = 𝑥1 + 𝑥2 + ⋯ + 𝑥𝑛 𝑖=1

𝑥𝑖 𝑥𝑗 = 𝑥1 𝑥2 + 𝑥1 𝑥3 + 𝑥1 𝑥4 + ⋯ + 𝑥1 𝑥𝑛 + 𝑥2 𝑥3 + 𝑥2 𝑥4 + ⋯ + 𝑥2 𝑥𝑛 + ⋯ + 𝑥𝑛−1 𝑥𝑛 1≤𝑖<𝑗 ≤𝑛

𝑥𝑖 𝑥𝑗 = 𝑥1 𝑥2 + 𝑥1 𝑥3 + 𝑥1 𝑥4 + 𝑥1 𝑥5 + 𝑥2 𝑥3 + 𝑥2 𝑥4 + 𝑥2 𝑥5 + 𝑥3 𝑥4 + 𝑥3 𝑥5 + 𝑥4 𝑥5 1≤𝑖<𝑗 ≤5

1≤𝑖<𝑗 ≤3

𝑥𝑖 𝑥1 𝑥1 𝑥2 = + + 𝑥𝑖 + 𝑥𝑗 𝑥1 + 𝑥2 𝑥1 + 𝑥3 𝑥2 + 𝑥3 𝑥𝑖 𝑥𝑗 = 𝑥1 𝑥2 + 𝑥1 𝑥3 + 𝑥2 𝑥1 + 𝑥2 𝑥3 + 𝑥3 𝑥1 + 𝑥3 𝑥2 = 2 𝑥1 𝑥2 + 𝑥1 𝑥3 + 𝑥2 𝑥3

1≤𝑖≠𝑗 ≤3

i


និយមន័យ: ផលបូកស៊ី មមទ្រ៊ី និង ស៊ី គ្លិច តាង 𝑃 𝑥, 𝑦, 𝑧 ជាអនុគម្ន៍មានបីអលថរ 𝑥, 𝑦, 𝑧។ តាង 𝑃 𝑥, 𝑦, 𝑧 = 𝑃 𝑥, 𝑦, 𝑧 + 𝑃 𝑦, 𝑧, 𝑥 + 𝑃 𝑧, 𝑥, 𝑦 cyclic

𝑃 𝑥, 𝑦, 𝑧 = 𝑃 𝑥, 𝑦, 𝑧 + 𝑃 𝑥, 𝑧, 𝑦 + 𝑃 𝑦, 𝑥, 𝑧 sym

+𝑃 𝑦, 𝑧, 𝑥 + 𝑃 𝑧, 𝑥, 𝑦 + 𝑃 𝑧, 𝑦, 𝑥 ឧទាហរណ៍ 𝑥3𝑦 = cyclic

𝑥 3 𝑦 z 0 = 𝑥 3 𝑦𝑧 0 + 𝑦 3 𝑧𝑥 0 + 𝑧 3 𝑥𝑦 0 cyclic

= 𝑥3 𝑦 + 𝑦3 𝑧 + 𝑧3 𝑥 𝑥3 = sym

𝑥 3 𝑦0 𝑧 0 = 𝑥 3 𝑦0 𝑧 0 + 𝑥 3 𝑧 0 𝑦0 + 𝑦 3 𝑥 0 𝑧 0 + 𝑦3 𝑧 0 𝑥0 + 𝑧 3 𝑥 0 𝑦0 + 𝑧 3 𝑦0 𝑥 0 sym

= 2 𝑥3 + 𝑦3 + 𝑧3 𝑥3 𝑦2𝑧 = 𝑥3 𝑦2𝑧 + 𝑥3 𝑧2𝑦 + 𝑦3 𝑥2 𝑧 + 𝑦3 𝑧2𝑥 + 𝑧3 𝑥2𝑦 + 𝑧3 𝑦2 𝑥 sym

𝑥2𝑦 = sym

𝑥 2 𝑦1 𝑧 0 = 𝑥 2 𝑦1 𝑧 0 + 𝑥 2 𝑧1 𝑦 0 + 𝑦 2 𝑥1 𝑧 0 + 𝑦 2 𝑧1 𝑥 0 + 𝑧 2 𝑥1 𝑦 0 + 𝑧 2 𝑦1 𝑥 0 sym

= 𝑥2 𝑦 + 𝑥2 𝑧 + 𝑦2 𝑥 + 𝑦2 𝑧 + 𝑧2 𝑥 + 𝑧2 𝑦 𝑥𝑦𝑧 = 𝑥𝑦𝑧 + 𝑥𝑧𝑦 + 𝑦𝑥𝑧 + 𝑦𝑧𝑥 + 𝑧𝑥𝑦 + 𝑧𝑦𝑥 = 6𝑥𝑦𝑧 sym

កមនសោមមួយមានលកខណៈឆលលុះ រឺ ស៊ី មមទ្រ៊ី កលនោម្ម្ួ យ មានលកខណៈសុលី ម្រទី លបើលយើ ងឆ្លលស់ អញ្ញាតណាម្ួ យនឹងអញ្ញាតណាម្ួ យលផេងលទៀត កលនោម្ល ោះលៅែដែល។ ឧទហរណ៍ 𝑃 = 𝑥 + 𝑦 + 𝑧, 𝑥, 𝑦, 𝑧 ∈ ℝ មានលកខណៈសុលី ម្រទីលធៀបនឹង ិ វាលៅែដែល។ លបើលយើ ងជ្ំនស 𝑥, 𝑦, 𝑧 លោយសារលបើលយើ ងជ្ំនស ួ 𝑥 លោយ 𝑦 និង 𝑦 លោយ 𝑥 វញ ួ 𝑥

ិ វាលៅែដែល។ លបើលយើ ងជ្ំនស ិ វាលៅ លោយ 𝑧 និង 𝑧 លោយ 𝑥 វញ ួ 𝑦 លោយ 𝑧 និង 𝑧 លោយ 𝑦 វញ ែដែល។

ii


ពហុ ធា 𝑃 = 𝑥 3 + 𝑦 3 + 𝑧 ម្ិនមានលកខណៈសុលី ម្រទីលទ លររោះ លបើលយើ ងជ្ំនស ួ 𝑧 លោយ 𝑥 និង 𝑥 លោយ 𝑧 លយើ ងទញបាន 𝑃′ = 𝑧 3 + 𝑦 3 + 𝑥 ≠ 𝑥 3 + 𝑦 3 + 𝑧 ។ ចំលរោះកលនោម្សុលី ម្រទី លយើ ងអាចសនមតថា 𝑥 ≥ 𝑦 ≥ 𝑧 រ ឺ 𝑥 ≤ 𝑦 ≤ 𝑧 បាន លបើលទោះជាលគម្ិនរបាប់ ដបបលនោះក៏លោយ រ ឺម្ួ យជាទូ លៅ 𝑥, 𝑦, 𝑧 ម្ិនចំបាច់ លផទ ៀងផ្ទទត់ លកខខណឌដបបលនោះក៏លោយ លលើ កដលងដត មានសម្ម តក ិ ម្ម ម្ករសាប់ លផេងពីលនោះ។ លហតុអវីកល៏ យើ ងអាចសនមតបានដបបលនោះ?។ លររោះ 𝑥 អាចលែរើ តួ

ជ្ំនស ូ 𝑦, 𝑧 បាន 𝑦 អាចលែរើ តួជ្ំនស ូ 𝑥, 𝑧 បាន និង 𝑧 អាចលែរើ តួជ្ំនស ូ 𝑥, 𝑦 បាន។ លបើមានម្ួ យកនុងចំនន ួ ទំងបីធជា ំ ងលគ លយើ ងតាងវាលោយ 𝑥1 ចំនន ួ ធំប ទ ប់ លោយ 𝑦1 និងតូចជាងលគលោយ 𝑧1 ។ ែូលចនោះ 𝑥1 ≥ 𝑦1 ≥ 𝑧1 ។ ែូលចនោះកលនោម្សុលី ម្រទី ែូចជា 𝑃 = 𝑥 + 𝑦 + 𝑧 = 𝑥1 + 𝑦1 + 𝑧1 ។ ែូលចនោះកលនោម្ 𝑃 លៅដតែដែលរាន់ ដតដថម្សនទសេន៍ 1 ពីលរោម្ដតបុ លណា ណ ោះ។

អនគ្មន៍ផត-អនគ្មន៍ម ៉ោ ង ិ លៅលរោម្ប ទ ត់ ភ្ជជប់ រគប់ អនុគម្ន៍ម្ួយផតចល ល ោះចំណុច 𝐴 និង 𝐵 លបើ ដខេលោងអនុគម្ន៍ល ោះឋត ិ លៅចល ល ោះ 𝐴 និង 𝐵 ។ វាលបាងលបើវាលៅខាងលលើ ។ ចំណុចទំងអស់ ឋត

* អនុគម្ន៍ 𝑓 ម្ួ យ ដែលកំណត់ លលើ 𝐼 ∈ ℝ លៅថា ផត លបើ ចំលរោះរគប់ 𝜆 ∈ 0; 1 និង រគប់ 𝑥, 𝑦 ∈ 𝐼 លគមាន 𝑓 𝜆𝑥 + 1 − 𝜆 𝑦 ≤ 𝜆𝑓 𝑥 + 1 − 𝜆 𝑓 𝑦 * អនុគម្ន៍ម្ួយផតលលើ 𝑎, 𝑏 លបើ 𝑓 ′ ′ 𝑥 > 0, ∀𝑥 ∈ 𝑎, 𝑏 * លបើ 𝑓 ផតចល ល ោះ 𝐴, 𝐵 ល ោះ តម្ម្ល ធប ំ ំ ផុតរបស់ 𝑓 លបើម្ិនលៅរតង់ 𝐴 គឺលៅរតង់ 𝐵 : 𝑓𝑚 𝑎𝑥 = max 𝑓𝐴 , 𝑓𝐵 * លបើ 𝑓 លបាងចល ល ោះ 𝐴, 𝐵 ល ោះ តម្ម្ល តច ូ បំ ផុតរបស់ 𝑓 លបើម្ិនលៅរតង់ 𝐴 គឺលៅរតង់ 𝐵 : 𝑓𝑚𝑖𝑛 = min 𝑓𝐴 , 𝑓𝐵

iii


đ??ľ

đ??ľ

đ??´

đ??´

áž˘áž“ážťáž‚áž˜á&#x;’áž“á&#x;?លបវងចល áž› á&#x;„á&#x;‡ đ??´đ??ľ

áž˘áž“ážťáž‚áž˜á&#x;’áž“á&#x;?áž•áž?áž…áž› áž› á&#x;„á&#x;‡ đ??´đ??ľ

đ??ľ

đ??´ đ??ś

đ??ˇ

áž˘áž“ážťáž‚áž˜á&#x;’áž“á&#x;?áž•áž?áž…áž› áž› á&#x;„á&#x;‡ đ??´đ??ś; áž˘áž“ážťáž‚áž˜á&#x;’áž“á&#x;?លបវងចល áž› á&#x;„á&#x;‡ đ??śđ??ˇ; áž•áž?áž…áž› áž› á&#x;„á&#x;‡ đ??ˇđ??ľ

iv


𝐵

𝐴 𝜆𝑓 𝑥 + 1 − 𝜆 𝑓 𝑦

𝑥 𝜆𝑥 + 1 − 𝜆 𝑦

𝑦 𝑓 𝜆𝑥 + 1 − 𝜆 𝑦

v


ម្គតិកា

ផ្ផែករ៊ី១ លំហាត់ ១. អនុគម្ន៍ងាយ ............................................................................................ 1 គណ

................................................................................................ 1

សម្ភ្ជព.............................................................................................. 1 សម្ីោរ ............................................................................................... 2 របព័នធសម្ីោរ...................................................................................... 7

ិ វសម្ភ្ជព ............................................................................................ 14 ិ ោរ ............................................................................................. 34 វសម្ី ិ ោរ .................................................................................... 37 របព័នធវសម្ី ២. អនុគម្ន៍អិចសបូណង់ដសយល និងលោ​ោរ ីត.................................................... 39 គណ

................................................................................................ 39

សម្ភ្ជព.............................................................................................. 40 សម្ីោរ ............................................................................................... 40 របព័នធសម្ីោរ...................................................................................... 44 ិ វសម្ភ្ជព ............................................................................................ 46 ិ ោរ ............................................................................................. 46 វសម្ី ិ ោរ .................................................................................... 50 របព័នធវសម្ី ៣. រតីលោណមារត ........................................................................................... 53 គណ

................................................................................................ 53

សម្ភ្ជព.............................................................................................. 54 សម្ីោរ ............................................................................................... 57 របព័នធសម្ីោរ...................................................................................... 64 vi


ិ វសម្ភ្ជព ............................................................................................ 65 ិ ោរ ............................................................................................. 66 វសម្ី ិ ោរ .................................................................................... 67 របព័នធវសម្ី ៤. ពហុ ធា ....................................................................................................... 69 ៥. សម្ីោរអនុគម្ន៍ ......................................................................................... 71

ផ្ផែករ៊ី២ ចមមលើយ ១. អនុគម្ន៍ងាយ ............................................................................................ 73 គណ

................................................................................................ 73

សម្ភ្ជព.............................................................................................. 75 សម្ីោរ ............................................................................................... 76 របព័នធសម្ីោរ...................................................................................... 85

ិ វសម្ភ្ជព ............................................................................................ 94

ិ ោរ ............................................................................................. 166 វសម្ី ិ ោរ .................................................................................... 169 របព័នធវសម្ី ២. អនុគម្ន៍អិចសបូណង់ដសយល និងលោ​ោរ ីត.................................................... 171 គណ

................................................................................................ 171

សម្ភ្ជព.............................................................................................. 172 សម្ីោរ ............................................................................................... 172 របព័នធសម្ីោរ...................................................................................... 174 ិ វសម្ភ្ជព ............................................................................................ 174 ិ ោរ ............................................................................................. 175 វសម្ី ិ ោរ .................................................................................... 177 របព័នធវសម្ី ៣. រតីលោណមារត ........................................................................................... 179 គណ

................................................................................................ 179

សម្ភ្ជព.............................................................................................. 183 vii


សម្ីោរ ............................................................................................... 185 របព័នធសម្ីោរ...................................................................................... 193

ិ វសម្ភ្ជព ............................................................................................ 195 ិ ោរ ............................................................................................. 201 វសម្ី ិ ោរ .................................................................................... 202 របព័នធវសម្ី ៤. ពហុ ធា ....................................................................................................... 203 ៥. សម្ីោរអនុគម្ន៍ ......................................................................................... 207

viii


១. អនុគមន៍ ងាយ គណ្នា ឹ ថា 𝑎 + 𝑏 + 𝑐 = 0 និង 𝑎2 + 𝑏 2 + 𝑐 2 = 1 ។ គណនា 𝑎4 + 𝑏 4 + 𝑐 4 ដោយដង

1. គណនា

𝑛 𝑛+1 ,𝑛 ∈ ℕ 2

2.

𝑆𝑛 = 1 + 3 + 6 + ⋯ +

3.

𝑆𝑛 = 13 + 23 + 33 + ⋯ + 𝑛3 , 𝑛 ∈ ℕ

4.

តាង 𝑆 = 2 + 1 22 + 1 24 + 1 … 21024 + 1 + 1 ។ ចូរគណនា 𝑆 1/1024 ។

5.

(កាណាដា ១៩៦៩) គណនា 1.1! + 2.2! + 3.3! + ⋯ + 𝑛 − 1 . 𝑛 − 1 ! + 𝑛. 𝑛! ដដល 𝑛! = 𝑛. 𝑛 − 1 … 3.2.1 ។

6.

គណនា 𝑆=

7.

1 1 1 1 + + ⋯+ + 1! 1999! 3! 1997! 1997! 3! 1999! 1!

ដគដោយចំនន ួ ពិត 𝑢, 𝑣, 𝑠, 𝑡 ដផទ ៀងផ្ទទត់ លក្ខខណឌ 𝑢 + 𝑣 + 𝑠 + 𝑡 = 𝑢7 + 𝑣 7 + 𝑠 7 + 𝑡 7 = 0 ចូរគណនា 𝑃 = 𝑡 𝑡 + 𝑢 𝑡 + 𝑣 𝑡 + 𝑠

សមភាព ចូរបង្ហាញថា

8. 9. 10.

𝑛 𝑛 + 1 2𝑛 + 1 12 + 22 + 32 + ⋯ + 𝑛2 = , 6 1 2 3 𝑛 𝑛+2 + 2 + 3 + ⋯+ 𝑛 = 2 − 𝑛 ,𝑛 ∈ ℕ 2 2 2 2 2 ចូរបង្ហាញថា ដបើ n ជាចំនន ួ គត់ ធម្ម ជាតិ ដនាោះ

លឹម សុ វណ្ណវិចិត្រ | I. គណ្នា

𝑛∈ ℕ

1


11.

1.2 + 2.5 + ⋯ + 𝑛 3𝑛 − 1 = 𝑛2 (𝑛 + 1) ចូរបង្ហាញថា ដបើ n ជាចំនន ួ គត់ ធម្ម ជាតិ ដនាោះ 2

2

2

1 + 3 + 5 + ⋯ + 2𝑛 − 1 12.

2

𝑛 4𝑛2 − 1 = 3

តាង 𝑎, 𝑏, 𝑐 ជាចំនន ួ ពិត ខុសពី −1 និង 1 ដដល 𝑎 + 𝑏 + 𝑐 = 𝑎𝑏𝑐 ។ ចូរបង្ហាញថា

𝑎 𝑏 𝑐 4𝑎𝑏𝑐 + + = 2 2 2 2 1−𝑎 1−𝑏 1−𝑐 1 − 𝑎 1 − 𝑏2 1 − 𝑐 2

13.

(កាណាដា ១៩៦៩)

ចូរបង្ហាញថា ដបើ 𝑎1 /𝑏1 = 𝑎2 /𝑏2 = 𝑎3 /𝑏3 និង 𝑝1 ; 𝑝2 ; 𝑝3 ម្ិនសូនយទំងអស់ រពម្គ្នន ដនាោះ 𝑛

𝑎1 𝑏1

=

ិ មា ចំដ ោះរគប់ ចំនន ួ គត់ វជ ជ ន𝑛។

𝑝1 𝑎1𝑛 + 𝑝2 𝑎2𝑛 + 𝑝3 𝑎3𝑛 𝑝1 𝑏1𝑛 + 𝑝2 𝑏2𝑛 + 𝑝3 𝑏3𝑛

សមីការ ដោ​ោះរាយសម្ីការ

14.

𝑥 3 + 2𝑥 2 − 𝑥 − 2 = 0

15.

𝑥 3 − 𝑥 2 − 8𝑥 + 12 = 0

16.

𝑥 3 − 9𝑥 2 + 27𝑥 − 27 = 0

17.

𝑥3 + 𝑥2 − 3 = 0

18.

6𝑥 3 + 7𝑥 2 − 1 = 0

19.

2𝑥 3 + 𝑥 2 + 5𝑥 − 3 = 0

20.

2𝑥 4 − 5𝑥 2 + 2 = 0

21.

𝑥 4 − 2𝑥 2 − 3 = 0

22. 23. 24. 2

𝑥−1

4

+ 𝑥+1

4

= 16

2𝑥 − 3 4 + 2𝑥 − 5 4 = 2 1 3 10 + 2 = 2 2 2𝑥 − 𝑥 + 1 2𝑥 − 𝑥 + 3 2𝑥 − 𝑥 + 7 ១. អនុគមន៍ងាយ | លឹម សុ វណ្ណវិចិត្រ


25.

𝑥 − 𝑎 𝑥 𝑥 + 𝑎 𝑥 + 2𝑎 = 3𝑎4

26.

32.

6𝑥 + 5 2 3𝑥 + 2 𝑥 + 1 = 35 4 2 𝑥2 + 2 − 8 𝑥 − −4=0 𝑥 𝑥 4𝑥 4 + 6𝑥 3 − 10𝑥 2 − 9𝑥 + 9 = 0 𝑥 + 1 5 81 = 𝑥5 + 1 11 2 𝑥 − 6𝑥 − 9 2 = 𝑥 3 − 4𝑥 2 − 9𝑥 25𝑥 2 2 𝑥 + = 11 𝑥+5 2 𝑥 2 − 16 𝑥 − 3 2 + 9𝑥 2 = 0

33.

𝑥 4 + 4𝑥 − 1 = 0

34.

𝑥 4 − 4𝑥 3 − 1 = 0

35.

𝑥 4 − 2𝑥 2 − 400𝑥 = 9999

27. 28. 29. 30. 31.

36.

𝑥 3 − 2𝑥 − 𝑥 2 + 2 𝑎 − 2𝑎2 𝑥 = 0

37.

𝑥2 − 𝑎

38.

2

− 6𝑥 2 + 4𝑥 + 2𝑎 = 0

ឹ ថាអងគ ខាងដឆវងោច ដោ​ោះរាយសម្ីការ 𝑥 4 − 4𝑥 3 − 10𝑥 2 + 37𝑥 − 14 = 0 ដោយដង

ោក្់ ជាផលគុ ណដដលមានដម្គុ ណជាចំនន ួ គត់ បាន។

39.

សម្ីការ 𝑥 3 + 𝑎𝑥 2 + 𝑏𝑥 + 𝑐 = 0 ដដល 𝑎, 𝑏, 𝑐 ជាចំនន ួ គត់ មានរ ឺសសនិទនម្ួ យ តាង

ដោយ 𝑥1 ។ ចូរបង្ហាញថា 𝑥1 ជាចំនន ួ គត់ និងថា 𝑐 ដចក្ោច់ នឹង 𝑥1 ។

40.

សម្ីការ 𝑥 4 + 𝑎𝑥 3 + 𝑏𝑥 2 + 𝑐𝑥 + 𝑑 = 0 ដដល 𝑎, 𝑏, 𝑐, 𝑑 ជាចំនន ួ គត់ មានរ ឺសសនិទនម្ួ យ

តាងដោយ 𝑥1 ។ ចូរបង្ហាញថា 𝑥1 ជាចំនន ួ គត់ និងថា 𝑑 ដចក្ោច់ នឹង 𝑥1 ។

41.

សនមតថា 𝑥1 , 𝑥2 , 𝑥3 ជារ ឺសននសម្ីការ 𝑎𝑥 3 + 𝑏𝑥 2 + 𝑐𝑥 + 𝑑 = 0, 𝑎 ≠ 0 ។ ចូរបង្ហាញថា

𝑥1 + 𝑥2 + 𝑥3 = − 𝑏 𝑎 , 𝑥1 𝑥2 + 𝑥2 𝑥3 + 𝑥3 𝑥1 = 𝑐 𝑎 , 𝑥1 𝑥2 𝑥3 = − 𝑑 𝑎 ។

42.

ដគដោយសម្ីការ 𝑥 3 + 𝑝𝑥 + 𝑞 = 0 ។ ចូរគណនាផលបូ ក្កាដរននរ ឺសរបស់ វា។

43*.

ដោ​ោះរាយសម្ីការ 𝑥 3 + 3𝑥 − 3 = 0 ។

ចូររក្រ ឺសសនិទនននសម្ីការ

44.

𝑥 4 + 2𝑥 3 − 16𝑥 2 − 2𝑥 + 15 = 0

លឹម សុ វណ្ណវិចិត្រ | III. សមីការ

3


45.

ЁЭСе 4 тИТ 7ЁЭСе 3 + 5ЁЭСе 2 + 4ЁЭСе + 12 = 0

46.

ЁЭСе 4 + ЁЭСе 3 тИТ 5ЁЭСе тИТ 5 = 0

47.

ЁЭСе4 + ЁЭСе3 тИТ 1 = 0

48.

6ЁЭСе 4 тИТ ЁЭСе 3 + 5ЁЭСе 2 тИТ ЁЭСе тИТ 1 = 0

сЮКсЯДсЯДсЯЗсЮЪсЮ╢сЮЩсЮЯсЮШсЯТсЮ╕сЮАсЮ╢сЮЪ

49.

ЁЭСе + 10 + ЁЭСе тИТ 2 = 6

50.

ЁЭСетИТ ЁЭСе+3=1

51.

15 тИТ ЁЭСе + 3 тИТ ЁЭСе = 6

52.

20 + ЁЭСе 20 тИТ ЁЭСе + = 6 ЁЭСе ЁЭСе

53.

ЁЭСе +1тИТ

ЁЭСе =ЁЭСО

54.

ЁЭСе 2 + 5ЁЭСе + 3 тИТ ЁЭСе 2 + 5ЁЭСе тИТ 2 = 1

55.

2ЁЭСе + 3 тИТ ЁЭСе + 1 = 1

56.

2ЁЭСе + 3 + ЁЭСе + 1 = 5

57.

2ЁЭСе тИТ 3 + 4ЁЭСе + 1 = 4

58.

ЁЭСе + 4 + 2ЁЭСе + 6 = 7

59.

2ЁЭСе тИТ 4 тИТ ЁЭСе + 5 = 1

60.

5ЁЭСе + 7 тИТ 2ЁЭСе + 3 = 3ЁЭСе + 4

61.

ЁЭСе2 + ЁЭСе + 4 + ЁЭСе2 + ЁЭСе + 1 =

62.

ЁЭСе 2 тИТ 9ЁЭСе + 24 тИТ 6ЁЭСе 2 тИТ 59ЁЭСе + 149 = 5 тИТ ЁЭСе

63. 64. 65.

2ЁЭСе 2 + 2ЁЭСе + 9

ЁЭСе + 12 ЁЭСе тИТ 64 = 0 ЁЭСетИТ4

=ЁЭСетИТ8 ЁЭСе+2 ЁЭСе тИТ 3 2 + 3ЁЭСе тИТ 22 =

ЁЭСе 2 тИТ 3ЁЭСе + 7

66.

2ЁЭСе 2 + 3ЁЭСе тИТ 5 2ЁЭСе 2 + 3ЁЭСе + 9 + 3 = 0

67.

ЁЭСе ЁЭСе 2 + 15 тИТ 2 = ЁЭСе

4

сЯб. сЮвсЮУсЮ╗сЮВсЮШсЮУсЯНсЮДсЮ╢сЮЩ | сЮЫсЮ╣сЮШ сЮЯсЮ╗ сЮЬсЮОсЯТсЮОсЮЬсЮ╖сЮЕсЮ╖сЮПсЯТсЮЪ

4

ЁЭСе 2 + 15


𝑥+1 𝑥−1 3 − = 𝑥−1 𝑥+1 2

68. 69. 70. 71.

𝑥+𝑎 𝑥−𝑎 +2 =3 𝑥−𝑎 𝑥+𝑎 4 2−𝑥+ =2 2−𝑥+3 4𝑥 2 + 12𝑥 1 + 𝑥 = 27 1 + 𝑥

72. 73*.

7−𝑥+ 𝑥−3=𝑎 4

4

𝑥−2+ 4−𝑥 =2

74.

𝑥 2 + 𝑎𝑥 − 2𝑎 = 𝑥 + 1

75.

2𝑥 − 1 − 𝑥 + 𝑎 = 0

76.

𝑎2 − 𝑥 𝑥 2 + 𝑎2 = 𝑎 − 𝑥

77.

𝑥− 𝑥−𝑎 =𝑎

78.

𝑥 + 𝑎2 + 2𝑎 − 3 + 𝑥 + 𝑎 + 2 − 2𝑎 + 2𝑎2 − 𝑎3 = 𝑎 1 − 𝑥

79. 80. 81. 82*.

𝑥−1

2

+ 𝑦+5

2

=0

𝑥 2 + 𝑦 2 − 2𝑥 + 6𝑦 + 10 = 0 𝑥+𝑦−𝑎

2

+ 𝑦−1

2

+ 𝑥+3

2

=0

(ទួរគី ១៩៩៨)

តាង 𝑎𝑛 ជាសវត វី ននចំនន ួ ពិត ក្ំនត់ ដោយ 𝑎1 = 𝑡 និង 𝑎𝑛+1 = 4𝑎𝑛 1 − 𝑎𝑛 ចំដ ោះរគប់ 𝑛 ≥ 1 ។ ដតើតនំ លរបស់ 𝑡 ខុសៗគ្ននចំនន ួ បុ នាមន ដដល 𝑎1998 = 0 ?

83*.

២០០១ ចូរក្ំនត់ រគប់ រតីធាតុននចំនន ួ ពិត 𝑎, 𝑏, 𝑐 ដដល 𝑎2 − 2𝑏 2 = 1, 2𝑏 2 − 3𝑐 2 = 1 និង 𝑎𝑏 + 𝑏𝑐 + 𝑐𝑎 = 1

84*.

(អន្តរជាតិ ១៩៦៣) ដោ​ោះរាយសម្ីការ

លឹម សុ វណ្ណវិចិត្រ | III. សមីការ

𝑥 2 − 𝑝 + 2 𝑥 2 − 1 = 𝑥 ដដល 𝑝 ជាបារដម្រតនិងជាចំនន ួ ពិត។ 5


85*.

(អន្តរជាតឡ ១៩៦៧) ិ ងលស ី ដគដោយសម្ីការ

𝑥 5 + 5𝜆𝑥 4 − 𝑥 3 + 𝜆𝛼 − 4 𝑥 2 − 8𝜆 + 3 𝑥 + 𝜆𝛼 − 2 = 0 ើ បីដោយសម្ីការមានរ ឺសដដលម្ិនោស្ស័យនឹង 𝜆 ដតម្ួ យគត់ ។ ក្) ចូរក្ំនត់ 𝛼 ដដម្ ើ បីដោយសម្ីការមានរ ឺសដដលម្ិនោស្ស័យនឹង 𝜆 ដតពីរគត់ ។ ខ) ចូរក្ំនត់ 𝛼 ដដម្

86*.

(អន្តរជាតិ ១៩៥៩)

ចូរដោ​ោះរាយសម្ីការខាងដរកាម្

ក្)

𝑥 + 2𝑥 − 1 + 𝑥 − 2𝑥 − 1 = 2

ខ)

𝑥 + 2𝑥 − 1 + 𝑥 − 2𝑥 − 1 = 1

គ)

𝑥 + 2𝑥 − 1 + 𝑥 − 2𝑥 − 1 = 2

87*. ដោ​ោះរាយសម្ីការ 𝑥+3−4 𝑥−1+ 𝑥+8−6 𝑥−1=1 88*. ដោ​ោះរាយសម្ីការ

𝑥 + 2 𝑥 + 2 𝑥 + ⋯ + 2 𝑥 + 2 3𝑥 = 𝑥 (មានរឌីកាល់ ចំនន ួ 𝑛 ដង)

89.

ដោ​ោះរាយសម្ីការ

𝑥 − 3 − 2 𝑥 − 4 + 𝑥 − 4 𝑥 − 4 = 𝑎, 90.

ដោ​ោះរាយសម្ីការ

91.

ដោ​ោះរាយសម្ីការខាងដរកាម្ចំដ ោះ 𝑥 ≥ −1

𝑎−𝑥

5

+ 𝑥−𝑏

2𝑥 2 + 4𝑥 = 92.

= 𝑎 − 𝑏 5; 𝑎 ≠ 𝑏 ។

𝑥+3 2

ដគដោយសម្ីការ 𝑥 2 − 3𝑥 + 1 = 𝑚 𝑥 4 + 𝑥 2 + 1 (*) ដដលក្នវងដនាោះ 𝑚 ជាបារដម្រត

និង 𝑥 ជាអញ្ញាត។ 6

5

១. អនុគមន៍ងាយ | លឹម សុ វណ្ណវិចិត្រ


ក្) ដោ​ោះរាយសម្ីការក្រណី 𝑚 = −

3 3

ើ បីដោយ សម្ីការ(*) មានរ ឺសចំនន ខ) ចូរក្ំនត់ តនំ ល 𝑚 ដដម្ ួ ម្ួ យ។

93.

ដោ​ោះរាយសម្ីការ 3

94.

7𝑥 + 1 −

3

𝑥2 − 𝑥 − 8 +

3

𝑥 2 − 8𝑥 − 1 = 2

ដោ​ោះរាយសម្ីការ

1 + 𝑥3 2 = 𝑥2 + 2 5

ត្រព័នស ធ មីការ ដោ​ោះរាយរបព័នស ធ ម្ីការ

95. 96. 97. 98. 99. 101. 103.

105. 107. 108. 109.

𝑥 2 − 𝑥𝑦 + 3𝑦 2 + 2𝑥 − 5𝑦 − 4 = 0 𝑥 + 2𝑦 = 4 2𝑥𝑦 − 𝑦 2 + 5𝑥 + 20 = 0 3𝑥 + 2𝑦 − 3 = 0 𝑥 2 − 4𝑦 2 = 200 𝑥 + 2𝑦 = 100 2 𝑥 + 9𝑦 2 + 6𝑥𝑦 − 6𝑥 − 18𝑦 − 40 = 0 𝑥 + 30 = 2𝑦 𝑥+𝑦 =5 100. 𝑥𝑦 = 6 𝑥 2 + 𝑦 2 = 10 102. 𝑥+𝑦 =4 1 1 104. + = 13 2 2 𝑥 𝑦 1 1 − =1 𝑥 𝑦 𝑥 5 + 𝑦 5 = 275 106. 𝑥+𝑦 =5 𝑥 + 𝑦 + 𝑥𝑦 = −11 𝑥 2 + 𝑦 2 + 𝑥𝑦 = 13 𝑥 2 𝑦 − 𝑥𝑦 2 = 30 𝑥 + 𝑥𝑦 − 𝑦 = 13 𝑥 + 0,2 2 + 𝑦 + 0,3 2 = 1 𝑥 + 𝑦 = 0,9

លឹម សុ វណ្ណវិចិត្រ | IV. ត្រព័នស ធ មីការ

𝑥−𝑦 =5 𝑥𝑦 = −4 𝑥 4 + 𝑦 4 = 17 𝑥+𝑦 =3 𝑥3 + 𝑦3 = 7 𝑥+𝑦 =1

𝑥 3 − 𝑦 3 = 63 𝑥𝑦 = 4

7


110.

𝑥+𝑦+

111.

𝑥 3 𝑦 + 𝑥 3 𝑦 2 + 2𝑥 2 𝑦 2 + 𝑥 2 𝑦 3 + 𝑥𝑦 3 = 30 𝑥 2 𝑦 + 𝑥𝑦 + 𝑥 + 𝑦 + 𝑥𝑦 2 = 11

112.

2𝑥 2 + 𝑥𝑦 − 45𝑦 2 = 0 2𝑥 + 9𝑦 2 = 4

113.

𝑥 2 − 5𝑥𝑦 = 16 2𝑥𝑦 + 𝑦 2 = 3 2𝑥 2 + 𝑦 2 + 3𝑥𝑦 = 12 1 𝑥 + 𝑦 2 − 𝑦2 = 7 2 𝑥 3 + 𝑥𝑦 2 = 10 𝑥 2 𝑦 − 𝑦 3 = −3 10𝑥 2 + 5𝑦 2 − 2𝑥𝑦 − 38𝑥 − 6𝑦 + 41 = 0 3𝑥 2 − 2𝑦 2 + 5𝑥𝑦 − 17𝑥 − 6𝑦 + 20 = 0 𝑦 2 𝑥 2 − 3 + 𝑥𝑦 + 1 = 0 𝑦 2 3𝑥 2 − 6 + 𝑥𝑦 + 2 = 0 𝑥 2 2 𝑦 119. 𝑥+𝑦−2 = + =3 𝑦 3 𝑥 3 𝑥 3 3 𝑦 + = 𝑥+𝑦−1 =9 2 𝑦 2 𝑥 𝑦 2 − 𝑥𝑦 + 2 = 0 𝑥𝑦 − 2 = 6 − 𝑥 2 121. 8 − 𝑥 2 = 𝑥 + 2𝑦 2 2 + 3𝑦 2 = 2𝑥𝑦 2𝑢 + 𝑣 + 𝑤 = 6 𝑥 + 2𝑦 − 𝑧 = 5 123. 3𝑢 + 2𝑣 + 𝑤 = 9 3𝑥 − 4𝑦 + 𝑧 = 1 2 2 2 3𝑢3 + 2𝑣 3 + 𝑤 3 = 27 𝑥 +𝑦 +𝑧 =6 𝑥𝑦 + 𝑥 + 𝑦 = 7 𝑥+𝑦+𝑧 =2 125. 𝑦𝑧 + 𝑦 + 𝑧 = −3 𝑥𝑦 + 𝑦𝑧 + 𝑧𝑥 = −5 𝑥𝑧 + 𝑥 + 𝑧 = −5 𝑥 2 + 𝑦 2 − 𝑧 2 = 12 𝑥 2 − 𝑦𝑧 = 14 𝑥 2 + 𝑥𝑦 + 𝑦 2 = 3 127. 𝑦 2 − 𝑥𝑧 = 28 𝑦 2 + 𝑦𝑧 + 𝑧 2 = 7 𝑧 2 − 𝑥𝑦 = −14 𝑧 2 + 𝑧𝑥 + 𝑥 2 = 19

114. 115. 116. 117. 118.

120. 122. 124. 126.

8

𝑥 1 = 𝑦 2 𝑥+𝑦 𝑥 1 =− 𝑦 2

១. អនុគមន៍ងាយ | លឹម សុ វណ្ណវិចិត្រ


128.

𝑥𝑦 + 𝑥𝑧 = 8 𝑦𝑧 + 𝑥𝑦 = 9 𝑥𝑧 + 𝑦𝑧 = −7

129.

130.

4𝑥𝑦 + 𝑦 2 + 2𝑧 2 = −3 4𝑥𝑧 + 𝑥 2 + 2𝑧 2 = 1 8𝑦𝑧 + 𝑦 2 + 2𝑧 2 = 1 7 4 5 − = 3 𝑥−7 𝑦+6

131.

132.

5 𝑥−7 133.

+

3 𝑦+6

4 𝑥+𝑦− 𝑥−𝑦 2 𝑥+𝑦− 𝑥−𝑦

134.

= −4 +4

5𝑥𝑦 =1 𝑥+𝑦 7𝑦𝑧 =1 𝑦+𝑧 6𝑥𝑧 = 𝑥 + 𝑧 𝑦 3 = 9𝑥 2 − 27𝑥 + 27 𝑧 3 = 9𝑦 2 − 27𝑦 + 27 𝑥 3 = 9𝑧 2 − 27𝑧 + 27

13 6 1 𝑥−𝑦− 4 𝑥+𝑦 4 𝑥−𝑦−

4

𝑥+𝑦

=1 =

9 4

𝑥+𝑦 2𝑥 − 1 +4 =5 2𝑥 − 1 𝑥+𝑦 𝑥 =𝑦+1

135.

𝑥−𝑦 𝑥 + 𝑦 10 + = 𝑥+𝑦 𝑥−𝑦 3 𝑥𝑦 − 2𝑥 − 2𝑦 = 2

136.

137. 138.

𝑥+𝑦+

𝑥+𝑦 12 = 𝑥−𝑦 𝑥−𝑦

𝑥𝑦 = 15 𝑦 𝑥 2 + 𝑦 2 − 2𝑎𝑦 − 3 = 0 𝑥 𝑥 2 + 𝑦 2 = 2𝑎𝑥 𝑥+𝑦 =2 𝑥𝑦 − 𝑧 2 = 1

ើ បីដោយរបព័នធសម្ីការខាងដរកាម្មានរ ឺសដតម្ួ យគត់ 139. ចូរក្ំនត់ តនំ ល 𝑎 ដដម្ លឹម សុ វណ្ណវិចិត្រ | IV. ត្រព័នស ធ មីការ

9


𝑦=

1

𝑥 𝑦 = 𝑎𝑥 + 1

140*. ដគដោយរបព័នធសម្ីការ 2 4 5 𝑥 + 𝑦 + 𝑧 = 61 3 5 6 𝑥 + 𝑦 + 𝑧 = 79 2 𝑧 ក្) ចូរគណនាផលបូ ក្ 5 𝑦 + 2 ; ខ) ក្នវងចំដនាម្រ ឺសជាចំនន ួ គត់ ធម្ម ជាតិទំងអស់ របស់ របព័នធ ចូរក្ំនត់ រ ឺសដដលមានតំនល 𝑥 ធំបំផុត។

141*. (អន្តរជាតិ ១៩៦១) ដោ​ោះរាយរបព័នស ធ ម្ីការ

𝑥+𝑦+𝑧 =𝑎 𝑥 2 + 𝑦 2 + 𝑧 2 = 𝑏2 𝑥𝑦 = 𝑧 2 ើ បីដោយ ដដល 𝑎 និង 𝑏 ជាចំនន ួ ពិតដដលដគដោយ។ ចូរក្ំនត់ លក្ខខណឌដលើ 𝑎 និង 𝑏 ដដម្ ិ មា ចំដលើ យវជ ជ ននិងមានតំនលខុសគ្នន។

142*. (អន្តរជាតិ ១៩៦៣) ដោ​ោះរាយរបព័នស ធ ម្ីការ

ដដលបារដម្រត 𝒚 ជាចំនន ួ ពិត។

𝑥5 + 𝑥2 𝑥1 + 𝑥3 𝑥2 + 𝑥4 𝑥3 + 𝑥5 𝑥4 + 𝑥1

= 𝑦𝑥1 = 𝑦𝑥2 = 𝑦𝑥3 = 𝑦𝑥4 = 𝑦𝑥5

143*. (អន្តរជាតិ ១៩៦៥) ដគដោយរបព័នធសម្ីការ

𝑎11 𝑥1 + 𝑎12 𝑥2 + 𝑎13 𝑥3 = 0 𝑎21 𝑥1 + 𝑎22 𝑥2 + 𝑎23 𝑥3 = 0 𝑎31 𝑥1 + 𝑎32 𝑥2 + 𝑎33 𝑥3 = 0 ដដលមានដម្គុ ណដផទ ៀងផ្ទទត់ លក្ខខណឌខាងដរកាម្ ិ មា ក្) 𝑎11 , 𝑎22 , 𝑎33 ជាចំនន ួ ពិតវជ ជ ន 10

១. អនុគមន៍ងាយ | លឹម សុ វណ្ណវិចិត្រ


ិ មា ខ) ដម្គុ ណដផេងដទៀតអវជ ជ នទំងអស់

ិ មា គ) សម្ីការនិម្ួយៗមានផលបូ ក្ដម្គុ ណវជ ជ ន។ ចូរបង្ហាញថា 𝑥1 = 𝑥2 = 𝑥3 = 0 ជាចំដលើ យដតម្ួ យគត់ របស់ របព័ន។ធ

144*. (អន្តរជាតិ ១៩៦៥)

ចូរក្ំនត់ ចំនន ួ ពិត 𝑥1 , 𝑥2 , 𝑥3 , 𝑥4 ដដល 𝑥1 + 𝑥2 𝑥3 𝑥4 𝑥2 + 𝑥1 𝑥3 𝑥4 𝑥3 + 𝑥1 𝑥2 𝑥4 𝑥4 + 𝑥1 𝑥2 𝑥3

=2 =2 =2 =2

145*. (អន្តរជាតិ ១៩៦៦)

ដោ​ោះរាយរបព័នស ធ ម្ីការ 𝑎1 − 𝑎2 𝑥2 + 𝑎1 − 𝑎3 𝑥3 + 𝑎2 − 𝑎1 𝑥1 + 𝑎2 − 𝑎3 𝑥3 + 𝑎3 − 𝑎1 𝑥1 + 𝑎3 − 𝑎2 𝑥3 + 𝑎4 − 𝑎1 𝑥1 + 𝑎4 − 𝑎2 𝑥3 + ដដល 𝑎1 , 𝑎2 , 𝑎3 និង 𝑎4 ជាចំនន ួ ពិតខុសគ្ននពីរៗ។

𝑎1 − 𝑎4 𝑎2 − 𝑎4 𝑎3 − 𝑎4 𝑎4 − 𝑎3

𝑥4 𝑥4 𝑥4 𝑥3

=1 =1 =1 =1

146*. (អន្តរជាតិ ឡងលស ី ១៩៦៧) ដោ​ោះរាយរបព័នស ធ ម្ីការ

𝑥2 + 𝑥 − 1 = 𝑦 𝑦2 + 𝑦 − 1 = 𝑧 𝑧2 + 𝑧 − 1 = 𝑥

147.

(អន្តរជាតិ ឡងលស ី ១៩៦៧) ដោ​ោះរាយរបព័នស ធ ម្ីការ

𝑥+𝑦 + 1−𝑥 =6 𝑥+𝑦+1 + 1−𝑦 =4

148*. (អន្តរជាតិ ឡងលីស១៩៦៧) ដោ​ោះរាយរបព័នស ធ ម្ីការ

លឹម សុ វណ្ណវិចិត្រ | IV. ត្រព័នស ធ មីការ

11


𝑥1 + 𝑥2 + ⋯ + 𝑥𝑛 = 𝑎 𝑥12 + 𝑥22 + ⋯ + 𝑥𝑛2 = 𝑎2 …… 𝑥1𝑛 + 𝑥2𝑛 + ⋯ + 𝑥𝑛𝑛 = 𝑎𝑛

149.

(អន្តរជាតិ ឡងលស ី ១៩៦៧) ដតើក្នវងក្រណី ណា ដដលរបព័នធ

𝑥 + 𝑦 + 𝑚𝑧 = 𝑎 𝑥 + 𝑚𝑦 + 𝑧 = 𝑏 𝑚𝑥 + 𝑦 + 𝑧 = 𝑐 មានចំដលើ យ? ចូរក្ំនត់ លក្ខខណឌដដលចំដលើ យដតម្ួ យគត់ របស់ របព័នខា ធ ងដលើ ជាសវត វី នពវ ន។ត

150*. (អន្តរជាតិ ១៩៦៧) ដគដោយសវត វី 𝒄𝒏 :

𝑐1 = 𝑎1 + 𝑎2 + ⋯ + 𝑎8 𝑐2 = 𝑎12 + 𝑎22 + ⋯ + 𝑎82 … …… 𝑐𝑛 = 𝑎1𝑛 + 𝑎2𝑛 + ⋯ + 𝑎8𝑛 … …… ឹ ថា ក្នវងចំដនាម្ 𝑐𝑛 ដដល 𝑎1 , 𝑎2 , … , 𝑎8 ជាចំនន ួ ពិត ដដលម្ិនសូនយទំងអស់ គ្នន។ ដោយដង ើ បីដោយ 𝑐𝑛 = 0 ។ មានចំនន ួ ដដលដសមើសន ូ យដរចើនរប់ ម្ិនអស់ ចូរក្ំនត់ តនំ លរបស់ 𝑛 ដដម្

151*. ដោ​ោះរាយរបព័នស ធ ម្ីការ

152*. ដោ​ោះរាយរបព័នស ធ ម្ីការ

𝑥 2 = 2𝑥 − 𝑦 𝑦 2 = 2𝑦 − 𝑧 𝑧 2 = 2𝑧 − 𝑡 𝑡 2 = 2𝑡 − 𝑥

𝑎−1 2 𝑎−1 𝑥22 + 𝑎𝑥2 + 2 ……………………

2

𝑥12 + 𝑎𝑥1 +

2 𝑥1000 + 𝑎𝑥1000 +

ដដល 𝑎 ជាចំនន ួ ពិត។ 12

១. អនុគមន៍ងាយ | លឹម សុ វណ្ណវិចិត្រ

= 𝑥2 2

= 𝑥3 𝑎−1 2

2

= 𝑥1


153*. ដោ​ោះរាយរបព័នស ធ ម្ីការ 2𝑥1 − 5𝑥2 + 3𝑥3 = 0 2𝑥2 − 5𝑥3 + 3𝑥4 = 0 …………… 2𝑥𝑛 − 5𝑥1 + 3𝑥2 = 0

ដដល 𝑛 ជាចំនន ួ គត់ ≥ 3 ។

154*. តាង 𝑥, 𝑦, 𝑧 ជាចំដលើយរបស់ របព័នស ធ ម្ីការ 𝑥 =𝑦 4−𝑦 𝑦 =𝑧 4−𝑧 𝑧 =𝑥 4−𝑥 ចូរក្ំនត់ តនំ លរបស់ ផលបូ ក្ 𝑆 = 𝑥 + 𝑦 + 𝑧 ។

155.

(កាណាដា ១៩៧០) ដោ​ោះរាយសម្ីការ 𝑥 + 𝑦𝑧 = 2 𝑦 + 𝑧𝑥 = 2 𝑧 + 𝑥𝑦 = 2

156.

(អន្តរជាតិ ១៩៦៨) ដគដោយរបព័នធសម្ីការ

𝑎𝑥 2 + 𝑏𝑥 + 𝑐 = 𝑦 𝑎𝑦 2 + 𝑏𝑦 + 𝑐 = 𝑧 𝑎𝑧 3 + 𝑏𝑧 + 𝑐 = 𝑥 ដដលក្នវងដនាោះ 𝑎 ≠ 0 ។ តាង Δ = 𝑏 − 1 2 − 4𝑎𝑐 ។ ចូរបង្ហាញថាដបើ Δ < 0 ដនាោះរបព័នធ សម្ីការគ្នមនរ ឺស។

157.

ើ បីដោយរបព័នធ ចូរក្ំនត់ 𝑎 ∈ ℝ ដដម្ 1 𝑎+1 2 2 𝑥 3 + 𝑎𝑥 2 𝑦 + 𝑥𝑦 2 = 1 𝑥 3 − 𝑎𝑦 3 = មានរ ឺសដផទ ៀងផ្ទទត់ 𝑥 + 𝑦 = 0

158.

ដោ​ោះរាយសម្ីការ 2 − 𝑥 3𝑥 − 2𝑧 = 3 − 𝑧 𝑦 3 + 3𝑦 2 = 𝑥 2 − 3𝑥 + 2 𝑦 2 + 𝑧 2 = 6𝑧 𝑧≤3

លឹម សុ វណ្ណវិចិត្រ | IV. ត្រព័នស ធ មីការ

(1) (2) (3) (4) 13


159.

ដគដោយ 𝑎, 𝑏, 𝑐 > 0 ។ដោ​ោះរាយសម្ីការ 𝑎 𝑏 − = 𝑐 − 𝑧𝑥 𝑥 𝑧 𝑏 𝑐 − = 𝑎 − 𝑥𝑦 𝑦 𝑥 𝑐 𝑎 − = 𝑏 − 𝑦𝑧 𝑧 𝑦

160.

ដោ​ោះរាយរបព័នស ធ ម្ីការ 2𝑥 + 𝑥 2 𝑦 = 𝑦 2𝑦 + 𝑦 2 𝑧 = 𝑧 2𝑧 + 𝑧 2 𝑥 = 𝑥

វិសមភាព ចូរបង្ហាញថា

161. 162. 163. 164. 165. 166.

1 10 2 2100

<

1 3 5 99 1 … < 2 4 6 100 10

2100 100 < < 50 10 10 2 1 3 5 2𝑛 − 1 1 . . … ≤ ,𝑛 ∈ ℕ 2 4 6 2𝑛 3𝑛 + 1 1 3 5 99 1 … < 2 4 6 100 12 ិ ម្ភាពខាងដរកាម្ពិត ចូរបង្ហាញថា ចំដ ោះរគប់ ចំនន ួ គត់ ធម្ម ជាតិ 𝑛 > 1 វស 1 1 1 13 + + ⋯+ > 𝑛+1 𝑛+2 𝑛 + 𝑛 24 ិ ម្ភាពខាងដរកាម្ពិត ចូរបង្ហាញថា ចំដ ោះរគប់ ចំនន ួ គត់ ធម្ម ជាតិ 𝑛 វស 1 𝑛 <3 𝑛 រ ឺ 2 ?។

2< 1+

167.

ដតើម្ួយណាធំជាង 1,000001

168.

ដតើម្ួយណាធំជាង 1 0001 000 រ ឺ 1001999 ?។

169.

(កាណាដា ១៩៦៩) ដតើម្ួយណាធំជាង

14

1.000.000

𝑐 + 1 − 𝑐 រ ឺ 𝑐 − 𝑐 − 1 ចំដ ោះ 𝑐 ≥ 1?។

១. អនុគមន៍ងាយ | លឹម សុ វណ្ណវិចិត្រ


170.

ឡ áž˜ážś ផលគឝ ណននចá&#x;†áž“áž“ áž˝ ពឡáž?វជ ជ áž“áž…á&#x;†áž“áž“ áž˝ đ?‘› áž˜ážśáž“áž?á&#x;†áž“លដáž&#x;មមá&#x;’ ážž áž˝ áž™á&#x;” ចសរបងá&#x;’ហវញáž?វផលបស កá&#x;’ážšáž”áž&#x;á&#x;‹ ážœážś មá&#x;’ážˇáž“áž˜ážśáž“ áž?á&#x;†áž“áž›áž?សចជវង đ?‘› ដទá&#x;”

171.

áž&#x;នមáž?áž?ážś đ?‘Ľ1 , đ?‘Ľ2 , đ?‘Ľ3 , ‌ , đ?‘Ľđ?‘› đ?‘› ∈ â„• ជវចá&#x;†áž“áž“ áž˝ ពឡáž?áž˜ážśáž“áž&#x;ញá&#x;’ញវដសចគá&#x;’áž“áž“ áž˜ážśáž“áž?á&#x;†áž“លធá&#x;†áž‡ážśáž„ −1 á&#x;” áž…ážźážš បងá&#x;’ហវញáž?ážś 1 + đ?‘Ľ1 1 + đ?‘Ľ2 ‌ 1 + đ?‘Ľđ?‘› ≼ 1 + đ?‘Ľ1 + đ?‘Ľ2 + â‹Ż + đ?‘Ľđ?‘› á&#x;”

172. 173.

ឡ មá&#x;’áž—ážśáž–áž ážśáž„ážŠážšáž€ážśáž˜á&#x;’ពឡáž? ចសរបងá&#x;’ហវញáž?ážś áž…á&#x;†ážŠ á&#x;„á&#x;‡ážšáž‚áž”á&#x;‹ áž…á&#x;†áž“áž“ áž˝ áž‚áž?á&#x;‹ ធមá&#x;’ម ជវáž?ឡ đ?‘› > 6 ážœáž&#x; đ?‘› đ?‘› đ?‘› đ?‘› > đ?‘›! > 2 3 2 ដគដá&#x;„áž™ đ?‘“ đ?‘Ľ = đ?‘Žđ?‘Ľ + 1998đ?‘Ľ + đ?‘? ដដល đ?‘Ž; đ?‘? ∈ ℤ; đ?‘Ž < 2000; đ?‘? < 2000 ដ ážž áž™ đ?‘“ áž˜ážśáž“ážš ážşáž&#x;ពីរដផá&#x; áž„áž‚á&#x;’áž“áž“áž‚ážş đ?‘Ľ1 ; đ?‘Ľ2 á&#x;” ចសរបងá&#x;’ហវញáž?ážś đ?‘Ľ1 − đ?‘Ľ2 ≼ 1 998 á&#x;”

174.

ឡ áž˜ážś ដគដá&#x;„យចá&#x;†áž“áž“ áž˝ ពឡáž?វជ ជ áž“ đ?‘Ž, đ?‘?, đ?‘Ľ, đ?‘Ś ដផទ á&#x;€áž„áž•á&#x;’áž‘áž‘áž?á&#x;‹ ចសរបងá&#x;’ហវញáž?ážś

đ?‘Ž + đ?‘? = 1; đ?‘Žđ?‘Ľ + đ?‘?đ?‘Ś = 2; đ?‘Žđ?‘Ľ 2 + đ?‘?đ?‘Ś 2 = 3 4 < đ?‘Žđ?‘Ľ 3 + đ?‘?đ?‘Ś 3 < 4,5

175.

គណនវáž?á&#x;†áž“áž›áž?ážźáž…áž”á&#x;† áž•ážťáž?áž“áž“

176.

2đ?‘Ľ − 1 đ?‘Ľ+3 ážž បីដá&#x;„áž™đ?‘“ đ?‘Ľ = đ?‘Žđ?‘Ľ 2 + đ?‘?đ?‘Ľ + đ?‘? ដផទ á&#x;€áž„áž•á&#x;’áž‘áž‘áž?á&#x;‹ đ?‘“ đ?‘Ľ ចសរកá&#x;’á&#x;†áž“áž?á&#x;‹ đ?‘Ž, đ?‘?, đ?‘? ដដមá&#x;’ đ?‘†= đ?‘Ľ +

≤ 1 áž…á&#x;†ážŠ á&#x;„á&#x;‡ážšáž‚áž”á&#x;‹

8

đ?‘Ľ ∈ −1; 1 នឡងដដល đ??ž = 3 đ?‘Ž2 + 2đ?‘? 2 áž˜ážśáž“áž?á&#x;†áž“លធá&#x;†áž”á&#x;†áž•ážťáž?á&#x;”

177.

ឡ áž˜ážś ដគដá&#x;„យចá&#x;†áž“áž“ áž˝ ពឡáž?មá&#x;’ážˇáž“áž˘ážœáž‡ ជ áž“ đ?‘Ž1 ; đ?‘Ž2 ; đ?‘Ž3 ; đ?‘Ž4 ; đ?‘Ž5 ដដលđ?‘Ž1 + đ?‘Ž2 + đ?‘Ž3 + đ?‘Ž4 + đ?‘Ž5 = 1 á&#x;” ចសរបងá&#x;’ហវញáž?ážś 1 1 ; min đ?‘Žđ?‘–2 − đ?‘Žđ?‘—2 ≤ đ?‘–≠đ?‘— 100 36 áž?ážśáž„ đ?‘Ž, đ?‘?, đ?‘? ជវរងá&#x;’áž ážœáž&#x;á&#x;‹ រជឝ áž„áž“áž“ážšáž?áž¸ážŠáž€ážśážŽáž˜á&#x;’áž˝ áž™á&#x;” ចសរបងá&#x;’ហវញáž?ážś đ?‘Ž đ?‘? đ?‘? + + <2 đ?‘?+đ?‘? đ?‘?+đ?‘Ž đ?‘Ž+đ?‘? min đ?‘Žđ?‘– − đ?‘Žđ?‘— đ?‘–≠đ?‘—

178.

2

≤

áž…á&#x;†áž“ á&#x;†

បរឞជរ áž˝ លកá&#x;’ហហណá&#x;’ឌ đ?‘Ž, đ?‘?, đ?‘? ជវរងវវáž&#x;á&#x;‹ áž?á&#x;’ជង ážť áž?á&#x;’រីបកវណá&#x;’ áž…ážźážšáž&#x;វកá&#x;’លបងáž?ážśáž„ đ?‘Ž = đ?‘Ľ + đ?‘Ś, đ?‘? = đ?‘Ś + đ?‘§, đ?‘? = đ?‘§ + đ?‘Ľ

លរម áž&#x;ážť វណá&#x;’ážŽážœážˇáž…ážˇáž?á&#x;’ážš | V. ážœážˇáž&#x;áž˜áž—ážśáž–

15


179.

ដគដá&#x;„áž™ đ?‘Ž, đ?‘?, đ?‘? ជវរងá&#x;’áž ážœáž&#x;á&#x;‹ រជឝ áž„áž“áž“ážšáž?áž¸ážŠáž€ážśážŽáž˜á&#x;’áž˝ áž™á&#x;” ចសរបងá&#x;’ហវញáž?ážś 3 đ?‘Žđ?‘? + đ?‘?đ?‘? + đ?‘?đ?‘Ž ≤ đ?‘Ž + đ?‘? + đ?‘?

180.

2

< 4 đ?‘Žđ?‘? + đ?‘?đ?‘? + đ?‘?đ?‘Ž

áž?ážśáž„ đ?‘Ž, đ?‘?, đ?‘? ជវរងá&#x;’áž ážœáž&#x;á&#x;‹ រជឝ áž„áž“áž“ážšáž?áž¸ážŠáž€ážśážŽáž˜á&#x;’áž˝ áž™á&#x;” ចសរបងá&#x;’ហវញáž?ážś 2đ?‘Ž2 + 2đ?‘? 2 − đ?‘? 2 2đ?‘?2 + 2đ?‘? 2 − đ?‘Ž2 2đ?‘? 2 + 2đ?‘Ž2 − đ?‘? 2 ≤ 2đ?‘Ž2 + đ?‘?đ?‘? 2đ?‘?2 + đ?‘?đ?‘Ž 2đ?‘? 2 + đ?‘Žđ?‘?

181. 182.

ážš áž?ážś đ?‘“ đ?‘Ľ = đ?‘Žđ?‘Ľ 2 + đ?‘?đ?‘Ľ + đ?‘? ≼ 0; ∀đ?‘Ľ ∈ â„?; đ?‘Ž < đ?‘? á&#x;” ចសរកá&#x;’á&#x;†áž“áž?á&#x;‹ min đ??š ដដល ដá&#x;„យដង đ?‘Ž+đ?‘?+đ?‘? đ??š= đ?‘?−đ?‘Ž

(អនá&#x;’áž?រជវáž?ឡ á&#x;Ąá&#x;Šá&#x;Śá&#x;Ą)

áž?ážśáž„ đ?‘Ž, đ?‘? នឡង đ?‘? ជវរងá&#x;’áž ážœáž&#x;á&#x;‹ រជឝ áž„áž“áž“ážšáž?áž¸ážŠáž€ážśážŽáž˜á&#x;’áž˝ áž™ ážŠážŠáž›áž˜ážśáž“ážšáž€á&#x;’លវនផទ đ?‘† á&#x;” ចសរបងá&#x;’ហវញáž?ážś đ?‘Ž2 + đ?‘? 2 + đ?‘? 2 ≼ 4đ?‘† 3 ដáž?ឞដពលណវដយឞ áž„áž˜ážśáž“áž&#x;មá&#x;’áž—ážśáž–?á&#x;”

183.

(អនá&#x;’áž?រជវáž?ឡ ឥងលáž&#x; ី á&#x;Ąá&#x;Šá&#x;Śá&#x;§)

ឡ មá&#x;’áž—ážśáž– ចសរបងá&#x;’ហវញáž?ážś ážšáž‚áž”á&#x;‹ áž‚ážź វច ážť ឡ áž‘á&#x;?ážš đ?‘“ នឡង đ?‘” áž&#x;áž?áž? ឡ កá&#x;’នវងបល áž„á&#x;‹ ážœáž&#x; đ?‘Žđ?‘“ 2 + đ?‘?đ?‘“đ?‘” + đ?‘?đ?‘”2 ≼ 0 ពឡáž? ដបឞនង ឡ áž˜ážśáž“ážŠáž?ដបឞ លកá&#x;’áž áž ážŽážŒážŠážźáž…áž?ដá&#x;…ដនá&#x;„á&#x;‡ážšáž?ážźážœáž”ážśáž“ážŠáž•áž‘ á&#x;€áž„áž•á&#x;’áž‘áž‘áž?á&#x;‹ á&#x;– đ?‘Ž ≼ 0; đ?‘? ≼ 0; 4đ?‘Žđ?‘? ≼ đ?‘? 2 á&#x;”

184.

ដគដá&#x;„យចá&#x;†áž“áž“ áž˝ áž‚áž?á&#x;‹ đ?‘› ≼ 2 នឡង áž…á&#x;†áž“áž“ áž˝ ពឡáž? đ?‘Ž1 ; đ?‘Ž2 ; ‌ ; đ?‘Žđ?‘› ដដល đ?‘Žđ?‘– − đ?‘Žđ?‘— ≼

185.

(អនá&#x;’áž?រជវáž?ឡ á&#x;˘á&#x; á&#x; á&#x; )

đ?‘–<đ?‘—

đ?‘› 2

đ?‘› đ?‘–=1 đ?‘Žđ?‘–

đ?‘›

đ?‘Žđ?‘– đ?‘–=1

ដគឹá&#x;’áž™ đ?‘Ž, đ?‘?, đ?‘? > 0 ដដល đ?‘Žđ?‘?đ?‘? = 1á&#x;” ចសរបងá&#x;’ហវញáž?ážś 1 1 1 đ?‘Žâˆ’1+ đ?‘?−1+ đ?‘?−1+ ≤1 đ?‘? đ?‘? đ?‘Ž

186.

(áž ážť áž„áž‚á&#x;’គី á&#x;Ąá&#x;Šá&#x;Šá&#x;Ś)

ឡ áž˜ážś áž?ážśáž„ đ?‘Ž នឡង đ?‘? ជវចá&#x;†áž“áž“ áž˝ ពឡáž?វជ ជ áž“ ដដល đ?‘Ž + đ?‘? = 1 á&#x;” ចសរបងá&#x;’ហវញáž?ážś đ?‘Ž2 đ?‘?2 1 + ≼ đ?‘Ž+1 đ?‘?+1 3

187. 16

(áž‚á&#x;’áž‘áž&#x; ážš áž” ី áž? áž‘) á&#x;Ą. áž˘áž“ážťáž‚áž˜áž“á&#x;?áž„ážśáž™ | លរម áž&#x;ážť វណá&#x;’ážŽážœážˇáž…ážˇáž?á&#x;’ážš

= 0 á&#x;” ចសរបងá&#x;’ហវញáž?ážś


ចូរបង្ហាញថា ក្) ចំដ ោះរគប់ ចំនន ួ ពិត 𝑎, 𝑏 ដគមាន ិ ។ ដសមគ្ន ើ ន ដបើ 𝑎 = 𝑏និង រាសម្ក្វញ

𝑎2 + 𝑏 2 ≥ 2𝑎𝑏 និង 4𝑎𝑏 ≤ 𝑎 + 𝑏

2

វា

ខ) ចំដ ោះរគប់ ចំនន ួ ពិត 𝑥 > 0 ដគមាន 𝑥 + 1 𝑥 ≥ 2 ។ វាដសមគ្ន ើ ន ដបើ 𝑥 = 1 និង ិ ។ រាសម្ក្វញ

188.

(កាណាដា ១៩៧១)

ិ មា តាង 𝑥 និង 𝑦 ជាចំនន ួ ពិតវជ ជ ន ដដល 𝑥 + 𝑦 = 1 ។ ចូរបង្ហាញថា 1 1 1+ 1+ ≥9 𝑥 𝑦

189.

190.

ដគដោយ២ចំនន ួ ពិត 𝑎, 𝑏 ម្ិនសូនយ។ ចូរក្ំនត់ តនំ លតូចបំ ផុតនន

(រុ ស្ុ ី ១៩៩៥)

𝑎6 𝑎4 𝑎2 𝑏 6 𝑏 4 𝑏 2 + + + + + 𝑏 6 𝑏 4 𝑏 2 𝑎6 𝑎4 𝑎2

ចូរបង្ហាញថា ចំដ ោះរគប់ ចំនន ួ ពិត 𝑥, 𝑦 > 0 𝑥 𝑦 1 + 2 ≤ 4 2 4 𝑥 +𝑦 𝑥 +𝑦 𝑥𝑦

191.

ចូរបង្ហាញថា ចំដ ោះរគប់ ចំនន ួ ពិត 𝑥, 𝑦 ដគមាន 𝑥2 + 𝑦2 + 1 > 𝑥 𝑦2 + 1 + 𝑦 𝑥2 + 1

192.

ដគឱ្យ 𝑥, 𝑦, 𝑧 > 0 ។ ចូរបង្ហាញថា 𝑥2 𝑦2 𝑧2 𝑥 𝑦 𝑧 + + ≥ + + 𝑦2 𝑧2 𝑥2 𝑧 𝑥 𝑦

193.

ចូរបង្ហាញថា ដបើ 𝑎, 𝑏, 𝑐 ជារង្ហវស់ រជុ ងរបស់ រតីដកាណម្ួ យ ដនាោះដគមាន

194.

𝑎+𝑏 𝑏+𝑐 𝑐+𝑎 ≥ 8 𝑎+𝑏−𝑐 𝑏+𝑐−𝑎 𝑐+𝑎−𝑏 ិ មា ដគឱ្យចំនន ួ ពិតវជ ជ ន 𝑥, 𝑦, 𝑧 ។ ចូរបង្ហាញថា 𝑥2 − 𝑧2 𝑦2 − 𝑥2 𝑧2 − 𝑦2 + + ≥0 𝑦+𝑧 𝑧+𝑥 𝑥+𝑦 ចូរក្ំណត់ ក្រណី សម្ភាព។

195.

(អាសុ ប ី ៉ា សុ ភ ី ច ិ ១៩៩៦)

ដគឱ្យ 𝑎, 𝑏, 𝑐 ជារង្ហវស់ រជុ ងននរតីដកាណម្ួ យ។ ចូរបង្ហាញថា 𝑎+𝑏−𝑐+ 𝑏+𝑐−𝑎+ 𝑐+𝑎−𝑏 ≤ 𝑎+ 𝑏+ 𝑐 លឹម សុ វណ្ណវិចិត្រ | V. វិសមភាព

17


196.

(áž˘ážśáž˜áž˜ážšáž… ឡ á&#x;Ąá&#x;Šá&#x;Šá&#x;§)

ដគដá&#x;„áž™ đ?‘Ž, đ?‘?, đ?‘? > 0 á&#x;” ចសរបងá&#x;’ហវញáž?ážś 1 1 1 1 + 3 + 3 ≤ 3 3 3 3 đ?‘Ž + đ?‘? + đ?‘Žđ?‘?đ?‘? đ?‘? + đ?‘? + đ?‘Žđ?‘?đ?‘? đ?‘? + đ?‘Ž + đ?‘Žđ?‘?đ?‘? đ?‘Žđ?‘?đ?‘?

197.

(អនá&#x;’áž?រជវáž?áž&#x; á&#x;Ąá&#x;Šá&#x;Šá&#x;Ś) ឡ áž?áž›áž&#x; ី

ដគដá&#x;„áž™ đ?‘Ž, đ?‘?, đ?‘? > 0 ដដល đ?‘Žđ?‘?đ?‘? = 1 á&#x;” ចសរបងá&#x;’ហវញáž?ážś đ?‘Žđ?‘? đ?‘?đ?‘? đ?‘?đ?‘Ž + 5 + 5 ≤1 5 5 5 đ?‘Ž + đ?‘? + đ?‘Žđ?‘? đ?‘? + đ?‘? + đ?‘?đ?‘? đ?‘? + đ?‘Ž5 + đ?‘?đ?‘Ž

198.

(ážœáž&#x;áž˜áž—ážśáž–áž€ážź áž&#x;ឝីáž&#x;á&#x;’ážœ ážś áž?) ឡ

ចសរបងá&#x;’ហវញáž?ážś ážšáž‚áž”á&#x;‹ áž…á&#x;†áž“áž“ áž˝ ពឡáž? đ?‘Ž1 , đ?‘Ž2 , ‌ , đ?‘Žđ?‘› , đ?‘?1 , đ?‘?2 , ‌ , đ?‘?đ?‘› ážŠáž‚áž˜ážśáž“ đ?‘Ž12 + â‹Ż + đ?‘Žđ?‘›2 đ?‘?12 + â‹Ż + đ?‘?đ?‘›2 ≼ đ?‘Ž1 đ?‘?1 + â‹Ż + đ?‘Žđ?‘› đ?‘?đ?‘› 2 ឡ áž‘á&#x;?ážš đ?‘Ž1 , ‌ , đ?‘Žđ?‘› នឡង đ?‘?1 , ‌ , đ?‘?đ?‘› កá&#x;’ážź លីដនដអá&#x;Šá&#x;‚ážšáž“ážšáž„áž‚á&#x;’áž“áž“ á&#x;” អងគ áž‘á&#x;†áž„á&#x;˘ážŠáž&#x;មគá&#x;’áž“ ážž áž“ ដបឞ ážœážťáž…

áž&#x;á&#x;† áž‚ážśáž›á&#x;‹

កá&#x;’áž“áž„ ážť គណá&#x;’ážˇážšážœážˇáž‘á&#x;’យវ ážœážˇáž&#x;áž˜áž—ážśáž–áž€á&#x;’ážźáž&#x;ី ឝáž&#x;ážśážśážš អន កá&#x;’áž áž›á&#x;‡áž”á&#x;…áž?ážśážœážˇáž&#x;áž˜áž—ážśáž–áž&#x;ážśážśážš ážœážˇáž&#x;áž˜áž—ážśáž–áž€á&#x;’ážźáž&#x;ី ážť រឺកá&#x;’á&#x;? ážœážˇáž&#x;áž˜áž—ážśáž– កá&#x;’ážźáž&#x;ី ឝរឝនវកá&#x;’ážźážœáž&#x;គីáž&#x;ážśážśážš (áž‚ážşáž”á&#x;…áž?ážśáž˜áž”á&#x;’មá&#x;‡ អស áž‚ážťáž&#x;ážśáž?ážśá&#x;†áž„ លវáž&#x; ី កá&#x;’ážźáž&#x;ី(ážť Augustin Louis Cauchy), ážœážˇáž…áž‘á&#x;’áž™á&#x;?ážš áž™á&#x;‰ážśáž€á&#x;’ážźáž”áž›ážœážˇáž… រឝនវកá&#x;’ážźážœáž&#x;គី (Viktor Yakovlevich Bunyakovsky, គណá&#x;’ážˇážšážœážˇáž‘á&#x;’យរស áž&#x; ážť á&#x;’ážť,ី á&#x;Ąá&#x;¨á&#x; á&#x;¤ď€­á&#x;Ąá&#x;¨á&#x;¨á&#x;Š)នឡង áž

រមá&#x;‰ážśáž“ áž˘ážśáž˜á&#x;‰ážśáž“ážŒážťáž&#x; áž&#x;ážśážśážš(Hermann Amandus Schwarz, គណá&#x;’ážˇážšážœážˇáž‘á&#x;’យអវ ážź លលម ážş á&#x;‰ážśáž„,á&#x;‹ á&#x;Ąá&#x;¨á&#x;¤á&#x;Łď€­á&#x;Ąá&#x;Šá&#x;˘á&#x;Ą))á&#x;”

199. 200. 201.

ចសរបងá&#x;’ហវញáž?ážś áž…á&#x;†ážŠ á&#x;„á&#x;‡ážšáž‚áž”á&#x;‹ đ?‘Ľ1 , đ?‘Ľ2 , ‌ , đ?‘Ľđ?‘› > 0 ážŠáž‚áž˜ážśáž“ 1 1 1 đ?‘Ľ1 + đ?‘Ľ2 + â‹Ż + đ?‘Ľđ?‘› + +â‹Ż+ ≼ đ?‘›2 đ?‘Ľ1 đ?‘Ľ2 đ?‘Ľđ?‘› ចសរបងá&#x;’ហវញáž?ážś áž…á&#x;†ážŠ á&#x;„á&#x;‡ážšáž‚áž”á&#x;‹ đ?‘Ž, đ?‘?, đ?‘? > 0 ážŠáž‚áž˜ážśáž“ đ?‘Ž + 2đ?‘? + 3đ?‘? 2 ≤6 đ?‘Ž2 + 2đ?‘? 2 + 3đ?‘? 2 ឡ áž˜ážś ដគដá&#x;„យចá&#x;†áž“áž“ áž˝ ពឡáž? đ?‘Ž, đ?‘?, đ?‘? វជ ជ áž“á&#x;„áž…á&#x;‹ áž ážśáž? ដដល đ?‘Ž + đ?‘? + đ?‘? = 1 á&#x;” ចសរបងá&#x;’ហវញáž?ážś 1 đ?‘Ž+ đ?‘Ž

202. 18

(áž˘ážśáž˜áž˜ážšáž… ឡ á&#x;Ąá&#x;Šá&#x;§á&#x;¨)

2

1 + đ?‘?+ đ?‘?

á&#x;Ą. áž˘áž“ážťáž‚áž˜áž“á&#x;?áž„ážśáž™ | លរម áž&#x;ážť វណá&#x;’ážŽážœážˇáž…ážˇáž?á&#x;’ážš

2

1 + đ?‘?+ đ?‘?

2

≼

100 3


ដគឱ្យចំនន ួ ពិត 𝑎, 𝑏, 𝑐, 𝑑, 𝑒 ដដល 𝑎+𝑏+𝑐+𝑑+𝑒 =8 𝑎2 + 𝑏 2 + 𝑐 2 + 𝑑2 + 𝑒 2 = 16 ចូរក្ំណត់ តនម្ល ធប ំ ំ ផុតរបស់ 𝑒 ។

203.

(អូស្ត្ស្វ ត លី ១៩៩៣)

ដគឱ្យចំនន ួ គត់ 𝑛 > 1, ចំនន ួ ពិត 𝑎1 , 𝑎2 , … , 𝑎𝑛 > 0 និង 𝑠 = 𝑎1 + 𝑎2 + ⋯ + 𝑎𝑛 ។ ចូរបង្ហាញថា

𝑛

𝑖=1 𝑛

𝑖=1 𝑛

𝑖=1

204.

𝑠 𝑛2 ≥ 𝑠 − 𝑎𝑖 𝑛 − 1 𝑠 − 𝑎𝑖 ≥ 𝑛 𝑛−1 𝑎𝑖 𝑎𝑖 𝑛 ≥ 𝑠 − 𝑎𝑖 𝑛 − 1

(ចន្ ិ ១៩៨៧ ១៩៨៨)

ក្) ដគដោយ 𝑎1 , 𝑎2 , 𝑎3 > 0 ដដល 𝑎12 + 𝑎22 + 𝑎32

2

> 2 𝑎14 + 𝑎24 + 𝑎34 ។ ចូរបង្ហាញថា

𝑎1 , 𝑎2 , 𝑎3 ជារង្ហវស់ រជុ ងននរតីដកាណម្ួ យ។ ខ) ដគដោយចំនន ួ គត់ 𝑛 ≥ 3 និង 𝑎1 , 𝑎2 , … , 𝑎𝑛 > 0 ដដល 𝑎12 + 𝑎22 + ⋯ + 𝑎𝑛2 2 > 𝑛 − 1 𝑎14 + 𝑎24 + ⋯ + 𝑎𝑛4 ចូរបង្ហាញថា រគប់ 𝑖, 𝑗, 𝑘 ខុសគ្ននពីរៗ ចំនន ួ 𝑎𝑖 , 𝑎𝑗 , 𝑎𝑘 ជារង្ហវស់ រជុ ងននរតីដកាណម្ួ យ។

205.

ដគដោយ 𝑎1 , 𝑎2 , … , 𝑎𝑛 > 0 ។ តាង 𝑆 = 𝑎1 + 𝑎2 + ⋯ + 𝑎𝑛 និង 𝑆2 = 𝑎12 + 𝑎22 + ⋯ +

𝑎𝑛2 ។ ចូរបង្ហាញថា 𝑆2 − 𝑎12 𝑆2 − 𝑎22 𝑆2 − 𝑎𝑛2 + + ⋯+ ≥ 𝑆1 𝑆1 − 𝑎1 𝑆1 − 𝑎2 𝑆1 − 𝑎𝑛

206.

(សង ឹ ហបរូ ី ២០០០)

ដគដោយ 𝑎, 𝑏, 𝑐, 𝑑 > 0 ដដល 𝑎2 + 𝑏 2 = 𝑐 2 + 𝑑2 3

207.

3

។ ចូរបង្ហាញថា

3

𝑐 𝑑 + ≥1 𝑎 𝑏 ដគដោយ 𝑥, 𝑦, 𝑧 > 1 ដដល 1 𝑥 + 1 𝑦 + 1 𝑧 = 2 ។ ចូរបង្ហាញថា 𝑥+𝑦+𝑧 ≥ 𝑥−1+ 𝑦−1+ 𝑧−1

លឹម សុ វណ្ណវិចិត្រ | V. វិសមភាព

19


208.

(ូណវ ážś á&#x;˘á&#x; á&#x; á&#x;Ą) ដគដá&#x;„áž™ đ?‘Ľ, đ?‘Ś, đ?‘§ > 0 ដដល đ?‘Ľđ?‘Śđ?‘§ ≼ đ?‘Ľđ?‘Ś + đ?‘Śđ?‘§ + đ?‘§đ?‘Ľ á&#x;” ចសរបងá&#x;’ហវញáž?ážś đ?‘Ľđ?‘Śđ?‘§ ≼ 3 đ?‘Ľ + đ?‘Ś + đ?‘§

209.

ដគដá&#x;„áž™ đ?‘Ž, đ?‘?, đ?‘? ដដល đ?‘Ž + đ?‘? + đ?‘? = đ?‘Žđ?‘?đ?‘? á&#x;” ចសរបងá&#x;’ហវញáž?ážś max đ?‘Ž, đ?‘?, đ?‘? ≼ 3

áž…á&#x;†áž“ á&#x;†

បរឞជរ áž˝ លកá&#x;’ហហណá&#x;’ឌ đ?‘Ž, đ?‘?, đ?‘? > 0 នឡង đ?‘Ž + đ?‘? + đ?‘? = đ?‘Žđ?‘?đ?‘? áž…ážźážšáž?ážśáž„ đ?‘Ž = tan đ?‘Ľ , đ?‘? = tan đ?‘Ś , đ?‘? = tan đ?‘§ đ?œ‹ ដសបចនá&#x;‡ đ?‘Ľ, đ?‘Ś, đ?‘§ អវចចវរá&#x;‹ áž‘á&#x;’យឝកá&#x;’áž‡ážśáž˜ážťáž€á&#x;’ ážśá&#x;† áž„ ážťáž“ ážšážšáž&#x;á&#x;‹ áž?á&#x;’រីបកវណá&#x;’áž?á&#x;’áž&#x;áž… áž˝ បវន មននá&#x;?áž™áž?ážś đ?‘Ľ, đ?‘Ś, đ?‘§ ∈ 0, 2 នឡង đ?‘Ľ+đ?‘Ś+đ?‘§ =đ?œ‹á&#x;”

210.

(អនá&#x;’áž?រជវáž?ឡ á&#x;Ąá&#x;Šá&#x;Šá&#x;Š)

ដគដá&#x;„យចá&#x;†áž“áž“ áž˝ áž‚áž?á&#x;‹ đ?‘› ≼ 2 á&#x;” កá&#x;’) ចសរកá&#x;’á&#x;†áž“áž?á&#x;‹ áž…á&#x;†áž“áž“ áž˝ ដáž?ážš đ??ś áž?ážźáž…áž”á&#x;† áž•ážťáž? ដដលចá&#x;†ážŠ á&#x;„á&#x;‡ážšáž‚áž”á&#x;‹ áž…á&#x;†áž“áž“ áž˝ ពឡáž? đ?‘Ľ1 , đ?‘Ľ2 , ‌ , đ?‘Ľđ?‘› ≼ 0; ážŠáž‚áž˜ážśáž“ 4

đ?‘›

đ?‘Ľđ?‘– đ?‘Ľđ?‘— đ?‘Ľđ?‘–2 + đ?‘Ľđ?‘—2 ≤ đ??ś 1≤đ?‘–<đ?‘— ≤đ?‘›

đ?‘Ľđ?‘– đ?‘–=1

áž ) áž…á&#x;†ážŠ á&#x;„á&#x;‡áž…á&#x;†áž“áž“ áž˝ ដáž?ážš đ??ś ដនá&#x;„á&#x;‡ ចសរកá&#x;’á&#x;†áž“áž?á&#x;‹ កá&#x;’រណី áž&#x;មá&#x;’áž—ážśáž–á&#x;”

211.

ដគដá&#x;„យចá&#x;†áž“áž“ áž˝ ពឡáž? đ?‘Ľ, đ?‘Ś, đ?‘§, đ?‘Ą ដដល đ?‘Ľ+đ?‘Ś+đ?‘§+đ?‘Ą =0 đ?‘Ľ2 + đ?‘Ś2 + đ?‘§2 + đ?‘Ą2 = 1 ចសរគណនវáž?á&#x;†áž“លធá&#x;†áž”á&#x;†áž•ážťáž?នឡងáž?ážźáž…áž”á&#x;† áž•ážťáž?áž“áž“ đ?‘ƒ = đ?‘Ľđ?‘Ś + đ?‘Śđ?‘§ + đ?‘§đ?‘Ą + đ?‘Ąđ?‘Ľ á&#x;”

212.

(អនá&#x;’áž?រជវáž?áž&#x; á&#x;˘á&#x; á&#x; á&#x;Ą) ឡ áž?áž›áž&#x; ី

áž?ážśáž„ đ?‘Ľ1 , đ?‘Ľ2 , ‌ , đ?‘Ľđ?‘› ជវចá&#x;†áž“áž“ áž˝ ពឡáž?á&#x;” ចសរបងá&#x;’ហវញáž?ážś đ?‘Ľ1 đ?‘Ľ2 đ?‘Ľđ?‘› < đ?‘› 2 + 2 2 +â‹Ż+ 2 1 + đ?‘Ľ1 1 + đ?‘Ľ1 + đ?‘Ľ2 1 + đ?‘Ľ1 + â‹Ż + đ?‘Ľđ?‘›2 213.

(áž˜áž˘á&#x;€ážšáž„ á&#x;‰ážś á&#x;‹ á&#x;Ąá&#x;Šá&#x;Šá&#x;¨)

ចសរបងá&#x;’ហវញáž?ážś áž…á&#x;†ážŠ á&#x;„á&#x;‡ážšáž‚áž”á&#x;‹ đ?‘Ľ; đ?‘Ś; đ?‘§ > 1 ដដល 1 đ?‘Ľ + 1 đ?‘Ś + 1 đ?‘§ = 2 ážŠáž‚áž˜ážśáž“ 20

á&#x;Ą. áž˘áž“ážťáž‚áž˜áž“á&#x;?áž„ážśáž™ | លរម áž&#x;ážť វណá&#x;’ážŽážœážˇáž…ážˇáž?á&#x;’ážš


đ?‘Ľ+đ?‘Ś+đ?‘§ ≼ đ?‘Ľâˆ’1+ đ?‘Śâˆ’1+ đ?‘§âˆ’1

214.

(ážœáž&#x;áž˜áž—ážśáž–áž?á&#x;† មរá&#x;€áž”) ឡ

ដគដá&#x;„áž™ đ?‘Ž1 ≤ đ?‘Ž2 ≤ â‹Ż ≤ đ?‘Žđ?‘› នឡង đ?‘?1 ≤ đ?‘?2 ≤ â‹Ż ≤ đ?‘?đ?‘› ជវáž&#x;ážœáž? វី áž“áž“áž…á&#x;†áž“áž“ áž˝ ពឡáž? ដកá&#x;’ឞនពីរá&#x;” áž?ážśáž„ đ?œŽ ជវចá&#x;†áž›ážśáž&#x;á&#x;‹ áž“áž“ 1,2, ‌ , đ?‘› á&#x;” ចសរបងá&#x;’ហវញáž?ážś ផលបស កá&#x;’ đ?‘†đ?œŽ =

đ?‘› đ?‘–=1 đ?‘Žđ?‘– đ?‘?đ?œŽ đ?‘–

áž˜ážśáž“áž?á&#x;†áž“លធá&#x;†áž”á&#x;†áž•ážťáž? ដពល đ?œŽ đ?‘– = đ?‘– នឡង áž?ážźáž…áž”á&#x;† áž•ážťáž?

ដពល đ?œŽ đ?‘– = đ?‘› − đ?‘– áž…á&#x;†ážŠ á&#x;„á&#x;‡ážšáž‚áž”á&#x;‹ đ?‘– á&#x;” áž˜ážśáž“áž“á&#x;?áž™áž?ážś đ?‘†đ?œŽ áž˜ážśáž“áž?á&#x;†áž“លធá&#x;†áž”á&#x;†áž•ážťáž?ដពល áž&#x;ážœáž? វី áž‘á&#x;†áž„á&#x;˘ážŠážšá&#x;€áž”áž?ážśáž˜á&#x;’áž›á&#x;† á&#x;„áž”á&#x;‹ ដសចគá&#x;’áž“áž“ ដ ážž áž™ áž?ážźáž…áž”á&#x;† áž•ážťáž?ដបឞ ដរá&#x;€áž”áž?ážśáž˜á&#x;’áž›á&#x;† á&#x;„áž”á&#x;‹ áž•áž‘ážœ យគá&#x;’áž“áž“á&#x;”

215.

(អនá&#x;’áž?រជវáž?ឡ á&#x;Ąá&#x;Šá&#x;§á&#x;Ľ)

ដគដá&#x;„យចá&#x;†áž“áž“ áž˝ ពឡáž? đ?‘Ľ1 ≼ đ?‘Ľ2 ≼ â‹Ż ≼ đ?‘Ľđ?‘› នឡង đ?‘Ś1 ≼ đ?‘Ś2 ≼ â‹Ż ≼ đ?‘Śđ?‘› á&#x;” áž?ážśáž„ đ?‘§1 , đ?‘§2 , ‌ , đ?‘§đ?‘› ជវចá&#x;†áž›ážśáž&#x;á&#x;‹ áž“áž“ đ?‘Ś1 , đ?‘Ś2 , ‌ , đ?‘Śđ?‘› á&#x;” ចសរបងá&#x;’ហវញáž?ážś đ?‘›

đ?‘›

đ?‘Ľđ?‘– − đ?‘Śđ?‘–

216.

≤

đ?‘–=1

đ?‘Ľđ?‘– − đ?‘§đ?‘–

2

đ?‘–=1

ចសរគណនវáž?á&#x;†áž“áž›áž?ážźáž…áž”á&#x;† áž•ážťáž?ážšáž”áž&#x;á&#x;‹ đ?‘“ đ?‘Ľ = áž…á&#x;†ážŠ á&#x;„á&#x;‡ đ?‘Ľ ∈ 0, đ?œ‹ 2 á&#x;”

217.

2

sin3 đ?‘Ľ cos3 đ?‘Ľ + cos đ?‘Ľ sin đ?‘Ľ

ចសរបងá&#x;’ហវញáž?ážś áž…á&#x;†ážŠ á&#x;„á&#x;‡ážšáž‚áž”á&#x;‹ áž…á&#x;†áž“áž“ áž˝ ពឡáž? đ?‘Ž, đ?‘?, đ?‘? ≼ 0 ážŠáž‚áž˜ážśáž“ đ?‘Ž3 + đ?‘? 3 + đ?‘? 3 ≼ đ?‘Ž2 đ?‘? + đ?‘? 2 đ?‘? + đ?‘? 2 đ?‘Ž ≼ 3đ?‘Žđ?‘?đ?‘?

218.

(អនá&#x;’áž?រជវáž?ឡ á&#x;Ąá&#x;Šá&#x;§á&#x;¨) ដគដá&#x;„áž™ đ?‘Žđ?‘˜

đ?‘˜âˆˆâ„•

ជវáž&#x;ážœáž? វី áž“áž“áž…á&#x;†áž“áž“ áž˝ áž‚áž?á&#x;‹ ធមá&#x;’ម ជវáž?ážˇáž˜á&#x;’ឡនáž&#x;ážźáž“áž™ ដ ážž áž™áž˜ážśáž“áž?ួហឝáž&#x;áž‚á&#x;’ននពីរá&#x;—á&#x;”

ចសរបងá&#x;’ហវញáž?ážś áž…á&#x;†ážŠ á&#x;„á&#x;‡ážšáž‚áž”á&#x;‹ đ?‘› ≼ 1 ážŠáž‚áž˜ážśáž“

đ?‘›

đ?‘˜=1

219.

đ?‘Žđ?‘˜ ≼ đ?‘˜2

đ?‘›

đ?‘˜=1

1 đ?‘˜

(áž…áž“á&#x;’ ឡ á&#x;Ąá&#x;Šá&#x;¨á&#x;¤/á&#x;Ąá&#x;Šá&#x;¨á&#x;Ľ)

ដគដá&#x;„áž™ đ?‘Ž1 , đ?‘Ž2 , ‌ , đ?‘Žđ?‘› > 0 á&#x;” ចសរបងá&#x;’ហវញáž?ážś

លរម áž&#x;ážť វណá&#x;’ážŽážœážˇáž…ážˇáž?á&#x;’ážš | V. ážœážˇáž&#x;áž˜áž—ážśáž–

21


220.

2 𝑎12 𝑎22 𝑎𝑛−1 𝑎𝑛2 + + ⋯+ + ≥ 𝑎1 + 𝑎2 + ⋯ + 𝑎𝑛 𝑎2 𝑎3 𝑎𝑛 𝑎1 ិ មា ដគដោយចំនន ួ ម្ិនអវជ ជ ន 𝑝, 𝑞, 𝑥, 𝑦, 𝑧 ។ ចូរបង្ហាញថា

𝑥 2000 𝑦 2000 𝑧 2000 𝑥 1999 + 𝑦1999 + 𝑧1999 + + ≥ 𝑝𝑦 + 𝑞𝑧 𝑝𝑧 + 𝑞𝑥 𝑝𝑥 + 𝑞𝑦 𝑝+𝑞

221.

(វសមភាព ិ

)

ចំដ ោះរគប់ សវត វី ដក្ើនននចំនន ួ ពិតពីរ តាងដោយ 𝑎1 ≤ 𝑎2 ≤ ⋯ ≤ 𝑎𝑛 និង 𝑏1 ≤ 𝑏2 ≤ ⋯ ≤ 𝑏𝑛 ដគមាន 𝑎1 + 𝑎2 + ⋯ + 𝑎𝑛 𝑏1 + 𝑏2 + ⋯ + 𝑏𝑛 𝑎1 𝑏1 + 𝑎2 𝑏2 + ⋯ + 𝑎𝑛 𝑏𝑛 ≤ 𝑛 𝑛 𝑛 ិ ដបើ 𝑏1 ≥ 𝑏2 ≥ ⋯ ≥ 𝑏𝑛 ដនាោះ ផទវ យដៅវញ 𝑎1 + 𝑎2 + ⋯ + 𝑎𝑛 𝑏1 + 𝑏2 + ⋯ + 𝑏𝑛 𝑎1 𝑏1 + 𝑎2 𝑏2 + ⋯ + 𝑎𝑛 𝑏𝑛 ≥ 𝑛 𝑛 𝑛

ិ ូ រស គណិ តវទ ុ េុី,១៨២១១៨៩៤ 222.

ីត) (វសមភាពនន្សប ិ

ិ មា ដគដោយ 𝑎, 𝑏, 𝑐 ជាចំនន ួ ពិតវជ ជ ន។ ចូរបង្ហាញថា 𝑎 𝑏 𝑐 3 + + ≥ 𝑏+𝑐 𝑐+𝑎 𝑎+𝑏 2 22

១. អនុគមន៍ងាយ | លឹម សុ វណ្ណវិចិត្រ


223.

ដគដោយ 𝑎, 𝑏, 𝑐, 𝑑 ≥ 0 ដដល 𝑎𝑏 + 𝑏𝑐 + 𝑐𝑑 + 𝑑𝑎 = 1 ។ ចូរបង្ហាញថា

224.

𝑎3 𝑏3 𝑐3 𝑑3 1 + + + ≥ 𝑏+𝑐+𝑑 𝑐+𝑑+𝑎 𝑑+𝑎+𝑏 𝑎+𝑏+𝑐 3 ដគដោយ 𝑎, 𝑏, 𝑐 > 0 និង 𝑛 ∈ ℕ∗ ។ ចូរបង្ហាញថា

225.

𝑎𝑛 𝑏𝑛 𝑐𝑛 𝑎𝑛−1 + 𝑏 𝑛−1 + 𝑐 𝑛−1 + + ≥ 𝑏+𝑐 𝑐+𝑎 𝑎+𝑏 2 ដគដោយ 𝑥, 𝑦, 𝑧 > 0 ដដល 𝑥𝑦𝑧 = 1 ។ ចូរបង្ហាញថា 𝑥3 𝑦3 𝑧3 3 + + ≥ 1+𝑦 1+𝑧 1+𝑧 1+𝑥 1+𝑥 1+𝑦 4

226.** (វសមភាពមធ្យមន្ពា ន្ត-មធ្យមធ្រណីមាគ្ត មន្-មធ្ រ ឺ វសមភាព ិ ិ កូសុី)

ិ មា ចំដ ោះរគប់ ចំនន ួ ពិតវជ ជ ន 𝑎1 , 𝑎2 , … , 𝑎𝑛 រ ឺសូនយ ដគមាន 𝑎1 + 𝑎2 + ⋯ + 𝑎𝑛 𝑛 𝑎1 𝑎2 … 𝑎𝑛 ≤ 𝑛 ដោយសម្ភាពដក្ើតមាន ទល់ ដត និងមានដត 𝑎1 = 𝑎2 = ⋯ = 𝑎𝑛 ដតបុ ដណា ណ ោះ។

227.

(អន្តរជាតិ ១៩៦៤)

តាង 𝑎, 𝑏, 𝑐 ជារង្ហវស់ រជុ ងននរតីដកាណម្ួ យ។ ចូរបង្ហាញថា 𝑎2 𝑏 + 𝑐 − 𝑎 + 𝑏 2 𝑐 + 𝑎 − 𝑏 + 𝑐 2 𝑎 + 𝑏 − 𝑐 ≤ 3𝑎𝑏𝑐

228.

(អន្តរជាតិ ឡងលស ី ១៩៦៧) ចូរបង្ហាញថា

2 1 2 1 1 𝑛 + 𝑛 + ≥ 𝑛! 𝑛 3 2 6 ិ មា (𝑛 ជាចំនន ួ គត់ វជ ជ ន) ដ ើ យបង្ហាញថាសម្ភាពោចដតក្នវងក្រណី 𝑛 = 1 ម្ួ យដតបុ ដណា ណ ោះ។

229.

(អន្តរជាតិ ឡងលស១៩៦៧) ី

ិ មា ិ ម្ភាពខាងដរកាម្ពិត ចូរបង្ហាញថា ចំដ ោះរគប់ ចំនន ួ ពិតវជ ជ ន វស 1 1 1 𝑎8 + 𝑏 8 + 𝑐 8 + + ≤ 𝑎 𝑏 𝑐 𝑎3 𝑏 3 𝑐 3

230.

(អន្តរជាតិ ឡងលីស ១៩៦៧) ិ ម្ភាព ចូររាយបញ្ញជក្់ វស

𝑥1 𝑥2 … 𝑥𝑘 𝑥1𝑛−1 + 𝑥2𝑛−1 + ⋯ + 𝑥𝑘𝑛−1 ≤ 𝑥1𝑛+𝑘−1 + 𝑥2𝑛+𝑘−1 + ⋯ + 𝑥𝑘𝑛+𝑘−1 លឹម សុ វណ្ណវិចិត្រ | V. វិសមភាព

23


ដដល 𝑥𝑖 > 0 𝑖 = 1,2, … , 𝑘 , 𝑘 ∈ ℕ, 𝑛 ∈ ℕ ។

231.

ចូរបង្ហាញថា ចំដ ោះរគប់ 𝑥1 , 𝑥2 , … , 𝑥𝑛 > 0 ដគមាន 1 1 1 𝑥1 + 𝑥2 + ⋯ + 𝑥𝑛 + +⋯+ ≥ 𝑛2 𝑥1 𝑥2 𝑥𝑛

232.** (រុ ស្ុ ី ២០០០)

ដគដោយ 𝑥, 𝑦, 𝑧 ∈ ℝ+ ដដល 𝑥𝑦𝑧 = 1 ។ ចូរបង្ហាញថា 𝑥 2 + 𝑦 2 + 𝑧 2 + 𝑥 + 𝑦 + 𝑧 ≥ 2 𝑥𝑦 + 𝑦𝑧 + 𝑧𝑥

233.

(សូមវៀត ១៩៦២) ដគដោយ 𝑎, 𝑏, 𝑐, 𝑑 > 0 ដដល 𝑎𝑏𝑐𝑑 = 1 ។ ចូរបង្ហាញថា 𝑎2 + 𝑏 2 + 𝑐 2 + 𝑑2 + 𝑎𝑏 + 𝑎𝑐 + 𝑎𝑑 + 𝑏𝑐 + 𝑏𝑑 + 𝑐𝑑 ≥ 10

234.

ដគដោយ 𝑎, 𝑏, 𝑐 ≥ 0 ។ ចូរបង្ហាញថា 𝑎+𝑏+𝑐

235.

3

≥ 𝑎3 + 𝑏 3 + 𝑐 3 + 24𝑎𝑏𝑐

ដគដោយ 𝑎, 𝑏, 𝑐 > 0 ។ ចូរបង្ហាញថា 𝑎𝑏𝑐 ≥ 𝑎 + 𝑏 − 𝑐 𝑏 + 𝑐 − 𝑎 𝑐 + 𝑎 − 𝑏

236.

ដគដោយ 𝑎1 , 𝑎2 , … , 𝑎𝑛 ∈ ℝ+ ដដល 𝑎1 + 𝑎2 + ⋯ + 𝑎𝑛 = 1 ។ ចូរបង្ហាញថា 𝑛

𝑎𝑘 1 − 𝑎𝑘

237.

238.

(អូគ្ទីស ២០០០)

𝑘=1

𝑛−1 ≤ 𝑛2𝑛

𝑛

ដគដោយចំនន ួ ពិត 𝑎, 𝑏 ដដល 𝑎 ≠ 0 ។ ចូរបង្ហាញថា 1 𝑏 𝑎2 + 𝑏 2 + 2 + ≥ 3 𝑎 𝑎 ∗ ចំដ ោះ 𝑛 ∈ ℕ តាង

ចូរបង្ហាញថា សវត វី 𝑈𝑛

1 𝑛 1 𝑛+1 𝑈𝑛 = 1 + ; 𝑉𝑛 = 1 + 𝑛 𝑛 ជាសវត វី ដក្ើន ដ ើ យសវត វី 𝑉𝑛 ជាសវត វី ចុោះ។

239.** (សូមវៀត ១៩៦៩) ដគដោយចំនន ួ គត់ 𝑛 ≥ 3 និង 𝑎1 , 𝑎2 , … , 𝑎𝑛 ∈ ℝ+∗ ។ ចូរបង្ហាញថា 𝑎1 𝑎2 𝑎𝑛−1 𝑎𝑛 𝑛 + + ⋯+ + > 𝑎2 + 𝑎3 𝑎3 + 𝑎4 𝑎𝑛 + 𝑎1 𝑎1 + 𝑎2 4 24

១. អនុគមន៍ងាយ | លឹម សុ វណ្ណវិចិត្រ


240.

(ចន្ ិ ១៩៨៩/១៩៩០)

ដគដោយ 𝑎1 , 𝑎2 , … , 𝑎𝑛 > 0 ដដល 𝑎1 𝑎2 … 𝑎𝑛 = 1 ។ ចូរបង្ហាញថា 2 + 𝑎1 2 + 𝑎2 … 2 + 𝑎𝑛 ≥ 3𝑛 ិ មា 241.** តាង 𝑎 និង 𝑏 ជាចំនន ួ ពិតវជ ជ ន។ ចូរបង្ហាញថា 1 1 2 + ≥ 1 + 𝑎𝑏 1 + 𝑎2 1 + 𝑏2 ដបើ (1) 0 < 𝑎, 𝑏 ≤ 1 រ ឺក្៏ (1) 𝑎𝑏 ≥ 3 ។

242.

ដគដោយ 𝑥1 , 𝑥2 , … , 𝑥𝑛+1 > 0 ដដល 1 1 1 + + ⋯+ =1 1 + 𝑥1 1 + 𝑥2 1 + 𝑥𝑛+1 ចូរបង្ហាញថា 𝑥1 𝑥2 … 𝑥𝑛+1 ≥ 𝑛𝑛+1 ។

243.

ដគដោយចំនន ួ គត់ 𝑛 > 1, 𝑥1 𝑥2 … 𝑥𝑛 ∈ ℝ+∗និង 𝑎1 , 𝑎2 , … , 𝑎𝑛 ∈ ℝ+ ។ តាង 𝑠 = 𝑥1 +

𝑥2 + ⋯ + 𝑥𝑛 ។ ចូរក្ំនត់ ចំនន ួ ដថរ 𝐶 𝑛 ធំបំផុត ដដល 𝑛

𝑗 =1

244.

𝑎𝑗 𝑠 − 𝑥𝑗 ≥𝐶 𝑛 𝑥𝑗

1 𝑛

𝑛

𝑎𝑗 𝑗 =1

ដគដោយ 𝑥, 𝑦, 𝑧 > 0 ដដល 𝑥 + 𝑦 + 𝑧 = 1 ។ ចូរបង្ហាញថា 1 1 1 1+ 1+ 1+ ≥ 64 𝑥 𝑦 𝑧

245.* (រូមា៉ាន្ី ១៩៩៧)

ដគដោយ ចំនន ួ គត់ 𝑛 ≥ 2 និង 𝑥1 , 𝑥2 , … , 𝑥𝑛 > 0 ដដល 𝑥1 𝑥2 … 𝑥𝑛 = 1 ។ ចូរគណនាតំនលតូចបំ ផុតរបស់ 𝑥𝑖9 + 𝑥𝑗9 1≤𝑖<𝑗 ≤𝑛

𝑥𝑖6 + 𝑥𝑖3 𝑥𝑗3 + 𝑥𝑗6

246.* ដគដោយ 𝑎, 𝑏, 𝑐, 𝑑 ≥ 0 ដដល 𝑎 + 𝑏 + 𝑐 + 𝑑 = 1 ។ ចូរបង្ហាញថា 1 176 + 𝑎𝑏𝑐𝑑 27 27 ដដល 𝑎1 + 𝑎2 + ⋯ + 𝑎𝑛 = 1 ។ ចូរបង្ហាញថា

𝑎𝑏𝑐 + 𝑏𝑐𝑑 + 𝑐𝑑𝑎 + 𝑑𝑎𝑏 ≤

247.* ដគដោយ 𝑎1 , 𝑎2 , … , 𝑎𝑛 ∈ ℝ+∗ 1

1 ≥ 𝑛2 𝑛 − 1 + 𝑛−3 𝑛 𝑎1 𝑎2 … 𝑎𝑛 លឹម សុ វណ្ណវិចិត្រ | V. វិសមភាព

𝑛

𝑘=1

1 𝑎𝑘 25


248.

ដគដោយ 𝑎1 , 𝑎2 , … , 𝑎𝑛 ∈ ℝ+∗ ដដល 𝑎1 + 𝑎2 + ⋯ + 𝑎𝑛 < 1 ។ ចូរបង្ហាញថា 𝑎1 𝑎2 … 𝑎𝑛 1 − 𝑎1 − 𝑎2 − ⋯ − 𝑎𝑛 1 ≤ 𝑛+1 𝑎1 + 𝑎2 + ⋯ + 𝑎𝑛 1 − 𝑎1 1 − 𝑎2 … 1 − 𝑎𝑛 𝑛

249.

(អាមមរចសតល ស ២០០២) ិ ី ចូរគណនាតំនលធំបំផុតរបស់

𝑆 = 1 − 𝑥1 1 − 𝑦1 + 1 − 𝑥2 1 − 𝑦2

ដបើ 𝑥12 + 𝑥22 = 𝑦12 + 𝑦22 = 𝑐 2 ។

250.

(អន្តរជាតិ ១៩៩៥)

ិ មា តាង 𝑎𝑏𝑐 ជាចំនន ួ វជ ជ ន ដដល 𝑎𝑏𝑐 = 1 ។ ចូរបង្ហាញថា 1 1 1 3 + 3 + 3 ≥ 3 𝑎 (𝑏 + 𝑐) 𝑏 (𝑐 + 𝑎) 𝑐 𝑎 + 𝑏 2

251.

252.

(អន្តរជាតិ ២០០១)

ិ មា តាង 𝑎, 𝑏, 𝑐 ជាចំនន ួ ពិតវជ ជ ន។ ចូរបង្ហាញថា 𝑎 𝑏 𝑐 + + ≥1 𝑎2 + 8𝑏𝑐 𝑏 2 + 8𝑐𝑎 𝑐 2 + 8𝑎𝑏 តាង 𝑎, 𝑏, 𝑐 ជាចំនន ួ ពិត។ ចូរបង្ហាញថា 𝑎𝑏 + 𝑏𝑐 + 𝑐𝑎 − 1

253.

254.

256. 26

≤ 𝑎2 + 1 𝑏 2 + 1 𝑐 2 + 1

(អន្តរជាតិ ១៩៨៤)

ិ មា តាង 𝑥, 𝑦, 𝑧 ជាចំនន ួ ពិតម្ិនអវជ ជ ន ដដល 𝑥 + 𝑦 + 𝑧 = 1 ។ ចូរបង្ហាញថា 7 0 ≤ 𝑥𝑦 + 𝑦𝑧 + 𝑧𝑥 − 2𝑥𝑦𝑧 ≤ 27 ិ មា ដគដោយចំនន ួ ពិតវជ ជ ន 𝑥, 𝑦, 𝑧 ។ ចូរបង្ហាញថា 1+

255.

2

𝑥 𝑦

1+

𝑦 𝑧

1+

𝑧 𝑥+𝑦+𝑧 ≥2 1+ 3 𝑥 𝑥𝑦𝑧

ដគដោយចំនន ួ ពិតបួ នដផេងគ្នន 𝑎, 𝑏, 𝑐, 𝑑 ដដល 𝑎 𝑏 𝑐 𝑑 + + + = 4; 𝑎𝑐 = 𝑏𝑑 𝑏 𝑐 𝑑 𝑎 ចូរគណនាតំនលធំបំផុតនន 𝑎 𝑏 𝑐 𝑑 + + + 𝑐 𝑑 𝑎 𝑏 ដគដោយ 𝑎, 𝑏, 𝑐 ជារង្ហវស់ រជុ ងននរតីដកាណម្ួ យមានបរ ិមារត ដសមើ 2 ។ ចូរបង្ហាញថា

១. អនុគមន៍ងាយ | លឹម សុ វណ្ណវិចិត្រ


257.

52 ≤ 𝑎2 + 𝑏 2 + 𝑐 2 + 2𝑎𝑏𝑐 < 2 27 ចំដ ោះរគប់ 𝑎, 𝑏, 𝑐 > 0។ ចូរបង្ហាញថា 𝑎2 𝑏 + 𝑏 2 𝑐 + 𝑐 2 𝑎 𝑎𝑏 2 + 𝑏𝑐 2 + 𝑐𝑎2 ≥ 𝑎𝑏𝑐 +

258.

3

𝑎3 + 𝑎𝑏𝑐 𝑏 3 + 𝑎𝑏𝑐 𝑐 3 + 𝑎𝑏𝑐

ិ មា 1) ចូរបង្ហាញថា ចំដ ោះរគប់ ចំនន ួ សនិទនវជ ជ ន 𝑝, 𝑞 ដដល 𝑝 + 𝑞 = 1 ដគមាន 𝑝𝑎 + 𝑞𝑏 ≥ 𝑎𝑝 𝑏 𝑞 ; ∀ 𝑎, 𝑏 > 0 ិ មា 2) ចូរបង្ហាញថា ចំដ ោះរគប់ ចំនន ួ ពិតវជ ជ ន 𝑝, 𝑞 ដដល 𝑝 + 𝑞 = 1 ដគមាន 𝑝𝑥 + 𝑞𝑦 ≥ 𝑥 𝑝 𝑦 𝑞 ; ∀ 𝑥, 𝑦 > 0

3)

(វសមភាពមន្-មធ្ទូ មៅ) ិ

ចូរបង្ហាញថា ចំដ ោះរគប់ ចំនន ួ ពិត 𝑎1 , 𝑎2 , … , 𝑎𝑛 > 0 ដដល 𝑎1 + 𝑎2 + ⋯ + 𝑎𝑛 = 1 ដគ មាន

𝑎

𝑎

𝑎

𝑎1 𝑥1 + 𝑎2 𝑥2 + ⋯ + 𝑎𝑛 𝑥𝑛 ≥ 𝑥1 1 𝑥2 2 … 𝑥𝑛 𝑛 ; ∀𝑥1 , 𝑥2 , … , 𝑥𝑛 > 0

4)

(វសមភាព ិ

)

ដគដោយ 𝑝, 𝑞 ∈ ℝ+∗ ដដល 1 𝑝 + 1 𝑞 = 1 និង បណា ត ចំនន ួ ពិត 𝑎1 , 𝑎2 , … , 𝑎𝑛 និង 𝑏1 , 𝑏2 , … , 𝑏𝑛 ចូរបង្ហាញថា 𝑎1 𝑏1 + 𝑎2 𝑏2 + ⋯ + 𝑎𝑛 𝑏𝑛

1

≤ 𝑎1 𝑝 + 𝑎2 𝑝 + ⋯ + 𝑎𝑛 𝑝 𝑝 × 𝑏1 𝑞 + 𝑏2 𝑞 + ⋯ + 𝑏𝑛 𝑞 ិ ទ័រ 𝑢 𝑎1𝑝 , 𝑎2𝑝 , … , 𝑎𝑛𝑝 និង វុច ិ ទ័រ 𝑣 𝑏1𝑞 , 𝑏2𝑞 , … , 𝑏𝑛𝑞 ដដលអងគ ទំងពីរដសមគ្ន ើ ន លុ ោះរតាដត វុច

1 𝑞

ិ មា ក្ូ លីដនដអ៊ែរនឹងគ្នន ដ ើ យ បណា ត 𝑎𝑖 𝑏𝑖 សុទធដតវជ ជ នរ ឺសូនយទំងអស់ រ ឺដបើម្ិនអញ្ច ឹ ង សុទធដតអ ិ មា វជ ជ នរ ឺសូនយទំងអស់ ។

259.

ដគដោយ 𝑎, 𝑏 ≥ 0 និង 𝑝, 𝑞 > 1 ដដល 1 𝑝 + 1 𝑞 = 1។ ចូរបង្ហាញថា

260.

(ប៉ាឡ ូ ញ ១៩៩៥) ូ

𝑎𝑝 𝑏 𝑞 + ≥ 𝑎𝑏 𝑝 𝑞

ដគដោយចំនន ួ គត់ 𝑛 ≥ 1 ចូរក្ំនត់ តំនលតូចបំ ផុតរបស់ ផលបូ ក្ 𝑥22 𝑥𝑛𝑛 + ⋯+ 2 𝑛 ដដល 𝑥1 , 𝑥2 , … , 𝑥𝑛 ជាចំនន ួ ពិត ដដល 1 𝑥1 + 1 𝑥2 + ⋯ + 1 𝑥𝑛 = 𝑛 ។ 𝑥1 +

លឹម សុ វណ្ណវិចិត្រ | V. វិសមភាព

27


(វសមភាព ិ

261.

)

តាង 𝑝 ∈ 1; +∞ ដ ើ យ 𝑎1 , 𝑎2 , … , 𝑎𝑛 និង 𝑏1 , 𝑏2 , … , 𝑏𝑛 ជាចំនន ួ ពិត។ ចូរបង្ហាញថា 𝑎1 + 𝑏1

𝑝

+ 𝑎2 + 𝑏2

𝑝

1 𝑝 𝑝

+ ⋯ + 𝑎𝑛 + 𝑏𝑛

1

1

≤ 𝑎1 𝑝 + 𝑎2 𝑝 + ⋯ + 𝑎𝑛 𝑝 𝑝 + 𝑏1 𝑝 + 𝑏2 𝑝 + ⋯ + 𝑏𝑛 𝑝 𝑝 ិ ទ័រ 𝑢 𝑎1 , 𝑎2 , … , 𝑎𝑛 និង វុច ិ ទ័រ 𝑣 𝑏1 , 𝑏2 , … , 𝑏𝑛 ដដលអងគ ទំងពីរដសមគ្ន ើ ន លុ ោះរតាដត វុច ក្ូ លីដនដអ៊ែរនឹងគ្នន ដ ើ យមានទិសដៅដូចគ្នន។

262.

ដគដោយ 𝑥, 𝑦, 𝑧 > 0 ។ ចូរបង្ហាញថា 4

4

𝑥3 4 𝑥3

263.

+

𝑥2

𝑥+𝑧

2 3

+

4 𝑦3

+

𝑦2

+

1 𝑧2 3

𝑦+𝑥

2 3

+

𝑧3 4 𝑧3

+

𝑧2

1 + 𝑥2 3

𝑧+𝑦

2 3

≤1

ដគដោយចំនន ួ ពិត 𝑎, 𝑏, 𝑐, 𝑑 ។ ចូរក្ំនត់ តនំ លតូចបំ ផុតរបស់ +2 𝑏−2 2+ 𝑐+3 2+ 𝑏+1 2+2 𝑐−2 + 𝑐+1 2+2 𝑑−2 2 + 𝑎+3 2 + 𝑑+1 2+2 𝑎−2 2+ 𝑏+3 2 ិ មា តាង 𝑥, 𝑦, 𝑧 ជាចំនន ួ ពិតវជ ជ ន

𝑆=

264.

+

1 𝑦2 3

4

𝑦3

1)

𝑎+1

2

+ 𝑑+3

2

ចូរបង្ហាញថា ដបើ 𝑥 + 𝑦 + 𝑧 = 𝑥𝑦𝑧 ដនាោះ 𝑥 1 + 𝑥2

2)

2

+

𝑦

+

𝑧

3 3 2

1 + 𝑧2 1 + 𝑦2 ចូរបង្ហាញថា ដបើ 0 < 𝑥, 𝑦, 𝑧 < 1 និង 𝑥𝑦 + 𝑦𝑧 + 𝑧𝑥 = 1 ដនាោះ 𝑥 𝑦 𝑧 3 3 + + ≥ 1 − 𝑥2 1 − 𝑦2 1 − 𝑧2 2

265.

(នបលារុស ១៩៩៩) ដគដោយសវត វី ននចំនន ួ ពិតពីរ 𝑥1 , 𝑥2 , … និង 𝑦1 , 𝑦2 , … ក្ំនត់ ដោយ 𝑥1 = 𝑦1 = 3; 𝑥𝑛+1 = 𝑥𝑛 + 1 + 𝑥𝑛2 ; 𝑦𝑛+1 =

𝑦𝑛

1 + 1 + 𝑦𝑛2 ចំដ ោះរគប់ 𝑛 ≥ 1 ។ ចូរបង្ហាញថា 2 < 𝑥𝑛 𝑦𝑛 < 3 ចំដ ោះរគប់ 𝑛 > 1 ។

266.

ិ ក្នវងចដនាលោះ −1; 1 ដដល 𝑛 ≥ 2 ។ ចូរ តាង 𝑥1 , 𝑥2 , … , 𝑥𝑛 ជាចំនន ួ ពិតខុសៗគ្ននចំនន ួ 𝑛 ឋត បង្ហាញថា

28

១. អនុគមន៍ងាយ | លឹម សុ វណ្ណវិចិត្រ


ដដល 𝑡𝑖 =

267.

𝑗 ≠𝑖

1 1 1 + + ⋯ + ≥ 2𝑛−2 𝑡1 𝑡2 𝑡𝑛

𝑥𝑗 − 𝑥𝑖 ។

ដយើ ងយក្ចំនន ួ ពិតបួ នដៅក្នវងចដនាលោះ

2− 6 2+ 6 ; 2 2

ដោយម្ិនបាច់ ដរ ើស។ ចូរបង្ហាញថា រតូវ

ដតមានពីរក្នវងចំដនាម្ដនាោះ តាងដោយ 𝑎 និង 𝑏 ដដល 𝑎 4 − 𝑏 2 − 𝑏 4 − 𝑎2 ≤ 2

268.

(វសមភាពយ ិ ិន្សន្ ិ )

តាង 𝑛 ≥ 1 ជាចំនន ួ គត់ ដ ើ យ 𝑓 ជាអនុគម្ន៍ផត ដលើ ដដន 𝐼 ។ ចូរបង្ហាញថា ចំដ ោះរគប់ ចំនន ួ ពិត 𝑙1 , 𝑙2 , … , 𝑙𝑛 ∈ ℝ+∗ ដដល 𝑙1 + 𝑙2 + ⋯ + 𝑙𝑛 = 1 ដគមាន 𝑓 𝑙1 𝑥1 + 𝑙2 𝑥2 + ⋯ + 𝑙𝑛 𝑥𝑛 ≤ 𝑙1 𝑓 𝑥1 + 𝑙2 𝑓 𝑥2 + ⋯ + 𝑙𝑛 𝑓 𝑥𝑛 ; ∀𝑥1 , 𝑥2 , … , 𝑥𝑛 ∈ 𝐼 ដ ើ យ ដបើ 𝑓 ផតោច់ ខាត ដនាោះក្ដនោម្ខាងដលើកាលយជាសម្ភាព ដពល 𝑥1 = 𝑥2 = ⋯ = 𝑥𝑛 ក្រណី 𝑓 ជាអនុគម្ន៍ដបាង : 𝑓 𝑙1 𝑥1 + 𝑙2 𝑥2 + ⋯ + 𝑙𝑛 𝑥𝑛 ≥ 𝑙1 𝑓 𝑥1 + 𝑙2 𝑓 𝑥2 + ⋯ + 𝑙𝑛 𝑓 𝑥𝑛 ; ∀𝑥1 , 𝑥2 , … , 𝑥𝑛 ∈ 𝐼

យ៉ាហា ូ ន្ យិន្សន្ ិ

ឈ្ ម ោះឈពញ យ៉ា ូ ហាន លុ យឌ្វច ិ វ ីលាម វ៉ាល់ដឺមមរ យិ នសិ ន (ឧសភា ១៨៥៩ - កុមភៈ ១៩២៥) គណិតវ ិទូ និង វ ិសវ ករជាតិដាណឺម៉ាក។ ឈគស្គាល់ គាត់ ឈដាយស្គរវ ិសមភាពយិ នសិ ន។ កនង ុ ឆ្នាំ ១៩១៥ គាត់ បានបង្ហាញថា រូបមនត វ ិសមភាពរបស់ គាត់ អាចឈ្បើបានឈលើ វ ិភាគកុាំផច លិ ។ លឹម សុ វណ្ណវិចិត្រ | V. វិសមភាព

29


269.

(កូមរ ៉ា ១៩៩៨)

270.

ដគដោយ 𝑎, 𝑏, 𝑐 > 0 ដដល 𝑎 + 𝑏 + 𝑐 = 𝑎𝑏𝑐 ។ ចូរបង្ហាញថា 1 1 1 3 + + ≤ 2 2 2 2 1+𝑎 1+𝑏 1+𝑐 ចូរក្ំនត់ ចំនន ួ ដថរ 𝑀 តូចបំ ផុត ដដលចំដ ោះរគប់ 𝑎, 𝑏 > 0 ដគមាន 1

1

𝑎3 + 𝑏 3 ≤ 𝑀 𝑎 + 𝑏

271.

ដគដោយ 𝑥1 , 𝑥2 , … , 𝑥𝑛 ∈ ℝ+∗ ដដល 𝑥1 + 𝑥2 + ⋯ + 𝑥𝑛 = 1។ ចូរបង្ហាញថា 𝑥1 1 − 𝑥1

272.

+

(អូគ្ទស ២០០០) ី

𝑥2 1 − 𝑥2

+ ⋯+

ដគដោយ 𝑎, 𝑏 > 0 និង 𝑛 ∈ ℤ ។ ចូរបង្ហាញថា 1+

273.

274.

1 3

(អន្តរជាតិ ២០០១)

𝑎 𝑏

𝑛

+ 1+

𝑏 𝑎

𝑥𝑛 1 − 𝑥𝑛

𝑛 𝑛−1

𝑛

≥ 2𝑛+1

ចូរបង្ហាញថា ចំដ ោះរគប់ ចំនន ួ ពិត 𝑙 ≥ 8 និង 𝑎, 𝑏, 𝑐 > 0 ដគមាន 𝑎 𝑏 𝑐 3 + + ≥ 1+𝜆 𝑎2 + 𝜆𝑏𝑐 𝑏 2 + 𝜆𝑐𝑎 𝑐 2 + 𝜆𝑎𝑏 +∗ ដគដោយចំនន ួ គត់ 𝑛 ≥ 1 ។ តាង 𝛼, 𝑡 ∈ 1; +∞ និង 𝛽 ∈ ℝ ។ តាង 𝑎1 , 𝑎2 , … , 𝑎𝑛 ∈ ℝ+∗ ដដល 𝑎1 + 𝑎2 + ⋯ + 𝑎𝑛 = 1 ។ ចូរបង្ហាញថា 𝑛

𝑎𝑖𝛼 𝑖=1

+

1

𝑡

𝛽

𝑎𝑖

≥𝑛

1 + 𝑛𝛽 𝑛𝛼

𝑡

ដោយអងគ ទំងពីរដសមគ្ន ើ ន ទល់ ដតនិងនាំដោយ 𝑎1 = 𝑎2 = ⋯ = 𝑎𝑛 = 1 𝑛 ។

275.

(អន្តរជាតស ១៩៩៨) ិ តលស ី

ដគដោយ 𝑥1 , 𝑥2 , … , 𝑥𝑛 ≥ 1 ។ ចូរបង្ហាញថា 1 1 1 + + ⋯+ ≥ 𝑥1 + 1 𝑥2 + 1 𝑥𝑛 + 1

276.

(អាមមរច ិ ១៩៨០)

ដគដោយ 𝑎, 𝑏, 𝑐 ∈ 0; 1 ។ ចូរបង្ហាញថា 30

១. អនុគមន៍ងាយ | លឹម សុ វណ្ណវិចិត្រ

𝑛 1 + 𝑥1 𝑥2 … 𝑥𝑛

1 𝑛


𝑎 𝑏 𝑐 + + + 1−𝑎 1−𝑏 1−𝑐 ≤ 1 𝑏+𝑐+1 𝑐+𝑎+1 𝑎+𝑏+1

277.

(អាមមរច ិ ១៩៧៧)

ដគដោយ 0 < 𝑝 ≤ 𝑎, 𝑏, 𝑐, 𝑑, 𝑒 ≤ 𝑞 ។ ចូរបង្ហាញថា 2

𝑎+𝑏+𝑐+𝑑+𝑒

278.

1 1 1 1 1 + + + + ≤ 25 + 6 𝑎 𝑏 𝑐 𝑑 𝑒

𝑝 𝑞 − 𝑞 𝑝

(អាសុ ប ី ៉ា សុ ភ ី ច ិ ២០០៤)

ចូរបង្ហាញថា ចំដ ោះរគប់ ចំនន ួ ពិត 𝑎, 𝑏, 𝑐 > 0 𝑎2 + 2 𝑏 2 + 2 𝑐 2 + 2 ≥ 9 𝑎𝑏 + 𝑏𝑐 + 𝑐𝑎

279.

(អន្តរជាតិ ១៩៨៣)

តាង 𝑎, 𝑏, 𝑐 ជារង្ហវស់ រជុ ងននរតីដកាណម្ួ យ។ ចូរបង្ហាញថា 𝑎2 𝑏 𝑎 − 𝑏 + 𝑏 2 𝑐 𝑏 − 𝑐 + 𝑐 2 𝑎 𝑐 − 𝑎 ≥ 0

280.

ិ មា ន្ិយមន្័យ- ដគដោយ ចំនន ួ គត់ 𝑛 ≥ 2 បណា ត ចំនន ួ ពិតវជ ជ ន 𝑎1 , 𝑎2 , … , 𝑎𝑛 និង បណា ត ិ មា ចំនន ួ ពិតវជ ជ ន 𝑙1 , 𝑙2 , … , 𝑙𝑛 ដដល

𝑛 𝑖=1 𝑙𝑖

= 1 ។ ដយើ ងក្ំនត់ អនុគម្ន៍ 𝑀 ដលើ ℝ∗ ដោយ 1 𝛼

𝑛

𝑙𝑖 𝑎𝑖𝛼

𝑀 𝑎 = 𝑖=1

ដគដៅ 𝑀 𝛼 ថា ម្ធយម្លំ ោប់ 𝛼 ននបណា ត ចំនន ួ 𝑎𝑖 ផេំនង ឹ ដម្គុ ណ 𝑙𝑖 ។

គ្ទស ដាប់ 𝛼 ិ ឹ ប ី ត ទ វសមភាពមធ្យមលំ

ដគដោយ 𝑎1 , 𝑎2 , … , 𝑎𝑛 > 0 ម្ិនដសមគ្ន ើ ន ទំងអស់ និង 𝑙1 , 𝑙2 , … , 𝑙𝑛 > 0 ដដល 𝑙1 + 𝑙2 + ⋯ + 𝑙𝑛 = 1 ។ ចូរបង្ហាញថា អនុគម្ន៍ 𝑀 𝛼 ដក្ើនោច់ ខាតដលើ ℝ មានន័យថា ចំដ ោះរគប់ 𝑎1 , 𝑎2 , … , 𝑎𝑛 > 0 និង 𝛼 < 𝛽 ដគមាន 𝑀 𝛼 ≤ 𝑀 𝛽 ដោយអងគ ទំងពីរដសមើ គ្នន ទល់ ដតនិងនាំដោយ 𝑎1 = 𝑎2 = ⋯ = 𝑎𝑛 ។

281.

ចូរបង្ហាញថា ចំដ ោះ 𝑎1 , 𝑎2 , … , 𝑎𝑛 > 0 ដគមាន min 𝑎1 , 𝑎2 , … , 𝑎𝑛 ≤ 𝑀 −1 ≤ 𝑀 0 ≤ 𝑀 1 ≤ 𝑀 2 ≤ max 𝑎1 , 𝑎2 , … , 𝑎𝑛 រឺ

លឹម សុ វណ្ណវិចិត្រ | V. វិសមភាព

31


min 𝑎1 , 𝑎2 , … , 𝑎𝑛 ≤

𝑛 1 1 1 + +⋯+ 𝑎1 𝑎2 𝑎𝑛

≤ 𝑎1 𝑎2 … 𝑎𝑛

1 𝑛

𝑎1 + 𝑎2 + ⋯ + 𝑎𝑛 𝑛

𝑎12 + 𝑎22 + ⋯ + 𝑎𝑛2 𝑛

ដ ើ យសម្ភាពដក្ើតមាន ដពល 𝑎1 = 𝑎2 = ⋯ = 𝑎𝑛 ។

282.

(មអៀរង ៉ា ់ ១៩៩៨) ដគដោយ 𝑥1 , 𝑥2 , 𝑥3 , 𝑥4 > 0 ដដល 𝑥1 𝑥2 𝑥3 𝑥4 = 1 ។ ចូរបង្ហាញថា 4

4

𝑥𝑖3

283.

𝑖=1

𝑥𝑖 ; 𝑖=1

𝑖=1

1 𝑥𝑖

ដគដោយ 𝑥, 𝑦, 𝑧 ≥ 0 ។ ចូរបង្ហាញថា 8 𝑥3 + 𝑦3 + 𝑧3

284.

≥ max

4

2

≥ 9 𝑥 2 + 𝑦𝑧 𝑦 2 + 𝑧𝑥 𝑧 2 + 𝑥𝑦

ដគដោយចំនន ួ គត់ 𝑛 > 1 និង 𝑥1 , 𝑥2 , … , 𝑥𝑛 > 0 ដដល 𝑥1 + 𝑥2 + ⋯ + 𝑥𝑛 = 1 ចូរបង្ហាញថា 𝑥1 1 − 𝑥1

285.

+

(ទួរគី ១៩៩៧)

𝑥2 1 − 𝑥2

+ ⋯+

𝑥𝑛 1 − 𝑥𝑛

𝑥1 + 𝑥2 + ⋯ + 𝑥𝑛 𝑛−1

ដគដោយចំនន ួ គត់ 𝑛 ≥ 2 ។ ចូរគណនាតំនលតូចបំ ផុតនន 𝑥15 𝑥25 𝑥𝑛5 + + ⋯+ 𝑥2 + 𝑥3 + ⋯ + 𝑥𝑛 𝑥1 + 𝑥3 + ⋯ + 𝑥𝑛 𝑥1 + 𝑥2 + ⋯ + 𝑥𝑛−1 2 2 2 ដដល 𝑥1 , 𝑥2 , … , 𝑥𝑛 > 0 ដ ើ យ 𝑥1 + 𝑥2 + ⋯ + 𝑥𝑛 = 1 ។

286.

(វសមភាព ិ

)

ិ មា តាង 𝑥; 𝑦; 𝑧 ជាចំនន ួ ពិតម្ិនអវជ ជ ន។ ចូរបង្ហាញថា ចំដ ោះរគប់ 𝑟 > 0 ដយើ ងមាន 𝑥𝑟 𝑥 − 𝑦 𝑥 − 𝑧 + 𝑦𝑟 𝑦 − 𝑧 𝑦 − 𝑥 + 𝑧𝑟 𝑧 − 𝑥 𝑧 − 𝑦 ≥ 0 រឺ 𝑥 𝑟 𝑥 − 𝑦 (𝑥 − 𝑧) ≥ 0 cyclic

32

១. អនុគមន៍ងាយ | លឹម សុ វណ្ណវិចិត្រ


287.

1) ដគដោយ 𝑎, 𝑏, 𝑐 > 0 ។ ចូរបង្ហាញថា 𝑎3 + 𝑏 3 + 𝑐 3 + 3𝑎𝑏𝑐 ≥ 𝑎𝑏 𝑎 + 𝑏 + 𝑏𝑐 𝑏 + 𝑐 + 𝑐𝑎 𝑐 + 𝑎 ≥ 2 𝑎𝑏 3/2 + 𝑏𝑐 3/2 + 𝑐𝑎 3/2

2) តាង 𝑡 ∈ 0; 3 ។ ចំដ ោះរគប់ 𝑎, 𝑏, 𝑐 ≥ 0 ចូរបង្ហាញថា 3 − 𝑡 + 𝑡 𝑎𝑏𝑐 2/𝑡 + 𝑎2 + 𝑏 2 + 𝑐 2 ≥ 2 𝑎𝑏 + 𝑏𝑐 + 𝑐𝑎 3) បង្ហាញថា 5 1 + 𝑎𝑏𝑐 4 + 𝑎2 + 𝑏 2 + 𝑐 2 ≥ 2 𝑎𝑏 + 𝑏𝑐 + 𝑐𝑎 2 2 2 + 𝑎𝑏𝑐 2 + 𝑎2 + 𝑏 2 + 𝑐 2 ≥ 2 𝑎𝑏 + 𝑏𝑐 + 𝑐𝑎 1 + 2𝑎𝑏𝑐 + 𝑎2 + 𝑏 2 + 𝑐 2 ≥ 2 𝑎𝑏 + 𝑏𝑐 + 𝑐𝑎

288. 1) 2)

289.

ដគដោយ 𝑎, 𝑏, 𝑐 > 0។ ចូរបង្ហាញថា 𝑏+𝑐 𝑐+𝑎 𝑎+𝑏 1 1 1 + + ≤ + + 𝑎2 + 𝑏𝑐 𝑏 2 + 𝑐𝑎 𝑐 2 + 𝑎𝑏 𝑎 𝑏 𝑐 𝑎2 + 𝑏𝑐 𝑏 2 + 𝑐𝑎 𝑐 2 + 𝑎𝑏 + + ≥𝑎+𝑏+𝑐 𝑏+𝑐 𝑐+𝑎 𝑎+𝑏 ិ មា 1) តាង 𝑎1 , 𝑎2 , 𝑏1 , 𝑏2 ជាចំនន ួ ពិតវជ ជ ន ដដល 𝑎1 + 𝑎2 = 𝑏1 + 𝑏2 និង max 𝑎1 , 𝑎2 ≥ ិ មា max 𝑏1 , 𝑏2 ។ តាង 𝑥 និង 𝑦 ជាចំនន ួ ពិតម្ិនអវជ ជ ន។ ចូរបង្ហាញថា 𝑥 𝑎 1 𝑦 𝑎 2 + 𝑥 𝑎 2 𝑦 𝑎 1 ≥ 𝑥 𝑏1 𝑦 𝑏2 + 𝑥 𝑏2 𝑦 𝑏1

2) (វសមភាព ិ

)

តាង 𝑎1 ; 𝑎2 ; 𝑎3 ; 𝑏1 ; 𝑏2 ; 𝑏3 ជាចំនន ួ ពិត ដដល 𝑎1 ≥ 𝑎2 ≥ 𝑎3 ≥ 0; 𝑏1 ≥ 𝑏2 ≥ 𝑏3 ≥ 0; 𝑎1 ≥ 𝑏1 ; 𝑎1 + 𝑎2 ≥ 𝑏1 + 𝑏2 ; 𝑎1 + 𝑎2 + 𝑎3 = 𝑏1 + 𝑏2 + 𝑏3 ក្រណី ដនោះ ដគថាសវត វី 𝑎1 ; 𝑎2 ; 𝑎3 លប់ ដលើ សត វី 𝑏1 ; 𝑏2 ; 𝑏3 ។ ិ មា ចូរបង្ហាញថា ចំដ ោះរគប់ ចំនន ួ ពិតវជ ជ ន 𝑥; 𝑦; 𝑧 ដគមាន 𝑥 𝑎 1 𝑦𝑎 2 𝑧𝑎 3 ≥ sym

290. 291.

𝑥 𝑏1 𝑦 𝑏2 𝑧 𝑏3 sym

ិ មា ដគដោយ 𝑎, 𝑏, 𝑐 ជាចំនន ួ វជ ជ ន ដដល 𝑎𝑏𝑐 = 1 ។ ចូរបង្ហាញថា 1 1 1 + + ≤1 𝑎+𝑏+1 𝑏+𝑐+1 𝑐+𝑎+1 ដគដោយ 𝑎, 𝑏, 𝑐, 𝑑 > 0 ។ ចូរបង្ហាញថា

3 3 𝑎 + 𝑏 3 + 𝑐 3 + 𝑑3 ≥ 𝑎𝑏 𝑐𝑑 + 𝑎𝑐 𝑏𝑑 + 𝑎𝑑 𝑏𝑐 + 𝑏𝑐 𝑑𝑎 + 𝑏𝑑 𝑐𝑎 + 𝑐𝑑 𝑎𝑏 2 លឹម សុ វណ្ណវិចិត្រ | V. វិសមភាព

33


292.

(មអៀរង ៉ា ់ ១៩៩៦)

ិ មា តាង 𝑥; 𝑦; 𝑧 ជាចំនន ួ ពិតវជ ជ ន។ ចូរបង្ហាញថា 1 1 𝑥𝑦 + 𝑦𝑧 + 𝑧𝑥 + 𝑥+𝑦 2 𝑦+𝑧 293.

294.

+ 2

1 𝑧+𝑥

2

9 4

ិ មា តាង 𝑥; 𝑦; 𝑧 ជាចំនន ួ ពិតវជ ជ ន ដដល 𝑥𝑦 + 𝑦𝑧 + 𝑧𝑥 = 1។ ចូរបង្ហាញថា 1 1 1 5 + + ≥ 𝑥+𝑦 𝑦+𝑧 𝑧+𝑥 2

(អន្តរជាតស ១៩៩០) ិ តលស ី

ចូរបង្ហាញថា ចំដ ោះរគប់ ចំនន ួ ពិត 𝑎, 𝑏, 𝑐 ដយើ ងមាន 𝑎2 + 𝑎𝑏 + 𝑏 2 𝑏 2 + 𝑏𝑐 + 𝑐 2 𝑐 2 + 𝑐𝑎 + 𝑎2 ≥ 𝑎𝑏 + 𝑏𝑐 + 𝑐𝑎

295.

(មអៀរង ៉ា ់ ១៩៩៦)

ដគដោយ 𝑥, 𝑦, 𝑧 > 0 ។ ចូរបង្ហាញថា 1 𝑥𝑦 + 𝑦𝑧 + 𝑧𝑥 𝑥+𝑦

296.

2

+

1 𝑦+𝑧

2

+

1 𝑧+𝑥

2

9 4

(ជប៉ាុន្ ១៩៩៧) ដគដោយ 𝑎, 𝑏, 𝑐 > 0 ។ ចូរបង្ហាញថា 𝑏+𝑐−𝑎 2 𝑐+𝑎−𝑏 2 𝑎+𝑏−𝑐 2 3 + + ≥ 𝑏 + 𝑐 2 + 𝑎2 𝑐 + 𝑎 2 + 𝑏2 𝑎 + 𝑏 2 + 𝑐2 5

វិសមីការ ិ ម្ីការ ដោ​ោះរាយវស

297.

𝑥 3 − 3𝑥 2 − 10𝑥 + 24 > 0

298.

𝑥 3 + 4𝑥 2 + 5𝑥 + 2 ≤ 0

299.

2𝑥 3 − 3𝑥 2 + 7𝑥 − 3 > 0

300.

𝑥 4 − 3𝑥 3 + 𝑥 2 + 3𝑥 − 2 ≥ 0

301.

𝑥 4 + 6𝑥 3 + 6𝑥 2 + 6𝑥 + 5 < 0

302.

3𝑥 4 − 10𝑥 2 + 3 > 0

303.

3𝑥 2 𝑥 − 4

34

2

< 32 − 5 𝑥 − 2

១. អនុគមន៍ងាយ | លឹម សុ វណ្ណវិចិត្រ

2

3


304. 305. 306.

ЁЭСе2 тИТ ЁЭСе

2

+ ЁЭСе2 тИТ ЁЭСе + 2 тЙе 0

ЁЭСе ЁЭСе + 1 ЁЭСе + 2 ЁЭСе + 3 < 48 ЁЭСе+1

4

> 2 1 + ЁЭСе4

307.

ЁЭСе 4 тИТ ЁЭСе 3 тИТ 10ЁЭСе 2 + 2ЁЭСе + 4 < 0

308.

ЁЭСе+4 2 1 2 309. < < ЁЭСетИТ2 ЁЭСе+1 ЁЭСе ЁЭСетИТ2 1 1 1 1 тИТ тЙд тИТ ЁЭСе+1 ЁЭСе ЁЭСетИТ1 ЁЭСетИТ2

310. 311.

ЁЭСе 2 тИТ ЁЭСе + 1 ЁЭСе 2 тИТ 3ЁЭСе + 1 1 + > 2ЁЭСе тИТ ЁЭСетИТ1 ЁЭСетИТ3 4ЁЭСе тИТ 8

312.

ЁЭСе 3 тИТ 2ЁЭСе 2 + 5ЁЭСе + 2 тЙе1 ЁЭСе 2 + 3ЁЭСе + 2

313.

1 1 1 1 + + + >0 ЁЭСе+5 ЁЭСетИТ7 ЁЭСетИТ5 ЁЭСе+7

314.

1 1 тЙд ЁЭСе 2 + ЁЭСе 2ЁЭСе 2 + 2ЁЭСе + 3

315.

1 4 4 1 1 тИТ + тИТ < ЁЭСе тИТ 1 ЁЭСе тИТ 2 ЁЭСе тИТ 3 ЁЭСе тИТ 4 30

316.

4ЁЭСе тИТ 17 10ЁЭСе тИТ 13 8ЁЭСе тИТ 30 5ЁЭСе тИТ 4 + > + ЁЭСетИТ4 2ЁЭСе тИТ 3 2ЁЭСе тИТ 7 ЁЭСетИТ1

317.

ЁЭСе2 +

319.

ЁЭСе2 5 < 2 ЁЭСе+1 4 ЁЭСе+1 4 128 > 2 ЁЭСе(ЁЭСе + 1) 15

320.

ЁЭСе3 тИТ

321.

ЁЭСе+6 ЁЭСетИТ4 ЁЭСетИТ6 ЁЭСе+4

322.

ЁЭСе3 тИТ ЁЭСе тЙд ЁЭСе

318.

ЁЭСе2 +

4ЁЭСе 2 ЁЭСетИТ2

2

тЙд5

1 1 тЙе4 ЁЭСетИТ 3 ЁЭСе ЁЭСе

сЮЫсЮ╣сЮШ сЮЯсЮ╗ сЮЬсЮОсЯТсЮОсЮЬсЮ╖сЮЕсЮ╖сЮПсЯТсЮЪ | VI. сЮЬсЮ╖сЮЯсЮШсЮ╕сЮАсЮ╢сЮЪ

2

2

2ЁЭСе 2 + 72 < 2 ЁЭСе тИТ 36 4 323. тЙе ЁЭСетИТ1 ЁЭСе+1 тИТ2

ЁЭСетИТ6 ЁЭСе+9 + ЁЭСе+6 ЁЭСетИТ9

35


324.

𝑥−1

𝑥2 − 𝑥 − 2 ≥ 0

326.

4 − 𝑥2 +

328. 330. 332.

𝑥 ≥0 𝑥

325.

𝑥−2 > −1 1 − 2𝑥

327.

1−𝑎

𝑥+1> 3−𝑥

329.

𝑥+2≥ 𝑥−𝑎

24 − 10𝑥 > 3 − 4𝑥

331.

𝑥 > 24 − 5𝑥

333.

2𝑥 + 1 < 1

𝑥 > 1−𝑥 1 3 1 1 − < − 𝑥2 4 𝑥 2

334.

4− 1−𝑥− 2−𝑥 > 0

335.

𝑥 2 + 𝑥 − 12 < 𝑥

336.

1 − 13 + 3𝑥 2 ≤ 2𝑥

337.

𝑥 2 + 𝑥 > 1 − 2𝑥

338.

4−𝑥 <

339.

𝑥+3> 𝑥−1+ 𝑥−2 𝑥−2

340.

2𝑥 − 3 − 1

342.

𝑥 + 2 − 5𝑥 > 4𝑥 − 2

344.

24 + 2𝑥 − 𝑥 2 <1 𝑥

345.

<4

2 𝑥 + 𝑥 2 + 4𝑥 + 3 < 3

341. 343.

𝑥 2 − 2𝑥

2− 𝑥+3 1 >− 𝑥−1 3 𝑥 + 1 + 1 < 4𝑥 2 + 3𝑥

𝑥+1+ 𝑥+3−2

346.

ចូរបង្ហាញថាព ុ ធា 𝑃 𝑥 = 𝑥 8 − 𝑥 5 + 𝑥 2 − 𝑥 + 1 > 0, ∀𝑥 ∈ ℝ ។

347.

សនមតថា 𝑃 𝑥 = 𝑎0 𝑥 𝑚 + 𝑎1 𝑥 𝑚 −1 + ⋯ + 𝑎𝑚 −1 𝑥 + 𝑎𝑚 និង 𝑄 𝑥 = 𝑏0 𝑥 𝑛 +

ិ ម្ភាព 𝑃 𝑥 𝑏1 𝑥 𝑛−1 + ⋯ + 𝑏𝑛−1 𝑥 + 𝑏𝑛 , 𝑚 ∈ ℕ, 𝑛 ∈ ℕ, 𝑄 𝑥 ≢ 0 ។ ចូរបង្ហាញថាវស 0 និង 𝑃 𝑥 𝑄 𝑥 > 0 សម្ម្ូ លគ្នន។ 36

១. អនុគមន៍ងាយ | លឹម សុ វណ្ណវិចិត្រ

𝑄 𝑥 >


348.

ឹ ថា 𝑓 −1 < 1, 𝑓 1 > −1 , 𝑓 3 < −4 ដដល 𝑓 𝑥 = 𝑎𝑥 2 + 𝑏𝑥 + 𝑐, 𝑎 ≠ 0 ដោយដង ចូរក្ំនត់ សញ្ញារបស់ 𝑎 ។

349.

(អន្តរជាតិ ១៩៦០) ិ ម្ីការ ដោ​ោះរាយវស

4𝑥 2 1 − 1 + 2𝑥

350.

2

< 2𝑥 + 9

(អន្តរជាតិ ១៩៦២) ិ ម្ីការ ដោ​ោះរាយវស

3−𝑥− 𝑥+1>

351.

ិ ម្ីការ ដោ​ោះរាយវស

1 2

9𝑥 2 + 16 ≥ 2 2𝑥 + 4 + 4 2 − 𝑥

ត្រព័នវធ ស ិ មីការ ិ ម្ីការ 352*. ដោ​ោះរាយរបព័នវធ ស 𝑥 + 𝑦 ≤ 2, 𝑥 + 𝑦 ≥ 1, 𝑥 ≥ 0, 𝑦≥0 ដដលក្នវងដនាោះ 𝑧 = 2𝑥 + 3𝑦 មានតំនលធំបំផុត។

353.

354.

ចូរក្ំនត់ រ ឺសគត់ ធម្ម ជាតិរបស់ របព័នធ

ចូរក្ំនត់ តនំ លរបស់ 𝑎 ដដលសំ នុំ

𝑥 =𝑦+2 𝑥𝑦 ≤ 17 𝑦+1 1 < 𝑥+2 2

𝑥; 𝑦 𝑥 2 + 𝑦 2 + 2𝑥 ≤ 1 ∩ មានចំនច ុ ដតម្ួ យគត់ ។ ចូរក្ំនត់ ចំនច ុ ដនាោះ។

355.

𝑥; 𝑦 𝑥 − 𝑦 + 𝑎 ≥ 0

ចូរក្ំនត់ 𝑥; 𝑦 ដដល 𝑥 2 − 2𝑥𝑦 + 12 = 0 𝑥 2 + 4𝑦 2 ≤ 60 𝑥∈ℤ

លឹម សុ វណ្ណវិចិត្រ | VII. ត្រព័នវធ ស ិ មីការ

37


356.

ិ ម្ីការ ដោ​ោះរាយរបព័នវធ ស 4 − 3𝑥 ≥ 𝑥 𝑥+ 𝑥−1<5

357.

ដោ​ោះរាយរបព័នស ធ ម្ីការ 3

𝑥 + 𝑥 2 13 − 𝑦 − 𝑧 + 𝑥 2𝑦 + 2𝑧 − 2𝑦𝑧 − 26 + 5𝑦𝑧 − 7𝑦 − 7𝑧 + 30 = 0 (1) 𝑥 3 + 𝑥 2 17 − 𝑦 − 𝑧 − 𝑥 2𝑦 + 2𝑧 + 2𝑦𝑧 − 26 − 3𝑦𝑧 + 𝑦 + 𝑧 − 2 = 0 (2) 2 𝑥 − 11𝑥 + 28 ≤ 0 (3) ើ បីដោយរបព័នខា 358. ចូរក្ំនត់ 𝑎 ដដម្ ធ ងដរកាម្មានរ ឺស 1−𝑎 𝑎+1 3𝑥 2 + 10𝑥𝑦 − 5𝑦 2 ≤ −2 𝑥 2 + 2𝑥𝑦 − 7𝑦 2 ≥

38

១. អនុគមន៍ងាយ | លឹម សុ វណ្ណវិចិត្រ


á&#x;˘. áž˘áž“ážťáž‚áž˜áž“á&#x;? អឝá&#x;Šážˇ áž…áž&#x;á&#x;’áž”á&#x;‰ážźážŽáž„á&#x;‹áž&#x;á&#x;’áž&#x;á&#x;‚លលá&#x;„កវរីáž?

. គណá&#x;’áž“ážś គណនវ

359. 362. 364.

360.

25log 5 3

log đ?‘Ž đ?‘? 2 + log đ?‘Ž 2 đ?‘? 4 log 2 25 log 2 5

366.

log 20,5 4

361.

đ?‘’ ln ln 3 363.

1 log 2 32

365.

log 3 5 . log 4 9 . log 5 2

367.

đ?‘Ž

log đ?‘Ž đ?‘?

−đ?‘?

ln đ?‘Žđ?‘? − ln đ?‘?

log đ?‘? đ?‘Ž

368. ចសរគណនវ log 30 8 áž” ážž log 30 3 = đ?‘?, log 30 5 = đ?‘‘ á&#x;” 369. ចសរគណនវ log 9 40 áž” ážž log 15 = đ?‘?, log 20 50 = đ?‘‘ á&#x;” 370. ចសរគណនវ log 0,175

4

áž” ážž log 196 = đ?‘?, log 56 = đ?‘‘ á&#x;”

371. គណនវ log 2 24 log 2 192 − log 96 2 log12 2

លរម áž&#x;ážť វណá&#x;’ážŽážœážˇáž…ážˇáž?á&#x;’ážš | I. គណá&#x;’áž“ážś

39


áž&#x;áž˜áž—ážśáž– 372. áž…ážźážš áž„á&#x;’ហវញáž?ážś áž” ážž đ?‘Ž = log12 18 , đ?‘? = log 24 54 បនវá&#x;„á&#x;‡ đ?‘Žđ?‘? + 5 đ?‘Ž − đ?‘? = 1 á&#x;”

III. áž&#x;áž˜áž¸áž€ážśážš áž” á&#x;„á&#x;‡áž&#x;á&#x;’ážšážśáž™áž&#x;áž˜áž¸áž€ážśážš

373.

4đ?‘Ľ+1,5 + 9đ?‘Ľ = 6đ?‘Ľ +1

374.

252đ?‘Ľâˆ’đ?‘Ľ

375.

22đ?‘Ľ + 2đ?‘Ľ

2 +1

2

+ 92đ?‘Ľâˆ’đ?‘Ľ

2 +2đ?‘Ľ+2

2 +1

= 34.152đ?‘Ľâˆ’đ?‘Ľ

= 25+4đ?‘Ľ

đ?‘Ľ

đ?‘Ľ

376.

5 2−7

377.

32đ?‘Ľ − 2.3đ?‘Ľ

378.

đ?‘Ľ2 . 2

2đ?‘Ľ+1−1

379.

4log 64

đ?‘Ľâˆ’3 +log 2 5

2

2

+6

2 +đ?‘Ľ+6

5 2+7

+ 32

+ 2đ?‘Ľ = 2

đ?‘Ľ+6

=7

=0

2đ?‘Ľ+1+1

+ đ?‘Ľ 2 . 2đ?‘Ľâˆ’2

= 50

380.

đ?‘Ľ log đ?‘Ľ

381.

log 3 đ?‘Ľ 2 + 4đ?‘Ľ + 12 = 2

382.

log 3 đ?‘Ľ + log 9 đ?‘Ľ + log 27 đ?‘Ľ = 5,5

383.

log 2 3 − đ?‘Ľ + log 2 1 − đ?‘Ľ = 3

384.

log đ?‘Ľ − 3 + log đ?‘Ľ + 6 = log 2 + log 5

385.

log đ?‘Ľ − 4 + log đ?‘Ľ + 3 = log 5đ?‘Ľ + 4

386.

ln đ?‘Ľ 3 + 1 − 0,5 ln đ?‘Ľ 2 + 2đ?‘Ľ + 1 = ln 3

387.

log 5 đ?‘Ľ − 2 + 2 log 5 đ?‘Ľ 3 − 2 + log 5 đ?‘Ľ − 2

388.

2 log 3 đ?‘Ľ − 2 + log 3 đ?‘Ľ − 4

389.

log 2 đ?‘Ľ + 2

390.

log 2

40

1−đ?‘Ľ

2

=9

2

+ log 2 đ?‘Ľ + 10

2 2

=0 = 4 log 2 3

đ?‘Ľâˆ’2 3đ?‘Ľ − 7 − 1 = log 2 đ?‘Ľâˆ’1 3đ?‘Ľ − 1

á&#x;˘. áž˘áž“ážťáž‚áž˜áž“á&#x;?អឝឡចáž&#x;á&#x;’áž”á&#x;‰ážźážŽá&#x;’áž„á&#x;‹áž&#x;á&#x;’áž&#x;á&#x;‚áž›-áž›á&#x;„កវរីរ | លរម áž&#x;ážť វណá&#x;’ážŽážœážˇáž…ážˇáž?á&#x;’ážš

−1

=4


đ?‘Ľâˆ’7 đ?‘Ľâˆ’1 + log 2 =1 đ?‘Ľâˆ’1 đ?‘Ľ+1

391.

2 log 2

392.

394.

log 3 5đ?‘Ľ − 2 − 2 log 3 3đ?‘Ľ + 1 = 1 − log 3 4 1 log 3đ?‘Ľ − 2 − 2 = log đ?‘Ľ + 2 − log 50 2 4 4 2 log 2 1 + + log 2 1 − = 2 log 2 −1 đ?‘Ľ đ?‘Ľ+4 đ?‘Ľâˆ’1

395.

log 2 đ?‘Ľ 4 + log đ?‘Ž đ?‘Ľ 2 = 1

396.

log 2 đ?‘Ľ − 1 − log

397.

log 2 6đ?‘Ľ 2 + 25đ?‘Ľ = 1 + log 2 đ?‘Žđ?‘Ľ + 4đ?‘Ž − 2

398.

log 3 đ?‘Ľ log 4 đ?‘Ľ log 5 đ?‘Ľ = log 3 đ?‘Ľ log 4 đ?‘Ľ + log 4 đ?‘Ľ log 5 đ?‘Ľ + log 5 đ?‘Ľ log 3 đ?‘Ľ

393.

399.

log 3

2

đ?‘Ľ + 3 = log 8 đ?‘Ľ − đ?‘Ž

3

+ log1/2 đ?‘Ľ − 3

3 đ?‘Ľ3 1 log 2 đ?‘Ľ − log 3 = + log 2 đ?‘Ľ đ?‘Ľ 3 2

400.

log 10đ?‘Ľ 2 log đ?‘Ľ = 1

401.

log 2 đ?‘Ľ − 1 2 đ?‘Ľ = 2 log 2 đ?‘Ľ + 3 − log 2 đ?‘Ľ log 2 2

402.

2 log 9 đ?‘Ľ + 9 log đ?‘Ľ 3 = 10

403.

log đ?‘Ľ (125đ?‘Ľ) log 225 đ?‘Ľ = 1

404.

log đ?‘Ľ 5 + log đ?‘Ľ 5đ?‘Ľ =

405.

log log đ?‘Ľ + log log đ?‘Ľ 3 − 2 = 0

406.

log 3đ?‘Ľ+7 9 + 12đ?‘Ľ + 4đ?‘Ľ 2 = 4 − log 2đ?‘Ľ+3 6đ?‘Ľ 2 + 23đ?‘Ľ + 21

407.

log 2 4 − đ?‘Ľ + log 4 − đ?‘Ľ log đ?‘Ľ +

408.

9 + log 2đ?‘Ľ 5 4

1 1 − 2 log 2 đ?‘Ľ + =0 2 2

đ?‘Ľ 2 log đ?‘Ľ 27 log 9 đ?‘Ľ = đ?‘Ľ + 4

លរម áž&#x;ážť វណá&#x;’ážŽážœážˇáž…ážˇáž?á&#x;’ážš | III. áž&#x;áž˜áž¸áž€ážśážš

41


7 =0 6

409.

log đ?‘Ľ 2 − log 4 đ?‘Ľ +

410.

log 0,5đ?‘Ľ đ?‘Ľ 2 − 14 log16đ?‘Ľ đ?‘Ľ 3 + 40 log 4đ?‘Ľ đ?‘Ľ = 0

411.

4 log đ?‘Ľ đ?‘Ľ + 2 log 4đ?‘Ľ đ?‘Ľ 2 = 3 log 2đ?‘Ľ đ?‘Ľ 3

412.

log 3đ?‘Ľ

413. 414. 415.

2

log1/

3 + log 23 đ?‘Ľ = 1 đ?‘Ľ 1+đ?‘Ľ

10 log đ?‘Ľ 2 − 3đ?‘Ľ + 2 = log đ?‘Ľ − 3 log1/

log đ?‘Ľ 2đ?‘Ž − đ?‘Ľ log đ?‘Ž đ?‘Ľ 1 + = log đ?‘Ľ 2 log đ?‘Ž 2 log đ?‘Ž 2 −1 2 log đ?‘Ž 2 đ?‘Ľ đ?‘Ž + log đ?‘Žđ?‘Ľ đ?‘Ž log 1 2đ?‘Ľ = 0 log 2đ?‘Ľ đ?‘Ž đ?‘Ž

416.

1 + log 0,04 đ?‘Ľ + 3 + log 0,2 đ?‘Ľ = 1

417.

2 − log đ?‘Ľ 9 = −

418. 419.

log đ?‘Ľ đ?‘Ľ 2 + 1 =

12 log 3 đ?‘Ľ log

đ?‘Ľ

đ?‘Ľ2 1 + đ?‘Ľ2

+4

log 2 đ?‘Ľ − 0,5 = log 2 đ?‘Ľ 1 đ?‘Ľ = 2 + log 16 − log 4 4 2

420.

log 3đ?‘Ľ − 24−đ?‘Ľ

421. 422.

1 log 3 log 9 đ?‘Ľ + + 9đ?‘Ľ = 2đ?‘Ľ 2 2 2 log 3 3đ?‘Ľ −13đ?‘Ľ+28 + = log 5 0,2 9

423.

log 2 + log 4đ?‘Ľâˆ’2 + 9 = 1 + log 2đ?‘Ľâˆ’2 + 1

424.

log 6.5đ?‘Ľ + 25.20đ?‘Ľ = đ?‘Ľ + log 5

425.

log

42

5

4đ?‘Ľ − 6 − log 5 2đ?‘Ľ − 2

2

=2

á&#x;˘. áž˘áž“ážťáž‚áž˜áž“á&#x;?អឝឡចáž&#x;á&#x;’áž”á&#x;‰ážźážŽá&#x;’áž„á&#x;‹áž&#x;á&#x;’áž&#x;á&#x;‚áž›-áž›á&#x;„កវរីរ | លរម áž&#x;ážť វណá&#x;’ážŽážœážˇáž…ážˇáž?á&#x;’ážš

1+đ?‘Ľ

10 − 2


426.

đ?‘Ľ 1 − log 5 = log 4đ?‘Ľ − 12

427.

log 2 4đ?‘Ľ + 1 = đ?‘Ľ + log 2 2đ?‘Ľ+3 − 6

428.

log 3 9đ?‘Ľ + 9 = đ?‘Ľ − log 1 28 − 2.3đ?‘Ľ

429.

log 2

430. 431.

3

8 −1 =đ?‘Ľâˆ’2 2đ?‘Ľ 1 đ?‘Ľ log1/3 2 − 1 = log1/3 2 log đ?‘Ľ+1

đ?‘Ľ+1 đ?‘Ľ

433.

3log đ?‘Ľ = 54 − đ?‘Ľ log 3

434.

log 2 (9 − 2đ?‘Ľ ) = 10log

435.

đ?‘Ľâˆ’1

436.

3đ?‘Ľ

437. 438.

−4

= 105+log đ?‘Ľ

log 2 đ?‘Ľâˆ’log đ?‘Ľ 2

2 −7,2đ?‘Ľ+3,9

3.2log đ?‘Ľ

đ?‘Ľ

= 100 đ?‘Ľ + 1

432.

log đ?‘Ľ+5 3

1 4

3đ?‘Ľâˆ’2

3−đ?‘Ľ

= đ?‘Ľâˆ’1

3

− 9 3 log 7 − đ?‘Ľ = 0 + 2.3log đ?‘Ľ

3đ?‘Ľâˆ’2

= 5.6^ log đ?‘Ľ 2 (3đ?‘Ľ − 2)

1 − log1/5 đ?‘Ľ + 2 = 3 − log1/5 đ?‘Ľ

439.

log 4 6 + đ?‘Ľ −

440.

5đ?‘Ľ + 12đ?‘Ľ = 13đ?‘Ľ

441.

3đ?‘Ľ + 4đ?‘Ľ + 5đ?‘Ľ = 6đ?‘Ľ

442.

2đ?‘Ľ = 1 − đ?‘Ľ

443.

log 2 4 − đ?‘Ľ = đ?‘Ľ − 3

លរម áž&#x;ážť វណá&#x;’ážŽážœážˇáž…ážˇáž?á&#x;’ážš | III. áž&#x;áž˜áž¸áž€ážśážš

đ?‘Ľâˆ’2

=

1 + log 2 2

đ?‘Ľâˆ’

đ?‘Ľâˆ’2

43


ážš áž?ážś đ?‘Ľ = 9 ជវរ ážşáž&#x;áž˜áž˝ យនá&#x;ƒáž&#x;áž˜áž¸áž€ážśážš 444. áž” យដង log đ?œ‹ (đ?‘Ľ 2 + 15đ?‘Ž2 ) − log đ?œ‹ (đ?‘Ž − 2) = log đ?œ‹ ចសរកá&#x;†á&#x;ƒáž?á&#x;‹ ážš ážşáž&#x;áž”áž&#x;á&#x; áž„áž” á&#x;€áž?áž“á&#x;ƒáž&#x;áž˜áž¸áž€ážśážšáž”á&#x;ƒá&#x;„á&#x;‡á&#x;”

8đ?‘Žđ?‘Ľ đ?‘Žâˆ’2

IV. áž?á&#x;’ážšáž–á&#x;?áž“áž&#x; áž’ áž˜áž¸áž€ážśážš áž” á&#x;„á&#x;‡áž&#x;á&#x;’ážšážśáž™áž&#x;á&#x;’ážš áž–á&#x;?á&#x;ƒáž&#x; áž’ áž˜áž¸áž€ážśážš

445.

log đ?‘Ž 2 đ?‘Ľ − log đ?‘Ž 4 đ?‘Ś = 3 log đ?‘Ž 6 đ?‘Ľ + log đ?‘Ž 8 đ?‘Ś = 4

446.

2đ?‘Ľ+đ?‘Śâˆ’1 + 2đ?‘Ľâˆ’đ?‘Ś +1 = 3 1 đ?‘Ľ log 2 +đ?‘Ś log 2−2 1 3 3 .3 + 3đ?‘Ľ log 3 2−đ?‘Ś log 3 2−2 = 7 7

447. 448. 449. 450.

101+log đ?‘Ľ+đ?‘Ś = 50 log đ?‘Ľ − đ?‘Ś + log đ?‘Ľ + đ?‘Ś = 2 − log 5 đ?‘Ľ log đ?‘Ľ 2 = log 3 đ?‘Ľ + đ?‘Ś đ?‘Ľ 2 + đ?‘Ś 2 = 65 log 2 đ?‘Ľ + đ?‘Ś − log 3 đ?‘Ľ − đ?‘Ś = 1 đ?‘Ľ2 − đ?‘Ś2 = 2 3đ?‘Ś 2 + 1 log 3 đ?‘Ľ = 1 đ?‘Ľ 2đ?‘Ś

451. 452. 453. 44

2 +10

= 27

4−đ?‘Ś log 2 đ?‘Ľ = 4 log 2 đ?‘Ľ + 2−2đ?‘Ś = 4 đ?‘Ś + log đ?‘Ľ = 1 đ?‘Ľ đ?‘Ś = 0,01 đ?‘Ľ log đ?‘Ś = 2 đ?‘Ľđ?‘Ś = 20 á&#x;˘. áž˘áž“ážťáž‚áž˜áž“á&#x;?អឝឡចáž&#x;á&#x;’áž”á&#x;‰ážźážŽá&#x;’áž„á&#x;‹áž&#x;á&#x;’áž&#x;á&#x;‚áž›-áž›á&#x;„កវរីរ | លរម áž&#x;ážť វណá&#x;’ážŽážœážˇáž…ážˇáž?á&#x;’ážš


454. 455.

2đ?‘Ľ . 8−đ?‘Ś = 2 2 1 1 log 9 + 0,5 = log 3 9đ?‘Ś đ?‘Ľ 2 log đ?‘Ž đ?‘Ľđ?‘Ś − 2

log đ?‘Ž

4 9

−1

= −1

đ?‘Ľ + đ?‘Ś = 5đ?‘Ž 456.

2 log đ?‘Ś đ?‘Ľ + log đ?‘Ľ đ?‘Ś = 5 đ?‘Ľđ?‘Ś = 8

457.

log đ?‘Ľ đ?‘Ś + log đ?‘Ś đ?‘Ľ = 2,5 đ?‘Ľ + đ?‘Ś = đ?‘Ž2 + đ?‘Ž

458.

đ?‘Ľ2 đ?‘Ś2 + = 28 đ?‘Ś đ?‘Ľ log 9 đ?‘Ľ − log 1 đ?‘Ś = 1,5

459.

log 2 đ?‘Ś = log 4 đ?‘Ľđ?‘Ś − 2 log 9 đ?‘Ľ 2 + log 3 (đ?‘Ľ − đ?‘Ś) = 1

9

460. 461. 462. 463. 464.

2

đ?‘Ś 2 +đ?‘Ľ 2

2đ?‘Ľ +đ?‘Ś = 4 đ?‘Ľđ?‘Ś = 2 đ?‘Ľ

−3đ?‘Ś/đ?‘Ľ

4đ?‘Ś = 16 đ?‘Ľ − 2đ?‘Ś = 12 − 8 đ?‘Ľ + đ?‘Ś = 4 + đ?‘Ś2 + 2 log đ?‘Ľ − 2 log 2 = log 1 + 0,5đ?‘Ś log 3 log 2 đ?‘Ľ + log1/3 log1/2 đ?‘Ś = 1 đ?‘Ľđ?‘Ś 2 = 4 đ?‘Ľ 1đ?‘Ś 9 = 92đ?‘Ś 3 đ?‘Ľ + 3đ?‘Ś 2đ?‘Ľ = −4 đ?‘Ľ đ?‘Ś

លរម áž&#x;ážť វណá&#x;’ážŽážœážˇáž…ážˇáž?á&#x;’ážš | V. ážœážˇáž&#x;áž˜áž—ážśáž–

45


V. ážœážˇáž&#x;áž˜áž—ážśáž– 465.

áž”áž?ážžáž˜áž˝áž™ážŽážśáž’á&#x;†áž‡ážśáž„ ážšážœážśáž„ 2300 á&#x;ƒážˇáž„ 3200 ?

466.

áž…ážźážš áž„á&#x;’ហវញáž?ážś log đ?‘› đ?‘› + 1 > log đ?‘›+1 đ?‘› + 2 áž…á&#x;†áž” á&#x;„á&#x;‡áž&#x;á&#x;’ážšáž‚ á&#x;‹ áž…á&#x;†á&#x;ƒá&#x;ƒ áž˝ áž‚áž?á&#x;‹ ធមម ជវáž?ឡ đ?‘› > 1 á&#x;”

467.

áž” áž™áž˜ážˇá&#x;ƒáž”áž&#x;á&#x;’ážš ážžáž?ážśážšážśáž„ áž…ážźážš áž„á&#x;’ហវញáž?ážś log 4 9 > log 9 25 á&#x;”

468.

បគបá&#x;„áž™ đ?‘Ľ1 , đ?‘Ľ2 , ‌ , đ?‘Ľđ?‘› > 0 á&#x;” áž…ážźážš áž„á&#x;’ហវញáž?ážś đ?‘Ľ

đ?‘Ľ

đ?‘Ľ

�1 1 . �2 2 ‌ �� � ≼ �1 . �2 ‌ ��

469.

đ?‘Ľ 1 +đ?‘Ľ 2 +â‹Ż+đ?‘Ľ đ?‘› đ?‘›

(áž˘ážśáž˜á&#x; ážšáž… ឡ áž&#x;áž?áž›áž&#x; ី á&#x;Ąá&#x;Šá&#x;Šá&#x;Š) បគបá&#x;„áž™ đ?‘Ľ, đ?‘Ś, đ?‘§ > 1á&#x;” áž…ážźážš áž„á&#x;’ហវញáž?ážś đ?‘Ľđ?‘Ľ

2 +2��

đ?‘Śđ?‘Ś

2 +2��

��

2 +2đ?‘Ľđ?‘Ś

≼ ���

VI. ážœážˇáž&#x;áž˜áž¸áž€ážśážš ឡ áž˜áž¸áž€ážśážš áž” á&#x;„á&#x;‡áž&#x;á&#x;’ážšážśáž™ážœáž&#x;

470.

log1/5 2đ?‘Ľ 2 + 5đ?‘Ľ + 1 < 0

471.

log1/3 đ?‘Ľ 2 + 2đ?‘Ľ > 0

472.

log1/2 đ?‘Ľ 2 − 4đ?‘Ľ + 6 < −2

473.

log1/3

474.

3đ?‘Ľ − 1 <1 đ?‘Ľ+2 35 − đ?‘Ľ 2 1 log 0,25 â‰Ľâˆ’ đ?‘Ľ 2

475.

log 5 (2đ?‘Ľ − 4) < log 5 (đ?‘Ľ + 3)

476.

log 0,1 đ?‘Ľ 2 + đ?‘Ľ − 2 > log 0,1 (đ?‘Ľ + 3)

477.

log đ?‘Ľ 2 − 3đ?‘Ľ + 4 − log đ?‘Ľ + 1 > 0

478.

log 1 đ?‘Ľ + 1 ≤ log 2 2 − đ?‘Ľ

46

2

á&#x;˘. áž˘áž“ážťáž‚áž˜áž“á&#x;?អឝឡចáž&#x;á&#x;’áž”á&#x;‰ážźážŽá&#x;’áž„á&#x;‹áž&#x;á&#x;’áž&#x;á&#x;‚áž›-áž›á&#x;„កវរីរ | លរម áž&#x;ážť វណá&#x;’ážŽážœážˇáž…ážˇáž?á&#x;’ážš

�� +�� +��


đ?‘Ľ 2 + 6đ?‘Ľ + 9 < − log 2 (đ?‘Ľ + 1) 2(đ?‘Ľ + 1)

479.

log1/2

480.

log(đ?‘Ľ − 2) + log 27 − đ?‘Ľ < 2

481.

log(đ?‘Ľ − 1) + log(đ?‘Ľ − 2) < log(đ?‘Ľ + 2)

482.

log 2 (2 − đ?‘Ľ) + log1/2 (đ?‘Ľ − 1) > log

483.

log1/5 (2đ?‘Ľ + 5) − log1/5 (16 − đ?‘Ľ 2 ) ≤ 1

484.

log 2 1 +

485.

log 7 đ?‘Ľ − log 7 (2đ?‘Ľ − 5) ≤ log 7 2 − log 7 (đ?‘Ľ − 3)

486.

log1/3 (đ?‘Ľ − 1) + log1/3 (đ?‘Ľ + 1) + log 3 (5 − đ?‘Ľ) < 1

487.

log 2 đ?‘Ľ 2 + log 2 đ?‘Ľ − 1

488.

log 2 đ?‘Ľ + 3 log đ?‘Ľ − 4 ≼ 0

489.

1 − log 4 đ?‘Ľ 1 ≤ 1 + log 2 đ?‘Ľ 2

490.

log1/3 đ?‘Ľ > log đ?‘Ľ 3 −

1 đ?‘Ľ + log1/2 1 + đ?‘Ľ 4

2

2

3

≼1

>2

491.

5 2 đ?‘Ľ5 log 2 đ?‘Ľ 4 − log1/2 4

492.

2 log 23 đ?‘Ľ − 3 log 3 đ?‘Ľ − 8 2 log 23 đ?‘Ľ − 3 log 3 đ?‘Ľ − 6 ≼ 3

493.

log 22 đ?‘Ľ + 3 log 2 đ?‘Ľ + 1 log 22 đ?‘Ľ + 3 log 2 đ?‘Ľ − 3 < 5

494.

1,25

1− log 2 đ?‘Ľ 2

1 2

< 0,64

− 20 log 2 đ?‘Ľ + 148 < 0

2+log 2 đ?‘Ľ

2đ?‘Ľ+3

495.

22đ?‘Ľ+1 − 21.

496.

0,1đ?‘Ľ+1 < 0,8 + 2.10đ?‘Ľ

លរម áž&#x;ážť វណá&#x;’ážŽážœážˇáž…ážˇáž?á&#x;’ážš | VI. ážœážˇáž&#x;áž˜áž¸áž€ážśážš

2

+2≼0

47


497.

2đ?‘Ľ + 2−đ?‘Ľ < 3

498.

34−3đ?‘Ľ − 35.33đ?‘Ľâˆ’2 + 6 ≼ 0

499.

2đ?‘Ľ

500.

3log đ?‘Ľ+2 < 3log đ?‘Ľ

501.

6 < 2đ?‘Ľ −1

1 2

2 +5

−2

log đ?‘Ľ 2

+ 2 > 3.2− log (−đ?‘Ľ)

502.

log 2 4đ?‘Ľ − 5.2đ?‘Ľ + 2 > 2

503.

log1/

5

504.

log

5đ?‘Ľ − 1 log

505.

log(1 + 2đ?‘Ľ+1 ) >

2

6đ?‘Ľ+1 − 36đ?‘Ľ ≼ −2

2

2 2 >2 5đ?‘Ľ − 1

đ?‘Ľ log 2 log 4 + log 3 log 8

3 − 2đ?‘Ľ <1 1−đ?‘Ľ

506.

log 2

507.

log 3 9đ?‘Ľ − 3 ≤ log 3 đ?‘Ľ −

508.

9đ?‘Ľ + 3đ?‘Ľ − 2 ≼ 9 − 3đ?‘Ľ

509.

log1/3 log 4 đ?‘Ľ 2 − 5 3đ?‘Ľ+6 log 1 log 2 2 đ?‘Ľ +2 3

510.

0,3

511.

log 4/3

48

1 3

>0

>1

đ?‘Ľ+3−đ?‘Ľ >0

á&#x;˘. áž˘áž“ážťáž‚áž˜áž“á&#x;?អឝឡចáž&#x;á&#x;’áž”á&#x;‰ážźážŽá&#x;’áž„á&#x;‹áž&#x;á&#x;’áž&#x;á&#x;‚áž›-áž›á&#x;„កវរីរ | លរម áž&#x;ážť វណá&#x;’ážŽážœážˇáž…ážˇáž?á&#x;’ážš


512.

log1/2

513.

đ?‘Ľ log 2 đ?‘Ľâˆ’2 >

514.

đ?‘Ľ

515.

5 − đ?‘Ľ − đ?‘Ľ + 1 > −3 đ?‘Ľ 4

log đ?‘Ľ 2 −3 log đ?‘Ľ+1

đ?‘Ľ log 2

đ?‘Ľ

> 1000

≼2

516.

log đ?‘Ž (đ?‘Ľ − 1) + log đ?‘Ž đ?‘Ľ > 2

517.

1 2 + < 1, 0 < đ?‘Ž < 1 5 − log đ?‘Ž đ?‘Ľ 1 + log đ?‘Ž đ?‘Ľ

518.

3 log đ?‘Ž đ?‘Ľ + 6 >1 log 2đ?‘Ž đ?‘Ľ + 2

519.

log đ?‘Ž 1 − 8đ?‘Žâˆ’đ?‘Ľ ≼ 2(1 − đ?‘Ľ)

520.

log x−3 đ?‘Ľ − 1 < 2

521.

log đ?‘Ľ đ?‘Ľ + 2 > 2

522.

log 2đ?‘Ľ (đ?‘Ľ 2 − 5đ?‘Ľ + 6) < 1

523.

log đ?‘Ľ+3 (đ?‘Ľ 2 − đ?‘Ľ) < 1

524.

log 3đ?‘Ľ+5 9đ?‘Ľ 2 + 8đ?‘Ľ + 2 > 2

525.

log 2�+4 � 2 + 1 ≤ 1

526.

log đ?‘Ľ

527.

log đ?‘Ľ 2 (3 − 2đ?‘Ľ) > 1

15 < −2 1 − 2đ?‘Ľ

លរម áž&#x;ážť វណá&#x;’ážŽážœážˇáž…ážˇáž?á&#x;’ážš | VI. ážœážˇáž&#x;áž˜áž¸áž€ážśážš

49


528.

log x 2 +3x (đ?‘Ľ + 3) < 1

529.

log 2 đ?‘Ľâˆ’2 21−đ?‘Ľ ≼ 0

530.

log log

2

3

2

531.

đ?‘Ľ

532.

đ?‘Ľ log 2 6

533. 534. 535.

1 đ?‘Ľ 2

đ?‘Ľ 2 −đ?‘Ľâˆ’2

đ?‘Ľ 2 − 10đ?‘Ľ + 22 > 0 <1

đ?‘Ľ 2 −18đ?‘Ľ+56

>1

log 5 đ?‘Ľ 2 + 3 <0 4đ?‘Ľ 2 − 16đ?‘Ľ log 0,3 đ?‘Ľ − 1 − ≼0 2đ?‘Ľ − đ?‘Ľ 2 + 8 log 0,5 đ?‘Ľ + 2 >0 2đ?‘Ľ − 1

536.

3đ?‘Ľ 2 − 2đ?‘Ľ − 1 <0 log 3 đ?‘Ľ − 1

537.

25.2đ?‘Ľ − 10đ?‘Ľ + 5đ?‘Ľ > 25

538.

log 5 đ?‘Ľ + log đ?‘Ľ

539.

1 1 ≤ �+1 log 4 � + 2 log 4 � + 3

đ?‘Ľ log 5 đ?‘Ľ 2 − log 3 đ?‘Ľ > 3 log 3 đ?‘Ľ

VII. áž?á&#x;’ážšáž–á&#x;?នវធ áž&#x; ឡ áž˜áž¸áž€ážśážš ឡ áž˜áž¸áž€ážśážš áž” á&#x;„á&#x;‡áž&#x;á&#x;’ážšážśáž™áž&#x;á&#x;’ážš áž–á&#x;?á&#x;ƒáž’ážœáž&#x;

50

á&#x;˘. áž˘áž“ážťáž‚áž˜áž“á&#x;?អឝឡចáž&#x;á&#x;’áž”á&#x;‰ážźážŽá&#x;’áž„á&#x;‹áž&#x;á&#x;’áž&#x;á&#x;‚áž›-áž›á&#x;„កវរីរ | លរម áž&#x;ážť វណá&#x;’ážŽážœážˇáž…ážˇáž?á&#x;’ážš


540.

đ?‘Ľâˆ’8 2−đ?‘Ľ log 0,3

10 7 log 2 5 − 1

≼0

2đ?‘Ľâˆ’3 − 31 > 0 541.

log 22 đ?‘Ľ − 3 log 2 đ?‘Ľ + 2 log 5

1 3 log 3 5 − 1

≼0

đ?‘Ľâˆ’ đ?‘Ľâˆ’2≼0 542.

1 8+log đ?‘Ž đ?‘Ľ 1 > 81 3 0<đ?‘Ľ<1

លរម áž&#x;ážť វណá&#x;’ážŽážœážˇáž…ážˇáž?á&#x;’ážš | VII. áž?á&#x;’ážšáž–á&#x;?នវធ áž&#x; ឡ áž˜áž¸áž€ážśážš

log 2đ?‘Ž đ?‘Ľ

51


52

២. អនុគមន៍អុិចស្ប៉ូណ្ង់ស្សែល-លោការីរ | លឹម សុ វណ្ណវិចិត្រ


á&#x;Ł. áž?á&#x;’រី កá&#x;„ážŽáž˜ážśáž?á&#x;’ážš

I. គណá&#x;’áž“ážś ážš áž?ážś sin đ?›ź + cos đ?›ź = đ?‘Ž គណនវ ក) sin đ?›ź − cos đ?›ź ; áž ) sin4 đ?›ź + cos4 đ?›ź 543. ដá&#x;„យដង ážš áž?ážś tan đ?›ź + cot đ?›ź = đ?‘? គណនវ ក) tan2 đ?›ź + cot 2 đ?›ź ; áž ) tan3 đ?›ź + cot 3 đ?›ź 544. ដá&#x;„យដង 545. គណនវ sin

đ?œ‹ 3

−đ?›ź

3 3đ?œ‹ 2

ដបឞ tan đ?›ź = − 4 ,

546. គណនវ cos 70° + đ?›ź 547. គណនវ sin đ?›ź + đ?›˝ − đ?›ž

< đ?›ź < 2đ?œ‹ á&#x;”

ដបឞ sin(40° + đ?›ź) = đ?‘?, 0 < đ?›ź < 45° á&#x;” ដបឞ sin đ?›ź =

12 , cos đ?›˝ 13

=

8 ; sin � 17

4 5

= ; 0 < �, �, � < 3

đ?œ‹ 2

á&#x;”

548. គណនវ sin 3đ?›ź , cos 3đ?›ź , tan 3đ?›ź ដបឞ cot đ?›ź = 4/3, đ?œ‹ < đ?›ź < 2 đ?œ‹ á&#x;” គណនវ 549.

sin � + sin 3� cos � + cos 3�

550.

cos 4đ?›ź − cos 2đ?›ź sin 2đ?›ź + sin 4đ?›ź

551.

sin đ?›ź − 3 sin 2đ?›ź + sin 3đ?›ź cos đ?›ź − 3 cos 2đ?›ź + cos 3đ?›ź

552.

2 sin 2đ?›ź + 2 cos2 đ?›ź − 1 cos đ?›ź − sin đ?›ź − cos 3đ?›ź + sin 3đ?›ź

553.

cos

554.

đ?œ‹ 2đ?œ‹ 6đ?œ‹ cos 0 + cos + cos + â‹Ż + cos 7 7 7

2đ?œ‹ 4đ?œ‹ 6đ?œ‹ + cos + cos 7 7 7

555. ដá&#x;„áž™ sin đ?›ź + cos đ?›ź = 1,4, 0 < đ?›ź <

đ?œ‹ 4

�

á&#x;” គណនវ tan 2 á&#x;”

ឡ áž˜ážś 556. áž˜ážťážťá&#x;† áž&#x;á&#x;’ážšáž˝áž…ážœáž‡á&#x;’ ជ áž“ đ?›ź, đ?›˝, đ?›ž ដផទ á&#x;€áž„áž•á&#x;’áž‘áž‘áž?á&#x;‹ áž‘ážťá&#x;†áž“វកá&#x;‹ áž‘ážťá&#x;†áž“áž„ đ?›˝ 1 đ?›ź đ?›ž 1 đ?›ź đ?›ź tan = cot , cot = 3 tan + cot 2 3 2 2 2 2 2 គណនវ đ?›ź + đ?›˝ + đ?›ž á&#x;”

លរម áž&#x;ážť វណá&#x;’ážŽážœážˇáž…ážˇáž?á&#x;’ážš | I. គណá&#x;’áž“ážś

53


គណនវដá&#x;„áž™áž˜ážˇáž“ážŠá&#x;’បឞáž?ážśážšážśáž„ 557.

cos 292°30′

559.

2 cos 40° − cos 20° sin 20°

560.

−2 2 sin 10° 2 sin 35° −

561.

cos2 73° + cos 2 47° + cos 73° cos 47°

562.

sin 6° − sin 42° − sin 66° + sin 78°

563.

cos2 33° − cos 2 57° sin 21° − cos 21°

565.

tan6 20° − 33 tan4 20° + 27 tan2 20° − 3

566.

cot 2 36° cot 2 72°

567.

558.

sec 5° cos 40° − 2 sin 5°

564.

6 cos 40° − 8 cos 3 40°

គណនវ đ??´ = tan6

568.

cosec 10° − 3 sec 10°

គណនវ

đ?‘ƒ = 1 − 4 cos 2

đ?œ‹ 1999

đ?œ‹ 5đ?œ‹ 7đ?œ‹ + tan6 + tan6 18 18 18

1 − 4 cos2

2đ?œ‹ 1999

1 − 4 cos2

3đ?œ‹ 999đ?œ‹ ‌ 1 − 4 cos2 1999 1999

II. áž&#x;áž˜áž—ážśáž– បងá&#x;’áž ážśáž‰ážšáž˜áž—ážśáž– 569.

sin6 � + cos6 � + 3 sin2 � cos 2 � = 1

570.

sin2 đ?›ź − tan2 đ?›ź = tan6 đ?›ź cos2 đ?›ź − cot 2 đ?›ź

571. ដá&#x;„áž™ đ?›ź, đ?›˝, đ?›ž áž‡ážśáž˜ážťážťá&#x;† áž“áž“á&#x;’áž?ីដá&#x;„ណá&#x;” ចសរបងá&#x;’ហវញáž?ážś sin đ?›ź sin đ?›˝ − cos đ?›ž = cos đ?›ź cos đ?›˝ á&#x;” បងá&#x;’áž ážśáž‰ážšáž˜áž—ážśáž– 572.

cos � + sin �

2

573.

= 1 + sin 2đ?›ź đ?œ‹ đ?›ź 1 − sin đ?›ź = 2 sin2 − 4 2

54

á&#x;Ł. áž?á&#x;’រីកá&#x;„ណá&#x;’áž˜ážśáž?á&#x;’ážš | លរម áž&#x;ážť វណá&#x;’ážŽážœážˇáž…ážˇáž?á&#x;’ážš


574.

1 − 2 sin2 đ?›ź =1 đ?œ‹ đ?œ‹ 2 cot 4 + đ?›ź cos2 4 − đ?›ź

575.

1 − 2 sin2 đ?›˝ 1 − tan đ?›˝ = 1 + sin 2đ?›˝ 1 + tan đ?›˝

576.

tan

577.

cos đ?›ź + sin đ?›ź = tan 2đ?›ź + sec 2đ?›ź cos đ?›ź − sin đ?›ź

578.

sin4 đ?›ź + 2 sin đ?›ź cos đ?›ź − cos4 đ?›ź = cos 2đ?›ź tan 2đ?›ź − 1

579.

sin đ?›ź − đ?›˝ sin đ?›ź + đ?›˝ = − cos 2 đ?›ź sin2 đ?›˝ 1 − tan2 đ?›ź cot 2 đ?›˝

580.

3 − 4 cos 2đ?›ź + cos 4đ?›ź = 8 sin4 đ?›ź

581.

1 1 3 cos4 � = cos 4� + cos 2� + 8 2 8

582.

sin4 � + cos4 � =

583.

4 sin6 � + cos 6 � = 1 + 3 cos2 2�

584.

8 sin8

đ?œ‹ 1 + sin 2đ?›ź +đ?›ź = 4 cos 2đ?›ź

1 1 + cos2 2� 2

� � + cos 8 = 1 + 6 cos2 � + cos4 � 2 2

បងá&#x;’áž ážśáž‰ážšáž˜áž—ážśáž– 2 sin đ?›ź + sin 4đ?›ź 585. = tan 2đ?›ź cos đ?›ź 2 cos đ?›ź + cos 3đ?›ź 586.

cos4 đ?›ź − sin4 đ?›ź + sin 2đ?›ź = 2 cos 2đ?›ź −

លរម áž&#x;ážť វណá&#x;’ážŽážœážˇáž…ážˇáž?á&#x;’ážš | II. áž&#x;áž˜áž—ážśáž–

đ?œ‹ 4 55


đ?œ‹ đ?œ‹ 3 + đ?›ź + cos 2 −đ?›ź = 3 3 2

587.

cos2 � + cos2

588.

sin2 � + cos

589.

log1/3 cos2 đ?›ź + đ?›˝ + cos 2 đ?›ź − đ?›˝ − cos 2đ?›ź cos 2đ?›˝ = 0

590.

cot 2 2đ?›ź − 1 − cos 8đ?›ź cot 4đ?›ź = sin 8đ?›ź 2 cot 2đ?›ź

591.

16 sin 10° sin 30° sin 50° sin 70° = 1

592.

sin2 � cos4 � =

593.

sin 9� + 3 sin 7� + 3 sin 5� + sin 3� = 8 sin 6� cos 3 �

594.

tan đ?›ź − đ?›˝ + tan đ?›˝ − đ?›ž + tan đ?›ž − đ?›ź = tan đ?›ź − đ?›˝ tan đ?›˝ − đ?›ž tan đ?›ž − đ?›ź

595. 596.

đ?œ‹ đ?œ‹ 1 − đ?›ź cos + đ?›ź = 3 3 4

1 1 1 1 + cos 2đ?›ź − cos 4đ?›ź − cos 6đ?›ź 16 32 16 32

đ?›ź đ?œ‹ 1 + sin đ?›ź − 1 − sin đ?›ź = 2 sin ; 0 ≤ đ?›ź ≤ 2 2 ចសរបងá&#x;’ហវញáž?ážś 2 + 2 + â‹Ż + 2 = 2 cos

đ?‘? 2đ?‘› + 1

(កនឝងដនវá&#x;„á&#x;‡áž˜ážśáž“តរ đ?‘› ដង) áž…ážťá&#x;†ážŠ á&#x;„á&#x;‡ đ?‘› ∈ â„• á&#x;” 597.

ឡ áž˜ážś ដគដá&#x;„áž™ áž…ážťá&#x;†áž“áž“ áž˝ áž‚áž?á&#x;‹ វជá&#x;’ ជ áž“ đ?‘› á&#x;” ចសរបងá&#x;’ហវញáž?ážś áž˜ážśáž“áž–áž ážť áž’ážś đ?‘‡đ?‘› áž˜áž˝ áž™ ដដល cos đ?‘›đ?‘Ľ = đ?‘‡đ?‘› cos đ?‘Ľ áž…ážťá&#x;†ážŠ á&#x;„á&#x;‡á&#x;’áž‚áž”á&#x;‹ áž…ážťá&#x;†áž“áž“ áž˝ ពឡáž? đ?‘Ľ á&#x;” áž–áž ážť áž’ážś đ?‘‡đ?‘› ដនá&#x;„á&#x;‡ ដគដá&#x;…áž?ážś áž–áž ážť áž’ážś

598.

ដគដá&#x;„យចឝá&#x;†áž“áž“ áž˝ ពឡáž? đ?‘Ľ ដផទ á&#x;€áž„áž•á&#x;’áž‘áž‘áž?á&#x;‹ 3+2 2 ចសរបងá&#x;’ហវញáž?ážś

599.

2+1

đ?‘Ľ

=

=

2−1

đ?‘Ľ

+3

đ?œ‹ 2 cos 9

ចសរបងá&#x;’ហវញáž?ážś sin2

56

đ?‘Ľ

đ?œ‹ 2đ?œ‹ 3đ?œ‹ 7 + sin2 + sin2 = 7 7 7 4

á&#x;Ł. áž?á&#x;’រីកá&#x;„ណá&#x;’áž˜ážśáž?á&#x;’ážš | លរម áž&#x;ážť វណá&#x;’ážŽážœážˇáž…ážˇáž?á&#x;’ážš

á&#x;”


III. áž&#x;មីá&#x;„ážš ដá&#x;„á&#x;„á&#x;‡á&#x;’ážśáž™ážšáž˜áž¸á&#x;„ážš 600.

cos 1,5đ?œ‹ + đ?‘Ľ = 2 sin đ?‘Ľ + đ?œ‹ cos đ?‘Ľ

601.

2 sin đ?‘Ľ cos đ?‘Ľ + 3 − 2 cos đ?‘Ľ − 3 sin đ?‘Ľ = 0

602.

sin 2đ?‘Ľ = cos đ?‘Ľ − sin đ?‘Ľ

604.

2 tan đ?‘Ľ cos đ?‘Ľ + 1 = 2 cos đ?‘Ľ + tan đ?‘Ľ

605.

603.

tan3 3đ?‘Ľ − 2 sin3 3đ?‘Ľ = 0

sin đ?‘Ľ + cos2 đ?‘Ľ = 1/4

606.

3 cos đ?‘Ľ = 2 sin2 đ?‘Ľ

607.

6 cos2 đ?‘Ľ + 13 sin đ?‘Ľ = 12

608.

3 cos2 đ?‘Ľ − 4 cos đ?‘Ľ − sin2 đ?‘Ľ − 2 = 0

609.

đ?‘Ľ đ?‘Ľ cos4 + sin2 = 1 5 5

610.

sin đ?‘Ľ −

611.

tan2 đ?‘Ľ − 4 tan đ?‘Ľ + 3 = 0

612.

2 tan đ?‘Ľ − 2 cot đ?‘Ľ = 3

613.

3 = 4 tan đ?‘Ľ cos2 đ?‘Ľ

614.

1 = cot đ?‘Ľ + 3 sin2 đ?‘Ľ

615.

2 cos đ?‘Ľ cos đ?‘Ľ − 8 tan đ?‘Ľ = 5

616.

đ?œ‹ đ?œ‹ cos − sin 4 6

2

2 cos đ?‘Ľ − 1 sin2 đ?‘Ľ = sin2 đ?‘Ľ 2 cos đ?‘Ľ − 1

đ?œ‹ sec đ?‘Ľ + tan đ?‘Ľ = sin cos đ?‘Ľ 4

617.

log 2 (3 sin đ?‘Ľ) − log 2 cos đ?‘Ľ − log 2 1 − tan đ?‘Ľ − log 2 (1 + tan đ?‘Ľ) = 1

618. 620.

sin4 2đ?‘Ľ + cos 4 2đ?‘Ľ = sin 2đ?‘Ľ cos 2đ?‘Ľ 3 sin4 đ?‘Ľ + cos 4 đ?‘Ľ − 2 sin 2đ?‘Ľ + sin2 2đ?‘Ľ = 0 4 sin4 đ?‘Ľ + cos 4 đ?‘Ľ + sin 2đ?‘Ľ + đ?‘Ž = 0

621.

sin2 đ?‘Ľ cos2 đ?‘Ľ

622.

tan 5đ?‘Ľ + 2 sin 10đ?‘Ľ = 5 sin 5đ?‘Ľ

623.

cos 2đ?‘Ľ − 3 sin đ?‘Ľ + 2 = 0

624.

cos 10đ?‘Ľ + 12 + 4 2 sin 5đ?‘Ľ + 6 = 4

625.

6 sin2 đ?‘Ľ + 2 sin2 2đ?‘Ľ = 5

619.

1

+

លរម áž&#x;ážť វណá&#x;’ážŽážœážˇáž…ážˇáž?á&#x;’ážš | III. áž&#x;មីá&#x;„ážš

2 −4=0 sin đ?‘Ľ cos đ?‘Ľ

57


đ?‘Ľ =0 2

626.

cos3 đ?‘Ľ + cos2 đ?‘Ľ − 4 cos2

627.

4 cos đ?‘Ľ 2 − 3 sin2 đ?‘Ľ = − 1 + cos 2đ?‘Ľ

628.

tan3 đ?‘Ľ − 1 +

629.

8 cos4 đ?‘Ľ − 8 cos 2 đ?‘Ľ − cos đ?‘Ľ + 1 = 0

630.

sin đ?‘Ľ = 5 cos đ?‘Ľ

631.

632.

sin đ?‘Ľ + cos đ?‘Ľ = 0

633.

634.

cos2 đ?‘Ľ − 4 sin đ?‘Ľ cos đ?‘Ľ = 0

635.

sin đ?‘Ľ + sin 2đ?‘Ľ = cos đ?‘Ľ + 2 cos2 đ?‘Ľ

636.

sin 2đ?‘Ľ − sin2 đ?‘Ľ = 2 sin đ?‘Ľ − 4 cos đ?‘Ľ

637.

tan đ?‘Ľ + sin đ?œ‹ + đ?‘Ľ = 2 sin2

638.

1 đ?œ‹ − 3 cot − đ?‘Ľ = 3 2 cos đ?‘Ľ 2

sin đ?‘Ľ − cos đ?‘Ľ = 0 sin đ?‘Ľ = sin đ?‘Ľ + 2 cos đ?‘Ľ

đ?‘Ľ 2

1 + sin 2đ?‘Ľ cos đ?‘Ľ − sin đ?‘Ľ = cos đ?‘Ľ + sin đ?‘Ľ

639.

3 cos đ?‘Ľ − sin đ?‘Ľ = 1 + cos 2đ?‘Ľ − sin 2đ?‘Ľ

640.

sin2 đ?‘Ľ + 3 sin đ?‘Ľ cos đ?‘Ľ + 2 cos 2 đ?‘Ľ = 0

641.

2 cos2 đ?‘Ľ + 3 sin 2đ?‘Ľ − 8 sin2 đ?‘Ľ = 0

642.

3 sin2 đ?‘Ľ + 5 cos 2 đ?‘Ľ − 2 cos 2đ?‘Ľ − 4 sin 2đ?‘Ľ = 0

643.

2 sin2 đ?‘Ľ − 5 sin đ?‘Ľ cos đ?‘Ľ − 8 cos 2 đ?‘Ľ = −2

644.

1 = 4 sin đ?‘Ľ + 6 cos đ?‘Ľ cos đ?‘Ľ

646.

sin2 đ?‘Ľ 1 + tan đ?‘Ľ = 3 sin đ?‘Ľ cos đ?‘Ľ − sin đ?‘Ľ + 3

647.

sin4 đ?‘Ľ + sin3 đ?‘Ľ cos đ?‘Ľ + sin2 đ?‘Ľ cos2 đ?‘Ľ + sin đ?‘Ľ cos3 đ?‘Ľ + cos4 đ?‘Ľ = 1

645.

648.

3 1 3 cos đ?‘Ľ + sin đ?‘Ľ = 2 2 2

58

á&#x;Ł. áž?á&#x;’រីកá&#x;„ណá&#x;’áž˜ážśáž?á&#x;’ážš | លរម áž&#x;ážť វណá&#x;’ážŽážœážˇáž…ážˇáž?á&#x;’ážš

649.

sin3 đ?‘Ľ + 4 cos 3 đ?‘Ľ = 0

3 sin đ?‘Ľ + cos đ?‘Ľ = 2


650.

sin 5đ?‘Ľ = 3 1 + cos 5đ?‘Ľ

651.

cos đ?‘Ľ + sin đ?‘Ľ = 1

652.

đ?‘Ľ sin đ?‘Ľ + cos đ?‘Ľ cot = − 3 2

653.

sin đ?‘Ľ tan 5đ?‘Ľ = cos đ?‘Ľ

654.

sin 2đ?‘Ľ + 3 cos 2đ?‘Ľ

2

− 5 = cos

đ?œ‹ − 2đ?‘Ľ 6

655.

cos 6đ?‘Ľ + tan2 đ?‘Ľ + cos 6đ?‘Ľ tan2 đ?‘Ľ = 1

656.

tan 2đ?‘Ľ + tan đ?‘Ľ = −1 1 − tan 2đ?‘Ľ tan đ?‘Ľ

658.

2 tan 3đ?‘Ľ − 3 tan 2đ?‘Ľ = tan2 2đ?‘Ľ tan 3đ?‘Ľ

659.

cot đ?‘Ľ + cot 15° + cot đ?‘Ľ + 25° = cot 15° cot đ?‘Ľ cot(đ?‘Ľ + 25°)

660.

đ?‘Ľ sin đ?‘Ľ + tan = 0 2

661.

1 + cos đ?‘Ľ + tan

662.

tan 2đ?‘Ľ + cot đ?‘Ľ = 4 sin 2đ?‘Ľ

663.

đ?‘Ľ 53 đ?‘Ľ 15 cot + 130 sin đ?‘Ľ = tan 2 3 2

664.

59 đ?‘Ľ đ?‘Ľ cos đ?‘Ľ + 6 sin đ?‘Ľ tan = 4 tan đ?‘Ľ cot 4 2 2

665.

2 sin2 đ?‘Ľ −

666.

cos 3đ?‘Ľ = −2 cos đ?‘Ľ

667.

cos 9đ?‘Ľ − 2 cos 6đ?‘Ľ = 2

668.

cos 4đ?‘Ľ = cos 2 3đ?‘Ľ

669.

670.

sin 6đ?‘Ľ + 2 = 2 cos 4đ?‘Ľ

671.

đ?‘Ľ 3 sin = sin đ?‘Ľ 3 3 đ?‘Ľ sin đ?‘Ľ + 3 sin đ?‘Ľ = 3 sin 2 2

672.

sin

673.

3 cos đ?‘Ľ + 3 sin đ?‘Ľ + sin 3đ?‘Ľ − cos 3đ?‘Ľ = 0

674.

đ?‘Ž cos đ?‘Ľ + đ?‘? sin đ?‘Ľ = đ?‘?, đ?‘Ž2 + đ?‘? 2 ≠0

675.

sin đ?‘Ľ + sin đ?‘Ľ +

657.

tan 3đ?‘Ľ − tan 2đ?‘Ľ =1 1 + tan 3đ?‘Ľ tan 2đ?‘Ľ

đ?‘Ľ =0 2

đ?œ‹ = 2 sin2 đ?‘Ľ − tan đ?‘Ľ 4

đ?œ‹ 3 3đ?œ‹ đ?‘Ľ + đ?‘Ľ = 2 sin + 4 2 4 2

លរម áž&#x;ážť វណá&#x;’ážŽážœážˇáž…ážˇáž?á&#x;’ážš | III. áž&#x;មីá&#x;„ážš

đ?œ‹ =0 4

676.

sin 4đ?‘Ľ − sin 2đ?‘Ľ = 0 59


đ?‘Ľ 2

677.

sin2 đ?‘Ľ − cos 2 đ?‘Ľ = cos

679.

cos 3đ?‘Ľ − 4đ?œ‹ = sin(đ?œ‹ − đ?‘Ľ)

681.

sin đ?‘Ľ −

678.

cos 2đ?‘Ľ − cos 6đ?‘Ľ = 0

680.

sin đ?œ‹đ?‘Ľ 2 = sin đ?œ‹(đ?‘Ľ 2 + 2đ?‘Ľ)

682.

đ?œ‹ 2đ?œ‹ đ?œ‹ − sin đ?‘Ľ + = cos đ?‘Ľ + 6 3 4 1 + sin 2đ?‘Ľ = sin 3đ?‘Ľ 683. cos 5đ?‘Ľ + cos 7đ?‘Ľ = cos đ?œ‹ + 6đ?‘Ľ − cos 3đ?‘Ľ 2

684.

cos đ?‘Ľ − cos 3đ?‘Ľ = 2 3 sin2 đ?‘Ľ

685.

sin đ?‘Ľ + sin 3đ?‘Ľ + 4 cos 3 đ?‘Ľ = 0

686.

cos đ?‘Ľ − cos 2đ?‘Ľ = sin 3đ?‘Ľ

687.

sin đ?‘Ľ + 2 sin 2đ?‘Ľ = − sin 3đ?‘Ľ

688.

sin đ?‘Ľ + sin 2đ?‘Ľ = −1 sin 3đ?‘Ľ

689.

690.

cot đ?‘Ľ sin 2đ?‘Ľ − cos 2đ?‘Ľ = 1

691.

tan 2đ?‘Ľ cos 3đ?‘Ľ + sin 3đ?‘Ľ + 2 sin 5đ?‘Ľ = 0

692.

2 sin đ?‘Ľ +

693.

sin đ?‘Ľ + sin 2đ?‘Ľ + sin 3đ?‘Ľ + sin 4đ?‘Ľ = 0

694.

cos 9đ?‘Ľ − cos 7đ?‘Ľ + cos 3đ?‘Ľ − cos đ?‘Ľ = 0

695.

cos 5đ?‘Ľ − sin 5đ?‘Ľ = sin 7đ?‘Ľ − cos 7đ?‘Ľ

696.

sin 7đ?‘Ľ + cos2 2đ?‘Ľ = sin2 2đ?‘Ľ + sin đ?‘Ľ

697.

cos 2đ?‘Ľ − sin 3đ?‘Ľ − cos 8đ?‘Ľ = sin 10đ?‘Ľ − cos 5đ?‘Ľ

698.

sin đ?‘Ľ + sin 2đ?‘Ľ + sin 3đ?‘Ľ = 1 + cos đ?‘Ľ + cos 2đ?‘Ľ

699.

5 sin đ?‘Ľ + 6 sin 2đ?‘Ľ + 5 sin 3đ?‘Ľ + sin 4đ?‘Ľ = 0

700.

cosec đ?‘Ľ − cosec 2đ?‘Ľ = cosec 4đ?‘Ľ

60

2 sin 10đ?‘Ľ + sin 2đ?‘Ľ = cos 2đ?‘Ľ

đ?œ‹ đ?‘Ľ đ?œ‹ đ?‘Ľ đ?œ‹ đ?‘Ľ đ?œ‹ + 2 cos + = 3 sin + + 3 cos + 3 2 4 4 8 4 8

á&#x;Ł. áž?á&#x;’រីកá&#x;„ណá&#x;’áž˜ážśáž?á&#x;’ážš | លរម áž&#x;ážť វណá&#x;’ážŽážœážˇáž…ážˇáž?á&#x;’ážš


701.

tan 3đ?‘Ľ − tan đ?‘Ľ = 0

702.

tan đ?‘Ľ + tan 2đ?‘Ľ − tan 3đ?‘Ľ = 0

703.

cos 3đ?‘Ľ cos 6đ?‘Ľ = cos 4đ?‘Ľ cos 7đ?‘Ľ

704.

sin 2đ?‘Ľ sin 6đ?‘Ľ = cos đ?‘Ľ cos 3đ?‘Ľ

705.

cos 3đ?‘Ľ sin 7đ?‘Ľ = cos 2đ?‘Ľ sin 8đ?‘Ľ

706.

sin 5đ?‘Ľ cos 3đ?‘Ľ = sin 6đ?‘Ľ sin 2đ?‘Ľ

707.

sin đ?œ‹đ?‘Ľ +

708.

sin2 đ?‘Ľ + sin2 2đ?‘Ľ = sin2 3đ?‘Ľ

709.

cos2 đ?‘Ľ + cos2 2đ?‘Ľ cos 2 3đ?‘Ľ + cos2 4đ?‘Ľ = 2

710.

sin 7đ?‘Ľ + sin 9đ?‘Ľ = 2 cos2

711.

sin2 2đ?‘Ľ + sin2 đ?‘Ľ =

712.

1 đ?‘Ľ sin đ?‘Ľ + sin 2đ?‘Ľ + sin 3đ?‘Ľ = cot 2 2

713.

cos đ?‘Ľ + cos 2đ?‘Ľ + cos 3đ?‘Ľ = −0,5

714.

sin3 đ?‘Ľ cos 3đ?‘Ľ + cos3 đ?‘Ľ sin 3đ?‘Ľ + 0,375 = 0

715.

cos3 đ?‘Ľ cos 3đ?‘Ľ + sin3 đ?‘Ľ sin 3đ?‘Ľ =

đ?œ‹ đ?œ‹ 1 sin đ?œ‹đ?‘Ľ − = 4 12 2

đ?œ‹ đ?œ‹ − đ?‘Ľ − cos 2 + 2đ?‘Ľ 4 4

9 16

718.

2 4 đ?œ‹ đ?œ‹ 1 sin đ?‘Ľ sin − đ?‘Ľ sin + đ?‘Ľ = 3 3 8 đ?œ‹ đ?œ‹ 8 cos đ?‘Ľ cos − đ?‘Ľ cos + đ?‘Ľ + 1 = 0 3 3 đ?œ‹ 2đ?œ‹ tan đ?‘Ľ tan đ?‘Ľ + tan đ?‘Ľ + = 3 3 3

719.

sin 3đ?‘Ľ = 4 sin đ?‘Ľ cos 2đ?‘Ľ

720.

sin 3đ?‘Ľ cos đ?‘Ľ = 1,5 tan đ?‘Ľ

721.

tan đ?‘Ľ cot 3đ?‘Ľ = 4

722.

6 tan đ?‘Ľ +

723.

sin đ?‘Ľ cos đ?‘Ľ sin 3đ?‘Ľ − cos 3đ?‘Ľ sin2 đ?‘Ľ = 6 cot đ?‘Ľ

716. 717.

លរម áž&#x;ážť វណá&#x;’ážŽážœážˇáž…ážˇáž?á&#x;’ážš | III. áž&#x;មីá&#x;„ážš

5 = tan 2đ?‘Ľ tan 3đ?‘Ľ

61


724.

2 sin 3đ?‘Ľ sin đ?‘Ľ + 3 2 − 1 cos 2đ?‘Ľ = 3

725.

2 cos 4đ?‘Ľ + 5 cos 2đ?‘Ľ − 1 = 2 sin2 đ?‘Ľ

726.

2 + cos 4đ?‘Ľ = 5 cos 2đ?‘Ľ + 8 sin6 đ?‘Ľ

727.

tan2 đ?‘Ľ + cos 4đ?‘Ľ = 0

728.

tan đ?‘Ľ + cot đ?‘Ľ − cos 4đ?‘Ľ = 3

729.

sin 2đ?‘Ľ − 12 sin đ?‘Ľ − cos đ?‘Ľ + 12 = 0

730.

1 1 5đ?œ‹ + = 4 sin đ?‘Ľ + 3đ?œ‹ sin đ?‘Ľ sin đ?‘Ľ − 4 2

731.

1 + tan đ?‘Ľ = 2 2 sin đ?‘Ľ

732.

sin đ?‘Ľ +

733.

sin đ?‘Ľ + sin2 đ?‘Ľ + cos3 đ?‘Ľ = 0

734.

sin

735.

sin2 đ?‘Ľ + 2 tan2 đ?‘Ľ +

736.

8 cos đ?‘Ľ + 6 sin đ?‘Ľ − cos 2đ?‘Ľ − 7 = 0

737.

sin4 đ?‘Ľ + sin4

738. 739.

62

đ?œ‹ 1 = 1 − sin đ?‘Ľ cos đ?‘Ľ 4 2

đ?‘Ľ đ?‘Ľ + cos = 2 sin đ?‘Ľ 2 2 4 3

tan đ?‘Ľ − sin đ?‘Ľ +

11 =0 12

đ?‘Ľ đ?œ‹ + + cos 4 đ?‘Ľ = 0,5 sin2 2đ?‘Ľ 2 8

đ?‘Ľ đ?‘Ľ cos − 2 sin đ?‘Ľ sin đ?‘Ľ + 1 + sin − 2 cos đ?‘Ľ cos đ?‘Ľ = 0 4 4 3sin 3đ?‘Ľ = cos 4đ?‘Ľ − sin 9đ?‘Ľ − cos 10đ?‘Ľ

á&#x;Ł. áž?á&#x;’រីកá&#x;„ណá&#x;’áž˜ážśáž?á&#x;’ážš | លរម áž&#x;ážť វណá&#x;’ážŽážœážˇáž…ážˇáž?á&#x;’ážš


740.

1 tan đ?‘Ľ + cot đ?‘Ľ = 9

1 −1−1 cos 2 đ?‘Ľ

741.

sin đ?‘Ľ + 3 cos đ?‘Ľ =

742.

2 cos 2đ?‘Ľ + 2 =

743.

sin đ?‘Ľ + 2 cos đ?‘Ľ = 0

2 + cos 2đ?‘Ľ + 3 sin 2đ?‘Ľ 3 1 + 4 cos 2đ?‘Ľ

4

744.

sin đ?‘Ľ + cos đ?‘Ľ = 0

745.

2 cos đ?‘Ľ = 2 + 2 sin 2đ?‘Ľ

746. 747.

cos2 đ?‘Ľ − cos 2 3đ?‘Ľ = sin 2đ?‘Ľ sin đ?‘Ľ + 3 cos đ?‘Ľ =

0,5 + cos

đ?œ‹ −đ?‘Ľ 6

748.

1 + 4 sin đ?‘Ľ cos đ?‘Ľ = cos đ?‘Ľ − sin đ?‘Ľ

749.

cos 2đ?‘Ľ − sin 4đ?‘Ľ = sin đ?‘Ľ − cos đ?‘Ľ

750.

1 + tan đ?‘Ľ = sin đ?‘Ľ + cos đ?‘Ľ 1 − tan đ?‘Ľ

751.

4 sin 3đ?‘Ľ + 3 = 2 sin 3đ?‘Ľ + 2

752.

13 − 18 tan đ?‘Ľ = 6 tan đ?‘Ľ − 3

753.

1 + 8 sin 2đ?‘Ľ cos2 2đ?‘Ľ = 2 sin 3đ?‘Ľ + đ?œ‹/4

754. 755.

2 3 sin đ?‘Ľ =

3 tan đ?‘Ľ 2 sin đ?‘Ľ − 1

− 3

3 cos đ?‘Ľ + sin đ?‘Ľ − 2 + cot 3đ?‘Ľ + sin2 đ?‘Ľ −

លរម áž&#x;ážť វណá&#x;’ážŽážœážˇáž…ážˇáž?á&#x;’ážš | III. áž&#x;មីá&#x;„ážš

1 3đ?‘Ľ 1 = sin + 4 2 2

63


756.

log 5 tan đ?‘Ľ = log 5 4 log 4 3 sin đ?‘Ľ

757.

log 9 sin 2đ?‘Ľ = log 3

758.

2cos 2đ?‘Ľ = 3.2cos

759.

cot 2đ?‘Ľ = tan 2đ?‘Ľ + 2 tan 2đ?‘Ľ+1

760.

đ?‘Ľ 3 sin 2đ?‘Ľ+2 = đ?‘Ľ

2đ?‘Ľ

sin đ?‘Ľ 5 −4

761. ដáž?ឞរមីá&#x;„ážš 1 + cos 2đ?‘Ľ + sin 2đ?‘Ľ = 0 áž“ážˇáž„ážšáž˜áž¸á&#x;„ážš 1+ ដសចគá&#x;’ននដដររ ឺដទ?

1 − tan2 đ?‘Ľ 2 tan đ?‘Ľ + =0 2 1 + tan đ?‘Ľ 1 + tan2 đ?‘Ľ

ážž បីដá&#x;„យរមីá&#x;„ážš 762. ចសរកឝá&#x;†áž“áž?á&#x;‹ áž?áž“ážťá&#x;† áž› đ?‘? ដដម

đ?‘? cos đ?‘Ľ − 2 sin đ?‘Ľ = 2 + 2 − đ?‘? áž˜ážśáž“áž…ážťá&#x;†ážŠáž›ážž áž™á&#x;”

ážž បីដá&#x;„យរមីá&#x;„រហវងដá&#x;’á&#x;„áž˜áž–ážˇáž?áž…ážťá&#x;†ážŠ á&#x;„á&#x;‡á&#x;’áž‚áž”á&#x;‹ đ?‘Ľ 763. ចសរកឝá&#x;†áž“áž?á&#x;‹ áž?áž“ážťá&#x;† áž› đ?‘Ž, đ?‘? ដដម

đ?‘Ž cos đ?‘Ľ − 1 + đ?‘? 2 = cos đ?‘Žđ?‘Ľ + đ?‘? 2 − 1

IV. áž?á&#x;’ážšáž–á&#x;?áž“á&#x;’áž&#x; áž’ មីá&#x;„ážš ដá&#x;„á&#x;„á&#x;‡á&#x;’ážśáž™á&#x;’បពá&#x;?áž“ážš áž’ មីá&#x;„ážš đ?‘Ľ − đ?‘Ś = 6,5đ?œ‹ 764. 3 cos2 đ?‘Ľ − 12 cos đ?‘Ś = −4 2 đ?‘Ľ+đ?‘Ś = đ?œ‹ 3 765. sin đ?‘Ľ =2 sin đ?‘Ś đ?‘Ľ + đ?‘Ś = đ?œ‹/4 766. cos đ?‘Ľ + cos đ?‘Ś = đ?‘Ž sin 3đ?‘Ľ cos 2đ?‘Ś = 2đ?‘Ž − cos 3đ?‘Ľ sin 2đ?‘Ś 767. cos đ?‘Ľ − đ?‘Ś = 0,5 3 sin đ?‘Ľ sin đ?‘Ś = 4 768. 3 cos đ?‘Ľ cos đ?‘Ś = 4 64

á&#x;Ł. áž?á&#x;’រីកá&#x;„ណá&#x;’áž˜ážśáž?á&#x;’ážš | លរម áž&#x;ážť វណá&#x;’ážŽážœážˇáž…ážˇáž?á&#x;’ážš


769. 770. 771. 772. 773. 774. 775. 776.

tan đ?‘Ľ + tan đ?‘Ś = 2 cos đ?‘Ľ cos đ?‘Ś = 0,5 1 sin đ?‘Ľ cos đ?‘Ś = 4 3 tan đ?‘Ľ = tan đ?‘Ś tan đ?‘Ľ − 2 sin đ?‘Ś = −2 5 tan đ?‘Ľ + 2 sin đ?‘Ś = −4 4 sin đ?‘Ś − 6 2 cos đ?‘Ľ = 5 + 4 cos2 đ?‘Ś cos 2đ?‘Ľ = 0 1 đ?‘Ž+3 − cos đ?‘Ś = sin đ?‘Ľ 3 đ?‘Ž sin đ?‘Ľ cos đ?‘Ś = − 3 1 − tan đ?‘Ś = 2đ?‘Ž + 2 cos đ?‘Ľ tan đ?‘Ś + đ?‘Ž2 + 2đ?‘Ž cos đ?‘Ľ = 0 sin2 −2đ?‘Ľ − 3 − 2 tan 5đ?‘Ś = 3 2 − 1 /2 3 − 2 sin −2đ?‘Ľ + tan2 5đ?‘Ś = 3 2 − 1 /2 ដá&#x;„á&#x;„á&#x;‡á&#x;’ážśáž™á&#x;’បពá&#x;?áž“ážš áž’ មីá&#x;„ážš tan2 đ?‘Ľ + tan2 đ?‘Ś + tan2 đ?‘§ = đ?‘š2 tan3 đ?‘Ľ + tan3 đ?‘Ś + tan3 đ?‘§ = đ?‘š3

V. ážœážˇáž&#x;áž˜áž—ážśáž– 777. 778.

đ?œ‹ đ?œ‹

đ?œ‹

ចសរបងá&#x;’ហវញáž?ážś áž…ážťá&#x;†ážŠ á&#x;„á&#x;‡á&#x;’áž‚áž”á&#x;‹ 0 ≤ đ?‘Ľ ≤ 2 ; 6 < đ?›ź < 3 ážŠáž‚áž˜ážśáž“ đ?œ‹ sin đ?‘Ľ đ?œ‹ cos đ?‘Ľ tan + tan >1 4 sin đ?›ź 4 cos đ?›ź ដគដá&#x;„áž™á&#x;’áž?ីដá&#x;„ážŽáž˜ážˇáž“ážŠáž€áž„ đ??´đ??ľđ??ś á&#x;” ចសរបងá&#x;’ហវញáž?ážś

3 tan2 đ??´ tan2 đ??ľ tan2 đ??ś − 5 tan2 đ??´ + tan2 đ??ľ + tan2 đ??ś ≤ 9 + tan2 đ??ľ tan2 đ??ś + tan đ??ś tan2 đ??´ 779. ចសរបងá&#x;’ហវញáž?ážś đ?œ‹ 1 − sin 14 đ?œ‹ đ?œ‹ > 3 cos 7 2 sin 14 2

780.

ដគដá&#x;„áž™ 2đ?‘› áž…ážťá&#x;†áž“áž“ áž˝ ពឡáž? đ?‘Ľ1 ; đ?‘Ľ2 ; ‌ ; đ?‘Ľ2đ?‘› ∈

លរម áž&#x;ážť វណá&#x;’ážŽážœážˇáž…ážˇáž?á&#x;’ážš | V. ážœážˇáž&#x;áž˜áž—ážśáž–

đ?œ‹ đ?œ‹ , 6 2

ចសរកឝá&#x;†áž“áž?á&#x;‹ áž?áž“ážťá&#x;† áž›áž?សចបឝá&#x;† áž•ážťáž?áž“áž“ đ?‘› ∈ ℕ∗ ដដល

65


2đ?‘›

đ?‘Œ= 781.

2đ?‘›

sin đ?‘Ľđ?‘– đ?‘–=1

គណនវáž?ážťá&#x;†áž“លធឝá&#x;†áž”ážťá&#x;†áž•ážťáž?áž“áž“

đ?‘–=1

1 = 1800 sin đ?‘Ľđ?‘–

đ??´ đ??ľ đ??ś đ?‘‡ = tan tan tan 4 4 4 ដដល đ??´, đ??ľ, đ??ś áž‡ážśáž˜ážťážťá&#x;† បីននá&#x;’áž?ីដá&#x;„ážŽáž˜áž˝ áž™á&#x;” 782. 783. 784. 785.

áž?ážśáž„ đ?‘› ជវចឝá&#x;†áž“áž“ áž˝ áž‚áž?á&#x;‹ ធមម ជវáž?ឡ នឡងចឝá&#x;†áž“áž“ áž˝ ពឡáž? đ?‘Ľ ដដល 0 < đ?‘› + 1 đ?‘Ľ < đ?œ‹/2 ចសរបងá&#x;’ហវញáž?ážś 1 − cos đ?‘› đ?‘Ľ 1 + cos đ?‘› đ?‘Ľ < tan đ?‘›đ?‘Ľ sin đ?‘Ľ ដគដá&#x;„áž™ đ??´, đ??ľ, đ??ś áž‡ážśáž˜ážťážťá&#x;† កនឝងននá&#x;’áž?ីដá&#x;„ážŽáž˜áž˝ áž™á&#x;” ចសរបងá&#x;’ហវញáž?ážś đ??´ đ??ľ đ??ľ đ??ś đ??ś đ??´ đ??´ đ??ľ đ??ś 27 tan tan2 + tan tan2 + tan tan2 < 4 cot + cot + cot 2 2 2 2 2 2 2 2 2 áž?ážśáž„ đ??´, đ??ľ, đ??ś áž‡ážśáž˜ážťážťá&#x;† áž“áž“á&#x;’áž?ីដá&#x;„ណ đ??´đ??ľđ??ś á&#x;” គណនវáž?ážťá&#x;†áž“លធឝá&#x;†áž”ážťá&#x;†áž•ážťáž?áž“áž“ sin đ??ś ដបឞ sin2 đ??´ + sin2 đ??ľ 1 = đ?‘š, đ?‘š> 2 sin đ??ś 2 ដគដá&#x;„áž™ áž˘áž“ážťáž‚áž˜áž“á&#x;? đ?‘“(đ?‘Ľ) ដដល đ?‘“ tan 2đ?‘Ľ = tan4 đ?‘Ľ + cot 4 đ?‘Ľ á&#x;” ចសរបងá&#x;’ហវញáž?ážś đ?‘“ sin đ?‘Ľ +

đ?‘“ cos đ?‘Ľ ≼ 196 á&#x;” 786.

ដគដá&#x;„áž™á&#x;’áž?ីដá&#x;„ណ đ??´đ??ľđ??ś áž˜ážśáž“ đ??´ > đ??ľ > đ??ś á&#x;” គណនវáž?ážťá&#x;†áž“áž›áž?សចបឝá&#x;† áž•ážťáž?áž“áž“ đ?‘Ś=

đ?‘Ľ − sin đ??´ đ?‘Ľ − sin đ??ľ + −1 đ?‘Ľ − sin đ??ś đ?‘Ľ − sin đ??ś

VI. ážœážˇáž&#x;មីá&#x;„ážš ឡ មីá&#x;„ážš ដá&#x;„á&#x;„á&#x;‡á&#x;’ážśáž™ážœážš 787.

2 sin2 đ?‘Ľ + 3 sin đ?‘Ľ − 3 > 0

788.

cos 2� + 5 cos � + 3 ≼ 0

789.

tan2 đ?‘Ľ + 2 − 3 tan đ?‘Ľ − 2 3 < 0

790.

cot 2 � + cot � ≼ 0

791.

2

792.

2 − 1 sin đ?‘Ľ − 2 cos 2đ?‘Ľ + 2 − 2 < 0 đ?œ‹ cos đ?œ‹đ?‘Ľ + sin đ?œ‹đ?‘Ľ − >0 4

66

á&#x;Ł. áž?á&#x;’រីកá&#x;„ណá&#x;’áž˜ážśáž?á&#x;’ážš | លរម áž&#x;ážť វណá&#x;’ážŽážœážˇáž…ážˇáž?á&#x;’ážš


3 8

793.

cos3 đ?‘Ľ sin 3đ?‘Ľ + cos 3đ?‘Ľ sin3 đ?‘Ľ <

794.

cos � cos 2� cos 3� ≤ 0

795. 796.

đ?‘Ľ đ?‘Ľ 1 sin4 + cos 4 > 3 3 2 5 sin6 đ?‘Ľ + cos 6 đ?‘Ľ > 8

797.

8sin6 đ?‘Ľ − cos 6 đ?‘Ľ > 0

798.

tan đ?‘Ľ tan 3đ?‘Ľ < −1

799.

3 sin 2đ?‘Ľ − 1 > sin đ?‘Ľ + cos đ?‘Ľ

800.

sin 2đ?‘Ľ > 2 sin2 đ?‘Ľ + 2 − 2 cos 2 đ?‘Ľ , 0 < đ?‘Ľ < 2đ?œ‹

801. 802.

sin đ?‘Ľ > cos 2 đ?‘Ľ 5 − 2 sin đ?‘Ľ ≼ 6 sin đ?‘Ľ − 1

803.

1 − cos đ?‘Ľ < tan đ?‘Ľ − sin đ?‘Ľ

804.

91+sin

2 đ?œ‹đ?‘Ľ

+ 30.9cos

2 đ?œ‹đ?‘Ľ

≤ 117

VII. áž?á&#x;’ážšáž–á&#x;?áž“á&#x;’វធ áž&#x; ឡ មីá&#x;„ážš ážž បីដá&#x;„áž™á&#x;’បពá&#x;?áž“áž’áž˜ážśáž“ážš ឺរដáž?áž˜áž˝ យគáž?á&#x;‹ 805. ចសរកឝá&#x;†áž“áž?á&#x;‹ đ?‘š ដដម 4 − 6đ?‘š sin3 đ?‘Ľ + 3 2đ?‘š − 1 sin đ?‘Ľ + 2 đ?‘š − 2 sin2 đ?‘Ľ . cos đ?‘Ľ − 4đ?‘š − 3 cos đ?‘Ľ = 0 0 ≤ đ?‘Ľ ≤ đ?œ‹/4

លរម áž&#x;ážť វណá&#x;’ážŽážœážˇáž…ážˇáž?á&#x;’ážš | VII. áž?á&#x;’ážšáž–á&#x;?áž“á&#x;’វធ áž&#x; ឡ មីá&#x;„ážš

67


68

៣. ត្រីកោណ្មាត្រ | លឹម សុ វណ្ណវិចិត្រ


៤. ពហុ ធា

គណ្នា 806. កំនត់ ពហុ ធា 𝑃 𝑥 ដែលផផទ ៀងផ្ទទត់ លកខខណ្ឌ 𝑃 𝑢2 − 𝑣 2 = 𝑃 𝑢 + 𝑣 . 𝑃(𝑢 − 𝑣) ចំផ ោះគ្គប់ 𝑢, 𝑣 ∈ ℝ ។

807. កំនត់ ពហុ ធា 𝑃 𝑥 ដែលផផទ ៀងផ្ទទត់ លកខខណ្ឌ 𝑃 𝑥 + 1 = 𝑃 𝑥 + 2𝑥 + 1 ចំផ ោះគ្គប់ 𝑥 ∈ ℝ ។

808. កំនត់ ពហុ ធា 𝑃 𝑥 ដែលផផទ ៀងផ្ទទត់ លកខខណ្ឌ 𝑃 𝑥+1

2

= 𝑃 𝑥 2 + 2𝑥 + 1

ចំផ ោះគ្គប់ 𝑥 ∈ ℝ ។

809. (កាណាដា ១៩៧០) ផគផោយពហុ ធា 𝑓 𝑥 = 𝑥 𝑛 + 𝑎1 𝑥 𝑛−1 + 𝑎2 𝑥 𝑛−2 + ⋯ + 𝑎𝑛−1 𝑥 + 𝑎𝑛 ឹ ថាមានចំនន ដែលមានផេគុ ណ្ 𝑎1 , 𝑎2 , … , 𝑎𝑛 ជាចំនន ួ គត់ ផហើ យផោយែង ួ គត់ បួនផផេងគ្នា 𝑎, 𝑏, 𝑐 និង 𝑑 ដែល 𝑓 𝑎 =𝑓 𝑏 =𝑓 𝑐 =𝑓 𝑑 =5 ចូរបង្ហាញថា គ្នានចំនន ួ គត់ 𝑘 េួ យដែល 𝑓 𝑘 = 8 ។

810.

ិ មា ផគផោយពហុ ធាែផឺ គ្កទី 𝑛 ≥ 1 មានផេគុ ណ្េិនអវជ្ ជ ន និង 𝑥1 , 𝑥2 , … , 𝑥𝑛 ∈ ℝ+∗ ។ ចូរបង្ហាញថា 𝑥2 𝑃 𝑥1

2

+ 𝑃

លឹម សុ វណ្ណវិចិត្រ | I. គណ្នា

𝑥3 𝑥2

2

+ ⋯+ 𝑃

𝑥𝑛 𝑥𝑛−1

2

+ 𝑃

𝑥1 𝑥𝑛

2

≥𝑛 𝑃 1

2

69


70

៤. ពហុ ធា | លឹម សុ វណ្ណវិចិត្រ


៥. សមី ការអនុគមន៍

. គណ្នា 811. ចូរកំនត់ គ្គប់ អនុគមន៍ 𝑓: ℝ → ℝ ដែលផផទ ៀងផ្ទទត់ លកខខណ្ឌ 𝑓 𝑓 𝑥 +1 =1−𝑥 𝑓 𝑓 𝑥 =𝑥 ចំផ ោះគ្គប់ 𝑥 ∈ ℝ ។

812. ផគផោយអនុគមន៍𝑓: ℕ∗ → ℝ ដែលផផទ ៀងផ្ទទត់ a) 𝑓 1 = 2 b) ចំផ ោះគ្គប់ 𝑛 > 1 𝑓 1 + 𝑓 2 + ⋯ + 𝑓 𝑛 = 𝑛2 𝑓 𝑛 ចូរកំនត់ 𝑓 𝑛 ។

ិ មា 813. ផគផោយចំនន ួ គត់ វជ្ ជ ន 𝑛 ។ ចូរកំនត់ អនុគមន៍ 𝑓: ℝ → ℝ ដែលផផទ ៀងផ្ទទត់ 𝑓 𝑥𝑓 𝑦

= 𝑥𝑛 𝑓 𝑓 𝑦

ចំផ ោះគ្គប់ 𝑥, 𝑦 ∈ ℝ ។

814. ផគផោយផេរចំនន ួ ពិតបី 𝑎, 𝑏, 𝑐 ដែលមិនសូនយទំងបីគ្ពមគ្នា។ ចូរកំនត់ អនុគមន៍ 𝑓: ℝ → ℝ ដែល 𝑎𝑓 𝑥 2 + 𝑦𝑧 + 𝑏𝑓 𝑦 2 + 𝑧𝑥 + 𝑐𝑓 𝑧 2 + 𝑥𝑦 = 0 ចំផ ោះគ្គប់ 𝑥, 𝑦, 𝑧 ∈ ℝ ។

ិ មា ើ បីផោយផគោចរកបាននូវអនុគមន៍ 𝑓: ℝ+ → ℝ+ 815. ចូរកំនត់ លកខខណ្ឌផលើ ចំនន ួ វជ្ ជ ន 𝑝, 𝑞 ផែម ដែល 𝑓 𝑥𝑓 𝑦

= 𝑥𝑝 𝑦𝑞

ចំផ ោះគ្គប់ 𝑥, 𝑦 ∈ ℝ+ ។

816. ចូរកំនត់ គ្គប់ អនុគមន៍ 𝑓: ℤ → ℤ ដែល 𝑓 𝑚 + 19 ≥ 𝑓 𝑚 + 19 𝑓 𝑚 + 99 ≤ 𝑓 𝑚 + 99 លឹម សុ វណ្ណវិចិត្រ | I. គណ្នា

71


ចំផ ោះគ្គប់ 𝑚 ∈ ℤ ។

817. ចូរកំនត់ អនុគមន៍ 𝑓: ℝ → ℝ ដែលផផទ ៀងផ្ទទត់ 𝑥𝑓 𝑥 + 𝑦 + 𝑦𝑓 𝑦 − 𝑥 = 𝑓 2 𝑥 + 𝑓 2 𝑦 ចំផ ោះគ្គប់ 𝑥, 𝑦 ∈ ℝ ។

818. ចូរកំនត់ អនុគមន៍ 𝑓: ℤ → ℤ ដែលផផទ ៀងផ្ទទត់ លកខខណ្ឌ 1) 𝑓 1 = 1 2) 𝑓 𝑥 + 𝑦 𝑓 𝑥 − 𝑓 𝑦 ចំផ ោះគ្គប់ 𝑥, 𝑦 ∈ ℤ ។

=𝑓 𝑥−𝑦 𝑓 𝑥 +𝑓 𝑦

819. (កាណាដា ១៩៦៩) តាង 𝑓 ជាអនុគមន៍មួយដែលមានលកខណ្ៈែូចតផៅផនោះ a) 𝑓 𝑛 មានន័យចំព b) 𝑓(𝑛) ជាចំនន ួ គត់ ;

ោះគ្រប់ ចំនួនរត់វជ្ជ ិ មាន 𝑛 ;

c) 𝑓 2 = 2; d) 𝑓 𝑚𝑛 = 𝑓 𝑚 𝑓 𝑛 ចំផ ោះគ្គប់ 𝑚 និង 𝑛 ; e) 𝑓 𝑚 > 𝑓 𝑛 ចំផ ោះគ្គប់ 𝑚 > 𝑛 ។ ចូរបង្ហាញថា 𝑓 𝑛 = 𝑛 ។

820. ផគផោយ 𝑓 𝑥 ជាអនុគមន៍ដែលផផទ ៀងផ្ទទត់ 0 < 2𝑓 2 𝑥𝑦 ≤ 𝑓 𝑥 𝑓 𝑦 3 + 𝑓 𝑥 3 𝑓 𝑦 , 𝑓 1999 > 0 ចូរបង្ហាញថា 𝑓 2000 > 0 ។

72

៥. សមីការអនុគមន៍ | លឹម សុ វណ្ណវិចិត្រ

∀𝑥, 𝑦 ∈ ℝ


១. អនុគមន៍ ងាយ គណ្នា 1. 1 2.  យយើ ងមាន : 𝑎2 + 𝑏 2 + 𝑐 2 = 1 ⟹ 𝑎2 + 𝑏 2 = 1 − 𝑐 2 : 𝑎 + 𝑏 + 𝑐 = 0 ⟺ 𝑎 + 𝑏 = −𝑐 ⟹ 𝑎 + 𝑏 ⟹ 1 − 𝑐 2 + 2𝑎𝑏 = 𝑐 2 ⟹ 𝑎𝑏 = : 𝑎4 + 𝑏 4 + 𝑐 4 = 𝑎2 + 𝑏 2 1 2

2

2

= 𝑐 2 ⟺ 𝑎2 + 𝑏 2 + 2𝑎𝑏 = 𝑐 2 ;

2𝑐 2 −1 2

+ 𝑐 4 − 2 𝑎𝑏

2

= 1 − 𝑐2

2

+ 𝑐4 − 2

1 2

2𝑐 2 −1 2

2

= 𝑐 4 − 2𝑐 2 +

1 − 2𝑐 4 + 2𝑐 2 − = ។

2. 3.

.

𝑛 𝑛 +1 (𝑛+2) 6 𝑛 𝑛 +1 2

2

 យយើ ងមាន

= 24 = 14 + 4.13 . 1 + 6.12 . 12 + 4.1.13 + 14 4 = 34 = 24 + 4.23 . 1 + 6.22 . 12 + 4.2.13 + 14 … 𝑛 + 1 4 = 𝑛4 + 4. 𝑛3 . 1 + 6. 𝑛2 . 12 + 4. 𝑛. 13 + 14 បូកសមភាពទាំងអស់ យនេះចូលគ្នន យយើ ងទញបានផលបូកដដលចង់ បាន។ 1+1 2+1

4

4.4  គុណ 𝑆 នឹង 2 − 1 យយើ ងទញបាន 𝑆

= (2 − 1) 2 + 1 22 + 1 24 + 1 … 21024 + 1 + 1 = 22 − 1 22 + 1 24 + 1 … 21024 + 1 + 1 = 24 − 1 24 + 1 … 21024 + 1 + 1 ⋮ = 21024 − 1 21024 + 1 + 1 = 22048 − 1 + 1 = 22048

ដូយចនេះ 𝑆 1/1024 = 4 ។

5. 𝑛 + 1 ! − 1  យយើ ងមាន 𝑘. 𝑘! = 𝑘 + 1 ! − 𝑘! ចាំ យ េះ 𝑘 = 1,2,3 … ។ ដូយចនេះ 1.1! + 2.2! + 3.3! + ⋯ + 𝑛 − 1 . 𝑛 − 1 ! + 𝑛. 𝑛! = 2! − 1! + 3! − 2! + 4! − 3! + ⋯ + 𝑛! − 𝑛 − 1 ! + លឹម សុ វណ្ណវិចិត្រ | I. គណ្នា

𝑛 + 1 ! − 𝑛! 73


= 𝑛+1 !−1 6.  យយើ ងមាន 2000 2000 2000 2000 + + ⋯+ + 1 3 1997 1999 2000 1 2000 1 = = 1 + 1 2000 = 21999 2 𝑖 2

𝑆. 2000! =

𝑖=1

រាំលក ឹ ៖ រូបមនតយទេធាញវូ តុន

21999 𝑆= 2000! 𝑛

𝑎+𝑏

𝑛

=

7. 0  សនមតថា 𝑢, 𝑣, 𝑠, 𝑡 ជារ ឺសបួនននពហុ ធា

𝑖=1

𝑛 𝑛−𝑖 𝑖 𝑎 𝑏 𝑖

𝑓 𝑥 = 𝑥 4 − 𝑎𝑥 3 + 𝑏𝑥 2 − 𝑐𝑥 + 𝑑 តាមទ្ទឹសតីបទដវែត យយើ ងទញបាន 𝑢 + 𝑣 + 𝑠 + 𝑡 = 𝑎; 𝑢𝑣 + 𝑣𝑠 + 𝑠𝑡 + 𝑡𝑢 + 𝑢𝑠 + 𝑣𝑡 = 𝑏; 𝑢𝑣𝑠 + 𝑣𝑠𝑡 + 𝑠𝑡𝑢 = 𝑐, 𝑢𝑣𝑠𝑡 = 𝑑 យោយ 𝑢 + 𝑣 + 𝑠 + 𝑡 = 0 យ េះ 𝑎 = 0 ។ ដូយចនេះ 𝑓 𝑥 = 𝑥 4 + 𝑏𝑥 2 − 𝑐𝑥 + 𝑑 ។ តាង យយើ ងមាន

𝑆𝑛 = 𝑢𝑛 + 𝑣 𝑛 + 𝑠 𝑛 + 𝑡 𝑛

𝑆1 = 𝑢 + 𝑣 + 𝑠 + 𝑡 = 0; 𝑆2 = 𝑢2 + 𝑣 2 + 𝑠 2 + 𝑡 2 = 𝑢 + 𝑣 + 𝑠 + 𝑡 2 − 2 𝑢𝑣 + 𝑣𝑠 + 𝑠𝑡 + 𝑡𝑢 + 𝑢𝑠 + 𝑣𝑡 = −2𝑏 យដើមបីគណ 𝑆3 យយើ ងព ិនិតែករណី២ខាងយទ្ោម។ 

ករណី 𝑢, 𝑣, 𝑠, 𝑡 សុ ទដធ តខុសព ីសូ នែទាំងអស់ ។ ដូយចនេះ 𝑑 𝑑 𝑢3 + 𝑏𝑢 − 𝑐 + = 0; 𝑣 3 + 𝑏𝑣 − 𝑐 + = 0 𝑢 𝑣 𝑑 𝑑 𝑠 3 + 𝑏𝑠 − 𝑐 + = 0; 𝑡 3 + 𝑏𝑡 − 𝑐 + = 0 𝑠 𝑡 1 1 1 1 ⟹ 𝑆3 + 𝑏𝑆1 − 4𝑐 + 𝑑 + + + =0 𝑢 𝑣 𝑠 𝑡 𝑐 ⟹ 𝑆3 − 4𝑐 + 𝑑 = 0 𝑑 ⟹ 𝑆3 = 3𝑐 ករណីកនង ុ ចាំ យ ម 𝑢, 𝑣, 𝑠, 𝑡មានមួយយសមើ សូនែ។ សនមតថាជា 𝑢 ។ ដូយចនេះ 𝑑 = 0 យហើ យ

𝑓 𝑥 = 𝑥 4 + 𝑏𝑥 2 − 𝑐𝑥 = 𝑥(𝑥 3 + 𝑏𝑥 − 𝑐) ដូយចនេះ 𝑣, 𝑠, 𝑡 ជារ ឺសននសមីោរ 𝑥 3 + 𝑏𝑥 − 𝑐 = 0 ។ ដូយចនេះ 𝑣 3 + 𝑏𝑣 − 𝑐 = 0; 𝑠 3 + 𝑏𝑠 − 𝑐 = 0; 𝑡 3 + 𝑏𝑡 − 𝑐 = 0 74

១. អនុគមន៍ងាយ | លឹម សុ វណ្ណវិចិត្រ


⟹ 𝑆3 + 𝑏 𝑢 + 𝑣 + 𝑠 + 𝑡 − 3𝑐 = 0; ⟹ 𝑆3 = 3𝑐 កនង ុ ករណីទាំងព ីរ យយើ ងមាន 𝑆3 = 3𝑐 ។ យយើ ងមាន 𝑢4 + 𝑏𝑢2 − 𝑐𝑢 + 𝑑 = 0; 𝑣 4 + 𝑏𝑣 2 − 𝑐𝑣 + 𝑑 = 0; 𝑠 4 + 𝑏𝑠 2 − 𝑐𝑠 + 𝑑 = 0; 𝑡 4 + 𝑏𝑡 2 − 𝑐𝑡 + 𝑑 = 0 ⟹ 𝑆4 + 𝑏𝑆2 − 𝑐𝑆1 + 𝑑 = 0 ⟹ 𝑆4 = 2𝑏 2 − 4𝑑 4 2 យយើ ងមាន 𝑥 + 𝑏𝑥 − 𝑐𝑥 + 𝑑 = 0 យោយគុណអងគទង ាំ ព ីរនឹង 𝑥 𝑛 យយើ ងទញបាន យយើ ងទញបាន

𝑥 𝑛+4 + 𝑏𝑥 𝑛+2 − 𝑐𝑥 𝑛+1 + 𝑑𝑥 𝑛 = 0 ⟹ 𝑆𝑛+4 + 𝑏𝑆𝑛+2 − 𝑐𝑆𝑛+1 + 𝑑𝑆𝑛 = 0

𝑆5 = −𝑏𝑆3 + 𝑐𝑆2 + 𝑑𝑆1 = −5𝑏𝑐 𝑆7 = −𝑏𝑆5 + 𝑐𝑆4 − 𝑑𝑆3 = 7𝑐 𝑏 2 − 𝑑 យោយ 𝑆7 = 0 យ េះ 𝑐 = 0 រ ឺ 𝑏 2 = 𝑑 ។ 

ករណី 𝑏 2 = 𝑑 យយើ ងទញបាន

0 ≤ 𝑆4 = 2𝑏 2 − 4𝑑 = −2𝑏 2 ≤ 0 ⟹ 𝑏 = 0 ⟹ 𝑑 = 0 ⟹ 𝑆4 = 0 ⟹ 𝑢 = 𝑣 = 𝑠 = 𝑡 = 0 ⟹ 𝑃 = 0 ករណី 𝑐 = 0 យ េះ 𝑢, 𝑣, 𝑠, 𝑡 ជារ ឺសទាំងបួនននសមីោរ 𝑥 4 + 𝑏𝑥 2 + 𝑑 = 0 ដូយចនេះវាទ្តូវដត 𝑡 = −𝑢 រ ឺ 𝑡 = −𝑣 រ ឺ 𝑡 = −𝑠។ ដូយចនេះ 𝑃 = 0 ។

សមភាព 1.2.3 6 2 2

8. ចាំ យ េះ 𝑛 = 1 យយើ ងទញបាន 12 = 𝑘2 = 𝑘+

𝑘 𝑘+1 2𝑘+1 ។ យយើ ងទញបាន 6 𝑘+1 𝑘+1+1 2 𝑘+1 +1 1 2= 6

ព ិត។ សនមតថាព ិតដល់ 𝑘 មានន័យថា 12 + 22 + ⋯ +

1 + 2 + ⋯ + 𝑘2 + 𝑘 + 1

2

=

𝑘 𝑘+1 2𝑘+1 6

+

។ ដូយចនេះតាមវ ិចារយោយកាំយនើន យយើ ងទញបានសមមភាពព ិត

ចាំ យ េះទ្គប់ 𝑛 ∈ ℕ ។

9. ទ្ាយបញ្ជាក់ តាមកាំយនើន។ 10. ទ្ាយបញ្ជាក់ តាមកាំយនើន។ 11. ទ្ាយបញ្ជាក់ តាមកាំយនើន។ 12. តាង 𝑎 = tan𝑥, 𝑏 = tan𝑦, 𝑐 = tan𝑧 ដដល 𝑥, 𝑦, 𝑧 ≠

𝑘𝜋 4

ចាំ យ េះទ្គប់ ចាំ នន ួ គត់ 𝑘។

លកខខណឌ 𝑎 + 𝑏 + 𝑐 = 𝑎𝑏𝑐 ោាយជា tan (𝑥 + 𝑦 + 𝑧) = 0 ។យយើ ងមាន លឹម សុ វណ្ណវិចិត្រ | II. សមភាព

75


tan 2� + 2� + 2� =

2 tan đ?‘Ľ + đ?‘Ś + đ?‘§ =0 1 − tan2 đ?‘Ľ + đ?‘Ś + đ?‘§

ដសយចនá&#x; á&#x;‡ tan 2đ?‘Ľ + tan 2đ?‘Ś + tan 2đ?‘§ = tan 2đ?‘Ľ tan 2đ?‘Ś tan 2đ?‘§ 2 tan đ?‘Ľ 2 tan đ?‘Ś 2 tan đ?‘§ â&#x;š + + 2 2 1 − tan đ?‘Ľ 1 − tan đ?‘Ś 1 − tan2 đ?‘§ 2 tan đ?‘Ľ 2 tan đ?‘Ś 2 tan đ?‘§ = 2 2 1 − tan đ?‘Ľ 1 − tan đ?‘Ś 1 − tan2 đ?‘§ ដសយចនá&#x; á&#x;‡áž&#x;áž˜áž—ážśáž–áž– ឡáž?á&#x;”

13.ď ° áž?ážśáž„ đ?‘˜ = đ?‘Ž1 /đ?‘?1 = đ?‘Ž2 /đ?‘?2 = đ?‘Ž3 /đ?‘?3 áž™ á&#x; á&#x;‡

đ?‘?1 đ?‘Ž1đ?‘› + đ?‘?2 đ?‘Ž2đ?‘› + đ?‘?3 đ?‘Ž3đ?‘› = đ?‘˜ đ?‘› đ?‘?1 đ?‘?1đ?‘› + đ?‘?2 đ?‘?2đ?‘› + đ?‘?3 đ?‘?3đ?‘› đ?‘Ž1 đ?‘› = đ?‘?1 đ?‘?1đ?‘› + đ?‘?2 đ?‘?2đ?‘› + đ?‘?3 đ?‘?3đ?‘› đ?‘?1 យវá&#x;† á&#x;„áž™áž&#x;áž˜áž—ážśáž–áž– ឡáž?á&#x;”

áž&#x;áž˜áž¸áž€ážśážš 1 1

14. – 2; −1; 1 . 15. – 3; 2 . 16. 3 . 17. 1 . 18. – 1; − 2 ; 3 . 19. − 2; −

1 1 ; 2; 2

1 2

. 20.

2 . ď Ź áž?ážśáž„ đ?‘Ľ 2 = đ?‘Ś á&#x;” 21. {− 3; 3} . 22. −1; 1 .

23. 2 . ď ° áž?ážśáž„ 2đ?‘Ľ − 3 = đ?‘Ś + đ?‘? នឡង 2đ?‘Ľ − 5 = đ?‘Ś − đ?‘? á&#x;”យយឞ ងទញបវន đ?‘? = 1á&#x;” ជវá&#x;†áž“áž&#x; áž˝ áž…ážźáž› áž&#x;មីá&#x;„ážš យយឞ ងទញបវន đ?‘Ś + 1

4

+ đ?‘Śâˆ’1

4

= 2 áž&#x;áž˜áž˜ážźáž›áž“ážšáž„ đ?‘Ś 4 + 6đ?‘Ś 2 = 0á&#x;” យយឞ ងទញបវន

យវá&#x;† á&#x;„áž™ đ?‘Ľ = 2 á&#x;”

đ?‘Ś=0 1

24. − 2 ; 1 . 25. −

đ?‘Ž

13+1 2

;

đ?‘Ž

13−1 2

đ?‘Ž ∈ â„? . ď Ź áž?ážśáž„đ?‘Ľ 2 + đ?‘Žđ?‘Ľ = đ?‘Ś á&#x;” 26. −

đ?&#x;?đ?&#x;?+đ?&#x;“ đ?&#x;?đ?&#x;?−đ?&#x;“ ; đ?&#x;” đ?&#x;”

2

27. − 2; 4 − 18; 2; 4 + 18 . ď Ź áž?ážśáž„ đ?‘Ľ − đ?‘Ľ = đ?‘Śá&#x;” áž&#x;មីá&#x;„ážšáž&#x;áž˜áž˜ážźáž›áž“ážšáž„ đ?‘Ś 2 − 8đ?‘Ś = 0á&#x;” 3

7+1 7−1 3 1 ; 4 ; 1 . ď Ź ដចកអងគទង ážśá&#x;† áž– ីរនរង đ?‘Ľ 2 áž” áž‘ áž”á&#x;‹ មកáž?ážśáž„ 2đ?‘Ľ − đ?‘Ľ = đ?‘Śá&#x;” 29. {2 ; 2} 4 5− 61 5+ 61 9 30. −1; 9; 2 ; 2 . ď Ź ដចកអងគទង ážśá&#x;† áž– ីរនរង đ?‘Ľ 2 áž” áž‘ áž”á&#x;‹ មកáž?ážśáž„ đ?‘Ľ − đ?‘Ľ = đ?‘Ś á&#x;” 2 đ?&#x;?− đ?&#x;?đ?&#x;? đ?&#x;?+ đ?&#x;?đ?&#x;? 10đ?‘Ľ 2 đ?‘Ľ2 đ?‘Ľ2 31. ; . ď Ź ដកអងគ áž‘ áž„ ážśá&#x;† áž– ីរនរ áž„ យយឞ áž„ ទញបវន + 10 đ?‘Ľ+5 − 11 = 0 á&#x;” đ?&#x;? đ?&#x;? đ?‘Ľ+5 đ?‘Ľ+5 2 đ?‘Ľ2 đ?‘Ľ2 32. − 7 − 1; 7 − 1 . ď Ź បទá&#x;’áž„áž˝áž˜áž&#x;មីá&#x;„រជវរវង đ?‘Ľâˆ’3 − 6 đ?‘Ľâˆ’3 − 16 = 0 á&#x;” 2 2−1+1 2 2−1−1 33.{− ; }.ď Ź បទá&#x;’áž„áž˝áž˜áž&#x;មីá&#x;„រជវ đ?‘Ľ 2 + 1 2 − 2 đ?‘Ľ − 1 2 = 0 á&#x;” 2 2 2 2

28. − 2 ; −

34. 76

2 2−1+1

;

2 2−1−1

. ď Ź áž?ážśáž„ đ?‘Ľ = 1 đ?‘Ś á&#x;”

á&#x;Ą. áž˘áž“ážťáž‚áž˜áž“á&#x;?áž„ážśáž™ | លរម áž&#x;ážť វណá&#x;’ážŽážœážˇáž…ážˇáž?á&#x;’ážš


35. −9; 11 .  បូក 4𝑥 2 + 400𝑥 + 1 យៅអងគទង ាំ ព ីរននសមីោរ។ 36. – 𝑎; 𝑎 − 𝑎2 + 2; 𝑎 + 𝑎2 + 2|𝑎 ∈ ℝ .  យយើ ងទញរក 𝑎 យយើ ងទញបាន 𝑎 =

𝑥 2 −2 2𝑥

រឺ

𝑎 = −𝑥 ។ ទញរក 𝑥 យយើ ងទញបានចាំ យលើយ។ 37. – 1 − 3 + 𝑎; – 1 + 3 + 𝑎 ចាំ យ េះ 𝑎 ∈ −3; −1 ; – 1 − 3 + 𝑎; – 1 + 3 + 𝑎; −1 − 1 + 𝑎; −1 + 1 + 𝑎 ចាំ យ េះ 𝑎 ∈ −1; ∞ ; គ្នមនរ ឺសចាំ យ េះ 𝑎 ∈ −∞; −3 . យោេះទ្ាយរក 𝑎 យោយសនមតថា 𝑥 ជាបា៉ារា៉ាដម៉ា ទ្ត : 𝑎2 − 2 𝑥 2 − 1 𝑎 + 𝑥 4 − 6𝑥 2 + 4𝑥 = 0; យយើ ងទញបាន 𝑎 = 𝑥 2 + 2𝑥 − 2 រ ឺ 𝑎 = 𝑥 2 − 2𝑥 ។ 38.

−1− 29 5− 17 −1+ 29 5+ 17 ; ; ; 2 2 2 2 2

. យយើ ងសរយសរអងគខាងយឆេងននសមីោរជា

𝑥 + 𝑎𝑥 + 𝑐

𝑥 2 + 𝑏𝑥 + 𝑑 = 0

ដូយចនេះ 𝑥 4 + 𝑎 + 𝑏 𝑥 3 + 𝑎𝑏 + 𝑐 + 𝑑 𝑥 2 + 𝑏𝑐 + 𝑎𝑑 𝑥 + 𝑐𝑑 ≡ 𝑥 4 − 4𝑥 3 − 10𝑥 2 + 37𝑥 − 14 យយើ ងទញបាន 𝑎 + 𝑏 = −4 𝑎𝑏 + 𝑐 + 𝑑 = −10 𝑏𝑐 + 𝑎𝑑 = 37 𝑐𝑑 = −14 យោយដឹងថា 𝑎, 𝑏, 𝑐, 𝑑 ជាចាំ នន ួ គត់ យ េះ ព ីសមីោរចុងយទ្ោយយគ យយើ ងទញបាន 𝑐; 𝑑 = −1; 14 ; 1; −14 ; −2; 7 ; 2; −7

។ យយើ ងយផទៀងផ្ទទត់ យឃើ ញថា 𝑐; 𝑑 = (2; −7)

យផទៀងផ្ទទត់ ។ យយើ ងទញបាន 𝑎 = −5; 𝑏 = 1។ ដូយចនេះសមីោរសមមូលនឹង 𝑥 2 − 5𝑥 + 2 = 0 រ ឺ 𝑥2 + 𝑥 − 7 = 0 ។

39. សនមតថា 𝑥1 = 𝑝/𝑞 ដដល 𝑝 ∈ ℤ, 𝑞 ∈ ℕ និង 𝑝 និង 𝑞 ជាចាំ នន ួ បឋមនឹងគ្នន។ ដូយចនេះ 𝑝3 𝑞3

𝑝2

𝑝

+ 𝑎 𝑞 2 + 𝑏 𝑞 + 𝑐 = 0 សមមូលនឹង 𝑝 𝑝2 + 𝑎𝑝𝑞 + 𝑏𝑞 2 = −𝑐𝑞 3 ។ អងគខាងាតាំននសមភាព

យនេះដចកោច់ នឹង 𝑞 ។ អងគខាងយឆេងដចកោច់ នឹង 𝑞

យាំ ោយនិងមានដត 𝑞 ≡ 1 ដតប៉ា ុយ្ណេះ យទ្ េះ

𝑝2 + 𝑎𝑝𝑞 + 𝑏𝑞 2 ដចកមិនោច់ នឹង 𝑞 ។ ដូយចនេះសមភាពមានរាង 𝑝 𝑝2 + 𝑎𝑝 + 𝑏 = −𝑐 យយើ ងទញ បាន 𝑐 ជាពហុ គុណនន 𝑝 = 𝑥1 ។

40. យទ្បើទ្ាយបញ្ជាក់ ដច ូ សាំ នួរទី 39.។ 41. យទ្បើសមភាព 𝑎𝑥 3 + 𝑏𝑥 2 + 𝑐𝑥 + 𝑑 = 𝑎 𝑥 − 𝑥1 𝑥 − 𝑥2 𝑥 − 𝑥3 . 42. −2𝑝 . លឹម សុ វណ្ណវិចិត្រ | III. សមីការ

77


43.

3

13+3 2

3

13−3 2

1

1

1

 តាង 𝑥 = 𝑦 − 1/𝑦 ។ សមីោរយៅជា 𝑦 3 − 𝑦 3 − 3 𝑦 − 𝑦 + 1

3 𝑦 − 𝑦 − 3 = 0 រ ឺ 𝑦 3 − 𝑦 3 − 3 = 0 ។ តាង 𝑦 3 = 𝑡 យយើ ងទញបាន 𝑡 2 − 3𝑡 − 1 = 0 ។

44. −5; −1; 1; 3 . 45. 2; 6 . 46. −1 . 47. គ្នមនរ ឺសសនិទន 1

48.{− 3 ; 1/2} . 49. 6 .  គុណអងគទង ាំ ព ីរននសមីោរនឹង 𝑥 + 10 − 𝑥 − 2 ≠ 0 យយើ ងទញបាន 𝑥 + 10 + 𝑥 − 2 = 6 𝑥 + 10 − 𝑥 − 2 = 2 បូកអងគទង ាំ ព ីរចូលគ្នន យយើ ងទញបាន 𝑥 = 6 ។ 1 1

50. សមីោរគ្នមនរ ឺស។ 51. −1 . 52. 12 . 53. − 4

−𝑎 𝑎

2 1 1

;4

−𝑎 𝑎

2

ចាំ យ េះ 𝑎 ∈ 0; 1

យហើ យគ្នមនរ ឺសចាំ យ េះតាំ នល 𝑎 យផេងយទៀត។ 54. −6; 1 . 55. −1; 3 . 56. 3 . 57. 2 58. 5 .

59. 20 . 60. −4/3 . 61. −1; 0 . 62. 5 . 63. 16 . 64. 9 . 65. −3; 6 .  តាង 𝑦 = 𝑥 2 − 3𝑥 + 7 ។ 66. −9/2; 3 . 67. 1 .  តាង 𝑦 = 𝑥

4

𝑥 2 + 15 ។ 68. 5/3 .

69. {5𝑎/3} ចាំ យ េះ 𝑎 ≠ 0; −∞; 0 ∪ 0; ∞ ចាំ យ េះ 𝑎 = 0 ។ 70. 1 . 71. 3; 9 72. 5 ± 𝑎 8 −

𝑎2 2

ចាំ យ េះ 𝑎 ∈ 2; 2 2 ; គ្នមនរ ឺសចាំ យ េះតាំ នល 𝑎 យផេងព ីយនេះ។ តាង

𝑢 = 7 − 𝑥, 𝑣 = 𝑥 − 3 ។ យយើ ងមានទ្បព័នស ធ មីោរ 𝑢+𝑣 =𝑎 7 − 𝑥 + 𝑥 − 3 = 𝑢2 + 𝑣 2 = 4, 𝑢 ≥ 0, 𝑣 ≥ 0 យោេះទ្ាយទ្បព័នស ធ មីោរយនេះ យយើ ងទញបាន 𝑎 + 8 − 𝑎2 2 𝑎 − 8 − 𝑎2 𝑣1 = 2 𝑢1 =

𝑎 − 8 − 𝑎2 2 𝑎 + 8 − 𝑎2 𝑣2 = 2 𝑢2 =

73. 3 . តាង 4 𝑥 − 2 = 𝑢 ≥ 0, 4 4 − 𝑥 = 𝑣 ≥ 0 យយើ ងទញបានទ្បព័ន ធ 𝑢+𝑣 = 2 𝑢4 + 𝑣 4 = 2

78

១. អនុគមន៍ងាយ | លឹម សុ វណ្ណវិចិត្រ

9− 97 8

.


យលឞកáž&#x;មីá&#x;„ážšáž‘áž¸áž˜áž˝áž™áž‡ážśáž&#x;á&#x; á&#x;? យគឝណá&#x;¤ នឡងយទá&#x;’បឞáž&#x;មីá&#x;„រទីá&#x;˘ យយឞ ងទញបវន đ?‘˘đ?‘Ł 2 đ?‘˘ + đ?‘Ł យហឞ យយá&#x;„áž™ đ?‘˘ + đ?‘Ł = 2 យយឞ ងទញបវន đ?‘˘đ?‘Ł

2

− 8 đ?‘˘đ?‘Ł + 7 = 0á&#x;” ដសយចនá&#x; á&#x;‡ đ?‘˘đ?‘Ł

1

2

− đ?‘˘đ?‘Ł = 7

= 1; ��

2

=

7á&#x;” ដសយចនá&#x; á&#x;‡áž™áž™ážž ងទញបវនទá&#x;’បពá&#x;?áž“áž&#x; áž’ មីá&#x;„ážš đ?‘˘+đ?‘Ł =2 đ?‘˘+đ?‘Ł =2 đ?‘˘đ?‘Ł = 1 đ?‘˘đ?‘Ł = 7 áž‘á&#x;’បពá&#x;?áž“áž&#x; áž’ មីá&#x;„ážšáž‘áž¸áž˜áž˝áž™áž˜ážśáž“ážš ážşáž&#x; đ?‘˘ = đ?‘Ł = 1 á&#x;” áž‘á&#x;’áž?ážźážœáž“ážšáž„đ?‘Ľ = 3 á&#x;” áž‘á&#x;’បពá&#x;?áž“áž‘ áž’ á&#x;˘ ី áž‚á&#x;’នមនរ ážşáž&#x;á&#x;”

74.

2đ?‘Ž+1

áž…ážśá&#x;† áž™ á&#x; á&#x;‡đ?‘Ž ∈ −∞; 3 âˆŞ (2; ∞); âˆ…áž…ážśá&#x;† áž™ á&#x; á&#x;‡đ?‘Ž ∈

đ?‘Ž −2

1 ;2 3

.

75. đ?‘Ž + 1 + 2đ?‘Ž; đ?‘Ž + 1 − 2đ?‘Ž áž…ážśá&#x;† áž™ á&#x; á&#x;‡ đ?‘Ž ∈ 0; 1/2 , đ?‘Ž + 1 + 2đ?‘Ž áž…ážśá&#x;† áž™ á&#x; á&#x;‡ đ?‘Ž ∈

1 ;∞ 2

,∅

áž…ážśá&#x;† áž™ á&#x; á&#x;‡ đ?‘Ž ∈ −∞; 0 .

76. −∞; 0 áž…ážśá&#x;† áž™ á&#x; á&#x;‡ đ?‘Ž = 0, 0; 3đ?‘Ž/4 áž…ážśá&#x;† áž™ á&#x; á&#x;‡ đ?‘Ž ∈ 0; ∞ , ∅ áž…ážśá&#x;† áž™ á&#x; á&#x;‡ đ?‘Ž ∈ −∞; 0 . 77. đ?‘Ž2 + đ?‘Ž; đ?‘Ž2 − đ?‘Ž + 1 áž…ážśá&#x;† áž™ á&#x; á&#x;‡ đ?‘Ž ∈ 0; 1 , đ?‘Ž2 + đ?‘Ž áž…ážśá&#x;† áž™ á&#x; á&#x;‡ đ?‘Ž ∈ 1; ∞ , ∅ áž…ážśá&#x;† áž™ á&#x; á&#x;‡ đ?‘Ž ∈ −∞; 0 .

78. 0 áž…ážśá&#x;† áž™ á&#x; á&#x;‡ đ?‘Ž = 1, ∅ áž…ážśá&#x;† áž™ á&#x; á&#x;‡ đ?‘Ž ≠1. 79. 1; −5 . 80.

1; −3 . 81. −3; 1 áž…ážśá&#x;† áž™ á&#x; á&#x;‡

đ?‘Ž ∈ −2 ; ∅ áž…ážśá&#x;† áž™ á&#x; á&#x;‡ đ?‘Ž ∉ −2 .

82.ď ° áž?ážśáž„ f(đ?‘Ľ) = 4đ?‘Ľ(1 − đ?‘Ľ) = 1 − 2đ?‘Ľ − 1

2

á&#x;” យយឞ ងយឃឞ ញáž?ážś យបឞ 0 ≤ f x ≤ 1 áž™ á&#x; á&#x;‡

0 ≤ đ?‘Ľ ≤ 1á&#x;” ដសយចនá&#x; á&#x;‡ យបឞ a1998 = 0 áž™ á&#x; á&#x;‡ áž‘á&#x;’áž?ážźážœážŠáž? 0 ≤ đ?‘Ą ≤ 1á&#x;” អី áž›ážźážœ យយឞ ងយក 0 ≤ đ?œƒ ≤

đ?œ‹ 2

ដដល

sin đ?œƒ = đ?‘Ą á&#x;” យយឞ áž„áž&#x;យងá&#x; áž?យឃឞ ញáž?ážś áž…ážśá&#x;† áž™ á&#x; á&#x;‡áž‘á&#x;’áž‚áž”á&#x;‹ đ?œ™ ∈ â„? យយឞ áž„áž˜ážśáž“ đ?‘“ sin2 đ?œ™ = 4 sin2 đ?œ™ 1 − sin2 đ?œ™ = 4 sin2 đ?œ™ cos2 đ?œ™ = sin2 2đ?œ™ áž™á&#x;„áž™ đ?‘Ž1 = sin2 đ?œƒ áž™ á&#x; á&#x;‡ យយឞ ងទញបវន ដសយចនá&#x; á&#x;‡ đ?‘Ž1998

đ?‘Ž2 = sin2 2đ?œƒ , đ?‘Ž3 = sin2 4đ?œƒ , â‹Ż , đ?‘Ž1998 = sin2 21997 đ?œƒ đ?‘˜đ?œ‹ = 0 áž‘áž›á&#x;‹ ដáž?នឡង យវá&#x;† á&#x;„áž™ sin 21997 θ = 0 á&#x;” áž˜ážśáž“áž“á&#x;?áž™áž?ážś đ?œƒ = 21997 áž…ážśá&#x;† áž™ á&#x; á&#x;‡áž…ážśá&#x;† áž“áž“ áž˝ áž‚áž?á&#x;‹ đ?‘˜

áž ážśá&#x; á&#x;‡ យហឞ áž™ áž?ážśá&#x;† នលរបáž&#x;á&#x;‹ đ?‘Ą ដដល đ?‘Ž1998 = 0 áž™áž&#x;មឞ áž“áž„ ážš sin2 đ?‘˜đ?œ‹/21997 áž˜ážśáž“áž?ážśá&#x;† នលរបáž&#x;á&#x;‹ đ?‘Ą ដបបយនá&#x; á&#x;‡ áž…ážśá&#x;† áž“áž“ áž˝ 2

1996

ដដល đ?‘˜ ∈ ℤ á&#x;” ដសយចនá&#x; á&#x;‡áž™áž™ážž áž„

+ 1 áž‚ážşáž˜ážśáž“áž?ážśá&#x;† áž“áž› sin đ?‘˜đ?œ‹/21997 áž…ážśá&#x;† áž™ á&#x; á&#x;‡ đ?‘˜ = 2

0,1,2, â‹Ż , 21996 á&#x;”

83. ď ° áž™á&#x;„áž™ đ?‘Ž2 − 2đ?‘? 2 = 1 áž™ á&#x; á&#x;‡ đ?‘Ž ≠0 á&#x;” áž™á&#x;„áž™ 2đ?‘? 2 − 3đ?‘? 2 = 1 áž™ á&#x; á&#x;‡ đ?‘? ≠0 á&#x;” យបឞ đ?‘? = 0 áž™ á&#x; á&#x;‡ đ?‘? =

1 2

នឡង đ?‘Ž = 2 á&#x;” đ?‘Ž, đ?‘?, đ?‘? =

2, 1/ 2, 0 ជវចវá&#x;† áž™áž›ážžáž™áž˜áž˝áž™ážšáž”áž&#x;á&#x;‹ áž‘á&#x;’បពá&#x;?áž“áž&#x; áž’ មីá&#x;„ážšá&#x;” យយឞ áž„

នរងបងá&#x;’ហវញáž?ážś áž‚á&#x;’áž“áž˜áž“áž…ážśá&#x;† យលឞយá&#x;’យផá&#x; ងយទá&#x;€áž?យទá&#x;” យយឞ áž„áž&#x;នមáž?áž?ážś áž˜ážśáž“áž…ážśá&#x;† áž“áž“ áž˝ áž– ឡáž? đ?‘Ž, đ?‘?, đ?‘? áž™á&#x;„áž™ đ?‘Žđ?‘?đ?‘? ≠0 យផទá&#x;€áž„áž•á&#x;’áž‘áž‘áž?á&#x;‹ áž&#x;មីá&#x;„ážšá&#x;” យយឞ ងយឃឞ ញáž?ážś យបឞ đ?‘Ž, đ?‘?, đ?‘? ជវចវá&#x;† យលឞយ áž™ á&#x; á&#x;‡ −đ?‘Ž, −đ?‘?, −đ?‘? កá&#x;?ជវចវá&#x;† យលឞយដដរá&#x;” ដសយចនá&#x; á&#x;‡áž‘á&#x;’áž?ážźážœážŠáž?áž˜ážśáž“áž…ážśá&#x;† áž“áž“ áž˝ ážœ ážˇáž‡ážśáž˜ážśáž“áž– ីរកនង ážť លរម áž&#x;ážť វណá&#x;’ážŽážœážˇáž…ážˇáž?á&#x;’ážš | III. áž&#x;áž˜áž¸áž€ážśážš

79


áž…ážśá&#x;† áž™ ម đ?‘Ž, đ?‘?, đ?‘? ážš ážş −đ?‘Ž, −đ?‘?, đ?‘? á&#x;” áž&#x;នមáž?áž?ážś đ?‘Ž, đ?‘? ážœ ážˇáž‡ážśáž˜ážśáž“á&#x;” áž?ážśáž„ đ?‘Ž = cot đ??´ , đ?‘? = cot đ??ľ នឡង đ?œ‹

đ?‘? = cot đ??ś ដដល 0 < đ??´, đ??ľ < 2 , 0 < đ??ś < đ?œ‹ á&#x;” áž™á&#x;„áž™ đ?‘Žđ?‘? + đ?‘?đ?‘? + đ?‘?đ?‘Ž = 1 áž™ á&#x; á&#x;‡ cot đ??´ cot đ??ľ + cot đ??ľ cot đ??ś + cot đ??ś cot đ??´ = 1 1 − cot đ??´ cot đ??ľ cot đ??ś = = − cot đ??´ + đ??ľ = cot đ?œ‹ − đ??´ − đ??ľ cot đ??´ + cot đ??ľ đ??´+đ??ľ+đ??ś =đ?œ‹ ដសយចនá&#x; á&#x;‡ đ??´, đ??ľ, đ??ś áž‡ážśáž˜ážťážśá&#x;†áž€áž„ áž“ážť áž‘á&#x;’áž?ីយá&#x;„ណá&#x;” យយឞ áž„áž˜ážśáž“ đ?‘Ž2 + 1 = 2 đ?‘? 2 + 1 = 3 đ?‘? 2 + 1 1 2 3 = = 2 2 sin đ??´ sin đ??ľ sin2 đ??ś 1 2 3 â&#x;š = = sin đ??´ sin đ??ľ sin đ??ś áž?ážśáž˜áž…ážśáž”á&#x;‹ áž&#x;ឝី áž“áž&#x; ážť យយឞ ងទញបវនáž?ážś áž‘á&#x;’áž‡ážťáž„ážˆáž˜áž“ážšáž„áž˜ážťážśá&#x;† đ??´, đ??ľ, đ??ś áž˜ážśáž“ážšáž„á&#x;’áž á&#x; áž&#x;á&#x;‹ đ?‘˜, đ?‘˜ 2, đ?‘˜ 3 យរá&#x;€áž„áž‚á&#x;’áž“áž“ áž…ážśá&#x;† áž™ á&#x; á&#x;‡ â&#x;š

áž…ážśá&#x;† áž“áž“ áž˝ áž– ឡáž?ážœ ážˇáž‡ážśáž˜ážśáž“ đ?‘˜ áž ážśá&#x; á&#x;‡á&#x;” ដសយចនá&#x; á&#x;‡áž‘á&#x;’áž?ីយá&#x;„ណ đ??´đ??ľđ??ś ដកងទá&#x;’áž?áž„á&#x;‹ đ??ś

យវá&#x;† á&#x;„áž™ đ?‘? = cot đ??ś = 0 áž•áž‘áž™ ážť áž– ីáž&#x;នមáž?

ដដល đ?‘? ≠0 á&#x;” ដសយចនá&#x; á&#x;‡á&#x;„ážšáž&#x;នមáž?ážšáž”áž&#x;á&#x;‹ យយឞ áž„áž ážťáž&#x; យហឞ áž™ đ?‘Ž, đ?‘?, đ?‘? =

2, 1/ 2, 0 ជវចវá&#x;† យលឞយដáž?áž˜áž˝áž™

áž‚áž?á&#x;‹ ážšáž”áž&#x;á&#x;‹ áž‘á&#x;’បពá&#x;?áž“á&#x;” áž’

84.

4−đ?‘? 2 4−2đ?‘?

4 3

áž…ážśá&#x;† áž™ á&#x; á&#x;‡ ≤ đ?‘? ≤ 2. ď ° យយឞ áž„áž˜ážśáž“ đ?‘Ľ ≼ 0á&#x;” យលឞកអងគទង ážśá&#x;† áž– ីរជវá&#x;„យរយយឞ ងទញបវន

5đ?‘Ľ 2 − đ?‘? − 4 + 4 đ?‘Ľ 2 − 1 đ?‘Ľ 2 − đ?‘? = đ?‘Ľ 2 â&#x;ş 4 đ?‘Ľ 2 − 1 đ?‘Ľ 2 − đ?‘? = đ?‘? + 4 − 4đ?‘Ľ 2 យបឞ 4đ?‘Ľ 2 ≤ đ?‘? + 4 យយឞ ងយលឞកអងគទង ážśá&#x;† áž– ីរននáž&#x;មីá&#x;„រជវá&#x;„យរមáž? ងយទá&#x;€áž? យយឞ ងទញបវន

áž˜ážśáž“áž“á&#x;?áž™áž?ážś đ?‘Ľ = 4−đ?‘? 2 4−2đ?‘?

−16 đ?‘? + 1 đ?‘Ľ 2 + 16đ?‘? = −8 đ?‘? + 4 đ?‘Ľ 2 + đ?‘? + 4 2 4−đ?‘? 2 â&#x;ş đ?‘Ľ2 = 4 4 − 2đ?‘? 4−đ?‘? á&#x;” យដឞមបីយá&#x;„áž™áž?ážśá&#x;† នលយនá&#x; á&#x;‡áž‡ážśáž…ážśá&#x;† យលឞយននáž&#x;មីá&#x;„ážšáž‘áž›á&#x;‹ ដáž? đ?‘? ≤ 2 នឡង

2 4−2đ?‘?

4

= 4đ?‘Ľ 2 ≤ đ?‘? + 4 áž&#x;áž˜áž˜ážźáž›áž“ážšáž„ 3 ≤ đ?‘? ≤ 2 á&#x;” យទá&#x;’á&#x;…áž– ីយនá&#x; á&#x;‡áž&#x;មីá&#x;„ážšáž‚á&#x;’áž“áž˜áž“áž…ážśá&#x;† យលឞយá&#x;”

85.ď ° យយឞ áž„áž&#x;ážšáž™áž&#x;ážšáž&#x;មីá&#x;„រដដលយá&#x;„យជវរវង đ?‘Ľ 5 − đ?‘Ľ 3 − 4đ?‘Ľ 2 − 3đ?‘Ľ − 2 + đ?œ† 5đ?‘Ľ 4 + đ?›źđ?‘Ľ 2 − 8đ?‘Ľ + đ?›ź = 0 ážš ážşáž&#x;ážšáž”áž&#x;á&#x;‹ áž&#x;មីá&#x;„រយនá&#x; á&#x;‡áž˜ážˇáž“á&#x;„áž&#x;á&#x;’áž&#x;á&#x;? យនរង đ?œ† យវá&#x;† á&#x;„យនឡងយបឞ ážœážśáž‡ážśážš ážşáž&#x;ážšáž˝áž˜ážšáž”áž&#x;á&#x;‹ áž&#x;មីá&#x;„ážš đ?‘Ľ 5 − đ?‘Ľ 3 − 4đ?‘Ľ 2 − 3đ?‘Ľ − 2 = 0 នឡង 5đ?‘Ľ 4 + đ?›źđ?‘Ľ 2 − 8đ?‘Ľ + đ?›ź = 0 áž&#x;មីá&#x;„ážšáž‘áž¸áž˜áž˝áž™áž&#x;áž˜áž˜ážźáž›áž“ážšáž„ đ?‘Ľ − 2 đ?‘Ľ 2 + đ?‘Ľ + 1 −1Âąđ?‘– 3 á&#x;” 2 64 − 5 យយឞ áž„áž˜ážśáž“

2

= 0 យហឞ áž™áž˜ážśáž“ážš ážşáž&#x;យផá&#x; áž„áž‚á&#x;’áž“áž“áž…ážśá&#x;† áž“áž“ áž˝ បី

đ?‘Ľ1 = 2; đ?‘Ľ2,3 = ក) áž…ážśá&#x;† áž™ á&#x; á&#x;‡ đ?›ź = 80

đ?‘Ľ1 = 2 ជវរ ážşáž&#x;ដáž?áž˜áž˝áž™áž‚áž?á&#x;‹ ážŠážŠáž›áž˜ážˇáž“á&#x;„áž&#x;á&#x;’áž&#x;á&#x;? យនរង đ?œ† á&#x;”

á&#x;Ą. áž˘áž“ážťáž‚áž˜áž“á&#x;?áž„ážśáž™ | លរម áž&#x;ážť វណá&#x;’ážŽážœážˇáž…ážˇáž?á&#x;’ážš


ខ) ចាំ យ េះ 𝛼 = −3 យយើ ងមានរ ឺសចាំ នន ួ ព ីរដដលមិនោស្ស័ យនឹង 𝜆 គឺ 𝑥1 = 𝜔 និង 𝑥2 = 𝜔2 ។ 1

86. លកខខណឌ 2𝑥 − 1 ≥ 0, ⟹ 𝑥 ≥ 2 និង 𝑥 ≥ 2𝑥 − 1 ⟹ 𝑥 − 1

≥ 0 ព ិតជានិច។ ច យយើ ង 2, 1/2 ≤ 𝑥 ≤ 1 មាន 𝑥 + 2𝑥 − 1 + 𝑥 − 2𝑥 − 1 = 𝑐 ⟺ 𝑐 2 = 2𝑥 + 2 𝑥 − 1 = 4𝑥 − 2, 𝑥 ≥ 1 1 2 ក) 𝑐 = 2។ សមីោរមានរ ឺស ≤ 𝑥 ≤ 1 ។ 2

2

ខ) 𝑐 = 1។ សមីោរគ្នមនរ ឺស។ 2

គ) 𝑐 2 = 4។ សមីោរយៅជា 4𝑥 − 2 = 4, ⟹ 𝑥 = 3/2 ។

87. 5; 10  យយើ ងមាន 𝑥+3−4 𝑥−1= 𝑥−1−4 𝑥−1+4 =

𝑥−1

2

−4 𝑥−1+4=

𝑥+8−6 𝑥−1= 𝑥−1−6 𝑥−1+9=

𝑥−1−3

𝑥−1−2

2

ដូយចនេះសមីោរយៅជា 𝑥−1−2 ⟹

2

+

𝑥−1 −2 +

𝑥−1−3

2

=1

𝑥−1 −3 = 1

យបើ 𝑥 − 1 ≥ 3 យ េះ 𝑥−1 −2+ 𝑥−1 −3= 1 ⟹ 𝑥 = 10 យបើ 𝑥 − 1 − 2 ≥ 0 និង 𝑥 − 1 − 3 ≤ 0 ⟹ 5 ≤ 𝑥 ≤ 10យ េះ 𝑥−1 −2− 𝑥−1 +3= 1 ⟹1=1 យបើ 𝑥 − 1 − 2 ≤ 0 ⟹ 𝑥 ≤ 5 យ េះ − 𝑥−1 +2− 𝑥−1 +3 = 1 ⟹𝑥 =5

88. 0; 3  តាង 3𝑥 = 𝑥 + 2𝑥 = 𝑦12 𝑥 + 2𝑦1 = 𝑦22 𝑥 + 2𝑦2 = 𝑦32 …………… 2 𝑥 + 2𝑦𝑛−2 = 𝑦𝑛−1 𝑥 + 2𝑦𝑛−1 = 𝑦𝑛2 ដដល 𝑦1 , 𝑦2 , … , 𝑦𝑛 ជាចាំ នន ួ ព ិតវ ិជាមាន។ សមីោរដដលយោយមានរាង 𝑦𝑛 = 𝑥

លឹម សុ វណ្ណវិចិត្រ | III. សមីការ

81

2


យយើ ងនឹងបង្ហាញថា 𝑦1 = 𝑥 ។ សនមតថា 𝑥 > 𝑦1 ។ ដូយចនេះ 𝑦1 > 𝑦2 > ⋯ > 𝑦𝑛 ។ មានន័យថា យបើ 𝑥 > 𝑦1 យ េះ 𝑥 > 𝑦𝑛 ផទយ ុ ព ីសមីោរ 𝑥 = 𝑦𝑛 ។ ដូចគ្ននចាំ យ េះករណី 𝑥 < 𝑦1 ។ ដូយចនេះ 𝑥 = 𝑦1 ។ យោយ 𝑦12 = 3𝑥 យ េះ 3𝑥 = 𝑥 2 យយើ ងទញបាន 𝑥 = 0; 𝑥 = 3 ។

89.  សមីោរសមមូលនឹង 𝑥−4−1 +

𝑥−4−2 =𝑎

តាង 𝑦 = 𝑥 − 4 ≥ 0 ។ សមីោរយៅជា 𝑦 − 1 + 𝑦 − 2 = 𝑎 ។ យបើ 𝑎 = 0 យ េះ 𝑦 = 1 និង 𝑦 = 2 មិនោច។ ដូយចនេះ 𝑎 > 0 ។ យបើ 0 ≤ 𝑦 < 1 យ េះ 1 − 𝑦 + 2 − 𝑦 = 𝑎; ⟹ 𝑦 = 1 < 𝑎 ≤ 3 ។ យយើ ងទញបាន 𝑥 = 4 +

3−𝑎 2 2

3−𝑎 2

។ យោយ 0 ≤ 𝑦 =

3−𝑎 2

< 1 យ េះ

យបើ 1 ≤ 𝑦 ≤ 2 យ េះ 𝑦 − 1 − 𝑦 + 2 = 𝑎; ⟹ 𝑎 = 1 ។ 1 ≤ 𝑦 ≤ 2 ទ្តូវនឹង 5 ≤ 𝑥 ≤ 8 ។ យបើ 𝑦 > 2 យ េះ 𝑦 − 1 + 𝑦 − 2 = 𝑎; ⟹ 𝑦 = 𝑥 =4+ ជាសរុប

𝑎+3 2 2

𝑎+3 2

។ 𝑦 > 2 ⟹ 𝑎 > 1 ។ យយើ ងទញបាន

យបើ 𝑎 < 1 សមីោរគ្នមនរ ឺស។ យបើ 𝑎 = 1 សមីោរមានរ ឺស 5 ≤ 𝑥 ≤ 8 ។ 3−𝑎 2 2 𝑎+3 2 ។ 2 5

យបើ 1 < 𝑎 ≤ 3 សមីោរមានរ ឺស 𝑥 = 4 + យបើ 𝑎 > 3 សមីោរមានរ ឺស 𝑥 = 4 +

រ ឺ𝑥 = 4 +

𝑎+3 2 2

90.  តាង 𝑢 = 𝑎 − 𝑥; 𝑣 = 𝑥 − 𝑏 ។ ដូយចនេះ 𝑢 + 𝑣 5 = 𝑎 − 𝑏 5 ; 𝑢 + 𝑣 = 𝑎 − 𝑏 ។ យយើ ងមាន 𝑢5 + 𝑣 5 = 𝑢 + 𝑣 𝑢 + 𝑣 2 − 2𝑢𝑣 2 − 𝑢𝑣 𝑢 + 𝑣 2 + 𝑢2 𝑣 2 5 ⟺ 𝑎−𝑏 = 𝑎−𝑏 𝑎 − 𝑏 2 − 2𝑢𝑣 2 − 𝑢𝑣 𝑎 − 𝑏 2 + 𝑢2 𝑣 2 4 4 ⟺ 𝑎 − 𝑏 = 𝑎 − 𝑏 + 5𝑢2 𝑣 2 − 5𝑢𝑣 𝑎 − 𝑏 2 ⟺ 5 𝑢𝑣 2 − 5𝑢𝑣 𝑎 − 𝑏 2 = 0 ដូយចនេះ 𝑢𝑣 = 0 រ ឺ 𝑢𝑣 = 𝑎 − 𝑏 ។ ករណី 𝑢𝑣 = 0 យ េះ 𝑢 = 0 រ ឺ 𝑣 = 0 ។ យបើ 𝑢 = 0 យ េះ 𝑥 = 𝑎 ។យបើ 𝑣 = 0 យ េះ 𝑥 = 𝑏 ។ ករណី 𝑢𝑣 = 𝑎 − 𝑏 យ េះ យយើ ងមាន 𝑢+𝑣 = 𝑎−𝑏 𝑢𝑣 = 𝑎 − 𝑏 𝑢, 𝑣 ជារ ឺសននសមីោរ 𝑡 2 − 𝑎 − 𝑏 𝑡 + 𝑎 − 𝑏 = 0 គ្នមនរ ឺស។ ដូយចនេះសមីោរមានរ ឺសព ីរ គឺ 𝑥1 = 𝑎; 𝑥2 = 𝑏 ។

91.  តាង 82

១. អនុគមន៍ងាយ | លឹម សុ វណ្ណវិចិត្រ


𝑦=

𝑥+3 − 1 ⟹ 𝑦 ≥ 0; 𝑦 + 1 = 2

𝑥+3 2

⟹ 2𝑦 2 + 4𝑦 + 2 = 𝑥 + 3 ⟹ 𝑥 = 2𝑦 2 + 4𝑦 − 1 ដូយចនេះ ចាំ យ េះ 𝑥 ≥ −1 យយើ ងទញបានថា អនុគមន៍ 𝑦 = 2𝑥 2 + 4𝑥 − 1 ជាអនុគមន៍ទ្ចាស 𝑥+3 2

ននអនុគមន៍ 𝑦 =

− 1 មយ៉ាងវ ិញយទៀត អនុគមន៍ 𝑦 = 2𝑥 2 + 4𝑥 − 1 យកើនយលើ −1; ∞

(អនុគមន៍ទាំងព ីរឆាេះុ គ្ននយធៀបនឹងប ទ ត់ 𝑦 = 𝑥) ដូយចនេះ 2𝑥 2 + 4𝑥 − 1 =

𝑥+3 −1 2

⟺ 2𝑥 2 + 4𝑥 − 1 = 𝑥 ⟺ 2𝑥 2 + 3𝑥 − 1 = 0 −3 + 17 ⟹ 𝑥= 4

92.  យយើ ងមាន

𝑥4 + 𝑥2 + 1 = 𝑥2 + 𝑥 + 1 𝑥2 − 𝑥 + 1 > 0 𝑥 2 − 3𝑥 + 1 = 2 𝑥 2 − 𝑥 + 1 − 𝑥 2 + 𝑥 + 1 តាង 𝑡 =

𝑥 2 −𝑥+1 𝑥 2 +𝑥+1

។ សមីោរយៅជា 2𝑡 2 − 𝑚𝑡 − 1 = 0

ពី𝑡 =

𝑥 2 −𝑥+1 𝑥 2 +𝑥+1

1

យយើ ងទញបាន 𝑡2 𝑥2 + 𝑥 + 1 = 𝑥2 − 𝑥 + 1 𝑡 − 1 𝑥 2 + 𝑡 2 + 1 𝑥 + 𝑡 2 − 1 = 0 (2) Δ = 𝑡2 + 1 2 − 4 𝑡2 − 1 2 = 3 − 𝑡 2 3𝑡 2 − 1 ≥ 0 3 ⟹ ≤𝑡≤ 3 3 2

ក) ករណី 𝑚 = − 3/3 ព ី(១) យយើ ងទញបាន

𝑡1 = −

3

; 𝑡2 =

2 3 យយើ ងយក 𝑡 = 1/ 3 ។ ព ី(២) យយើ ងទញបាន 𝑥 = 1 ។

លឹម សុ វណ្ណវិចិត្រ | III. សមីការ

1 3

83


1 2

ខ) កនង ុ សមីោរ(១) យយើ ងមាន Δ = 𝑚2 + 8 > 0 ដូយចនេះ(១)មានរ ឺសព ីរ 𝑡1 ; 𝑡2 ។ យោយ 𝑡1 𝑡2 = − < 0 យ េះ 𝑡1 < 0 < 𝑡2 ។ មានដតរ ឺស 𝑡2 មួយគត់ ដដលោចយោយសមីោរ(២) មានរ ឺស យទ្ េះ 3 3

≤ 𝑡 ≤ 3។ សមីោរ(*) មានរ ឺសដតមួយគត់ យបើ សមីោរ(២)មានរ ឺសដតមួយគត់ ដដរ។

យបើ 𝑡 = 1 យ េះ (១) យាំ ោយ 𝑚 = 1 យហើ យ (២)

យាំ ោយ 𝑥 = 0 ។

យបើ 𝑡 2 − 1 ≠ 0 យ េះ Δ = 3 − 𝑡 2 3𝑡 2 − 1 = 0 ⟹ 𝑡1 = យបើ 𝑡 =

3 ; 3

3 ; 𝑡2 3

= 3។

3 ⟹𝑥=1 3 5 3 ⟹𝑥=1 3

1 ⟹𝑚=−

យបើ 𝑡 = 3; 1 ⟹ 𝑚 = ដូយចនេះ 𝑚 = −

3 ; 3 3

93.  តាង 𝑦 =

1;

5 3 3

។ 3

7𝑥 + 1; 𝑧 = − 𝑥 2 − 𝑥 − 8; 𝑡 = 𝑦+𝑧+𝑡 =2 ⟹ 𝑦+𝑧+𝑡

3

3

𝑥 2 − 8𝑥 − 1 ។ ដូយចនេះ

=8

(1)

យយើ ងមាន 𝑦 3 + 𝑧 3 + 𝑡 3 = 7𝑥 + 1 − 𝑥 2 − 𝑥 − 8 + 𝑥 2 − 8𝑥 − 1 = 8 (2) ព ី(១)និង(២) យយើ ងទញបាន 𝑦 + 𝑧 + 𝑡 3 − 𝑦3 + 𝑧3 + 𝑡3 = 3 𝑦 + 𝑧 𝑧 + 𝑡 𝑡 + 𝑦 = 0 𝑦 = −𝑧 𝑦+𝑧 =0 𝑧 + 𝑡 = 0 ⟹ ⟺ 𝑧 = −𝑡 𝑡 = −𝑦 𝑡+𝑦 =0 3

3

7𝑥 + 1 =

𝑥2 − 𝑥 − 8 =

3

3

3

3

𝑥2 − 𝑥 − 8

𝑥 2 − 8𝑥 − 1

⟺ 7𝑥 + 1 = 𝑥 2 − 𝑥 − 8 ⟺ 𝑥 = −1 ; 𝑥 = 9 ⟺ 𝑥 2 − 𝑥 − 8 = 𝑥 2 − 8𝑥 − 1 ⟺ 𝑥=1 ⟺ 7𝑥 + 1 = − 𝑥 2 − 8𝑥 − 1

7𝑥 + 1 = − 𝑥 2 − 8𝑥 − 1 ⟺ 𝑥 = 0; 𝑥 = 1 94.  តាង 𝑢 = 𝑥 + 1 ≥ 0; 𝑣 = 𝑥 2 − 𝑥 + 1 > 0 ។ សមីោរសមមូលនឹង 𝑢 𝑢 2 5𝑢𝑣 = 2 𝑣 2 + 𝑣 2 ⟺ 5 = 2 𝑣 𝑣 𝑢 =2 𝑣 ⟺ 𝑢 1 = 𝑣 2 𝑢 = 2 ⟺ 𝑥 + 1 = 2 𝑥2 − 𝑥 + 1 𝑣 84

១. អនុគមន៍ងាយ | លឹម សុ វណ្ណវិចិត្រ


𝑥 > −1 ⟺ : គ្នមនរ ឺស 4𝑥 2 − 5𝑥 + 3 = 0 𝑢 1 = ⟺ 2 𝑥 + 1 = 𝑥2 − 𝑥 + 1 𝑣 2 𝑥 > −1 5 ± 37 ⟺ 𝑥= 2 5± 37 𝑥= 2 ។

សមីោរមានរ ឺសព ីរ

ត្រព័នស ធ មីការ 4

95.

− 9 ; 20/9 ; 2; 1 . 96.

98.

−19,6 ; 5,2 ; −14; 8

71

−1; 3 ;

; −25/7 .97.

21

51; 24,5 .

. ោក់ អងគខាងយឆេងននសមីោរទីមួយជារាង 𝑥 + 3𝑦

2

6 𝑥 + 3𝑦 − 40 យហើ យតាង 𝑥 + 3𝑦 = 𝑡 ។

99. 2; 3 ; 3; 2 . 100. 4; −1 ; 1; −4 . 101. 1; 3 ; 3; 1 . 102. 1; 2 ; 2; 1 1 103. − 2 ; −1/3 ; 1/3; 1/2 .104. −1; 2 ; 2; −1 105. 3; 2 ; 2; 3 106. −1; −4 ; 4; 1 . 107.

−3; 4 ; 4; −3 . 108.

. 109. 0,6; 0,3 ; 0,4; 0,5 .

28; −5 − 28 ; 5; 2 ; −2; −5 1

5 + 28; −5 + 28 ; 5 −

1

110. −1; 2 ; − 4 ; − 4 111. 2; 1 ; 1; 2 ;

5+ 21 5− 21

;

2

មក −𝑢 + 𝑣 = 𝑧 និង 𝑢𝑣 = 𝑡 ។

112.

25+5 61 5+ 61 9

;

9

;

;

2

5− 21 5+ 21

;

2

−25+5 61 5− 61 9

;

9

. តាង 𝑥 + 𝑦 = 𝑢, 𝑥𝑦 = 𝑣 ប ទ ប់

2

; −6; −4/3 ; 3/2; 1/3 .  តាង

𝑥 = 𝑦𝑡 . 113.

−1; 3 ; 1; −3 ; 16/ 11; 1/ 11 ; − 16

114.

1; 2 ; −1; −2 ;

115.

3

116.

2; 1

2+ 19 14+4 19

;−3

11 ; −1/ 11 .

2; 2 ; − 2; − 2 . 3

14+4 19

;

−2+ 19 3

−14+4 19

;3

3 −14+4 19

; 2; −1 .

.  គុណសមីោរទីព ីរនឹង 2 រួចបូកចូលសមីោរទីមួយ ប ទ ប់ មកគុណសមីោរទីព ីរ

ដដដលនឹង −2 រួចបូកចូលសមីោរទីមួយចាស់ យយើ ងទញបានទ្បព័នថ ធ មី

លឹម សុ វណ្ណវិចិត្រ | IV. ត្រព័នស ធ មីការ

85


16𝑥 2 + 8𝑥𝑦 + 𝑦 2 − 72𝑥 − 18𝑦 + 81 = 0 4𝑥 2 − 12𝑥𝑦 + 9𝑦 2 − 4𝑥 + 6𝑦 + 1 = 0 4𝑥 + 𝑦 2 − 18 4𝑥 + 𝑦 + 81 = 0 ⟺ 2𝑥 − 3𝑦 2 − 2 2𝑥 − 3𝑦 + 1 = 0 4𝑥 + 𝑦 = 9 ⟺ 2𝑥 − 3𝑦 = 1

117.

0; 1/ 3 ; 0; −1/ 3 ; 1; 1 ; −1; −1 . 118.

119. 121. 122.

2; 6 ; 1; 3 .120. 2 2; − 2 ; −2 2; 2 . 6; 6/3 ; − 6; − 6/3 2; 1; −1 ; 31 15 ; 17 15 ; −2/3 . 123. 1; 2; 2 ; 2; 1; 1

124. 125.

3; −2; 1 ; −2; 3; 1 ; ; 2 ; −1 ; ; 2 2 2 3; 1; −2 ; −5; −3; 0 . 126. 3; 5; −1 ; −3; −5; 1

127.

2; −1; 3 ; −2; 1; −3 ; −

128. 130.

−4; −3; 1 ; 4; 3; −1 . 129. 1/2; 1/3; 1/4 −1; 1; 0 ; 1; −1; 0 . 131. 3; 3; 3 .

132.

16; 30 . 133.

135.

− 2 ; − 5 ; 5; 4 .

3+ 17 3− 17

1

2

−∞; 3 ;

; 13

41; 40 . 134.

5

;− 13

7

; −9/7 ; 1; 3 .

3− 17 3+ 17

11 13

;

7

5

;− 13

; −1 .

; 13

11 13

.

.

𝑐; 𝑐 − 1 |𝑐 ∈ ℝ ∖

1 2

.

2

3 109+9

136.

7

2

;

3 109−9 2

; −5; −3 . 137.

0; 𝑎 + 𝑎2 + 3

ចាំ យ េះ 𝑎 ∈

0; 𝑎 + 𝑎2 + 3 ; 0; −𝑎 − 𝑎2 − 3 ; 0; −𝑎 + 𝑎2 − 3

ចាំ យ េះ 𝑎 ∈

3; ∞ ។

138.

1; 1; 0 .  សមីោរទីមួយ យយើ ងទញបាន 𝑥 = 2 − 𝑦 ។ ជាំនស ួ ចូលសមីោរទីព ីរ យយើ ង

ទញបាន 2𝑦 − 𝑦 2 − 𝑧 2 = 1 ⟹ 𝑧2 + 𝑦 − 1 2 = 0 ⟹ 𝑧 = 0; 𝑦 = 1 1

139. − 4 ∪ 0; ∞ .

140. ក) 25.  គុណសមីោរទីមួយនឹង 𝛼 និងសមីោរទីព ីរនឹង 𝛽 ដដល 𝛼𝛽 ≠ 0 ប ទ ប់ មកបូកចូល គ្នន យយើ ងទញបាន៖ 2𝛼 4𝛼 5𝛼 +𝛽 𝑥+ +𝛽 𝑦+ + 𝛽 𝑧 = 61𝛼 + 79𝛽 3 5 6

86

១. អនុគមន៍ងាយ | លឹម សុ វណ្ណវិចិត្រ


យយើ ងយក 2 𝑦 5

𝑧

2𝛼 3

+ 𝛽 = 0;

4𝛼 5

2 5

+𝛽 = ;

5𝛼 6

1 2

+ 𝛽 = ។ យយើ ងទញបាន 𝛼 = 3, 𝛽 = −2 ។ ដូយចនេះ

+ 2 = 61𝛼 + 79𝛽 = 15 ។

ខ) (27; 10; 42) .  យោយារ 𝑥, 𝑦, 𝑧 ជាចាំ នន ួ គត់ ធមម ជាតិ យ េះ 𝑥 = 3𝑘; 𝑦 = 5𝑙; 𝑧 = 6𝑚 ដដល 𝑘; 𝑙; 𝑚 ∈ ℕ ។ ទ្បព័នស ធ មីោរសមមូលនឹង 4𝑙 + 5𝑚 = 61 − 2𝑘 5𝑙 + 6𝑚 = 79 − 3𝑘 យយើ ងទញបាន 𝑙 = 29 − 3𝑘 និង 𝑚 = 2𝑘 − 11 ។ យោយ 𝑘; 𝑙; 𝑚 ∈ ℕយ េះ 𝑘>0 29 − 3𝑘 > 0 2𝑘 − 11 > 0 យយើ ងទញបាន 𝑘 = 9 ។

141. ចាំ យ េះ 𝑎 = 0 យយើ ងទញបាន 𝑥, 𝑦, 𝑧 = 0,0,0 ។ ចាំ យ េះ 𝑎 ≠ 0 យយើ ងទញបាន 𝑥, 𝑦, 𝑧 ∈

𝑡1 , 𝑡2 , 𝑧0 ; 𝑡2 , 𝑡1 , 𝑧0

ដដល

𝑎2 − 𝑏 2 𝑎2 + 𝑏 2 ± 3𝑎2 − 𝑏 2 3𝑏2 − 𝑎2 ; 𝑡1,2 = 2𝑎 4𝑎 យដើមបីយោយចាំ យលើយវ ិជាមាននិងខុសគ្នន លកខខណឌចា​ាំបាច់ នឹងទ្គប់ ទ្គ្នន់ គ ឺ 3𝑏 2 > 𝑎2 > 𝑏 2 និង 𝑎 > 0 𝑧0 =

142. បូកសមីោរទាំងអស់ បញ្ចូលគ្នន យយើ ងទញបាន 2 𝑥1 + 𝑥2 + 𝑥3 + 𝑥4 + 𝑥5 = 𝑦 𝑥1 + 𝑥2 + 𝑥3 + 𝑥4 + 𝑥5 ។ យបើ 𝑦 = 2 យ េះយយើ ងទញបាន 𝑥1 − 𝑥2 = 𝑥2 − 𝑥3 = ⋯ = 𝑥5 − 𝑥1 ដូយចនេះ 𝑥1 = 𝑥2 = ⋯ = 𝑥5 ជាចាំ យលើយរបស់ ទ្បព័ន។ ធ យបើ 𝑦 ≠ 2 យ េះ 𝑥1 + 𝑥2 + 𝑥3 + 𝑥4 + 𝑥5 = 0។ បូកសមីោរទីបីខាងយដើមចូលគ្នន យយើ ងទញបាន 𝑥2 = 𝑦 𝑥1 + 𝑥2 + 𝑥3 ។យោយ 𝑥1 + 𝑥3 = 𝑦𝑥2 យ េះ 𝑥2 = 𝑦 2 + 𝑦 𝑥2 , ⟹ 𝑦 2 + 𝑦 − 1 𝑥2 = 0 ។ យបើ 𝑦 2 + 𝑦 − 1 ≠ 0 យ េះ 𝑥2 = 0 ដូចគ្ននយយើ ងទញបាន 𝑥1 = 𝑥2 = ⋯ = 𝑥5 = 0 ។ យបើ 𝑦 2 + 𝑦 − 1 = 0 យ េះ បូកសមីោរទី មួយនិងទីព ីរ យយើ ងទញបាន 𝑥5 + 𝑥2 + 𝑥1 + 𝑥3 = 𝑦𝑥1 + 𝑦𝑥2 ⟹ 𝑥3 + 𝑥5 = 𝑦𝑥1 + 𝑦𝑥2 − 𝑥2 − 𝑥1 ⟹ 𝑥3 + 𝑥5 = −𝑦 2 𝑥2 − 𝑦 2 𝑥1 ⟹ 𝑥3 + 𝑥5 = −𝑦 𝑥1 + 𝑥3 − 𝑦 𝑥5 + 𝑥2 ⟹ 𝑥3 + 𝑥5 = 𝑦𝑥4 ។ ដូចគ្នន យយើ ងទញបានសមីោរទី៥។ មានន័យថា សមីោរទី៤និងទី៥ ោស្ស័ យលីយនដអ៊ែ រ នឹងសមីោរបីខាងយលើ។ យក 𝑥1 = 𝑢; 𝑥5 = 𝑣 ។ យយើ ងទញបាន 𝑥2 = 𝑦𝑢 − 𝑣; 𝑥3 = 𝑦 2 𝑢 − 𝑦𝑣 − 𝑢; 𝑥4 = 𝑦 3 𝑢 − 𝑦 2 𝑣 − 2𝑦𝑢 + 𝑣 ។

143. សនមតថា 𝑥1 , 𝑥2 , 𝑥3 ជាចាំ យលើយរបស់ ទ្បព័ន។ ធ យយើ ងោចសនមតថា 𝑥1 ≥ 𝑥2 ≥ 𝑥3 ។ សនមតថា 𝑥1 > 0 ។ ព ីសមីោរទីមួយ យយើ ងទញបាន លឹម សុ វណ្ណវិចិត្រ | IV. ត្រព័នស ធ មីការ

87


đ?‘Ľ2 đ?‘Ľ3 + đ?‘Ž13 ≼ đ?‘Ľ1 . đ?‘Ž11 − đ?‘Ž12 − đ?‘Ž13 đ?‘Ľ1 đ?‘Ľ1 យវá&#x;† á&#x;„áž™ đ?‘Ľ1 = đ?‘Ľ2 = đ?‘Ľ3 = 0 á&#x;”

0 = đ?‘Ľ1 . đ?‘Ž11 + đ?‘Ž12 áž˜ážˇáž“á&#x;„áž…á&#x;” ដសយចនá&#x; á&#x;‡ đ?‘Ľ1 = 0

>0

144.ď ° áž™áž”ážžáž˜ážśáž“áž…ážśá&#x;† áž“áž“ áž˝ á&#x;’áž˜áž˝áž™áž™áž&#x;មឞ áž&#x;ážźáž“á&#x;‚ áž§áž”áž˜ážśáž?ážś đ?‘Ľ1 = 0 áž™ á&#x; á&#x;‡ đ?‘Ľ1 + đ?‘Ľ2 đ?‘Ľ3 đ?‘Ľ4 = 2 â&#x;š đ?‘Ľ2 đ?‘Ľ3 đ?‘Ľ4 = 2; đ?‘Ľ2 + đ?‘Ľ1 đ?‘Ľ3 đ?‘Ľ4 = 2 â&#x;š đ?‘Ľ2 = 2; ‌ ; đ?‘Ľ3 = đ?‘Ľ4 = 2 áž˜ážˇáž“á&#x;„áž…á&#x;” ដសយចនá&#x; á&#x;‡áž‚á&#x;’áž“áž˜áž“áž…ážśá&#x;† áž“áž“ áž˝ á&#x;’áž˜áž˝áž™á&#x;„áž…áž™áž&#x;មឞ đ?‘?

áž&#x;ážź áž“á&#x;‚បវនយទá&#x;” áž?ážśáž„ đ?‘Ľ1 đ?‘Ľ2 đ?‘Ľ3 đ?‘Ľ4 = đ?‘? á&#x;” áž&#x;មម áž?ក ឡ មមáž&#x;áž˜áž˜ážźáž›áž“ážšáž„ đ?‘Ľđ?‘– + đ?‘Ľ = 2, đ?‘– = 1,2,3,4 á&#x;” áž&#x;មីá&#x;„ážš đ?‘–

đ?‘? đ?‘Ľ

đ?‘Ľ + = 2 áž˜ážśáž“ážš ážşáž&#x;áž™á&#x;‰ážśáž„យទá&#x;’áž…ážžáž“áž…ážśá&#x;† áž“áž“ áž˝ áž– ីរáž?ážśáž„áž™á&#x;„áž™ đ?‘Ś នឡង đ?‘§ á&#x;” ដសយចនá&#x; á&#x;‡ đ?‘Ľđ?‘– áž“ážˇáž˜áž˝áž™á&#x;—á&#x;„áž…áž™áž&#x;មឞ đ?‘Ś ážš ážşáž™áž&#x;មឞ đ?‘§ á&#x;” យយឞ áž„áž˜ážśáž“áž”áž¸áž€ážšážŽáž¸á&#x;– đ?‘Ľ1 = đ?‘Ľ2 = đ?‘Ľ3 = đ?‘Ľ4 = đ?‘Ś á&#x;” ដសយចនá&#x; á&#x;‡ đ?‘Ś + đ?‘Ś 3 = 2

យវá&#x;† á&#x;„áž™ đ?‘Ś = 1 á&#x;”

đ?‘Ľ1 = đ?‘Ľ2 = đ?‘Ľ3 = đ?‘Ś, đ?‘Ľ4 = đ?‘§ á&#x;” ដសយចនá&#x; á&#x;‡ đ?‘§ + đ?‘Ś 3 = đ?‘Ś + đ?‘Ś 2 đ?‘§ = 2 á&#x;” យយឞ ងទញបវន đ?‘Ś; đ?‘§ = −1; 3 ; 1; 2 đ?‘Ľ1 = đ?‘Ľ2 = đ?‘Ś, đ?‘Ľ3 = đ?‘Ľ4 = đ?‘§ á&#x;” ករណីយនá&#x; á&#x;‡áž™áž™ážž ងទញបវន đ?‘Ś = đ?‘§ = 1 á&#x;” ដសយចនá&#x; á&#x;‡ážš ážşáž&#x; đ?‘Ľ1 ; đ?‘Ľ2 ; đ?‘Ľ3 ; đ?‘Ľ4 áž‚ážş

1; 1; 1; 1 ; −1; −1; −1; 3

នឡងចវá&#x;† លវáž&#x;á&#x;‹ ážšáž”áž&#x;á&#x;‹ ážœážśá&#x;”

145.ď ° áž?ážśáž„ đ??ż1 = đ?‘Ž1 − đ?‘Ž2 đ?‘Ľ2 + đ?‘Ž1 − đ?‘Ž3 đ?‘Ľ3 + đ?‘Ž1 − đ?‘Ž4 đ?‘Ľ4 នឡងដសចគá&#x;’áž“áž“áž…ážśá&#x;† áž™ á&#x; á&#x;‡ đ??ż2 , đ??ż3 នឡង đ??ż4 á&#x;” áž&#x;នមáž?áž?ážś đ?‘Ž1 < đ?‘Ž2 < đ?‘Ž3 < đ?‘Ž4 á&#x;” áž™á&#x;…កនង ážť ករណីយនá&#x; á&#x;‡ 2 đ?‘Ž1 − đ?‘Ž2 đ?‘Ž2 − đ?‘Ž3 đ?‘Ľ2 = đ?‘Ž3 − đ?‘Ž2 đ??ż1 − đ?‘Ž1 − đ?‘Ž3 đ??ż2 + đ?‘Ž1 − đ?‘Ž2 đ??ż3 = đ?‘Ž3 − đ?‘Ž2 − đ?‘Ž1 − đ?‘Ž3 + đ?‘Ž1 − đ?‘Ž2 = 0 2 đ?‘Ž2 − đ?‘Ž3 đ?‘Ž3 − đ?‘Ž4 đ?‘Ľ3 = đ?‘Ž4 − đ?‘Ž3 đ??ż2 − đ?‘Ž2 − đ?‘Ž4 đ??ż3 + đ?‘Ž2 − đ?‘Ž3 đ??ż4 = đ?‘Ž4 − đ?‘Ž3 − đ?‘Ž2 − đ?‘Ž4 + đ?‘Ž2 − đ?‘Ž3 = 0 ដសយចនá&#x; á&#x;‡ យយឞ ងទញបវន đ?‘Ľ2 = đ?‘Ľ3 = 0 យហឞ áž™ យវá&#x;† á&#x;„áž™ đ?‘Ľ1 = đ?‘Ľ4 = 1/ đ?‘Ž1 − đ?‘Ž4 á&#x;” ážš ážşáž&#x;យនá&#x; á&#x;‡áž™áž•áž‘á&#x;€áž„áž•á&#x;’áž‘áž‘áž?á&#x;‹ áž‘á&#x;’បពá&#x;?áž“áž&#x; áž’ មីá&#x;„រដដលយá&#x;„áž™á&#x;” áž…ážśá&#x;† យលឞយយផá&#x; ងយទá&#x;€áž?ជវចវá&#x;† លវáž&#x;á&#x;‹ áž“áž“áž…ážśá&#x;† áž™áž›ážžáž™áž˜áž˝áž™áž™áž“á&#x; á&#x;‡á&#x;”

146.ď ° áž™áž”ážžáž˜áž˝áž™áž€áž“áž„ ážť áž…ážśá&#x;† áž™ ម đ?‘Ľ, đ?‘Ś, đ?‘§ áž™áž&#x;មឞ 1 ážš ážş −1 áž™ á&#x; á&#x;‡ −1, −1, −1 នឡង 1,1,1 á&#x;” áž‘á&#x;’បពá&#x;?áž“áž‚á&#x;’áž“ áž’ មន áž…ážśá&#x;† យលឞយយផá&#x; áž„áž– ីយនá&#x; á&#x;‡áž™áž‘á&#x;” áž?ážśáž„ đ?‘“ đ?‘Ą = đ?‘Ą 2 + đ?‘Ą − 1 á&#x;” យបឞកង áž“ážť áž…ážśá&#x;† áž™ ម đ?‘Ľ, đ?‘Ś, đ?‘§ áž˜ážśáž“áž˜áž˝áž™áž’ážśá&#x;† ជវង 1 áž&#x;នមáž? áž?ážś đ?‘Ľ > 1 យយឞ áž„áž˜ážśáž“ đ?‘Ľ < đ?‘“ đ?‘Ľ = đ?‘Ś < đ?‘“ đ?‘Ś = đ?‘Ś < đ?‘“ đ?‘Ś = đ?‘§ < đ?‘“ đ?‘§ = đ?‘Ľ áž˜ážˇáž“á&#x;„áž…á&#x;” ដសយចនá&#x; á&#x;‡ đ?‘Ľ, đ?‘Ś, đ?‘§ ≤ 1 á&#x;” 5

ូលស ážœáž&#x;នមáž?áž?ážś áž˜ážśáž“áž˜áž˝áž™áž€áž“áž„ ážť áž…ážśá&#x;† áž™ ម đ?‘Ľ, đ?‘Ś, đ?‘§ áž&#x;នមáž?áž?ážś đ?‘Ľ áž˜ážśáž“áž?ážśá&#x;† áž“áž›áž?សចជវង −1 á&#x;” áž™á&#x;„áž™ min đ?‘“ = − 4 5

áž™ á&#x; á&#x;‡ យយឞ áž„áž˜ážśáž“ đ?‘Ľ = đ?‘“ đ?‘§ ∈ − 4 ; −1 á&#x;” áž™á&#x;„áž™ đ?‘“ នឡង đ?‘“ −1; 0

5

− 4 ; −1

= −1; −11/16 ⊆ −1; 0

5 4

5 4

= − ; −1 ដសយចនá&#x; á&#x;‡ đ?‘Ś = đ?‘“ đ?‘Ľ ∈ −1; 0 , đ?‘§ = đ?‘“ đ?‘Ś ∈ − ; −1 នឡង

đ?‘Ľ = đ?‘“ đ?‘§ ∈ −1,0 áž•áž‘áž™ ážť áž– ីá&#x;„ážšáž&#x;នមáž?á&#x;” ដសយចនá&#x; á&#x;‡ −1 ≤ đ?‘Ľ, đ?‘Ś, đ?‘§ ≤ 1 á&#x;” យបឞ −1 < đ?‘Ľ, đ?‘Ś, đ?‘§ < 1 áž™ á&#x; á&#x;‡ đ?‘Ľ > đ?‘“ đ?‘Ľ = đ?‘Ś > đ?‘“ đ?‘Ś = đ?‘§ > đ?‘“ đ?‘§ = đ?‘Ľ áž˜ážˇáž“á&#x;„áž…á&#x;” 88

á&#x;Ą. áž˘áž“ážťáž‚áž˜áž“á&#x;?áž„ážśáž™ | លរម áž&#x;ážť វណá&#x;’ážŽážœážˇáž…ážˇáž?á&#x;’ážš


147. −2; −1 ; −

14 13 3

;

3

.

148. តាង 𝑆𝑘 = 𝑥1𝑘 + 𝑥2𝑘 + ⋯ + 𝑥𝑛𝑘 និង តាង 𝜎𝑘 , 𝑘 = 1,2, … , 𝑛 ជាពហុ ធាសុី យមទ្ទីបថមទី𝑘 នន 𝑥1 , 𝑥2 , … , 𝑥𝑛 ។ ពហុ ធាសុី យមទ្ទីបថមទី𝑘 ជា 𝜎𝑘 𝑥1 , 𝑥2 , … , 𝑥𝑛 = ∑𝑥𝑖1 𝑥𝑖2 … 𝑥𝑖𝑘 ដដល 𝑖1 , 𝑖2 , … , 𝑖𝑘 ∈ 1,2, … , 𝑛 ។ ឧទហរណ៍ 𝜎1 𝑥1 , 𝑥2 , 𝑥3 = 𝑥1 + 𝑥2 + 𝑥3 ; 𝜎2 𝑥1 , 𝑥2 , 𝑥3 = 𝑥1 𝑥2 + 𝑥2 𝑥3 + 𝑥3 𝑥1 ; 𝜎3 𝑥1 , 𝑥2 , 𝑥3 = 𝑥1 𝑥2 𝑥3 ។ ទ្បព័នស ធ មីោរដដលយោយ 𝑆𝑘 = 𝑎𝑘 , 𝑘 = 1,2, … , 𝑛 ។ តាមរូបមនតញវូ តុន យយើ ងទញបាន

𝑘𝜎𝑘 = 𝑆1 𝜎𝑘−1 − 𝑆2 𝜎𝑘−2 + ⋯ + −1 𝑘 𝑆𝑘−1 𝜎1 + −1 𝑘−1 𝑆𝑘 , 𝑘 = 1,2, … , 𝑛 𝜎1 = 𝑎 𝑘𝜎2 = 𝑆1 𝜎1 + −1 1 𝑆2 = 𝑎. 𝑎 − 𝑎2 = 0, 𝜎2 = 0 … ⟹ 𝜎1 = 𝑎; 𝜎𝑘 = 0 ចាំ យ េះ 𝑘 = 2, … , 𝑛 តាមទ្ទឹសតីបទដវែត 𝑥1 , 𝑥2 , … , 𝑥𝑛 ជារ ឺសននពហុ ធា 𝑥 𝑛 − 𝑎𝑥 𝑛−1 មានន័យថា រ ឺសទាំងយ េះយសមើ នង ឹ 𝑎, 0, … , 0 និងចាំ លាស់ ទាំងអស់ ។

149. យបើ 𝑚 ∉ −2; 1 យ េះទ្បព័នមា ធ នរ ឺសដតមួយគត់ 𝑏+𝑎− 1+𝑚 𝑐 𝑎+𝑐− 1+𝑚 𝑏 𝑏+𝑐− 1+𝑚 𝑎 ;𝑦 = ;𝑧 = 2+𝑚 1−𝑚 2 + 𝑚 (1 − 𝑚) 2 + 𝑚 (1 − 𝑚) ចាំ នន ួ 𝑥, 𝑦, 𝑧 ជាសេុី តនព េនត យបើនង ិ មានដតយបើ 𝑎, 𝑏, 𝑐 ជាសេុី តនព េនតដដរ។ 𝑥=

យបើ 𝑚 = 1 យ េះទ្បព័នមា ធ នចាំ យលើយ យបើនង ិ មានដតយបើ 𝑎 = 𝑏 = 𝑐។ ករណី 𝑚 = −2 ទ្បព័នមា ធ ន ចាំ យលើយ យបើនង ិ មានដតយបើ 𝑎 + 𝑏 + 𝑐 = 0 ។ យៅកនង ុ ករណីទាំងព ីរយនេះ ទ្បព័នមា ធ នចាំ យលើយយទ្ចើនរាប់ មិនអស់ ។

150. យយើ ងមាន 𝑐𝑛 > 0 ចាំ យ េះទ្គប់ 𝑛 គូ។ ដូយចនេះ 𝑐𝑛 = 0 ោចបានដតចាំ យ េះ 𝑛 យសសដត ប៉ា ុយ្ណេះ។ សនមតថា 𝑎1 ≤ 𝑎2 ≤ ⋯ ≤ 𝑎8 យហើ យថា 𝑎1 ≤ 0 ≤ 𝑎8 ។ យបើ 𝑎1 < 𝑎8 យ េះមាន 𝑛0 ដដលចាំ យ េះទ្គប់ ចាំ នន ួ យសស 𝑛 > 𝑛0 យយើ ងមាន 7 𝑎1

𝑛

< 𝑎8𝑛 ។ ដូយចនេះ

𝑎1𝑛 + 𝑎2𝑛 + ⋯ + 𝑎8𝑛 > 7𝑎1𝑛 + 𝑎8𝑛 > 0 ផទយ ុ ព ីលកខខណឌដដលថា 𝑐𝑛 = 0 ចាំ យ េះតាំ នល 𝑛 យទ្ចើនរាប់ មិនអស់ ។ ដូចគ្ននករណី 𝑎1 > 𝑎8 ក៏មិនោចដដរ។ យយើ ងទញបាន 𝑎1 = −𝑎8 ។ តាមរយបៀបដូច គ្នន យយើ ងទញបាន 𝑎2 = −𝑎7 ; 𝑎3 = −𝑎6 និង 𝑎4 = −𝑎5 ។ យហើ យ 𝑐𝑛 = 0 ចាំ យ េះទ្គប់ ចាំ នន ួ យសស 𝑛។

151. តាង 𝑋 = 1 − 𝑥; 𝑌 = 1 − 𝑦; 𝑍 = 1 − 𝑧; 𝑇 = 1 − 𝑡 ។ យយើ ងទញបាន

លឹម សុ វណ្ណវិចិត្រ | IV. ត្រព័នស ធ មីការ

89


đ?‘‹2 = đ?‘Œ đ?‘Œ2 = đ?‘? đ?‘?2 = đ?‘‡ đ?‘‡2 = đ?‘‹ 4 2 8 2 យយឞ ងទញបវន đ?‘‹ = đ?‘Œ = đ?‘? â&#x;š đ?‘‹ = đ?‘? = đ?‘‡ â&#x;š đ?‘‹16 = đ?‘‡ 2 = đ?‘‹ â&#x;š đ?‘‹ đ?‘‹15 − 1 = 0 á&#x;” យយឞ ងទញបវន - đ?‘‹ = 0; â&#x;š đ?‘Œ = đ?‘? = đ?‘‡ = 0 â&#x;š đ?‘Ľ = đ?‘Ś = đ?‘§ = đ?‘Ą = 1 - đ?‘‹ = 1; â&#x;š đ?‘Œ = đ?‘? = đ?‘‡ = 1 â&#x;š đ?‘Ľ = đ?‘Ś = đ?‘§ = đ?‘Ą = 0

152.ď ° បសកáž&#x;មីá&#x;„ážšáž‘ážśá&#x;†áž„អáž&#x;á&#x;‹ បញá&#x;’ចសលគá&#x;’áž“áž“ យយឞ ងទញបវន 1000

đ?‘Ľđ?‘–2

0= đ?‘–=1

đ?‘Žâˆ’1 đ?‘Žâˆ’1 +2 đ?‘Ľđ?‘– + 2 2

យយឞ ងទញបវន �1 = �2 = ⋯ = �1000 =

đ?‘Žâˆ’1 2

1000

2

=

á&#x;”

đ?‘Ľđ?‘– + đ?‘–=1

đ?‘Žâˆ’1 2

2

153.ď ° áž?ážśáž„ đ?‘Ą1 = đ?‘Ľ1 − đ?‘Ľ2 ; đ?‘Ą2 = đ?‘Ľ2 − đ?‘Ľ3 ; ‌ ; đ?‘Ąđ?‘› = đ?‘Ľđ?‘› − đ?‘Ľ1 á&#x;” áž‘á&#x;’បពá&#x;?áž“áž&#x; áž’ មីá&#x;„ážšáž&#x;áž˜áž˜ážźáž›áž“ážšáž„ 2đ?‘Ą1 = 3đ?‘Ą2 2đ?‘Ą2 = 3đ?‘Ą3 ‌‌ 2đ?‘Ąđ?‘› = 3đ?‘Ą1 គឝណអងគនង ážš អងគននáž&#x;មីá&#x;„ážš យយឞ ងទញបវន 2đ?‘› đ?‘Ą1 đ?‘Ą2 ‌ đ?‘Ąđ?‘› = 3đ?‘› đ?‘Ą1 đ?‘Ą2 ‌ đ?‘Ąđ?‘› â&#x;š đ?‘Ą1 đ?‘Ą2 ‌ đ?‘Ąđ?‘› = 0á&#x;” ដសយចនá&#x; á&#x;‡ áž‘á&#x;’áž?ážźážœáž˜ážśáž“ đ?‘Ąđ?‘– áž˜áž˝áž™ážŠážŠáž›áž™áž&#x;មឞ áž&#x;ážźáž“á&#x;‚ á&#x;” យហឞ យយá&#x;„áž™ 2đ?‘Ąđ?‘– = 3đ?‘Ąđ?‘–+1 áž™ á&#x; á&#x;‡ យយឞ ងទញបវន đ?‘Ą1 = đ?‘Ą2 = â‹Ż = đ?‘Ąđ?‘› = 0 យហឞ áž™ đ?‘Ľ1 = đ?‘Ľ2 = â‹Ż = đ?‘Ľđ?‘› á&#x;”

154.ď ° យយឞ áž„áž˜ážśáž“ 3đ?‘† = đ?‘Ľ 2 + đ?‘Ś 2 + đ?‘§ 2 ≼ 0 â&#x;š đ?‘† = đ?‘Ľ + đ?‘Ś + đ?‘§ ≼ 0 á&#x;” ដសយចនá&#x; á&#x;‡áž€áž“áž„ ážť áž…ážśá&#x;† áž™ ម đ?‘Ľ, đ?‘Ś, đ?‘§ áž‘á&#x;’áž?ážźážœážŠáž?áž˜ážśáž“áž˜áž˝áž™áž‡ážśáž…ážśá&#x;† áž“áž“ áž˝ áž˜ážˇáž“áž˘ážœ ážˇáž‡ážśáž˜ážśáž“á&#x;” áž&#x;នមáž?áž?ážś đ?‘Ľ ≼ 0 á&#x;” យយឞ ងទញបវន đ?‘Ś 4−đ?‘Ś =đ?‘Ľ ≼0 â&#x;š0≤đ?‘Ś ≤4 đ?œ‹ ដសចគá&#x;’ននយយឞ ងទញបវន 0 ≤ đ?‘Ľ, đ?‘Ś, đ?‘§ ≤ 4 á&#x;” áž?ážśáž„ đ?‘Ľ = 4 sin2 đ?›ź ដដល 0 ≤ đ?›ź ≤ 2 á&#x;” យយឞ ងទញបវន đ?‘§ = 4 sin2 2đ?›ź ; đ?‘Ś = 4 sin2 4đ?›ź á&#x;” យហឞ áž™ đ?‘Ľ = 4 sin2 8đ?›ź á&#x;” ដសយចនá&#x; á&#x;‡ 4 sin2 8đ?›ź = 4 sin2 đ?›ź â&#x;š cos 16đ?›ź = cos 2đ?›ź យវá&#x;† á&#x;„áž™ 1) 16đ?›ź = 2đ?›ź + 2đ?‘˜đ?œ‹ â&#x;š đ?›ź = -đ?‘˜=0

đ?‘˜đ?œ‹ 7

đ?‘˜âˆˆâ„¤ ; đ?œ‹ 0 ≤ đ?›ź ≤ â&#x;š đ?‘˜ = {0; 1; 2; 3} 2 đ?‘Ľ = đ?‘Ś = 0; â&#x;š đ?‘† = 0

-đ?‘˜ = 1; 2; 3 90

á&#x;Ą. áž˘áž“ážťáž‚áž˜áž“á&#x;?áž„ážśáž™ | លរម áž&#x;ážť វណá&#x;’ážŽážœážˇáž…ážˇáž?á&#x;’ážš


đ?œ‹ 2đ?œ‹ 3đ?œ‹ + sin2 +sin2 =7 7 7 7 đ?‘˜đ?œ‹ 2) 16đ?›ź = −2đ?›ź + 2đ?‘˜đ?œ‹ â&#x;š đ?›ź = 9 đ?‘˜ ∈ ℤ ; đ?œ‹ 0 ≤ đ?›ź ≤ â&#x;š đ?‘˜ = {0; 1; 2; 3; 4} 2 -đ?‘˜=0 â&#x;š đ?‘† = 4 sin2

đ?‘Ľ = đ?‘Ś = 0; â&#x;š đ?‘† = 0 - đ?‘˜ = 1; 2; 4 â&#x;š đ?‘† = 4 sin2 -đ?‘˜=3

đ?œ‹ 2đ?œ‹ 4đ?œ‹ + sin2 +sin2 =6 9 9 9

đ?œ‹ 2đ?œ‹ 4đ?œ‹ + sin2 +sin2 =9 3 3 3 1; 1; 1 ; −2; −2; −2 ď °áž™áž€áž&#x;មីá&#x;„ážšáž‘áž¸áž˜áž˝áž™ážŠáž€áž&#x;មីá&#x;„រទីព ីរ áž&#x;មីá&#x;„រទីព ីរដកáž&#x;មីá&#x;„ážš â&#x;š đ?‘† = 4 sin2

155.

ទីបី យយឞ ងទញបវន đ?‘Ľ − đ?‘Ś 1 − đ?‘§ = 0; (1) đ?‘Ś − đ?‘§ 1 − đ?‘Ľ = 0 (2) 1 â&#x;š đ?‘Ľ = đ?‘Ś ážš ážşđ?‘§ = 1 2 â&#x;š đ?‘Ś = đ?‘§ ážš ážşđ?‘Ľ = 1 យយឞ ងទញបវនបួនករណី đ?‘Ľ = đ?‘Ś = đ?‘§ ; đ?‘Ľ = đ?‘Ś; đ?‘Ľ = 1 ; đ?‘§ = 1; đ?‘Ś = đ?‘§ ; đ?‘§ = 1; đ?‘Ľ = 1 ករណីទវá&#x;†áž„បួនយនá&#x; á&#x;‡ áž‘á&#x;’áž?ážźážœáž“ážšáž„ đ?‘Ľ = đ?‘Ś = đ?‘§ = 1 ážš ážş đ?‘Ľ = đ?‘Ś = đ?‘§ = −2 á&#x;”

156. ď ° យយឞ áž„áž&#x;ឡ កវករណី đ?‘Ž > 0 យទá&#x;’ á&#x; á&#x;‡ đ?‘Ž ≠0 á&#x;” ករណី đ?‘Ž < 0 គឝណអងគáž&#x;មីá&#x;„ážšáž“ážšáž„áž&#x;ញá&#x;’ជវដកá&#x;” បសកáž&#x;មីá&#x;„ážšáž‘ážśá&#x;†áž„បីននទá&#x;’បពá&#x;?áž“áž” áž’ ញá&#x;’áž…ážź លគá&#x;’áž“áž“ យយឞ ងទញបវន đ?‘Žđ?‘Ľ 2 + đ?‘? − 1 đ?‘Ľ + đ?‘? + đ?‘Žđ?‘Ś 2 + đ?‘? − 1 đ?‘Ś + đ?‘? + đ?‘Žđ?‘§ 2 + đ?‘? − 1 đ?‘§ + đ?‘? = 0 áž?ážśáž„ đ?‘“ đ?‘Ą = đ?‘Žđ?‘Ą 2 + đ?‘? − 1 đ?‘Ą + đ?‘? á&#x;” យយឞ áž„áž˜ážśáž“ យបឞ Δ = đ?‘? − 1

2

đ?‘“ đ?‘Ľ +đ?‘“ đ?‘Ś +đ?‘“ đ?‘§ =0 (∗) − 4đ?‘Žđ?‘? < 0 áž™ á&#x; á&#x;‡ đ?‘“ đ?‘Ą > 0 áž…ážśá&#x;† áž™ á&#x; á&#x;‡áž‘á&#x;’áž‚áž”á&#x;‹ đ?‘Ą ∈ â„? យទá&#x;’ á&#x; á&#x;‡ đ?‘Ž > 0á&#x;” ដសយចនá&#x; á&#x;‡ đ?‘“ đ?‘Ľ +đ?‘“ đ?‘Ś +đ?‘“ đ?‘§ >0

áž•áž‘áž™ ážť áž– ីáž&#x;មីá&#x;„ážš(*)á&#x;”

157. ď ° ជវá&#x;†áž“áž&#x; áž˝ đ?‘Ś = −đ?‘Ľ ចសលកនង ážť áž‘á&#x;’បពá&#x;?áž“ áž’ យយឞ ងទញបវន 1 đ?‘Ž+1 2 2đ?‘Ľ 3 − đ?‘Žđ?‘Ľ 3 = 1 đ?‘Ľ 3 + đ?‘Žđ?‘Ľ 3 =

លរម áž&#x;ážť វណá&#x;’ážŽážœážˇáž…ážˇáž?á&#x;’ážš | IV. áž?á&#x;’ážšáž–á&#x;?áž“áž&#x; áž’ áž˜áž¸áž€ážśážš

2

(1) (2) 91


សមីោរទី២ យយើ ងទញបាន 𝑥 ≠ 0 ។ បូកសមីោរទាំងព ីរចូលគ្នន យយើ ងទញបាន 1 3𝑥 3 = 𝑎 + 1 2 + 1 > 0; ⟹ 𝑥 > 0 2 យកសមីោរទី១ដចកនឹងសមីោរទី២ យយើ ងទញបាន 𝑎+1 1 = 𝑎 + 1 2 ⟹ 𝑎 = 0; −1; 1 2−𝑎 2 ចាំ យ េះ 𝑎 = 0 យយើ ងមាន 1 1 𝑥=3 3 𝑥 = 2 ⟺ 2 1 3 2 𝑥 + 𝑥𝑦 = 1 𝑦=±3 2 1 1 រ ឺសដដលយផទៀងផ្ទទត់ គ ឺ 𝑥 = 3 ; 𝑦 = − 3 ។ 2

2

ចាំ យ េះ 𝑎 = −1 យយើ ងទញបាន 3

3

𝑥 +𝑦 =0 ⟹ 𝑥 3 − 𝑥 2 𝑦 + 𝑥𝑦 2 = 1 ចាំ យ េះ 𝑎 = 1 យយើ ងទញបាន

𝑥=

1 3

3 1

𝑦=−3

3

𝑥3 − 𝑦3 = 2 𝑥3 − 𝑦3 = 2 ⟹ 𝑥 3 + 𝑥 2 𝑦 + 𝑥𝑦 2 = 1 2𝑥 3 + 2𝑥 2 𝑦 + 2𝑥𝑦 2 = 2 3 2 2 ⟹ 𝑥 + 2𝑥 𝑦 + 2𝑥𝑦 + 𝑦 3 = 0 ⟹ 𝑥 + 𝑦 𝑥 2 − 𝑥𝑦 + 𝑦 2 + 2𝑥𝑦 𝑥 + 𝑦 = 0 ⟹ 𝑥 + 𝑦 𝑥 2 + 𝑥𝑦 + 𝑦 2 = 0 ⟹𝑥+𝑦 =0 ⟹ 𝑥 = 1; 𝑦 = −1 ដូយចនេះ 𝑎 = −1; 0; 1 ទ្បព័នមា ធ នរ ឺសយផទៀងផ្ទទត់ លកខខណឌ។

158. ទ្បព័នស ធ មមូលនឹង 3𝑥 2 − 2 3 + 𝑧 𝑥 + 3𝑧 + 3 = 0 1 𝑦 3 + 3𝑦 2 = 𝑥 2 − 3𝑥 + 2 2 𝑦 2 = −𝑧 2 + 6𝑧 3 𝑧≤3 4 ′ 2 សមីោរ(១) មានរ ឺស យបើ Δ = 3 + 𝑧 − 3 3𝑧 + 3 ≥ 0 ⟹ 𝑧 ≤ 0 ; 𝑧 ≥ 3 (៥)។ សមីោរទី៣ មានរ ឺសយបើ −𝑧 2 + 6𝑧 ≥ 0 ⟹ 0 ≤ 𝑧 ≤ 6 (៦)។ ព ី(៤) (៥)(៦)យយើ ងទញបាន 𝑧 = 0; 𝑧 = 3 ។ យបើ 𝑧 = 0 យ េះ តាម(៣) យយើ ងទញបាន 𝑦 = 0; និងព ី(២) យយើ ងទញបាន 𝑥 = 1; 𝑥 = 2 ។ យផទៀងផ្ទទត់ តាម(១) យយើ ងទញបាន 𝑥 = 1 ។ 92

១. អនុគមន៍ងាយ | លឹម សុ វណ្ណវិចិត្រ


យបើ 𝑧 = 3 យ េះ តាម(៣) យយើ ងទញបាន 𝑦 = ±3 ។ ព ី(១) យយើ ងទញបាន 𝑥 = 2 ។ ចាំ យ េះ 𝑥 = 2 យផទៀងផ្ទទត់ តាម(២) យយើ ងទញបាន 𝑦 = −3 ។ ដូយចនេះ 𝑥; 𝑦; 𝑧 = 1; 0; 0 ; 2; −3; 3 159.  គុណសមីោរទីមួយទីព ីរនិងទីបី នឹង 𝑐, 𝑎, 𝑏 យរៀងគ្នន យយើ ងទញបាន 𝑎𝑐 𝑏𝑐 − = 𝑐 2 − 𝑐𝑧𝑥 𝑥 𝑧 𝑎𝑏 𝑎𝑐 − = 𝑎2 − 𝑎𝑥𝑦 𝑦 𝑥 𝑏𝑐 𝑎𝑏 − = 𝑏 2 − 𝑏𝑦𝑧 𝑧 𝑦 បូកសមីោរទាំងបីបញ្ចូ លគ្នន យយើ ងទញបាន 𝑎2 + 𝑏 2 + 𝑐 2 = 𝑎𝑥𝑦 + 𝑏𝑧𝑦 + 𝑐𝑥𝑧 បាំ បាត់ ភាគដបងននទ្បព័នយធ ដើម

(1)

𝑎𝑧 − 𝑏𝑥 = 𝑐𝑥𝑧 − 𝑧 2 𝑥 2 𝑏𝑥 − 𝑐𝑦 = 𝑎𝑥𝑦 − 𝑥 2 𝑦 2 𝑐𝑦 − 𝑎𝑧 = 𝑏𝑧𝑦 − 𝑦 2 𝑧 2 បូកបញ្ចូ លគ្នន យយើ ងទញបាន 𝑥 2 𝑦 2 + 𝑦 2 𝑧 2 + 𝑧 2 𝑥 2 = 𝑎𝑥𝑦 + 𝑏𝑧𝑦 + 𝑐𝑥𝑧 បូក(១)នឹង(២) យយើ ងទញបាន

(2)

𝑎2 + 𝑏 2 + 𝑐 2 + 𝑥 2 𝑦 2 + 𝑦 2 𝑧 2 + 𝑧 2 𝑥 2 = 2 𝑎𝑥𝑦 + 𝑏𝑧𝑦 + 𝑐𝑥𝑧 ⟹ 𝑥𝑦 − 𝑎 2 + 𝑧𝑦 − 𝑏 2 + 𝑥𝑧 − 𝑐 2 = 0 𝑥𝑦 = 𝑎 ⟹ 𝑧𝑦 = 𝑏 𝑥𝑧 = 𝑐 ⟹

𝑥; 𝑦; 𝑧 =

𝑎𝑐 𝑎𝑏 𝑏𝑐 𝑎𝑐 𝑎𝑏 𝑏𝑐 ; ; ; − ;− ;− 𝑏 𝑐 𝑎 𝑏 𝑐 𝑎

160.  ទ្បព័នមា ធ នរ ឺសង្ហយ 0; 0; 0 ។ យបើ 𝑥 = ±1 យ េះ ±2 + 𝑦 = 𝑦 មិនោច។ ដូចគ្ននយយើ ង ទញបាន 𝑥 ≠ ±1; 𝑦 ≠ ±1; 𝑧 ≠ ±1 ។ យយើ ងមាន 2𝑥 4 1 − 𝑥2 2𝑦 𝑧= 5 1 − 𝑦2 2𝑧 𝑥= (6) 1 − 𝑧2 𝑦=

2𝑥 + 𝑥 2 𝑦 = 𝑦 2𝑦 + 𝑦 2 𝑧 = 𝑧 2𝑧 + 𝑧 2 𝑥 = 𝑥

លឹម សុ វណ្ណវិចិត្រ | IV. ត្រព័នស ធ មីការ

93


đ?œ‹ 4

đ?œ‹ 2

áž?ážśáž„ đ?‘Ľ = tan đ?›ź ≠¹1 â&#x;š đ?›ź ≠¹ + đ?‘˜ , đ?‘˜ ∈ ℤ á&#x;” áž– ី(á&#x;¤) យយឞ ងទញបវន đ?‘Ś = tan 2đ?›ź នឡងព ី(á&#x;Ľ) យយឞ ងទញបវន đ?‘§ = tan 4đ?›ź យហឞ យជវចឝងយទá&#x;’á&#x;„áž™ đ?‘Ľ = tan 8đ?›ź = tan đ?›ź á&#x;” ដសយចនá&#x; á&#x;‡ đ?œ‹ 8đ?›ź = đ?›ź + đ?‘›đ?œ‹ â&#x;š đ?›ź = đ?‘› ; đ?‘› ∈ ℤ 7 đ?œ‹ 2đ?œ‹ 4đ?œ‹ ដសយចនá&#x; á&#x;‡áž‘á&#x;’បពá&#x;?áž“áž˜ážś áž’ áž“ážš ážşáž&#x; đ?‘Ľ; đ?‘Ś; đ?‘§ = tan đ?‘› ; tan đ?‘› ; tan đ?‘› |đ?‘› ∈ ℤ 7

7

7

ážœážˇáž&#x;áž˜áž—ážśáž– 1 3 5

99

2 4 6

98

161. ď ° áž?ážśáž„ đ??´ = 2 . 4 . 6 ‌ . 100 ; đ??ľ = 3 . 5 . 7 ‌ 99 á&#x;” áž™á&#x;„áž™ áž™ á&#x; á&#x;‡ đ??ľ > đ??´á&#x;” យយឞ áž„áž˜ážśáž“

2 1 4 3 6 5 98 97 99 > ; > ; > ;‌; > ;1 > 3 2 5 4 7 6 99 98 100

1 2 3 4 . . 2 3 4 5 1 â&#x;š đ??´ < 10 á&#x;”

đ??´đ??ľ = 1

ដសយចនá&#x; á&#x;‡ đ??´2 < đ??´đ??ľ = 100

5 6 98 99 1 . ‌ . = 6 7 99 100 100

áž” áž‘ áž”á&#x;‹ មកយទá&#x;€áž? 3 5 7 99 đ??ľ < 2đ??´ = . . ‌ 4 6 8 100 2 3 4 5 6 7 98 99 យទá&#x;’ á&#x; á&#x;‡ 3 < 4 ; 5 < 6 ; 7 < 8 ; ‌ ; 99 < 100 á&#x;” ដសយចនá&#x; á&#x;‡ 1 1 đ??´. 2đ??´ > đ??´đ??ľ = â&#x;šđ??´> 100 10 2 162. ď ° យយឞ áž„áž˜ážśáž“ 1 50 1.2.3 ‌ 100 đ??ś100 = 50 100 2 2 1.2.3 ‌ 50 . 250 1.2.3 ‌ 50 1.2.3 ‌ 100 1.3.5. .99 = = 2.4.6 ‌ 100 2.4.6 ‌ 100 2.4.6 ‌ 100 យហឞ áž™áž?ážśáž˜áž&#x;ážśá&#x;† នួរ161. យយឞ áž„áž‘áž‰áž”ážśáž“ážœ ឡáž&#x;áž˜áž—ážśáž–áž– ឡáž?á&#x;”

163. ď ° ដវá&#x;†áž”សងយយឞ ងនរងបងá&#x;’ហវញáž?ážś áž…ážśá&#x;† áž™ á&#x; á&#x;‡áž‘á&#x;’áž‚áž”á&#x;‹ đ?‘› ≼ 1 យយឞ áž„áž˜ážśáž“ 1 3 5 2đ?‘› − 1 1 . . ‌ ≤ 2 4 6 2đ?‘› 3đ?‘› + 1 1 1 áž…ážśá&#x;† áž™ á&#x; á&#x;‡ đ?‘› = 1 យយឞ áž„áž˜ážśáž“ 2 = 3.1+1á&#x;” áž” áž‘ áž”á&#x;‹ មកយទá&#x;€áž?áž&#x;នមáž?áž?ážśáž– ឡáž?ដលá&#x;‹ đ?‘› áž˜ážśáž“áž“á&#x;?áž™áž?ážś 1 3 5 2đ?‘› − 1 1 . . ‌ ≤ 2 4 6 2đ?‘› 3đ?‘› + 1 2đ?‘›+1 យយឞ ងគឝណអងគáž&#x;áž„á&#x;’áž áž áž„áž“áž“ážœ ឡáž&#x;áž˜áž—ážśáž–áž™áž“á&#x; á&#x;‡áž“ážšáž„ 2đ?‘›+2 យយឞ ងទញបវន

94

á&#x;Ą. áž˘áž“ážťáž‚áž˜áž“á&#x;?áž„ážśáž™ | លរម áž&#x;ážť វណá&#x;’ážŽážœážˇáž…ážˇáž?á&#x;’ážš


1 3 5 2𝑛 − 1 2𝑛 + 1 2𝑛 + 1 . . … . ≤ 2 4 6 2𝑛 2𝑛 + 2 2𝑛 + 2 3𝑛 + 1 យយើ ងមាន 2

2𝑛 + 1 2 12𝑛3 + 28𝑛2 + 20𝑛 + 4 2𝑛 + 2 3𝑛 + 1 2𝑛 + 1 2 = 12𝑛3 + 28𝑛2 + 19𝑛 + 4 + 𝑛 2𝑛 + 1 2 1 = < 2𝑛 + 1 2 3𝑛 + 4 + 𝑛 3𝑛 + 4 2𝑛 + 1 1 ⟹ < 2𝑛 + 2 3𝑛 + 1 3𝑛 + 4 1 3 5 2𝑛 − 1 2𝑛 + 1 1 ⟹ . . … . < 2 4 6 2𝑛 2𝑛 + 2 3𝑛 + 4 164.  អនុវតតនវ៍ ិសមភាពកនង ុ សាំ នួរទី163. ចាំ យ េះ n = 50 យយើ ងទញបាន 1 3 5 2𝑛 − 1 1 . . … < 2 4 6 2𝑛 3𝑛 + 1 1 3 5 2 × 50 − 1 1 . . … < 2 4 6 2 × 50 3 × 50 + 1 1 3 5 99 1 1 1 . . … < = < 2 4 6 100 151 12,288 12 ព ិត។ 2𝑛 + 1

=

165.  ទ្ាយបញ្ជាក់ តាមកាំយនើន។ 166. ជាដាំបូង យយើ ងនឹងបង្ហាញថា ចាំ យ េះទ្គប់ ចាំ នន ួ គត់ ធមម ជាតិ𝑘 ≤ 𝑛 យយើ ងមាន 𝑘 1 𝑘 𝑘 𝑘2 ≤ 1+ < 1+ + 2 𝑛 𝑛 𝑛 𝑛 ចាំ យ េះ 𝑘 = 1 វ ិសមភាពព ិត។ សនមតថាវ ិសមភាពព ិតចាំ យ េះតាំ នល 𝑘 ្មួយ។ យយើ ងមាន 1+

1 𝑘 1 𝑘 1 1+ ≥ 1+ 1+ 𝑛 𝑛 𝑛 𝑛 𝑘+1 𝑘 𝑘+1 =1+ + 2 > 1+ 𝑛 𝑛 𝑛 ព ិត (មិនចា​ាំបាច់ ទល់ ដត 𝑘 ≤ 𝑛 យទ)។ 1+

1 𝑛

𝑘+1

= 1+

យយើ ងមាន 1 𝑘+1 1 𝑘 1 𝑘 𝑘2 1+ = 1+ 1+ < 1+ + 2 𝑛 𝑛 𝑛 𝑛 𝑛 2 2 𝑘 + 1 𝑘 + 2𝑘 + 1 𝑘 + 1 𝑘 =1+ + − 2 + 3 𝑛 𝑛2 𝑛 𝑛 លឹម សុ វណ្ណវិចិត្រ | V. វិសមភាព

1+

1 𝑛

95


𝑘+1 𝑘 + 1 2 𝑛 𝑘 + 1 − 𝑘2 + − 𝑛 𝑛2 𝑛3 2 𝑘+1 𝑘+1 <1+ + 𝑛 𝑛2 2 យទ្ េះ 𝑛 𝑘 + 1 > 𝑘 យបើ 𝑛 ≥ 𝑘 ។ ដូយចនេះយោយយក 𝑘 = 𝑛 យយើ ងទញបាន =1+

1 1 2=1+ < 1+ 𝑛 𝑛 167.  តាមសាំ នួរ 166. យយើ ងទញបាន 1,000 001

1 000 000

𝑛

= 1+

168. យយើ ងមាន

𝑛 𝑛2 <1+ + 2 =3 𝑛 𝑛 1 1 000 000

1 000 000

1001999 1001 1000 1 1 = . = 1+ 1000 1000 1000 1001 1000 3.1 < <1 1001 (តាមសាំ នួរ 166.)។ ដូយចនេះ 10001 000 > 1001999 ។

>2 1000

.

1 1001

169. យយើ ងមាន ⟺

𝑐+1− 𝑐 < 𝑐− 𝑐−1 𝑐+1− 𝑐−1<2 𝑐

𝑐+1− 𝑐−1

𝑐2

2

< 4𝑐

−1<𝑐

ព ិត។

170. តាងចាំ នន ួ ព ិតទាំងយ េះយោយ 𝑥1 , 𝑥2 , … , 𝑥𝑛 ដដល 𝑥1 𝑥2 … 𝑥𝑛 = 1 ។ យយើ ងទ្តូវបង្ហាញថា 𝑥1 + 𝑥2 + ⋯ + 𝑥𝑛 ≥ 𝑛 ។ ចាំ យ េះ 𝑛 = 1 វ ិសមភាពព ិត។ សនមតថាព ិតចាំ យ េះ 𝑛 = 𝑘 មានន័យថា ចាំ យ េះទ្គប់ ចាំ នន ួ ព ិត 𝑥1 , 𝑥2 , … , 𝑥𝑘 ដដល 𝑥1 𝑥2 … 𝑥𝑘 = 1 យយើ ងមាន 𝑥1 + 𝑥2 + ⋯ + 𝑥𝑘 ≥ 𝑘 ។ ចាំ យ េះ 𝑛 = 𝑘 + 1 យបើ 𝑥1 = 𝑥2 = ⋯ = 𝑥𝑘+1 = 1 យ េះ𝑥1 + 𝑥2 + ⋯ + 𝑥𝑘+1 ≥ 𝑘 + 1 វ ិសមភាព ព ិត។ ករណីយផេងព ីយនេះ យោយារ ផលគុណននចាំ នន ួ ទាំងយនេះមានតាំ នលយសមើ មួយ យហើ យចាំ នន ួ ទាំងយនេះ មិនមានតាំ នលយសមើ មួយដូចគ្ននទាំងអស់ ដូយចនេះមានចាំ នន ួ ខាេះតូចជាងមួយចាំ នន ួ ខាេះធាំ ជាងមួយ។ សនមត 𝑥1 < 1, 𝑥𝑘+1 > 1។ តាង 𝑦1 = 𝑥1 𝑥𝑘+1 ។ យយើ ងមាន 𝑦1 𝑥2 … 𝑥𝑘 = 1 ដូយចនេះ 𝑦1 + 𝑥2 + ⋯ + 𝑥𝑘 ≥ 𝑘 ។ យយើ ងមាន 𝑥1 + 𝑥2 + ⋯ + 𝑥𝑘+1 = 𝑦1 + 𝑥2 + ⋯ + 𝑥𝑘 − 𝑦1 + 𝑥1 + 𝑥𝑘+1 ≥ 𝑘 − 𝑦1 + 𝑥1 + 𝑥𝑘+1 = 𝑘 + 1 − 1 − 𝑥1 𝑥𝑘+1 + 𝑥1 + 𝑥𝑘+1 = 𝑘 + 1 + 𝑥𝑘+1 − 1 1 − 𝑥1 > 𝑘 + 1 96

១. អនុគមន៍ងាយ | លឹម សុ វណ្ណវិចិត្រ


វ ិសមភាពព ិតដល់ 𝑘 + 1 ។

171.  ទ្ាយបញ្ជាក់ តាមកាំយនើន។ ចាំ យ េះ 𝑛 = 1 វ ិសមភាពព ិត។ សនមតថាព ិតចាំ យ េះ 𝑛 = 𝑘 មាន ន័យថា 1 + 𝑥1 1 + 𝑥2 … 1 + 𝑥𝑘 ≥ 1 + 𝑥1 + 𝑥2 + ⋯ + 𝑥𝑘 ។ គុណអងគទង ាំ ព ីរននវ ិសមភព នឹង 1 + 𝑥𝑘+1 យយើ ងទញបាន 1 + 𝑥1 1 + 𝑥2 … 1 + 𝑥𝑘 1 + 𝑥𝑘+1 ≥ 1 + 𝑥1 + 𝑥2 + ⋯ + 𝑥𝑘 1 + 𝑥𝑘+1 ។ យយើ ងមាន 1 + 𝑥1 + 𝑥2 + ⋯ + 𝑥𝑘 1 + 𝑥𝑘+1 ≥ 1 + 𝑥1 + 𝑥2 + ⋯ + 𝑥𝑘 + 𝑥𝑘+1 ⟺ 𝑥1 + 𝑥2 + ⋯ + 𝑥𝑘 𝑥𝑘+1 ≥ 0 ព ិតយទ្ េះ 𝑥1 , 𝑥2 , … , 𝑥𝑘+1 មានសញ្ជាដូចគ្នន។ ដូយចនេះ 1 + 𝑥1 1 + 𝑥2 … 1 + 𝑥𝑘 1 + 𝑥𝑘+1 ≥ 1 + 𝑥1 + 𝑥2 + ⋯ + 𝑥𝑘 + 𝑥𝑘+1 ។

172.  ចាំ យ េះ 𝑛 = 6 យយើ ងមាន 26 = 64 < 6! = 720 < 36 = 729 សនមតថាវ ិសមភាពព ិតចាំ យ េះ ចាំ នន ួ គត់ ធមម ជាតិ 𝑛 មួយ។ យយើ ងនឹងបង្ហាញថា ព ិតចាំ យ េះ 𝑛 + 1 ដដរ។ យយើ ងមាន

𝑛 2

𝑛

> 𝑛! >

⟹ 𝑛+1 ដូយចនេះ យយើ ងទ្គ្នន់ ដតបង្ហាញថា

𝑛 𝑛 3 𝑛 𝑛 > 𝑛+1 !> 𝑛+1 2

𝑛 + 1 𝑛+1 𝑛 > 𝑛+1 2 2 ជាោរយស្សច។ វ ិសមភាពទាំងព ីរសមមូលនឹង

ព ិត តាមសាំ នួរ 166. ។

𝑛

និង

𝑛 3

𝑛

𝑛+1 >

𝑛 3

𝑛

𝑛+1 3

𝑛+1

𝑛 + 1 𝑛+1 𝑛 + 1 𝑛+1 2 3 >𝑛+1> 𝑛 𝑛 𝑛 𝑛 2 3 1 𝑛+1 𝑛 1 𝑛+1 𝑛 2 3 ⟺ 2 >1>3 𝑛 𝑛 𝑛 𝑛 2 3 1 1 𝑛 1 1 𝑛 ⟺ 1+ >1> 1+ 2 𝑛 3 𝑛 1 𝑛 ⟺2< 1+ <3 𝑛

173.  យយើ ងមាន Δ′ = 9992 − 𝑎𝑐 > 0; Δ′ ∈ ℤ យទ្ េះ 𝑎, 𝑐 ∈ ℤ និង 𝑥1 − 𝑥2 =

2 Δ′ 𝑎

យបើ Δ′ ≥ 2 យ េះ 𝑥1 − 𝑥2 = លឹម សុ វណ្ណវិចិត្រ | V. វិសមភាព

2 Δ′ 2 Δ′ 2 2 1 > ≥ > 𝑎 2000 2000 998 97


យបើ Δ′ = 1 យ េះ 𝑎𝑐 = 9992 − 1 = 998.1000 = 24 . 53 . 449 ។ យយើ ងមាន 22 . 53 . 449 = 1996 < 2000 ជាតួដចកមួយនន 24 . 53 . 449 = 𝑎𝑐 ⟹ តួដចកធាំ បាំផត ុ នន 𝑎𝑐 = 24 . 53 . 449 យហើ យ ដដលតូចជាង 2000 យសមើ នង ឹ 1996 ។ ដូយចនេះ 𝑎 ≤ 1996 (យហតុអេីបានយកតួដចកតូចជាង2000 ? យទ្ េះ 𝑎 < 2000, 𝑐 < 2000)។ យយើ ងទញបាន 2 Δ′ 2 1 = ≥ 𝑎 𝑎 998 174.  លកខខណឌដដលយោយ យាំ ោយ 𝑥 ≠ 𝑦 និង 𝑦−2 2−𝑥 𝑎= ;𝑏 = 𝑦−𝑥 𝑦−𝑥 ជាំនស ួ ចូលលកខខណឌទីបី យយើ ងទញបាន 𝑥1 − 𝑥2 =

𝑦 − 2 𝑥2 2 − 𝑥 𝑦2 + = 2 𝑥 + 𝑦 − 𝑥𝑦 𝑦−𝑥 𝑦−𝑥 តាង 𝐶 = 𝑎𝑥 3 + 𝑏𝑦 3 > 0 ។ យយើ ងមាន 𝑦−2 3 2−𝑥 3 𝐶= 𝑥 + 𝑦 𝑦−𝑥 𝑦−𝑥 = −𝑥𝑦 𝑥 + 𝑦 + 2 𝑥 2 + 𝑥𝑦 + 𝑦 2 = −𝑥𝑦 𝑥 + 𝑦 + 2 𝑥 + 𝑦 2 − 2𝑥𝑦 = 𝑥 + 𝑦 −𝑥𝑦 + 2 𝑥 + 𝑦 − 2𝑥𝑦 = 3 𝑥 + 𝑦 − 2𝑥𝑦; តាម (1) ដូយចនេះ 2 𝑥 + 𝑦 − 𝑥𝑦 = 3 3 𝑥 + 𝑦 − 2𝑥𝑦 = 𝐶 យយើ ងទញបាន 𝑥 + 𝑦 = 6 − 𝐶; 𝑥𝑦 = 9 − 2𝐶 ។ 3 = 𝑎𝑥 2 + 𝑏𝑦 2 =

យយើ ងមាន 𝑥 + 𝑦

2

> 4𝑥𝑦 ⟹

6−𝐶

2

> 4 9 − 2𝐶

⟹𝐶>4 ។

យយើ ងមាន 𝑥𝑦 > 0 ⟹ 9 − 2𝐶 > 0; ⟹ 𝐶 < 4,5 ។

175.   ករណី 𝑥 < −3 យ េះ 𝑆= 𝑥 +  ករណី −3 < 𝑥 < 0 ករណីយនេះ  ករណី 𝑥 > 1 2

2𝑥−1 𝑥+3

< 0 ដូយចនេះ 2𝑥 − 1 7 7 1 𝑆≥ = −2 + > −2 + = 𝑥+3 𝑥+3 3 3

យយើ ងមាន 98

2𝑥 − 1 > 𝑥 >3 𝑥+3

១. អនុគមន៍ងាយ | លឹម សុ វណ្ណវិចិត្រ

1


đ?‘†= đ?‘Ľ + ď Ź ករណី 0 ≤ đ?‘Ľ ≤ 1 2

2đ?‘Ľ − 1 1 ≼ đ?‘Ľ > đ?‘Ľ+3 2

ករណីយនá&#x; á&#x;‡áž™áž™ážž áž„áž˜ážśáž“ đ?‘† ≼ 1 3 យទá&#x;’ á&#x; á&#x;‡ 2đ?‘Ľ − 1 đ?‘Ľ 2 + đ?‘Ľ + 1 = đ?‘Ľ+3 đ?‘Ľ+3 2 1 đ?‘Ľ +đ?‘Ľ+1 1 đ?‘†â‰Ľ â&#x;ş ≼ 3 đ?‘Ľ+3 3 â&#x;ş 3đ?‘Ľ 2 + 3đ?‘Ľ + 3 ≼ đ?‘Ľ + 3 â&#x;ş đ?‘Ľ ≼ 0 áž– ឡáž?á&#x;”áž&#x;ញá&#x;’ជវយáž&#x;មឞ យកឞáž?áž˜ážśáž“ យពល đ?‘Ľ = 0 á&#x;” đ?‘†=đ?‘Ľâˆ’

ដសយចនá&#x; á&#x;‡ min đ?‘† = 1/3 យពល đ?‘Ľ = 0 á&#x;”

176. ď ° áž&#x;នមáž?áž?ážś đ?‘Ž ≼ 0 á&#x;” យបឞ đ?‘Ž < 0 áž™ á&#x; á&#x;‡áž‚ឝណពហឝ áž’ážśđ?‘“(đ?‘Ľ) áž“ážšáž„ −1 យយឞ ងទញបវនពហឝ ធវដដល áž™áž˜áž‚ážťážŽážœ ážˇáž‡ážśáž˜ážśáž“ យហឞ យយá&#x;…ដáž?យផទá&#x;€áž„áž•á&#x;’áž‘áž‘áž?á&#x;‹ áž&#x;មម áž?ក ឡ មមដដដលá&#x;” យយឞ áž„áž&#x;នមáž?ផងដដរáž?ážś đ?‘? ≼ 0 á&#x;” យបឞđ?‘? < 0 áž™ á&#x; á&#x;‡áž™áž™ážž ងយក đ?‘“(−đ?‘Ľ) យទá&#x;’ á&#x; á&#x;‡áž&#x;មម áž?ក ឡ មមáž&#x;ážťáž¸áž™áž˜áž‘á&#x;’ទីយធá&#x;€áž”áž“ážšáž„ đ?‘Ľ á&#x;” ជវá&#x;†áž“áž&#x; áž˝ đ?‘Ľ = 0; đ?‘Ľ = Âą1 áž…ážźáž›đ?‘“ đ?‘Ľ យយឞ ងទញបវន −1 ≤ đ?‘? ≤ 1 1 −1 ≤ đ?‘Ž + đ?‘? + đ?‘? ≤ 1 2 −1 ≤ đ?‘Ž − đ?‘? + đ?‘? ≤ 1 3 (2) នឡង(3)

យវá&#x;† á&#x;„áž™

áž™á&#x;„áž™ đ?‘Ž ≼ 0; đ?‘? ≼ 0; đ?‘? ≤ 1 áž™ á&#x; á&#x;‡

−1 − đ?‘? ≤ đ?‘Ž + đ?‘? ≤ 1 − đ?‘? −1 − đ?‘? ≤ đ?‘Ž − đ?‘? ≤ 1 + đ?‘? 0≤đ?‘Ž+đ?‘? ≤2 −2 ≤ đ?‘Ž − đ?‘? ≤ 2 2 2 â&#x;š đ?‘Ž2 + 2đ?‘Žđ?‘? + đ?‘? 2 ≤ 4 đ?‘Ž − 2đ?‘Žđ?‘? + đ?‘? ≤ 4 â&#x;š đ?‘Ž2 + đ?‘? 2 ≤ 4 (4)

យយឞ áž„áž˜ážśáž“ 8 2 2 8 8 32 đ?‘Ž + đ?‘? 2 − đ?‘? 2 ≤ đ?‘Ž2 + đ?‘? 2 ≤ . 4 = 3 3 3 3 3 đ??ž áž˜ážśáž“áž?ážśá&#x;† នលធវá&#x;† បវá&#x;†áž•áž? ážť យពល đ?‘? = 0; đ?‘Ž2 + đ?‘? 2 = 4 â&#x;š đ?‘? = 0; đ?‘Ž = Âą2 á&#x;” đ??ž=

យបឞ đ?‘? = 0; đ?‘Ž = 2; 2 â&#x;š đ?‘? = −1 á&#x;” យបឞ đ?‘? = 0; đ?‘Ž = −2; 2 â&#x;š đ?‘? = 1 á&#x;” ដសយចនá&#x; á&#x;‡ đ?‘Ž, đ?‘?, đ?‘? =

2; 0; −1 ; −2; 0; 1

á&#x;”

177. ď ° áž&#x;នមáž?áž?ážś đ?‘Ž1 ≤ đ?‘Ž2 ≤ đ?‘Ž3 ≤ đ?‘Ž4 ≤ đ?‘Ž5 á&#x;” áž&#x;នមáž?áž?ážś minđ?‘–≠đ?‘— đ?‘Žđ?‘– − đ?‘Žđ?‘— លរម áž&#x;ážť វណá&#x;’ážŽážœážˇáž…ážˇáž?á&#x;’ážš | V. ážœážˇáž&#x;áž˜áž—ážśáž–

2

1

> 100 áž™ á&#x; á&#x;‡ 99


1 10 1 𝑎3 − 𝑎2 > 10 1 𝑎4 − 𝑎3 > 10 1 𝑎5 − 𝑎4 > 10

1 10 2 ⟹ 𝑎3 > 10 3 ⟹ 𝑎4 > 10 4 ⟹ 𝑎2 > 10 1+2+3+4 ⟹ 𝑎1 + 𝑎2 + 𝑎3 + 𝑎4 + 𝑎5 > =1 10 𝑎2 − 𝑎1 >

មិនទ្តឹមទ្តូវ។

សនមតថា min𝑖≠𝑗 𝑎𝑖2 − 𝑎𝑗2 >

1 36

⟹ 𝑎2 >

យ េះ 1 36 1 𝑎32 − 𝑎22 > 36 1 𝑎42 − 𝑎32 > 36 1 𝑎52 − 𝑎42 > 36 𝑎22 − 𝑎12 >

⟹ 𝑎3 ⟹ 𝑎4 ⟹ 𝑎5

⟹ 𝑎1 + 𝑎2 + 𝑎3 + 𝑎4 + 𝑎5 > មិនទ្តឹមទ្តូវ។

1 6 2 > 6 3 > 6 4 > 6 2+ 3+ 4 >1 10

⟹ 𝑎2 >

1+

178.   ដំននាោះត្ាយទី១ យយើ ងមាន 𝑎 𝑏 𝑐 2𝑎 2𝑏 2𝑐 + + = + + 𝑏+𝑐 𝑐+𝑎 𝑎+𝑏 𝑏+𝑐 + 𝑏+𝑐 𝑐+𝑎 + 𝑐+𝑎 𝑎+𝑏 + 𝑎+𝑏 2𝑎 2𝑏 2𝑐 < + + =2 𝑎+ 𝑏+𝑐 𝑏+ 𝑐+𝑎 𝑐+ 𝑎+𝑏

 ដំននាោះត្ាយទី២

តាង 𝑎 = 𝑥 + 𝑦; 𝑏 = 𝑦 + 𝑧; 𝑐 = 𝑧 + 𝑥។ 𝑎+𝑏 >𝑐 ⟺𝑥+𝑦+𝑦+𝑧 >𝑧+𝑥 ⟹𝑦 >0 𝑎+𝑐 >𝑏 ⟺𝑥+𝑦+𝑧+𝑥 >𝑦+𝑧 ⟹𝑥 >0 𝑏+𝑐 > 𝑎 ⟺𝑦+𝑧+𝑧+𝑥 > 𝑥+𝑦 ⟹𝑧 > 0 ដូយចនេះ យយើ ងបាំ ដលង លកខខណឌ 𝑎, 𝑏, 𝑐 ជារង្ហេស់ ទ្ជុងទ្តីយោណមក 𝑥, 𝑦, 𝑧 > 0 ។ យយើ ងមាន 𝑥+𝑦 𝑦+𝑧 𝑧+𝑥 𝑥+𝑦 𝑦+𝑧 𝑧+𝑥 + + < + + =2 𝑦+𝑧+𝑧+𝑥 𝑧+𝑥+𝑥+𝑦 𝑥+𝑦+𝑦+𝑧 𝑦+𝑧+𝑥 𝑧+𝑥+𝑦 𝑥+𝑦+𝑧 100

១. អនុគមន៍ងាយ | លឹម សុ វណ្ណវិចិត្រ


ព ិត។

179.  តាង 𝑎 = 𝑥 + 𝑦; 𝑏 = 𝑦 + 𝑧; 𝑐 = 𝑧 + 𝑥 យ េះ 𝑥; 𝑦; 𝑧 > 0 ។ វ ិសមភាពខាងយឆេងសមមូលនឹង 𝑎 + 𝑏 + 𝑐 2 ≥ 3 𝑎𝑏 + 𝑏𝑐 + 𝑐𝑎 ⟺ 4 𝑥+𝑦+𝑧 2 ≥ 3 𝑥+𝑦 𝑦+𝑧 + 𝑦+𝑧 𝑧+𝑥 + 𝑧+𝑥 𝑥+𝑦 ⟺ 𝑥 2 + 𝑦 2 + 𝑧 2 ≥ 𝑥𝑦 + 𝑦𝑧 + 𝑧𝑥 1 ⟺ 𝑥−𝑦 2+ 𝑦−𝑧 2+ 𝑧−𝑥 2 ≥0 2 ព ិត។ អងគទង ាំ ព ីរយសមើ គ្ននយពល x = y = z មានន័យថា a = b = c ។ ដូចគ្ននវ ិសមភាពខាងាតាំសមមូលនឹង 𝑥𝑦 + 𝑦𝑧 + 𝑧𝑥 > 0 ព ិត យទ្ េះ 𝑥, 𝑦, 𝑧 > 0 ។

180.  យយើ ងមាន 𝑎2 + 𝑏 2 > 2𝑎𝑏 𝑎 + 𝑏 > 𝑐 ⟹ 𝑎2 + 𝑏 2 + 2𝑎𝑏 > 𝑐 2 ⟹ 𝑎2 + 𝑏 2 + 𝑎2 + 𝑏 2 > 𝑎2 + 𝑏 2 + 2𝑎𝑏 > 𝑐 2 ⟹ 2𝑎2 + 2𝑏 2 − 𝑐 2 > 0 ដូចគ្ននយយើ ងទញបាន 2𝑏 2 + 2𝑐 2 − 𝑎2 > 0 2𝑐 2 + 2𝑎2 − 𝑏 2 > 0 យយើ ងនឹងបង្ហាញថា 2𝑎2 + 2𝑏 2 − 𝑐 2 2𝑐 2 + 2𝑎2 − 𝑏 2 ≤ 2𝑎2 + 𝑏𝑐

2

យយើ ងមាន 2𝑎2 + 𝑏𝑐 2 − 2𝑎2 + 2𝑏 2 − 𝑐 2 2𝑐 2 + 2𝑎2 − 𝑏 2 = 4𝑎2 𝑏𝑐 − 2𝑎2 𝑏2 + 𝑐 2 − 4𝑏 2 𝑐 2 + 2𝑏 4 + 2𝑐 4 = 2 𝑏 2 − 𝑐 2 2 − 2𝑎2 𝑏 − 𝑐 2 =2 𝑏−𝑐 2 𝑏+𝑐+𝑎 𝑏+𝑐−𝑎 ≥ 0 ដូចគ្ននយយើ ងទញបាន 0 < 2𝑎2 + 2𝑏 2 − 𝑐 2 2𝑐 2 + 2𝑎2 − 𝑏 2 ≤ 0 < 2𝑏 2 + 2𝑐 2 − 𝑎2 2𝑎2 + 2𝑏 2 − 𝑐 2 ≤ 0 < 2𝑐 2 + 2𝑎2 − 𝑏 2 2𝑏2 + 2𝑐 2 − 𝑎2 ≤ គុណអងគនង ឹ អងគយយើ ងទញបានវ ិសមភាព។ អងគទង ាំ ព ីរយសមើ គ្ននយពល 𝑎

2𝑎2 + 𝑏𝑐 2 2𝑏 2 + 𝑎𝑐 2 2𝑐 2 + 𝑎𝑏 2 =𝑏=𝑐។

181. យយើ ងទញបាន 𝑎 ≥ 0; 𝑏 2 − 4𝑎𝑐 ≤ 0 ⟹ 𝑏 2 ≤ 4𝑎𝑐 = 4.2𝑎. ⟹ 2𝑏 ≤ 4𝑎 + 𝑐 ⟹ 2𝑏 − 4𝑎 ≤ 𝑐 ⟹ 3𝑏 − 3𝑎 ≤ 𝑎 + 𝑏 + 𝑐 លឹម សុ វណ្ណវិចិត្រ | V. វិសមភាព

𝑐 𝑐 ≤ 2𝑎 + 2 2

2

101


đ?‘Ž+đ?‘?+đ?‘? ≼3 đ?‘?−đ?‘Ž â&#x;š min đ??š = 3 â&#x;š

182. ď ° យយឞ áž„áž˜ážśáž“

đ?›ź 2 đ?‘Ž2 = đ?‘? 2 + đ?‘? 2 − 2đ?‘?đ?‘? cos đ?›ź 3 sin đ?›ź + cos đ?›ź = cos đ?›ź − 60° 2 đ?‘† = đ?‘?đ?‘? sin

យយឞ áž„áž˜ážśáž“

đ?‘Ž2 + đ?‘? 2 + đ?‘? 2 ≼ 4đ?‘† 3 â&#x;ş đ?‘? 2 + đ?‘? 2 ≼ đ?‘?đ?‘? 3 sin đ?›ź + cos đ?›ź â&#x;ş đ?‘? − đ?‘? 2 + 2đ?‘?đ?‘? 1 − cos đ?›ź − 60° ≼ 0 យយឞ áž„áž˜ážśáž“áž&#x;áž˜áž—ážśáž–áž‘áž›á&#x;‹ ដáž? đ?‘? = đ?‘? នឡង đ?›ź = 60° áž˜ážśáž“áž“á&#x;?áž™áž?ážś áž‘á&#x;’áž?ីយá&#x;„ណជវទá&#x;’áž?ីយá&#x;„ណáž&#x;មá&#x;? áž„á&#x; á&#x;”

183. ď ° áž&#x;នមáž?áž?ážś đ?‘Ž ≼ 0; đ?‘? ≼ 0; 4đ?‘Žđ?‘? ≼ đ?‘? 2 á&#x;” យបឞ đ?‘Ž = 0 áž™ á&#x; á&#x;‡ đ?‘? = 0 យហឞ យវ ឡáž&#x;áž˜áž—ážśáž–áž™á&#x;…ជវ đ?‘?đ?‘”2 ≼ 0 á&#x;” យបឞ đ?‘Ž > 0 áž™ á&#x; á&#x;‡ 2 đ?‘? 4đ?‘Žđ?‘? − đ?‘? 2 2 đ?‘” + đ?‘” ≼0 2đ?‘Ž 4đ?‘Ž ូលស វយយឞ áž„áž&#x;នមáž?áž?ážś đ?‘Žđ?‘“ 2 + đ?‘?đ?‘“đ?‘” + đ?‘?đ?‘”2 ≼ 0 áž…ážśá&#x;† áž™ á&#x; á&#x;‡áž‘á&#x;’áž‚áž”á&#x;‹ áž‚ážźážœáž…áž‘á&#x;? ážť ឡ ážš đ?‘“, đ?‘” á&#x;” ជវá&#x;†áž“áž&#x; áž˝ đ?‘“ áž™á&#x;„áž™ đ?‘Ąđ?‘” đ?‘Ą ∈ â„?

đ?‘Žđ?‘“ 2 + đ?‘?đ?‘“đ?‘” + đ?‘?đ?‘”2 = đ?‘Ž đ?‘“ +

យយឞ ងទញបវន đ?‘Žđ?‘Ą 2 + đ?‘?đ?‘Ą + đ?‘? đ?‘”2 ≼ 0 áž– ឡáž?áž…ážśá&#x;† áž™ á&#x; á&#x;‡áž‘á&#x;’áž‚áž”á&#x;‹ áž…ážśá&#x;† áž“áž“ áž˝ áž– ឡáž? đ?‘Ą ដសយចនá&#x; á&#x;‡ យវá&#x;† á&#x;„áž™ đ?‘Ž ≼ 0, đ?‘? ≼ 0, 4đ?‘Žđ?‘? ≼ đ?‘? 2 á&#x;”

184. ď °áž™áž™ážž áž„áž˜ážśáž“

đ?‘›

−đ?‘Žđ?‘– =

�� � =1;� ≠� �

đ?‘› đ?‘Žđ?‘– =

đ?‘› − 1 đ?‘Žđ?‘– − (−đ?‘Žđ?‘– | =

đ?‘Žđ?‘– − đ?‘Žđ?‘— đ?‘— =1;đ?‘— ≠đ?‘–

=

đ?‘Žđ?‘– − đ?‘Žđ?‘— đ?‘–≠đ?‘—

≤

đ?‘Žđ?‘– − đ?‘Žđ?‘— đ?‘–≠đ?‘—

đ?‘›

â&#x;š

đ?‘›

�� ≤ 2 �=1

đ?‘Žđ?‘– − đ?‘Žđ?‘— đ?‘–<đ?‘—

â&#x;š ážœ ឡáž&#x;áž˜áž—ážśáž–áž– ឡáž?á&#x;” áž…ážśá&#x;† áž™ á&#x; á&#x;‡ đ?‘Ž1 = đ?‘Ž2 = â‹Ż = đ?‘Žđ?‘›âˆ’1 = đ?‘Ľ នឡង đ?‘Žđ?‘› = − đ?‘› − 1 đ?‘Ľ យយឞ ងទញបវនអងគទង ážśá&#x;† á&#x;˘áž™áž&#x;មឞ áž‚á&#x;’áž“áž“á&#x;” ដសយចនá&#x; á&#x;‡ ážœ ឡáž&#x;áž˜áž—ážśáž–áž‡ážśážœ ឡáž&#x;áž˜áž—ážśáž–áž?សចជវងតយáž&#x;មឞ á&#x;” 102

á&#x;Ą. áž˘áž“ážťáž‚áž˜áž“á&#x;?áž„ážśáž™ | លរម áž&#x;ážť វណá&#x;’ážŽážœážˇáž…ážˇáž?á&#x;’ážš


185.   ដំន

ោះត្ាយទី១

ជាដាំ បូងយយើ ងសនមតថា កតាតនិមួយៗននអងគខាងាតាំរបស់ វ ិសមភាពមានតនមាវ ិជាមានឬសូ នែ។ យយើ ងមាន 1 1 1 1 =𝑏 1− + =𝑏 1− +𝑎 𝑐 𝑏 𝑏𝑐 𝑏 1 1 1 1 𝑎−1+ 𝑏−1+ =𝑏 𝑎− 1− 𝑎+ 1− 𝑏 𝑐 𝑏 𝑏 2 1 = 𝑏 𝑎2 − 1 − 𝑏 2 ≤ 𝑏𝑎 𝑏−1+

ដូចគ្នន យយើ ងទញបាន 1 1 𝑐−1+ ≤ 𝑐𝑏 2 𝑐 𝑎 1 1 𝑎−1+ 𝑐−1+ ≤ 𝑎𝑐 2 𝑏 𝑎 1 1 1 2 𝑎−1+ 𝑏−1+ 𝑐−1+ ≤ 𝑎𝑏𝑐 𝑏 𝑐 𝑎 𝑏−1+

⟹ ព ិត។

2

=1

1

ករណីមានកតាត្មួយអវ ិជាមាន ឧទហរណ៍ 𝑎 − 1 + 𝑏 < 0 យ េះ 𝑎 < 1 យហើ យ 𝑏 > 1។ កនង ុ 1 𝑐

1 𝑎

ករណីយនេះ 𝑏 − 1 + > 0 និង 𝑐 − 1 + > 0 ។ ដូយចនេះយបើមានកតាត្មួយអវ ិជាមាន យ េះ មាន ដតកតាតមួយយ េះប៉ា ុយ្ណេះ ដដលអវ ិជាមាន ដូយចនេះផលគុណននកតាតទាំងបីយនេះអវ ិជាមាន ដូយចនេះតូចជាង១។

 ដំន

ោះត្ាយទី២

វ ិសមភាពដដលឱ្ែសមមូលនឹង 𝑎 − 𝑎𝑏𝑐

1 3

𝑎𝑏𝑐 + 𝑏

2 3

𝑏 − 𝑎𝑏𝑐

1 3

𝑎𝑏𝑐 + 𝑐

2 3

𝑐 − 𝑎𝑏𝑐

1 3

𝑎𝑏𝑐 + 𝑎

2 3

≤ 𝑎𝑏𝑐

ជាំនស ួ 𝑎 = 𝑥 3 ; 𝑏 = 𝑦 3 ; 𝑐 = 𝑧 3 ដដល 𝑥, 𝑦, 𝑧 > 0 យយើ ងទញបាន 𝑥𝑦𝑧 2 𝑥𝑦𝑧 2 𝑥𝑦𝑧 2 3 3 𝑦 − 𝑥𝑦𝑧 + 𝑧 − 𝑥𝑦𝑧 + ≤ 𝑥3 𝑦3𝑧3 𝑦3 𝑧3 𝑥3 𝑥 2 𝑦 − 𝑦2 𝑧 + 𝑧 2 𝑥 𝑦2 𝑧 − 𝑧 2 𝑥 + 𝑥 2 𝑦 𝑧 2 𝑥 − 𝑥 2 𝑦 + 𝑦2 𝑧 ≤ 𝑥 3 𝑦3 𝑧 3

𝑥 3 − 𝑥𝑦𝑧 + ⟺

⟺ 3𝑥 3 𝑦 3 𝑧 3 +

𝑥6 𝑦3 ≥ 𝑐𝑦𝑐𝑙𝑖𝑐

លឹម សុ វណ្ណវិចិត្រ | V. វិសមភាព

𝑥4𝑦4𝑧 + 𝑐𝑦𝑐𝑙𝑖𝑐

𝑥5 𝑦2𝑧2 𝑐𝑦𝑐𝑙𝑖𝑐

103


⟺ 3 𝑥2 𝑦 𝑦2𝑧 𝑧2𝑥 +

𝑥2𝑦

3

𝑥2𝑦

𝑐𝑦𝑐𝑙𝑖𝑐

2

𝑦2𝑧

𝑠𝑦𝑚

តាង 𝑢 = 𝑥 𝑦; 𝑣 = 𝑦 𝑧; 𝑤 = 𝑧 𝑥 ។ យយើ ងមាន 𝑢, 𝑣, 𝑤 > 0 2

2

2

𝑢3 ≥

⟹ 3𝑢𝑣𝑤 + 𝑐𝑦𝑐𝑙𝑖𝑐

𝑢2 𝑣 𝑠𝑦𝑚

𝑢 𝑢−𝑣 𝑢−𝑤 ≥0 𝑐𝑦𝑐𝑙𝑖𝑐

ព ិត តាមវ ិសមភាព

186. យោយយទ្បើលកខខណឌ 𝑎 + 𝑏 = 1 យយើ ងបាំ ដលងវ ិសមភាពដដលយោយយៅជាវ ិសមភាព អូ ម៉ាូដសន 1 𝑎2 𝑏2 ≤ + 3 𝑎 + 𝑏 (𝑎 + 𝑎 + 𝑏 𝑎+𝑏 𝑏+ 𝑎+𝑏 2 ⟺ 𝑎−𝑏 𝑎+𝑏 ≥ 0 ព ិត។ សមភាពយកើតមាន យពល 𝑎 = 𝑏 = 1 2 ។

⟺ 𝑎2 𝑏 + 𝑎𝑏 2 ≤ 𝑎3 + 𝑏 3

187. 188. យោយ 0 < 𝑥 < 1 យ េះវ ិសមភាពដដលឱ្ែោចសរយសរជា 1 1 1+ ≥9 𝑥 1−𝑥 𝑥 + 1 1 − 𝑥 + 1 ≥ 9𝑥 1 − 𝑥 2 + 𝑥 − 𝑥 2 ≥ 9𝑥 − 9𝑥 2 2𝑥 − 1 2 ≥ 0 1+

ព ិត។

189.  យយើ ងមាន 𝑎6 𝑎4 𝑎2 𝑏 6 𝑏 4 𝑏 2 𝑎6 𝑏 6 𝑎4 𝑏 4 𝑎2 𝑏 2 + + + + + = + + + + + 𝑏 6 𝑏 4 𝑏 2 𝑎6 𝑎4 𝑎2 𝑏 6 𝑎6 𝑏 4 𝑎4 𝑏 2 𝑎2 ≥2+2+2 =6 អងគទង ាំ ២យសមើ គ្នន លុ េះទ្តាដត 𝑎 = 𝑏 ។

190.  យយើ ងមាន 𝑥4

104

𝑥 𝑦 𝑥 𝑦 + 2 ≤ + 2 4 +𝑦 𝑥 +𝑦 2 𝑥4 𝑦2 2 𝑥2 𝑦4 𝑥 𝑦 1 ≤ 2 + = 2𝑥 𝑦 2𝑥𝑦 2 𝑥𝑦

១. អនុគមន៍ងាយ | លឹម សុ វណ្ណវិចិត្រ


191.  យយើ ងមាន 0 ≤ 𝑥 − 𝑦2 + 1

2

+ 𝑦 − 𝑥2 + 1

2

⟹ 𝑥2 + 𝑦2 + 1 ≥ 𝑥 𝑦2 + 1 + 𝑦 𝑥2 + 1 អងគទង ាំ ២យសមើ គ្នន យបើ 𝑥=

𝑦2 + 1

𝑦 = 𝑥2 + 1 ⟹ 𝑥 2 + 𝑦 2 = 𝑥 2 + 𝑦 2 + 2 មិនោច។ ដូយចនេះអងគទ១ ី ធាំ ជាងអងគទ២ ី ោច់ ខាត មានន័យថា 𝑥2 + 𝑦2 + 1 > 𝑥 𝑦2 + 1 + 𝑦 𝑥2 + 1

192.  យយើ ងមាន 𝑥2 𝑦2 𝑧2 𝑥 𝑦 𝑧 + + ≥ + + 𝑦2 𝑧2 𝑥2 𝑧 𝑥 𝑦 𝑥 2 𝑥 𝑦 2 𝑦 2 𝑦 𝑧 2 𝑧 2 𝑧 𝑥 ⟺ −2 + + −2 + + −2 + 𝑦 𝑧 𝑧 𝑧 𝑥 𝑥 𝑥 𝑦 𝑦 𝑥 𝑦 2 𝑦 𝑧 2 𝑧 𝑥 2 ⟺ − + − + − ≥0 𝑦 𝑧 𝑧 𝑥 𝑥 𝑦 ព ិតជានិច។ ច អងគទង ាំ ២យសមើគ្នន មានដត x = y = z ។

2

≥0

193. តាង 𝑎 = 𝑥 + 𝑦, 𝑏 = 𝑦 + 𝑧, 𝑐 = 𝑧 + 𝑥 យ េះ 𝑥, 𝑦, 𝑧 > 0 ។ វ ិសមភាពសមមូលនឹង 𝑥 + 2𝑦 + 𝑧 𝑥 + 𝑦 + 2𝑧 2𝑥 + 𝑦 + 𝑧 ≥ 64𝑥𝑦𝑧 យយើ ងមាន 𝑥 + 2𝑦 + 𝑧 = 𝑥 + 𝑦 + 𝑦 + 𝑧 ≥ 2 𝑥𝑦 + 2 𝑦𝑧 ≥ 4 𝑥𝑦 2 𝑧 1/4 ដូចគ្នន 𝑥 + 𝑦 + 2𝑧 ≥ 4 𝑥𝑦𝑧 2 2𝑥 + 𝑦 + 𝑧 ≥ 4 𝑥 2 𝑦𝑧

1 4 1/4

ដូយចនេះ 𝑥 + 2𝑦 + 𝑧 𝑥 + 𝑦 + 2𝑧 2𝑥 + 𝑦 + 𝑧 ≥ 64 𝑥𝑦 2 𝑧. 𝑥𝑦𝑧 2 . 𝑥^2𝑦𝑧

1 4

= 64𝑥𝑦𝑧

ព ិត។

លឹម សុ វណ្ណវិចិត្រ | V. វិសមភាព

105


194. តាង 𝑎 = 𝑥 + 𝑦; 𝑏 = 𝑦 + 𝑧; 𝑐 = 𝑧 + 𝑥 យ េះ 𝑎; 𝑏; 𝑐 > 0 ។ យយើ ងមាន

ដតថា

𝑥 2 − 𝑧 2 𝑦 2 − 𝑥 2 𝑧𝑥 2 − 𝑦 2 𝑎−𝑏 𝑐 𝑏−𝑐 𝑎 𝑐−𝑎 𝑏 + + = + + 𝑦+𝑧 𝑧+𝑥 𝑥+𝑦 𝑏 𝑐 𝑎 𝑎𝑐 𝑏𝑎 𝑏𝑐 = + + −𝑎−𝑏−𝑐 𝑏 𝑐 𝑎

1 𝑎𝑐 𝑏𝑎 𝑏2 + 𝑐 2 + = 𝑎 ≥𝑎 2 𝑏 𝑐 2𝑏𝑐 យៅជាយសមើ គ្នន ទល់ ដត 𝑏 = 𝑐 ។ ដូចគ្ននដដរ 1 𝑎𝑐 𝑏𝑐 + ≥𝑐 2 𝑏 𝑎 1 𝑏𝑎 𝑏𝑐 + ≥𝑏 2 𝑐 𝑎 𝑎𝑐 𝑏𝑎 𝑏𝑐 បូកអងគននវ ិសមភាពទាំងយនេះបញ្ចូ លគ្នន យយើ ងទញបាន 𝑏 + 𝑐 + 𝑎 − 𝑎 − 𝑏 − 𝑐 ≥ 0 ។ ដូយចនេះ វ ិសមភាពព ិត។ សញ្ជាយសមើ យកើតមានយពល𝑎 = 𝑏 = 𝑐 មានន័យថា 𝑥 = 𝑦 = 𝑧 ។

195. តាង 𝑥 = 𝑎 + 𝑏 − 𝑐, 𝑦 = 𝑏 + 𝑐 − 𝑎, 𝑧 = 𝑐 + 𝑎 − 𝑏 ។ ដូយចនេះ 𝑥, 𝑦, 𝑧 > 0 និង 𝑎 =

𝑥+𝑧 ,𝑏 2

=

𝑦+𝑥 2

,𝑐 =

𝑧+𝑦 2

។ យយើ ងមាន

2 𝑥 + 𝑦 = 𝑥 + 𝑦 + 𝑥 + 𝑦 ≥ 𝑥 + 𝑦 + 2 𝑥𝑦 = ⟹ 𝑥 + 𝑦 ≤ 2. 𝑥 + 𝑦

𝑥+ 𝑦

2

យោយអងគទង ាំ ព ីរយសមើ គ្នន យបើ x = y ។ ដូយចនេះ 𝑎 + 𝑏 − 𝑐 + 𝑏 + 𝑐 − 𝑎 = 𝑥 + 𝑦 ≤ 2. 𝑥 + 𝑦 = 2 𝑏 ដូចគ្នន 𝑏+𝑐−𝑎+ 𝑐+𝑎−𝑏 ≤2 𝑐 𝑐+𝑎−𝑏+ 𝑎+𝑏−𝑐 ≤2 𝑎 បូកអងគនង ឹ អងគននវ ិសមភាព យយើ ងទញបានវ ិសមភាពព ិត។ សមភាពយកើតមាន លុ េះទ្តាដត 𝑥 = 𝑦 = 𝑧 មានន័យថា 𝑎 = 𝑏 = 𝑐 ។

196.   ដំន

ោះត្ាយទី១

យយើ ងមាន 𝑎 − 𝑏 𝑎2 − 𝑏 2 ≥ 0 ⟹ 𝑎3 + 𝑏 3 ≥ 𝑎𝑏 𝑎 + 𝑏 យោយអងគទង ាំ ព ីរយសមើ គ្នន ទល់ ដត និងមានដត 𝑎 = 𝑏 ។ ដូយចនេះ 106

១. អនុគមន៍ងាយ | លឹម សុ វណ្ណវិចិត្រ


1 1 𝑐 ≤ = 3 + 𝑏 + 𝑎𝑏𝑐 𝑎𝑏 𝑎 + 𝑏 + 𝑎𝑏𝑐 𝑎𝑏𝑐 𝑎 + 𝑏 + 𝑐 1 𝑎 ≤ 3 3 𝑏 + 𝑐 + 𝑎𝑏𝑐 𝑎𝑏𝑐 𝑎 + 𝑏 + 𝑐 1 𝑏 ≤ 3 3 𝑐 + 𝑎 + 𝑎𝑏𝑐 𝑎𝑏𝑐 𝑎 + 𝑏 + 𝑐 𝑎3

ដូយចនេះ 𝑎3

 ដំន

1 1 1 𝑐+𝑏+𝑎 1 + 3 + 3 ≤ = 3 3 3 + 𝑏 + 𝑎𝑏𝑐 𝑏 + 𝑐 + 𝑎𝑏𝑐 𝑐 + 𝑎 + 𝑎𝑏𝑐 𝑎𝑏𝑐 𝑎 + 𝑏 + 𝑐 𝑎𝑏𝑐

ោះត្ាយទី២

វ ិសមភាពយនេះសមមូលនឹង 𝑎3 + 𝑏 3 + 𝑎𝑏𝑐 𝑏 3 + 𝑐 3 + 𝑎𝑏𝑐 𝑎𝑏𝑐 𝑠𝑦𝑚

≤ 2 𝑎3 + 𝑏 3 + 𝑎𝑏𝑐 𝑏 3 + 𝑐 3 + 𝑎𝑏𝑐 𝑐 3 + 𝑎3 + 𝑎𝑏𝑐 𝑎7 𝑏𝑐 + 3𝑎4 𝑏 4 𝑐 + 4𝑎5 𝑏2 𝑐 2 + 𝑎3 𝑏 3 𝑐 3

⟺ 𝑠𝑦𝑚

𝑎3 𝑏 3 𝑐 3 + 2𝑎6 𝑏3 + 3𝑎4 𝑏 4 𝑐 + 𝑎7 𝑏𝑐 + 2𝑎5 𝑏 2 𝑐 2

≤ 𝑠𝑦𝑚 6 3

2𝑎 𝑏 − 2𝑎5 𝑏 2 𝑐 2 ≥ 0

⟺ 𝑠𝑦𝑚

𝑎6 𝑏 3 𝑐 0 ≥

⟺ 𝑠𝑦𝑚

𝑎5 𝑏 2 𝑐 2 𝑠𝑦𝑚

ព ិតតាមវ ិសមភាព Muirhead យទ្ េះ 6; 3; 0 លុ បយលើ 5; 2; 2 ។ សេុី ត 6; 3; 0 លុ បយលើ 5; 2; 2 យទ្ េះ 6 > 5; 6 + 3 = 9 > 5 + 2 = 7; 6 + 3 + 0 = 5 + 2 + 2 ។

197.  យយើ ងមាន 𝑎5 + 𝑏 5 = 𝑎 + 𝑏 𝑎4 − 𝑎3 𝑏 + 𝑎2 𝑏 2 − 𝑎𝑏 3 + 𝑏 4 = 𝑎 + 𝑏 𝑎 − 𝑏 𝑎3 − 𝑏 3 + 𝑎2 𝑏 2 ≥ 𝑎 + 𝑏 𝑎2 𝑏 2 យទ្ េះ 𝑎 − 𝑏 𝑎3 − 𝑏 3 ≥ 0 ។ 𝑎𝑏 𝑎𝑏 ⟹ 5 ≤ 5 𝑎 + 𝑏 + 𝑎𝑏 𝑎 + 𝑏 𝑎2 𝑏2 + 𝑎𝑏 1 = 𝑎𝑏 𝑎 + 𝑏 + 1 លឹម សុ វណ្ណវិចិត្រ | V. វិសមភាព

107


1 𝑎𝑏 𝑎 + 𝑏 + 𝑎𝑏𝑐 1 = 𝑎𝑏 𝑎 + 𝑏 + 𝑐 𝑐 = 𝑎+𝑏+𝑐 តាមរយបៀបដូចគ្នន ចាំ យ េះតួយផេងយទៀត។ យយើ ងទញបាន 𝑎𝑏 𝑏𝑐 𝑐𝑎 𝑐 𝑎 𝑏 + 5 + 5 ≤ + + =1 5 5 5 5 𝑎 + 𝑏 + 𝑎𝑏 𝑏 + 𝑐 + 𝑏𝑐 𝑐 + 𝑎 + 𝑐𝑎 𝑎 + 𝑏 + 𝑐 𝑎 + 𝑏 + 𝑐 𝑎 + 𝑏 + 𝑐 សមភាពយកើតមានយពល 𝑎 = 𝑏 = 𝑐 ។ =

198. តាង

𝑛

𝑛

𝑎𝑖2

𝐴=

𝑏𝑖2

𝑖=1 𝑛

𝑖=1 2

𝐵=

𝑎𝑖 𝑏𝑖 𝑖=1 𝑛

𝑛

𝑎𝑖2 𝑏𝑖2

𝐴−𝐵 = 𝑖=1

𝑎𝑖2 𝑏𝑗2

+ 𝑖≠𝑗

𝑖=1

𝑎𝑖2 𝑏𝑗2 − 2

= 𝑖≠𝑗

𝑎𝑖2 𝑏𝑖2 + 2

𝑎𝑖 𝑏𝑖 𝑎𝑗 𝑏𝑗 1≤𝑖<𝑗 ≤𝑛

𝑎𝑖 𝑏𝑖 𝑎𝑗 𝑏𝑗 1≤𝑖<𝑗 ≤𝑛

𝑎𝑖2 𝑏𝑗2 +

= 1≤𝑖<𝑗 ≤𝑛

𝑎𝑗2 𝑏𝑖2 − 2 1≤𝑖<𝑗 ≤𝑛

𝑎𝑖 𝑏𝑖 𝑎𝑗 𝑏𝑗 1≤𝑖<𝑗 ≤𝑛

𝑎𝑖2 𝑏𝑗2 + 𝑎𝑗2 𝑏𝑖2 − 2𝑎𝑖 𝑏𝑖 𝑎𝑗 𝑏𝑗

= 1≤𝑖<𝑗 ≤𝑛

=

𝑎𝑖 𝑏𝑗 − 𝑎𝑗 𝑏𝑖

2

≥0

1≤𝑖<𝑗 ≤𝑛

⟹ សាំ យណើព ិត។

199. តាមវ ិសមភាពកូសុី ាេត 𝑥1 + 𝑥2 + ⋯ + 𝑥𝑛

1 1 1 + + ⋯+ ≥ 𝑥1 𝑥2 𝑥𝑛

200. តាមវ ិសមភាពកូសុីាេត 𝑎+𝑏+𝑏+𝑐+𝑐+𝑐 108

2

𝑛

𝑥𝑖 . 𝑖=1

1 𝑥𝑖

2

= 𝑛2

≤ 1 + 1 + 1 + 1 + 1 + 1 𝑎2 + 𝑏 2 + 𝑏 2 + 𝑐 2 + 𝑐 2 + 𝑐 2

១. អនុគមន៍ងាយ | លឹម សុ វណ្ណវិចិត្រ


𝑎 + 2𝑏 + 3𝑐 2 ≤6 𝑎2 + 2𝑏 2 + 3𝑐 2

201. តាមវ ិសមភាពកូសុីាេត 1 2 1 2 1 1 1 ⟹3 + 𝑏+ + 𝑐+ ≥ 𝑎+ + 𝑏+ + 𝑐+ 𝑏 𝑐 𝑎 𝑏 𝑐 2 2 1 1 1 1 1 1 = 𝑎+𝑏+𝑐+ + + = 1+ + + 𝑎 𝑏 𝑐 𝑎 𝑏 𝑐 1 𝑎+ 𝑎

2

2

2

≥ 1+

3 1 3

តាមវ ិសមភាពកូសុី

𝑎𝑏𝑐 2 9 ≥ 1+ តាមវ ិសមភាពកូសុី 𝑎+𝑏+𝑐 2 = 1 + 9 = 100 1 2 1 ⟹ 𝑎+ + 𝑏+ 𝑎 𝑏

2

+ 𝑐+

1 𝑐

2

100 3

202. តាមវ ិសមភាពកូសុី ាេត 2

𝑎+𝑏+𝑐+𝑑

≤ 1 + 1 + 1 + 1 𝑎2 + 𝑏 2 + 𝑐 2 + 𝑑 2 = 4 𝑎2 + 𝑏 2 + 𝑐 2 + 𝑑 2

⟹ 8 − 𝑒 2 ≤ 4 16 − 𝑒 2 ⟹ 𝑒 5𝑒 − 16 ≤ 0 16 ⟹ 0≤𝑒≤ 5 យដើមបីឱ្ែ 𝑒 = 16 5 លកខខណឌគឺ 𝑎 = 𝑏 = 𝑐 = 𝑑 និង 𝑎 + 𝑏 + 𝑐 + 𝑑 = 24 5 ⟹ 𝑎 = 𝑏 = 𝑐 = 𝑑 = 6/5។

203.  តាមវ ិសមភាពកូសុី ាេត យយើ ងមាន 𝑛

យយើ ងមាន

𝑖=1 𝑛

𝑖=1

𝑠 − 𝑎𝑖 = 𝑠

 មយ៉ាងវ ិញយទៀត

𝑛

𝑖=1

លឹម សុ វណ្ណវិចិត្រ | V. វិសមភាព

1 1− 𝑠

𝑠 𝑠 − 𝑎𝑖

𝑛

𝑖=1

𝑠 − 𝑎𝑖 ≥ 𝑛2 𝑠

𝑛

𝑛

𝑎𝑖 = 𝑛 − 1 ⟹ 𝑖=1

𝑖=1

𝑠 𝑛2 ≥ 𝑠 − 𝑎𝑖 𝑛−1

ព ិត។

109


𝑛

𝑖=1

តាមវ ិសមភាពកូសុីាេត

យោយ

𝑛

𝑖=1 𝑛

𝑛

𝑎𝑖 𝑠 = 1 ⟹ ព ិត។

𝑠 − 𝑎𝑖 = −𝑛 + 𝑎𝑖

𝑖=1

𝑖=1

𝑛

𝑠 𝑎𝑖

𝑖=1

𝑛

𝑖=1

𝑠 𝑎𝑖

(∗)

𝑎𝑖 ≥ 𝑛2 𝑠 𝑛

𝑠 ≥ 𝑛2 ; (∗) ⟹ 𝑎𝑖

𝑖=1

𝑠 − 𝑎𝑖 ≥ −𝑛 + 𝑛2 = 𝑛 𝑛 − 1 𝑎𝑖

 យយើ ងមាន 𝑛

𝑛

𝑎𝑖2

1 𝑖=1

ដតថា

𝑎𝑖

𝑛

𝑎𝑖2 ≥

𝑖=1 𝑛

𝑎𝑖 𝑠 − 𝑎𝑖 𝑖=1 𝑛

ព ិត។

𝑖=1

𝑛

=

2

𝑛

𝑖=1

𝑎𝑖 ≥ 𝑠 − 𝑎𝑖

𝑎𝑖 𝑠2 ⟹= ≥ 𝑛 𝑠 − 𝑎𝑖 ∑𝑖=1 𝑎𝑖 𝑠 − 𝑎𝑖 𝑖=1 𝑠2 𝑛 ≥ 2 =𝑛−1 𝑠 𝑠2 − 𝑛

𝑖=1

𝑠2 𝑛

2

𝑛

𝑎𝑖

= 𝑠2

𝑖=1

=

𝑠2 𝑠 ∑𝑛𝑖=1 𝑎𝑖 − ∑𝑛𝑖=1 𝑎𝑖2

204. ក) សាំ នួរយនេះគ្នមនលកខណៈព ិយសស្យលើលាំោប់ 𝑎1 ; 𝑎2 ; 𝑎3 យទ មានន័យថា យបើយយើ ងផ្ទាស់ 𝑎1 យៅ 𝑎2 ; 𝑎2 យៅ 𝑎1 ។ល។យ េះសមម តក ិ មមយៅដដដលគ្នមនដទ្បទ្បួល។ តាមលកខណៈយនេះ យយើ ងោច សនមតថា 𝑎1 ≤ 𝑎2 ≤ 𝑎3 ។ ដូយចនេះ យយើ ងទ្គ្នន់ ដតបង្ហាញថា 𝑎1 + 𝑎2 > 𝑎3 យៅបានយហើ យ។ យយើ ងមាន = 𝑎12 + 𝑎22 + 𝑎32 2 > 2 𝑎14 + 𝑎24 + 𝑎34 ⟺ = 𝑎1 + 𝑎2 + 𝑎3 𝑎1 + 𝑎2 − 𝑎3 −𝑎1 + 𝑎2 + 𝑎3 𝑎1 − 𝑎2 + 𝑎3 > 0 កតាតនិមួយៗសុ ទដធ តវ ិជាមាន យលើកដលងដត 𝑎1 + 𝑎2 − 𝑎3 ដដលមិនទន់ ដង ឹ ។ ដត ផលគុណរបស់ កតាត ទាំងអស់ វ ិជាមាន ដូយចនេះ 𝑎1 + 𝑎2 − 𝑎3 ទ្តូវដតវ ិជាមាន។ ខ) ករណី 𝑛 = 3 បានទ្ាយបញ្ជាក់ រច ួ កនង ុ សាំ នួរក)។ យយើ ងសនមតថា 𝑛 ≥ 4 ។

110

១. អនុគមន៍ងាយ | លឹម សុ វណ្ណវិចិត្រ


តាមដូចយរៀបរាប់ កនង ុ សាំ នួរក) យយើ ងទ្គ្នន់ ដតបង្ហាញថា 𝑎1 ; 𝑎2 ; 𝑎3 ជារង្ហេស់ ទ្ជុងននទ្តីយោណ ជាោរ ទ្គប់ ទ្គ្នន់ យហើ យ។ តាមវ ិសមភាពកូសុី ាេត 𝑛 − 1 𝑎14 + 𝑎24 + ⋯ + 𝑎𝑛4 < 𝑎12 + 𝑎22 + ⋯ + 𝑎𝑛2 𝑎12 + 𝑎22 + 𝑎32 𝑎12 + 𝑎22 + 𝑎32 = + + 2 2 ≤ 𝑛−1

𝑎12 + 𝑎22 + 𝑎32 4

⟺ 2 𝑎14 + 𝑎24 + 𝑎34 < 𝑎12 + 𝑎22 + 𝑎32

2

2

2

2

𝑛

𝑎𝑘2 𝑘=4

𝑎12 + 𝑎22 + 𝑎32 + 4

𝑛

2

𝑎𝑘4

+ 𝑘=4

យហើ យតាម សាំ នួរ ក) យយើ ងទញបានថាសាំ យនើព ិត។

205. តាមវ ិសមភាពកូសុី ាេត ចាំ យ េះទ្គប់ 𝑖 យយើ ងមាន 2 2 𝑆2 − 𝑎𝑖2 = 𝑎12 + 𝑎22 + ⋯ + 𝑎𝑖−1 + 𝑎𝑖+1 + ⋯ + 𝑎𝑛2 1 1 ≥ 𝑎1 + 𝑎2 + ⋯ + 𝑎𝑖−1 + 𝑎𝑖+1 + ⋯ + 𝑎𝑛 2 = 𝑆 − 𝑎𝑖 𝑛−1 𝑛−1 1 2 𝑆2 − 𝑎𝑖 1 ⟺ ≥ 𝑆 − 𝑎𝑖 𝑆1 − 𝑎𝑖 𝑛 − 1 1 𝑛 𝑛 𝑆2 − 𝑎𝑖2 1 ⟹ ≥ 𝑆1 − 𝑎𝑘 = 𝑆1 𝑆1 − 𝑎𝑖 𝑛 − 1 𝑘=1

2

𝑘=1

206. តាង 𝐸 = 𝑎, 𝑏, 𝑐, 𝑑 ∈ ℝ+ 𝑎2 + 𝑏 2 = 𝑐 2 + 𝑑2

3

និង 𝑓 ជាអនុគមន៍ កាំនត់ យលើ 𝐸

យោយ 𝑐 3 𝑑3 + −1 𝑎 𝑏 យយើ ងយឃើ ញថា ចាំ យ េះ ∀ 𝜆 ∈ ℝ+: 𝑎, 𝑏, 𝑐, 𝑑 ∈ 𝐸 និង 𝑓 𝑎, 𝑏, 𝑐, 𝑑 ≥ 0 𝑓 𝑎, 𝑏, 𝑐, 𝑑 =

𝑎2 + 𝑏 2 = 𝑐 2 + 𝑑 2

3

⟺ 𝜆3 𝑎 2 + 𝜆3 𝑏 2 = 𝜆𝑐 2 + 𝜆𝑑 2 3 ⟺ 𝑎13 + 𝑏13 = 𝑐12 + 𝑑12 3 3 3 𝑐 𝑑 𝜆𝑐 3 𝜆𝑑 3 𝑓 𝑎, 𝑏, 𝑐, 𝑑 = + − 1 = 3 + 3 − 1 = 𝑓 𝜆3 𝑎, 𝜆3 𝑏, 𝜆𝑐, 𝜆𝑑 𝑎 𝑏 𝜆 𝑎 𝜆 𝑏 = 𝑓 𝑎1 , 𝑏1 , 𝑐1 , 𝑑1 យ េះ 𝑎1 = 𝜆3 𝑎, 𝑏1 = 𝜆3 𝑏, 𝑐1 = 𝜆𝑐, 𝑑1 = 𝜆𝑑 ∈ 𝐸 និង 𝑓 𝑎, 𝑏, 𝑐, 𝑑 ≥ 0 ⟺ 𝑓 𝑎1 , 𝑏1 , 𝑐1 , 𝑑1 ≥ 0 ។ លកខណៈខាងយលើយនេះព ិតចាំ យ េះ∀ 𝜆 ∈ ℝ+ ដូយចនេះយយើ ង ោចយទ្ជើសយរ ើសយក 𝜆 ដដលយផទៀងផ្ទទត់ 𝑎12 + 𝑏12 = 1 ។ ដតយយើ ងោចជាំនស ួ សាំ យនរ 𝑎1 , 𝑏1 , 𝑐1 , 𝑑1

លឹម សុ វណ្ណវិចិត្រ | V. វិសមភាព

111


យោយ 𝑎, 𝑏, 𝑐, 𝑑 យោយគ្នមនបញ្ជាអេី ទង ាំ អស់ ។ ដូយចនេះ មានន័យថាយយើ ងោចយក 𝑎, 𝑏, 𝑐, 𝑑 ∈ 𝐸 និង 𝑎2 + 𝑏 2 = 1 យោយមិនយធេយើ ោយបាត់ ភាពទូយៅយឡើយ។ ដូយចនេះ 𝑐 2 + 𝑑2 = 1 ។ តាមវ ិសមភាពកូសុី ាេត

យាំ ោយ 𝑐 3 𝑑3 + 𝑎 𝑏

𝑎𝑐 + 𝑏𝑑 ≥ 𝑐 2 + 𝑑2

2

=1

ដតយយើ ងមាន 𝑎2 + 𝑐 2 𝑏 2 + 𝑑 2 1 2 + = 𝑎 + 𝑏2 + 𝑐 2 + 𝑑2 = 1 2 2 2 𝑐 3 𝑑3 1 ⟹= + ≥ ≥1 𝑎 𝑏 𝑎𝑐 + 𝑏𝑑 207. តាមវ ិសមភាពកូសុី ាេត = 𝑎𝑐 + 𝑏𝑑 ≤

𝑥−1+ 𝑦−1+ 𝑧−1 ⟺

2

≤ 𝑥+𝑦+𝑧

𝑥−1+ 𝑦−1+ 𝑧−1≤

𝑥+𝑦+𝑧

𝑥−1 𝑦−1 𝑧−1 + + 𝑥 𝑦 𝑧 𝑥−1 𝑦−1 𝑧−1 + + 𝑥 𝑦 𝑧

យយើ ងមាន 𝑥−1 𝑦−1 𝑧−1 1 1 1 + + =3− + + =1 𝑥 𝑦 𝑧 𝑥 𝑦 𝑧 ដូយចនេះ 𝑥−1+ 𝑦−1+ 𝑧−1≤

𝑥+𝑦+𝑧

𝑥−1 𝑦−1 𝑧−1 + + = 𝑥 𝑦 𝑧

វ ិសមភាពព ិត។ យយើ ងមានសមភាព លុ េះទ្តាដត 𝑥−1 𝑧−1 𝑧−1 1 1 1 3 = 2 = 2 ; + + =2 ⟹𝑥=𝑦=𝑧= 2 𝑥 𝑧 𝑧 𝑥 𝑦 𝑧 2

208. តាង 𝑎 = 1 𝑥 ; 𝑏 = 1 𝑦 ; 𝑐 = 1 𝑧 ។ លកខខណឌ 𝑥𝑦𝑧 ≥ 𝑥𝑦 + 𝑦𝑧 + 𝑧𝑥 ⟺

1 1 1 + + ≤1 ⟹𝑎+𝑏+𝑐 ≤1 𝑥 𝑦 𝑧

វ ិសមភាព 𝑥𝑦𝑧 ≥ 3 𝑥 + 𝑦 + 𝑧 យយើ ងមាន 112

១. អនុគមន៍ងាយ | លឹម សុ វណ្ណវិចិត្រ

⟺ 𝑎𝑏 + 𝑏𝑐 + 𝑐𝑎 ≤

1 3

𝑥+𝑦+𝑧


1 ≼ đ?‘Ž + đ?‘? + đ?‘? 2 = đ?‘Ž2 + đ?‘? 2 + đ?‘? 2 + 2 đ?‘Žđ?‘? + đ?‘?đ?‘? + đ?‘?đ?‘Ž យហឞ áž™áž?ážśáž˜ážœ ឡáž&#x;áž˜áž—ážśáž–áž€ážźáž&#x;ឝី ážśá&#x; áž? យយឞ áž„áž˜ážśáž“ đ?‘Žđ?‘? + đ?‘?đ?‘? + đ?‘?đ?‘Ž ≤ đ?‘Ž2 + đ?‘? 2 + đ?‘? 2 đ?‘? 2 + đ?‘? 2 + đ?‘Ž2 â&#x;š đ?‘Ž2 + đ?‘? 2 + đ?‘? 2 ≼ đ?‘Žđ?‘? + đ?‘?đ?‘? + đ?‘?đ?‘Ž ដសយចនá&#x; á&#x;‡ យវá&#x;† á&#x;„áž™ 1 ≼ 3 đ?‘Žđ?‘? + đ?‘?đ?‘? + đ?‘?đ?‘Ž áž– ឡáž?á&#x;”

209.ď ° áž?ážśáž„ đ?‘Ž = tan đ?‘Ľ ; đ?‘? = tan đ?‘Ś ; đ?‘? = tan đ?‘§ áž™ á&#x; á&#x;‡ đ?‘Ľ, đ?‘Ś, đ?‘§ áž‡ážśáž˜ážťážśá&#x;†áž“áž“áž‘á&#x;’áž?ីយá&#x;„ណáž&#x;á&#x;’áž&#x;áž˝ áž…á&#x;” ដសយចនá&#x; á&#x;‡ ážœ ឡáž&#x;áž˜áž—ážśáž–áž&#x;áž˜áž˜ážźáž›áž“ážšáž„ max tan đ?‘Ľ , tan đ?‘Ś , tan đ?‘§ ≼ 3 á&#x;” ដáž?áž?ážś áž˜ážśáž“áž˜ážťážśá&#x;†áž˜áž˝áž™ážšáž”áž&#x;á&#x;‹ áž‘á&#x;’áž?ីយá&#x;„ណ ឧទហរណá&#x;? đ?‘Ľ ដដលធវá&#x;† ជវងរ ážşáž™áž&#x;មឞ đ?œ‹ 3 (យបឞ áž?សចជវងđ?œ‹ 3 áž‘ážśá&#x;†áž„អáž&#x;á&#x;‹ áž‚á&#x;’áž“áž“ áž™ á&#x; á&#x;‡ áž•áž›áž”ážźáž€áž˜ážťážśá&#x;†áž€áž“áž„ ážť áž‘á&#x;’áž?ីយá&#x;„ណ đ?œ‹ 3

áž˜ážśáž“áž?ážśá&#x;† áž“áž›áž?សចជវង đ?œ‹)á&#x;” áž™á&#x;„áž™áž˘áž“ážťáž‚áž˜áž“á&#x;? tan

យកឞនយលឞ 0, đ?œ‹ 2 áž™ á&#x; á&#x;‡ tan đ?‘Ľ ≼ tan = 3 á&#x;”

210.ď ° យយឞ áž„áž˜ážśáž“ đ?‘Ľ1 + đ?‘Ľ2 + â‹Ż + đ?‘Ľđ?‘› 4 = đ?‘Ľ12 + đ?‘Ľ22 + â‹Ż + đ?‘Ľđ?‘›2 + 2đ?‘Ľ1 đ?‘Ľ2 + 2đ?‘Ľ1 đ?‘Ľ3 + â‹Ż ≼ 4 đ?‘Ľ12 + đ?‘Ľ22 + â‹Ż + đ?‘Ľđ?‘›2 2đ?‘Ľ1 đ?‘Ľ2 + 2đ?‘Ľ1 đ?‘Ľ3 + â‹Ż ážœ ឡáž&#x;áž˜áž—ážśáž–áž€ážźáž&#x;ឝី 1 = 8 đ?‘Ľ12 + đ?‘Ľ22 + â‹Ż + đ?‘Ľđ?‘›2

2

đ?‘Ľđ?‘– đ?‘Ľđ?‘— đ?‘–<đ?‘—

đ?‘Ľđ?‘– đ?‘Ľđ?‘— đ?‘Ľ12 + đ?‘Ľ22 + â‹Ż + đ?‘Ľđ?‘›2

=8 đ?‘–<đ?‘—

đ?‘Ľđ?‘– đ?‘Ľđ?‘— đ?‘Ľđ?‘–2 + đ?‘Ľđ?‘—2

≼8

(2)

đ?‘–<đ?‘—

យយឞ áž„áž˜ážśáž“áž&#x;áž˜áž—ážśáž–

áž‘áž›á&#x;‹ ដáž?áž“ážˇáž„áž˜ážśáž“ážŠáž? áž˜ážśáž“áž™á&#x;‰ážśáž„áž™á&#x;„áž…á&#x;’áž&#x;á&#x;‹ đ?‘Ľđ?‘– áž…ážśá&#x;† áž“áž“ áž˝ đ?‘› − 2 ដដលយáž&#x;មឞ áž&#x;ážźáž“á&#x;‚á&#x;”

ឧទហរណá&#x;? đ?‘Ľ3 = â‹Ż = đ?‘Ľđ?‘› = 0 á&#x;” កនង ážť លកហហណឌយនá&#x; á&#x;‡ យយឞ áž„áž˜ážśáž“áž&#x;áž˜áž—ážśáž–(1) áž‘áž›á&#x;‹ ដáž?áž“ážˇáž„áž˜ážśáž“ážŠáž? 2đ?‘Ľ1 đ?‘Ľ2 = đ?‘Ľ12 + đ?‘Ľ22 áž˜ážśáž“áž“á&#x;?áž™áž?ážś đ?‘Ľ1 = đ?‘Ľ2 á&#x;” ដសយចនá&#x; á&#x;‡ đ??ś = 1 8 áž™á&#x;„យអងគទង ážśá&#x;† áž– ីរយáž&#x;មឞ áž‚á&#x;’áž“áž“ áž‘áž›á&#x;‹ ដáž?áž“ážˇáž„áž˜ážśáž“ážŠáž? áž˜ážśáž“áž™á&#x;‰ážśáž„áž™á&#x;„áž…á&#x;’áž&#x;á&#x;‹ đ?‘Ľđ?‘– áž…ážśá&#x;† áž“áž“ áž˝ đ?‘› − 2 áž?áž˝ ដដលយáž&#x;មឞ áž&#x;ážźáž“á&#x;‚ យហឞ áž™áž?ួព ីរយទá&#x;€áž?áž™áž&#x;មឞ áž‚á&#x;’áž“áž“á&#x;”

211.ď ° យយឞ áž„áž˜ážśáž“ đ?‘ƒ2 = đ?‘Ľđ?‘Ś + đ?‘Śđ?‘§ + đ?‘§đ?‘Ą + đ?‘Ąđ?‘Ľ ដសយចនá&#x; á&#x;‡ −1 ≤ đ?‘ƒ ≤ 1 á&#x;” 1

2

≤ �2 + �2 + �2 + �2 �2 + �2 + �2 + �2 = 1

1

យបឞ đ?‘Ľ = đ?‘§ = 2 ; đ?‘Ś = đ?‘Ą = − 2 áž™ á&#x; á&#x;‡ đ?‘Ľ + đ?‘Ś + đ?‘§ + đ?‘Ą = 0; đ?‘Ľ 2 + đ?‘Ś 2 + đ?‘§ 2 + đ?‘Ą 2 = 1 á&#x;” យហឞ áž™ 1

1

1

đ?‘ƒ = 2. −2 + −2

1 2

លរម áž&#x;ážť វណá&#x;’ážŽážœážˇáž…ážˇáž?á&#x;’ážš | V. ážœážˇáž&#x;áž˜áž—ážśáž–

+

1 2

1

1

−2 + −2

1 2

= −1 á&#x;” ដសយចនá&#x; á&#x;‡ đ?‘ƒmin = −1 á&#x;”

113


យយឞ áž„áž˜ážśáž“ đ?‘ƒ = đ?‘Ľ + đ?‘§ đ?‘Ś + đ?‘Ą = − đ?‘Ś + đ?‘Ą

2

≤ 0 á&#x;” យបឞ đ?‘Ľ = đ?‘Ś = 1 2 ; đ?‘§ = đ?‘Ą = − 1 2 áž™ á&#x; á&#x;‡

đ?‘ƒ = 0 á&#x;” ដសយចនá&#x; á&#x;‡ đ?‘ƒmax = 0 á&#x;” ជវáž&#x;ážšážťáž” đ?‘ƒmin = −1 ឧទហរណá&#x;?áž‘á&#x;’áž?áž„á&#x;‹ đ?‘Ľ = đ?‘§ = 1 2 ; đ?‘Ś = đ?‘Ą = − 1 2 á&#x;” đ?‘ƒmax = 0 ឧទហរណá&#x;?áž‘á&#x;’áž?áž„á&#x;‹ đ?‘Ľ = đ?‘Ś = 1 2 ; đ?‘§ = đ?‘Ą = − 1 2 á&#x;”

212.ď ° យយឞ áž„áž?ážśáž„ đ?‘Ľ1 = tan đ?›ź1 , đ?‘Ľ2 = sec đ?›ź1 tan đ?›ź2 នឡង áž™á&#x;„áž™

đ?œ‹ − 2

đ?‘Ľđ?‘˜ = sec đ?›ź1 sec đ?›ź2 ‌ sec đ?›źđ?‘˜âˆ’1 tan đ?›źđ?‘˜ < đ?›źđ?‘˜ < , 1 ≤ đ?‘˜ ≤ đ?‘› á&#x;” យយឞ ងគួរកáž?á&#x;‹ áž&#x;ážśá&#x;†áž‚á&#x;’áž“áž›á&#x;‹ áž?ážśá&#x;„រជវá&#x;†áž“áž&#x; áž˝ យនá&#x; á&#x;‡áž™áž‘á&#x;’ á&#x; á&#x;‡áž›ážśá&#x;† á&#x;„áž”á&#x;‹ ážšáž”áž&#x;á&#x;‹ tan đ?›ź áž‚ážş đ?œ‹ 2

−∞, ∞ នឡង sec đ?›ź áž ážťáž&#x;áž– ីáž&#x;ážź áž“á&#x;‚ជវនឡចá&#x;” áž… ដសយចនá&#x; á&#x;‡áž?ួទđ?‘˜ ី ážšáž”áž&#x;á&#x;‹ អងគហវងយឆá&#x; áž„áž“áž“ážœ ឡáž&#x;áž˜áž—ážśáž–áž™áž&#x;មឞ áž“áž„ ážš sec đ?›ź1 ‌ sec đ?›źđ?‘˜âˆ’1 tan đ?›źđ?‘˜ 1 + tan2 đ?›ź1 + â‹Ż + sec 2 đ?›ź1 ‌ sec 2 đ?›źđ?‘›âˆ’1 tan2 đ?›źđ?‘› = cos đ?›ź1 cos đ?›ź2 ‌ cos đ?›źđ?‘˜ sin đ?›źđ?‘˜ ដសយចនá&#x; á&#x;‡ážœ ឡáž&#x;áž˜áž—ážśáž–ážŠážŠáž›áž™á&#x;„áž™áž&#x;áž˜áž˜ážźáž›áž“ážšáž„ cos đ?›ź1 sin đ?›ź1 + cos đ?›ź1 cos đ?›ź2 sin đ?›ź2 + â‹Ż + cos đ?›ź1 cos đ?›ź2 ‌ cos đ?›źđ?‘› sin đ?›źđ?‘› < đ?‘› â&#x;ş đ?‘?1 đ?‘ 1 + đ?‘?1 đ?‘?2 đ?‘ 2 + â‹Ż + đ?‘?1 đ?‘?2 ‌ đ?‘?đ?‘› đ?‘ đ?‘› < đ?‘› ដដល đ?‘?đ?‘– = cos đ?›źđ?‘– នឡង đ?‘ đ?‘– = sin đ?›źđ?‘– áž…ážśá&#x;† áž™ á&#x; á&#x;‡ 1 ≤ đ?‘– ≤ đ?‘› á&#x;” áž…ážśá&#x;† áž™ á&#x; á&#x;‡2 ≤ đ?‘– ≤ đ?‘› áž™á&#x;„យវរ đ?‘?đ?‘–2 + đ?‘ đ?‘–2 = cos2 đ?›źđ?‘– + sin2 đ?›źđ?‘– = 1 យយឞ ងទញបវនáž?ážś 2 2 2 đ?‘?12 đ?‘?22 ‌ đ?‘?đ?‘–−1 đ?‘ đ?‘–2 + đ?‘?12 đ?‘?22 ‌ đ?‘?đ?‘–−1 đ?‘?đ?‘–2 = đ?‘?12 đ?‘?22 ‌ đ?‘?đ?‘–−1 2 2 2 â&#x;š đ?‘ 12 + đ?‘?12 đ?‘ 22 + â‹Ż + đ?‘?12 đ?‘ 22 ‌ đ?‘?đ?‘›âˆ’2 đ?‘ đ?‘›âˆ’1 + đ?‘?12 đ?‘ 22 ‌ đ?‘?đ?‘›âˆ’1 = 1 (∗) áž?ážśáž˜(*) នឡងáž?ážśáž˜ážœ ឡáž&#x;áž˜áž—ážśáž–áž€ážźáž&#x;ឝីវá&#x; áž? យយឞ ងទញបវន

đ?‘?1 đ?‘ 1 + đ?‘?1 đ?‘?2 đ?‘ 2 + â‹Ż + đ?‘?1 đ?‘?2 ‌ đ?‘?đ?‘› đ?‘ đ?‘› ≤

2 2 2 2 đ?‘ 12 + đ?‘?12 đ?‘ 22 + â‹Ż + đ?‘?12 đ?‘?22 ‌ đ?‘?đ?‘›âˆ’2 đ?‘ đ?‘›âˆ’1 + đ?‘?12 đ?‘?22 ‌ đ?‘?đ?‘›âˆ’1 . đ?‘?12 + đ?‘?22 + â‹Ż + đ?‘?đ?‘›âˆ’1 + đ?‘?đ?‘›2 đ?‘ đ?‘›2

=

2 đ?‘?12 + đ?‘?22 + â‹Ż + đ?‘?đ?‘›âˆ’1 + đ?‘?đ?‘›2 đ?‘ đ?‘›2

= cos2 đ?›ź1 + cos 2 đ?›ź2 + â‹Ż + cos2 đ?›źđ?‘›âˆ’1 + cos 2 đ?›źđ?‘› sin2 đ?›źđ?‘› ≤ đ?‘› ážœ ឡáž&#x;áž˜áž—ážśáž–áž…ážťáž„áž™áž‘á&#x;’á&#x;„áž™ á&#x;„វយជវáž&#x;áž˜áž—ážśáž– áž™á&#x;…យពលដដល cos đ?›ź1 = cos đ?›ź2 = â‹Ż = cos đ?›źđ?‘›âˆ’1 = cos đ?›źđ?‘› sin đ?›źđ?‘› = 1 1 áž&#x;áž˜áž—ážśáž–áž™áž“á&#x; á&#x;‡áž˜ážˇáž“á&#x;„ចយទ យទá&#x;’ á&#x; á&#x;‡ cos đ?›źđ?‘› sin đ?›źđ?‘› = 2 sin 2đ?›źđ?‘› < 1á&#x;” ដសយចនá&#x; á&#x;‡ យយឞ áž„áž˜ážśáž“ážœ ឡáž&#x;áž˜áž—ážśáž– á&#x;„áž…á&#x;‹ áž ážśáž?á&#x;”

213.ď ° áž?ážśáž„ đ?‘Ž = 1 đ?‘Ľ , đ?‘? = 1 đ?‘Ś , đ?‘? = 1/đ?‘§ áž™ á&#x; á&#x;‡ đ?‘Ž, đ?‘?, đ?‘? ∈ 0; 1 នឡង đ?‘Ž + đ?‘? + đ?‘? = 2á&#x;” ážœ ឡáž&#x;áž˜áž—ážśáž–ážŠážŠáž›áž™á&#x;„áž™áž&#x;áž˜áž˜ážźáž›áž“ážšáž„

114

á&#x;Ą. áž˘áž“ážťáž‚áž˜áž“á&#x;?áž„ážśáž™ | លរម áž&#x;ážť វណá&#x;’ážŽážœážˇáž…ážˇáž?á&#x;’ážš


1 1 1 + + ≥ 𝑎 𝑏 𝑐 ⟺

⟺ ⟺

1 𝑎+𝑏+𝑐 2

1 1 1 + + ≥ 𝑎 𝑏 𝑐

1−𝑎 1−𝑏 1−𝑐 + + 𝑎 𝑏 𝑐 𝑎+𝑏+𝑐 𝑎+𝑏+𝑐 𝑎+𝑏+𝑐 −𝑎 −𝑏 −𝑐 2 2 2 + + 𝑎 𝑏 𝑐

1 1 1 + + ≥ 𝑎 𝑏 𝑐

𝑏+𝑐−𝑎 𝑐+𝑎−𝑏 𝑎+𝑏−𝑐 + + 𝑎 𝑏 𝑐 1 1 1 𝑏+𝑐−𝑎 + 𝑐+𝑎−𝑏 + 𝑎+𝑏−𝑐 + + 𝑎 𝑏 𝑐 𝑎+𝑏+𝑐

2

𝑏+𝑐−𝑎 𝑐+𝑎−𝑏 𝑎+𝑏−𝑐 + + 𝑎 𝑏 𝑐

ព ិត តាមវ ិសមភាពកូសុីាេត។

214. យោយចាំ នន ួ ចាំ លាស់ ននលាំ ោប់ 𝑏1 , 𝑏2 , … , 𝑏𝑛 មានចាំ នន ួ កាំនត់ (ទាំងអស់ មាន 𝑛! រយបៀប) យ េះ មានមួយដបបដដល 𝑆 មានតាំ នលធាំ បាំផត ុ (ដូចគ្នន តូចបាំ ផត ុ )។ តាង 𝑖 < 𝑗 ជាសនទសេន៍ព ីរ, តាង 𝜎 ជាចាំ លាស់ នន 1,2, … , 𝑛 យហើ យសនមតថា 𝜎 𝑖 > 𝜎 𝑗 ។ ដូយចនេះ𝑏𝜎

𝑗

≥ 𝑏𝜎

𝑖

យទ្ េះសេុី ត 𝑏𝑘 យកើន។

តាង 𝜎′ជាចាំ លាស់ នន 1,2, … , 𝑛 ដដលដូចគ្នននឹង 𝜎 ដដរ ដតខុសទ្តង់ 𝑖, 𝑗 ដដល 𝜎 ′ 𝑗 = 𝜎 𝑖 និង 𝜎 ′ 𝑖 = 𝜎 𝑗 ។ ដូយចនេះ 𝑆𝜎′ − 𝑆𝜎 = 𝑎𝑗 − 𝑎𝑖 𝑏𝜎  យបើ 𝑎𝑖 < 𝑎𝑗 និង 𝑏𝜎

𝑗

 យបើ 𝑎𝑖 = 𝑎𝑗 រ ឺ 𝑏𝜎

= 𝑏𝜎

𝑗

< 𝑏𝜎 𝑖

𝑖

𝑖

− 𝑏𝜎

𝑗

យ េះ 𝑆𝜎′ > 𝑆𝜎 មានន័យថា 𝑆𝜎 មិនដមនធាំ បាំផត ុ យទ

យ េះ 𝑆𝜎′ = 𝑆𝜎

ដូយចនេះ ជាំនស ួ 𝜎 យោយ 𝜎′ ផលបូកមិនថយចុេះយទ រ ឺក៏ោចធាំ ជាងមុន។ យបើ 𝜎 1 ≠ 1 ឧទហរណ៍ 𝜎 = 3; 1; 2; … ; 𝑛 ; 𝜎 1 = 3; 𝜎 2 = 1។ យយើ ងមានចាំ លាស់ 𝜎′ មួយ ដដល 𝜎 ′ 1 = 1 ឧទហរណ៍ 𝜎′ = 1; 3; 2; … ; 𝑛 ; 𝜎′ 1 = 1; 𝜎′ 2 = 3 ដដលតួទង ាំ អស់ ដូចគ្នន នឹង 𝜎 យលើកដលងដតតួទម ី ួយនឹងទីព ីរយចញ។ តាមលកខណៈខាងយលើ យយើ ងទញបាន 𝑆𝜎 ≤ 𝑆𝜎′ ។ មាន ន័យថា យរៀប 𝜎 = 3; 1; 2; … ; 𝑛 ជា 𝜎 = 1; 3; 2; … ; 𝑛 យយើ ងទញបាន 𝑆𝜎 ធាំ ជាងមុន។ មានន័យ ថាផលបូក 𝑎1 𝑏3 + 𝑎2 𝑏1 + 𝑎3 𝑏2 + ⋯ + 𝑎𝑛 𝑏𝑛 ≤ 𝑎1 𝑏1 + 𝑎2 𝑏3 + 𝑎3 𝑏2 + ⋯ + 𝑎𝑛 𝑏𝑛

លឹម សុ វណ្ណវិចិត្រ | V. វិសមភាព

115


áž” áž‘ áž”á&#x;‹ មកយទá&#x;€áž?យយឞ áž„áž…ážśáž”á&#x;‹ យផដម ážž ឆá&#x;’លវáž&#x;á&#x;‹ áž&#x;áž“áž‘áž&#x;á&#x; áž“á&#x;?áž…ážśáž”á&#x;‹ áž– ីទីá&#x;˘áž™á&#x;… ឧទហរណá&#x;? đ?œŽ = 1; 3; 2; ‌ ; đ?‘› á&#x;” យយឞ áž„áž˜ážśáž“ đ?œŽ 2 = 3, đ?œŽ 3 = 2 á&#x;” យយឞ áž„áž˜ážśáž“áž…ážśá&#x;† លវáž&#x;á&#x;‹ đ?œŽâ€˛ áž˜áž˝áž™ážŠážŠáž› đ?œŽ ′ 2 = 2 ឧទហរណá&#x;? đ?œŽâ€˛ = 1; 2; 3; ‌ ; đ?‘› ; đ?œŽâ€˛ 2 = 2; đ?œŽâ€˛ 3 = 3 ដដលáž?ួទង ážśá&#x;† អáž&#x;á&#x;‹ ដសចគá&#x;’áž“áž“ áž“ážšáž„ đ?œŽ យលឞកដលងដáž?áž?ួទព ី ីរនឡង ទីបីយចញá&#x;” áž?ážśáž˜áž›áž€áž ážŽá&#x;ˆáž វងយលឞ យយឞ ងទញបវន đ?‘†đ?œŽ ≤ đ?‘†đ?œŽâ€˛ á&#x;” áž˜ážśáž“áž“á&#x;?áž™áž?ážś យរá&#x;€áž” đ?œŽ = 1; 3; 2; ‌ ; đ?‘› ជវ đ?œŽ = 1; 2; 3; ‌ ; đ?‘› យយឞ ងទញបវន đ?‘†đ?œŽ áž’ážśá&#x;† áž‡ážśáž„áž˜ážťáž“á&#x;” áž˜ážśáž“áž“á&#x;?áž™áž?វផលបសក đ?‘Ž1 đ?‘?1 + đ?‘Ž2 đ?‘?3 + đ?‘Ž3 đ?‘?2 + â‹Ż + đ?‘Žđ?‘› đ?‘?đ?‘› ≤ đ?‘Ž1 đ?‘?1 + đ?‘Ž2 đ?‘?2 + đ?‘Ž3 đ?‘?3 + â‹Ż + đ?‘Žđ?‘› đ?‘?đ?‘› ដសចគá&#x;’áž“áž“ áž…ážśá&#x;† áž™ á&#x; á&#x;‡ đ?‘– = 3, ‌ , đ?‘› − 1 យយឞ ងទញបវន đ?œŽ ′ đ?‘– = đ?‘– ដដលយពលយ á&#x; á&#x;‡áž•áž›áž”ážźáž€áž˜ážśáž“áž?ážśá&#x;† នលធវá&#x;† á&#x;” ដសយចនá&#x; á&#x;‡áž•áž›áž”ážźáž€áž˜ážśáž“áž?ážśá&#x;† នលធវá&#x;† បវá&#x;†áž•áž? ážť យពល đ?œŽ đ?‘– = đ?‘– á&#x;” áž‘á&#x;’វយបញá&#x;’ជវកá&#x;‹ ដច ážź áž‚á&#x;’ននករណីáž?សចបវá&#x;† áž•áž? ážť á&#x;”

215.ď ° ážœ ឡáž&#x;áž˜áž—ážśáž–áž&#x;áž˜áž˜ážźáž›áž“ážšáž„ đ?‘›

đ?‘Ľđ?‘– − đ?‘Śđ?‘– đ?‘–=1

đ?‘› 2

≤

đ?‘Ľ2đ?‘–

đ?‘›

− 2đ?‘Ľđ?‘– đ?‘Śđ?‘– + đ?‘Ś2đ?‘–

đ?‘–=1

đ?‘›

−2đ?‘Ľđ?‘– đ?‘§đ?‘– + đ?‘§2đ?‘–

−2đ?‘Ľđ?‘– đ?‘Śđ?‘– + đ?‘Ś2đ?‘– ≤ đ?‘–=1

đ?‘Ľ2đ?‘– − 2đ?‘Ľđ?‘– đ?‘§đ?‘– + đ?‘§2đ?‘–

≤

đ?‘–=1 đ?‘›

â&#x;ş

2

đ?‘–=1

đ?‘›

â&#x;ş

đ?‘Ľđ?‘– − đ?‘§đ?‘–

đ?‘–=1

áž™á&#x;„áž™ ∑đ?‘›đ?‘–=1 đ?‘Śđ?‘–2 = ∑đ?‘›đ?‘–=1 đ?‘§đ?‘–2 áž™ á&#x; á&#x;‡ážœ ឡáž&#x;áž˜áž—ážśáž–áž&#x;áž˜áž˜ážźáž›áž“ážšáž„ đ?‘›

đ?‘›

�� �� ≼ �=1

ážœ ឡáž&#x;áž˜áž—ážśáž–áž™áž‘á&#x;’á&#x;„យយនá&#x; á&#x;‡áž– ឡáž? áž?ážśáž˜ážœ ឡáž&#x;áž˜áž—ážśáž–áž?ážśá&#x;† យរá&#x;€áž”á&#x;”

�� �� �=1

216.ď ° áž˜ážśáž“ đ?‘Ľ ∈ 0, đ?œ‹ 2 á&#x;” áž?ážśáž„ đ?‘Ž1 = sin3 đ?‘Ľ ; đ?‘Ž2 = cos3 đ?‘Ľ នឡង đ?‘?1 = 1 cos đ?‘Ľ ; đ?‘?2 = 1/ sin đ?‘Ľ á&#x;” យយឞ ងយឃឞ ញáž?ážś យបឞ đ?‘Ž1 ≤ đ?‘Ž2 áž™ á&#x; á&#x;‡ đ?‘?1 ≤ đ?‘?2 យហឞ áž™ យបឞ đ?‘Ž1 ≼ đ?‘Ž2 áž™ á&#x; á&#x;‡ đ?‘?1 ≼ đ?‘?2 á&#x;” ដសយចនá&#x; á&#x;‡ áž?ážśáž˜ážœ ឡáž&#x;áž˜áž—ážśáž–áž?ážśá&#x;† យរá&#x;€áž” 1 1 + cos3 đ?‘Ľ = sin2 đ?‘Ľ + cos2 đ?‘Ľ = 1 sin đ?‘Ľ cos đ?‘Ľ មយá&#x;‰ážśáž„ážœ ឡញយទá&#x;€áž? យយឞ ងដរងáž?ážś đ?‘“ đ?œ‹ 4 = 1,0 ដសយចនá&#x; á&#x;‡ áž?ážśá&#x;† áž“áž›áž?សចបវá&#x;† áž•áž? ážť ážšáž”áž&#x;á&#x;‹ đ?‘“ áž‚ážşáž™áž&#x;មឞ 1 á&#x;” đ?‘“ đ?‘Ľ = đ?‘Ž1 đ?‘?1 + đ?‘Ž2 đ?‘?2 ≼ đ?‘Ž1 đ?‘?2 + đ?‘Ž2 đ?‘?1 = sin3 đ?‘Ľ

116

á&#x;Ą. áž˘áž“ážťáž‚áž˜áž“á&#x;?áž„ážśáž™ | លរម áž&#x;ážť វណá&#x;’ážŽážœážˇáž…ážˇáž?á&#x;’ážš


217. យោយវ ិសមភាពយនេះគ្នមនលកខណៈព ិយសសចាំ យ េះតាំយរៀប 𝑎, 𝑏, 𝑐 យយើ ងោចសនមតថា 𝑎 ≥ 𝑏 ≥ 𝑐 ។ ដូយចនេះ 𝑎2 ≥ 𝑏 2 ≥ 𝑐 2 និង 𝑎𝑏 ≥ 𝑎𝑐 ≥ 𝑏𝑐 ។ តាមវ ិសមភាពតាំ យរៀប យយើ ងទញបាន 𝑎3 + 𝑏 3 + 𝑐 3 = 𝑎2 𝑎 + 𝑏 2 𝑏 + 𝑐 2 𝑐 ≥ 𝑎2 𝑏 + 𝑏 2 𝑐 + 𝑐 2 𝑎 𝑎2 𝑏 + 𝑏 2 𝑐 + 𝑐 2 𝑎 = 𝑎𝑏 𝑎 + 𝑎𝑐 𝑐 + 𝑏𝑐 𝑏 ≥ 𝑎𝑏 𝑐 + 𝑎𝑐 𝑏 + 𝑏𝑐 𝑎 = 3𝑎𝑏𝑐

218. យគយោយ 𝑛 ≥ 1។ តាមវ ិសមភាពតាំយរៀប យយើ ងមាន ∑𝑛𝑘=1 𝑘 𝑘2 មានតាំនលតូចបាំ ផត ុ យពល 𝑎

𝑎1 ≤ 𝑎2 ≤ ⋯ ≤ 𝑎𝑛 ។ យៅកនង ុ លកខខណឌដបបយនេះ (𝑎1 ≤ 𝑎2 ≤ ⋯ ≤ 𝑎𝑛 ) យហើ យ យោយារ 𝑎1 , 𝑎2 , … , 𝑎𝑛 ជាចាំ នន ួ គត់ វ ិជាមាន ដដលខុសគ្ននព ីរៗ យ េះ យយើ ងទញបាន 𝑎𝑖 ≥ 𝑖 ចាំ យ េះទ្គប់ 𝑖 ។ ដូយចនេះ យយើ ងទញបាន

𝑛

𝑘=1

𝑎𝑘 ≥ 𝑘2

𝑛

𝑘=1

1 𝑘

219.  រនរៀរទីមួយ យយើ ងយរៀប 𝑎𝑖 តាមលាំ ោប់ មួយដដល 𝑎𝑘 1 ≤ 𝑎𝑘 2 ≤ ⋯ ≤ 𝑎𝑘 𝑛 ។ ដូយចនេះ 1 1 1 ≥ ≥⋯≥ 𝑎𝑘 1 𝑎𝑘 2 𝑎𝑘 𝑛 ដូយចនេះតាមវ ិសមភាពតាំ យរៀប យយើ ងទញបាន

 រនរៀរទីពីរ

2 𝑎12 𝑎22 𝑎𝑛−1 𝑎𝑛2 1 1 1 + +⋯+ + = 𝑎12 + 𝑎22 + ⋯ + 𝑎𝑛2 𝑎2 𝑎3 𝑎𝑛 𝑎1 𝑎2 𝑎3 𝑎1 1 1 1 ≥ 𝑎𝑘2 1 + 𝑎𝑘2 2 + ⋯ + 𝑎𝑘2 𝑛 𝑎𝑘 1 𝑎𝑘 2 𝑎𝑘 𝑛 = 𝑎1 + 𝑎2 + ⋯ + 𝑎𝑛

ចាំ យ េះទ្គប់ 𝑖 ∈ 1,2, … , 𝑛 យយើ ងមាន

𝑛

𝑎𝑖2 + 𝑎𝑖+1 ≥ 2𝑎𝑖 𝑎𝑖+1 𝑛 𝑎𝑖2 + 𝑎𝑖+1 ≥ 2 𝑎𝑖+1

𝑖=1

យោយយក 𝑎𝑛+1 = 𝑎1

𝑛

𝑛

𝑎𝑖+1 = 𝑖=1

លឹម សុ វណ្ណវិចិត្រ | V. វិសមភាព

𝑖=1

𝑛

𝑎𝑖 ⟹ 𝑖=1

𝑖=1

𝑛

𝑎𝑖 𝑖=1

𝑎𝑖2 ≥ 𝑎𝑖+1

𝑛

𝑎𝑖 𝑖=1

117


220. យយើ ងមាន 𝑥 1999 + 𝑦1999 + 𝑧1999 =

𝑥 1998 𝑝𝑦 + 𝑞𝑧 .

2

𝑥 2000 𝑦 2000 + 𝑦1998 𝑝𝑧 + 𝑞𝑥 . 𝑝𝑦 + 𝑞𝑧 𝑝𝑧 + 𝑞𝑥 2

𝑧 2000 𝑝𝑥 + 𝑞𝑦 . 𝑝𝑥 + 𝑞𝑦

+ 𝑧1998

𝑥 2000 𝑦 2000 𝑧 2000 + + 𝑝𝑦 + 𝑞𝑧 𝑝𝑧 + 𝑞𝑥 𝑝𝑥 + 𝑞𝑦 𝑥 2000 𝑦 2000 = 𝑝 𝑥 1998 𝑦 + 𝑦1998 𝑧 + 𝑧1998 𝑥 + 𝑞 𝑥 1998 𝑧 + 𝑦1998 𝑥 + 𝑧1998 𝑦 + 𝑝𝑦 + 𝑞𝑧 𝑝𝑧 + 𝑞𝑥 2000 𝑧 + (1) 𝑝𝑥 + 𝑞𝑦 យយើ ងមាន ≤ 𝑥 1998 𝑝𝑦 + 𝑞𝑧 + 𝑦1998 𝑝𝑧 + 𝑞𝑥 + 𝑧1998 𝑝𝑥 + 𝑞𝑦 .

𝑥 1998 𝑦 + 𝑦1998 𝑧 + 𝑧1998 𝑥 ≤ 𝑥 1999 + 𝑦1999 + 𝑧1999 (2) យទ្ េះតាមវ ិសមភាពតាំ យរៀប និងយោយារ(២)មានលកខណៈសុី យមទ្ទីយធៀបនឹង 𝑥, 𝑦, 𝑧 ដដលយយើ ងោច សនមតថា 𝑥 ≤ 𝑦 ≤ 𝑧 បាន យ េះ យ េះយោយយក 𝑏𝜎

𝑖 3

𝑆𝜎 =

𝑎1 = 𝑥 1998 ≤ 𝑎2 = 𝑦1998 ≤ 𝑎3 = 𝑧1998 ; 𝑏1 = 𝑥 ≤ 𝑏2 = 𝑦 ≤ 𝑏3 = 𝑧 = 𝑏2 ; 𝑏3 ; 𝑏1 យយើ ងទញបាន 𝑎𝑖 𝑏𝜎

𝑖

= 𝑥 1998 𝑦 + 𝑦1998 𝑧 + 𝑧1998 𝑥 ≤ 𝑎1 𝑏1 + 𝑎2 𝑏2 + 𝑎3 𝑏3

1

= 𝑥 1999 + 𝑦1999 + 𝑧1999 ព ិត។ ដូយចនេះវ ិសមភាព(១)យៅជា 𝑥 1999 + 𝑦1999 + 𝑧1999

2

𝑥 2000 𝑦 2000 𝑧 2000 ≤ 𝑝+𝑞 𝑥 +𝑦 +𝑧 + + 𝑝𝑦 + 𝑞𝑧 𝑝𝑧 + 𝑞𝑥 𝑝𝑥 + 𝑞𝑦 𝑥 2000 𝑦 2000 𝑧 2000 𝑥 1999 + 𝑦1999 + 𝑧1999 ⟺ + + ≥ 𝑝𝑦 + 𝑞𝑧 𝑝𝑧 + 𝑞𝑥 𝑝𝑥 + 𝑞𝑦 𝑝+𝑞 1999

1999

1999

221. តាមវ ិសមភាពតាំយរៀប យយើ ងមាន 𝑎1 𝑏1 + 𝑎2 𝑏2 + ⋯ + 𝑎𝑛 𝑏𝑛 = 𝑎1 𝑏1 + 𝑎2 𝑏2 + ⋯ + 𝑎𝑛 𝑏𝑛 𝑎1 𝑏1 + 𝑎2 𝑏2 + ⋯ + 𝑎𝑛 𝑏𝑛 ≥ 𝑎1 𝑏2 + 𝑎2 𝑏3 + ⋯ + 𝑎𝑛 𝑏1 … 𝑎1 𝑏1 + 𝑎2 𝑏2 + ⋯ + 𝑎𝑛 𝑏𝑛 ≥ 𝑎1 𝑏𝑛 + 𝑎2 𝑏1 + ⋯ + 𝑎𝑛 𝑏𝑛−1 118

១. អនុគមន៍ងាយ | លឹម សុ វណ្ណវិចិត្រ


បូកអងគនង ឹ អងគ យយើ ងទញបាន 𝑛 𝑎1 𝑏1 + 𝑎2 𝑏2 + ⋯ + 𝑎𝑛 𝑏𝑛 ≥ 𝑎1 + 𝑎2 + ⋯ + 𝑎𝑛 𝑏1 + 𝑏2 + ⋯ + 𝑏𝑛 ⟹ វ ិសមភាពព ិត។ ទ្ាយបញ្ជាក់ ដច ូ គ្ននករណីសេុី ត 𝑏𝑛 យរៀបតាមលាំ ោប់ ទ្ចាសមកវ ិញ។

222.  រនរៀរទី១ តាមលកខណៈសុី យមទ្ទី យយើ ងោចសនមតថា 𝑎 ≥ 𝑏 ≥ 𝑐 1 1 1 ⟹ ≤ ≤ 𝑎+𝑏 𝑎+𝑐 𝑏+𝑐 តាមវ ិសមភាព យយើ ងទញបាន 𝑎 𝑏 𝑐 1 1 1 + + = 𝑎. + 𝑏. + 𝑐. 𝑏+𝑐 𝑐+𝑎 𝑎+𝑏 𝑏+𝑐 𝑐+𝑎 𝑎+𝑏 1 1 1 1 ≥ 𝑎+𝑏+𝑐 + + 3 𝑏+𝑐 𝑐+𝑎 𝑎+𝑏 1 1 1 1 = 𝑏+𝑐 + 𝑐+𝑎 + 𝑎+𝑏 + + 6 𝑏+𝑐 𝑐+𝑎 𝑎+𝑏

ព ិត។

3 1 3 1 1 1 ≥ .3 𝑏 + 𝑐 𝑐 + 𝑎 𝑎 +𝑏 .3 6 𝑏 +𝑐𝑐 +𝑎𝑎 +𝑏 3 = 2

 រនរៀរទី២ តាមលកខណៈសុី យមទ្ទី យយើ ងោចសនមតថា 𝑎 ≥ 𝑏 ≥ 𝑐 1 1 1 ⟹ ≤ ≤ 𝑎+𝑏 𝑎+𝑐 𝑏+𝑐 តាមវ ិសមភាពតាំ យរៀប យយើ ងទញបាន 𝑎 𝑏 𝑐 1 1 1 + + = 𝑎. + 𝑏. + 𝑐. 𝑏+𝑐 𝑐+𝑎 𝑎+𝑏 𝑏+𝑐 𝑐+𝑎 𝑎+𝑏 1 1 1 ≥ 𝑎. + 𝑏. + 𝑐. 𝑐+𝑎 𝑎+𝑏 𝑏+𝑐 និង 𝑎 𝑏 𝑐 1 1 1 + + ≥ 𝑎. + 𝑏. + 𝑐. 𝑏+𝑐 𝑐+𝑎 𝑎+𝑏 𝑎+𝑏 𝑏+𝑐 𝑎+𝑐 បូកអងគនង ឹ អងគ យយើ ងទញបាន

លឹម សុ វណ្ណវិចិត្រ | V. វិសមភាព

119


𝑎 𝑏 𝑐 + + 𝑏+𝑐 𝑐+𝑎 𝑎+𝑏 1 1 1 1 1 1 ≥ 𝑎. + 𝑏. + 𝑐. + 𝑎. + 𝑏. + 𝑐. =3 𝑐+𝑎 𝑎+𝑏 𝑏+𝑐 𝑎+𝑏 𝑏+𝑐 𝑎+𝑐 ⟹ វ ិសមភាពព ិត។ 2

223. តាង 𝑎3 𝑏3 𝑐3 𝑑3 + + + 𝑏+𝑐+𝑑 𝑐+𝑑+𝑎 𝑑+𝑎+𝑏 𝑎+𝑏+𝑐 𝑥 = 𝑏 + 𝑐 + 𝑑, 𝑦 = 𝑐 + 𝑑 + 𝑎, 𝑧 = 𝑑 + 𝑎 + 𝑏, 𝑡 = 𝑎 + 𝑏 + 𝑐 𝑥 + 𝑦 + 𝑧 + 𝑡 = 3(𝑎 + 𝑏 + 𝑐 + 𝑑) យោយផលបូក 𝑆 មានលកខណៈសុី យមទ្ទីយធៀបនឹង 𝑎, 𝑏, 𝑐, 𝑑 (ឧទហរណ៍ ជាំនស ួ 𝑎 យោយ 𝑏 និង 𝑏 𝑆=

យោយ 𝑎 យយើ ងទញបាន 𝑆 យៅដដដល) យ េះ យយើ ងោចសនមតថា 𝑎 ≥ 𝑏 ≥ 𝑐 ≥ 𝑑 ⟹ 𝑎𝑛 ≥ 𝑏 𝑛 ≥ 𝑐 𝑛 ≥ 𝑑𝑛 ចាំ យ េះទ្គប់ 𝑛 ∈ ℕ និង 1 1 1 1 ≥ ≥ ≥ 𝑥 𝑦 𝑧 𝑡 តាមវ ិសមភាពតាំ យរៀប យយើ ងមាន 𝑎2 + 𝑏 2 + 𝑐 2 + 𝑑 2 ≥ 𝑎𝑏 + 𝑏𝑐 + 𝑐𝑑 + 𝑑𝑎 = 1។ តាមវ ិសមភាព

យយើ ងមាន 1 1 1 1 𝑆 = 𝑎3 + 𝑏 3 + 𝑐 3 + 𝑑 3 𝑥 𝑦 𝑧 𝑡 1 3 1 1 1 1 ≥ 𝑎 + 𝑏3 + 𝑐 3 + 𝑑3 + + + 4 𝑥 𝑦 𝑧 𝑡

និង 1 𝑎3 + 𝑏 3 + 𝑐 3 + 𝑑 3 ≥ 4 1 ≥ 1 𝑎+𝑏+𝑐+𝑑 4 1 ⟹ 𝑆≥ 𝑎+𝑏+𝑐+𝑑 16 ≥

1 𝑥+𝑦+𝑧+𝑡 48

𝑎2 + 𝑏 2 + 𝑐 2 + 𝑑 2 𝑎 + 𝑏 + 𝑐 + 𝑑 1 = 𝑎+𝑏+𝑐+𝑑 4 1 1 1 1 + + + 𝑥 𝑦 𝑧 𝑡

4 1111 1 1 1 1 1 4 1 + + + ≥ . 4 𝑥𝑦𝑧𝑡 . 4 = 𝑥 𝑦 𝑧 𝑡 48 𝑥𝑦𝑧 𝑡 3

224. តាមលកខណៈសុី យមទ្ទី យយើ ងោចសនមតថា 𝑎 ≥ 𝑏 ≥ 𝑐 ។ ដូយចនេះ 𝑎𝑛 ≥ 𝑏 𝑛 ≥ 𝑐 𝑛 ចាំ យ េះទ្គប់ 𝑛 ∈ ℕ និង តាមវ ិសមភាពតាំ យរៀប យយើ ងមាន 120

1 1 1 ≤ ≤ 𝑎+𝑏 𝑎+𝑐 𝑏+𝑐

១. អនុគមន៍ងាយ | លឹម សុ វណ្ណវិចិត្រ


𝑎𝑛 𝑏𝑛 𝑐𝑛 𝑎𝑛 𝑏𝑛 𝑐𝑛 + + ≥ + + 𝑏+𝑐 𝑐+𝑎 𝑎+𝑏 𝑎+𝑏 𝑏+𝑐 𝑐+𝑎

និង

𝑎𝑛 𝑏𝑛 𝑐𝑛 𝑎𝑛 𝑏𝑛 𝑐𝑛 + + ≥ + + 𝑏+𝑐 𝑐+𝑎 𝑎+𝑏 𝑐+𝑎 𝑎+𝑏 𝑏+𝑐 បូកអងគនង ិ អងគ យយើ ងទញបាន 𝑎𝑛 𝑏𝑛 𝑐𝑛 1 𝑎𝑛 + 𝑏 𝑛 𝑏 𝑛 + 𝑐 𝑛 𝑐 𝑛 + 𝑎𝑛 + + ≥ + + 𝑏+𝑐 𝑐+𝑎 𝑎+𝑏 2 𝑎+𝑏 𝑏+𝑐 𝑐+𝑎 តាមវ ិសមភាព 1 𝑎 + 𝑏 ≥ 𝑎𝑛−1 + 𝑏 𝑛−1 𝑎 + 𝑏 2 𝑛

ដូចគ្នន

𝑛

𝑎𝑛 + 𝑏 𝑛 1 𝑛−1 ⟹ ≥ 𝑎 + 𝑏 𝑛−1 𝑎+𝑏 2

𝑏 𝑛 + 𝑐 𝑛 1 𝑛−1 ≥ 𝑏 + 𝑐 𝑛−1 𝑏+𝑐 2 𝑐 𝑛 + 𝑎𝑛 1 𝑛−1 ≥ 𝑐 + 𝑎𝑛−1 𝑐+𝑎 2 𝑎𝑛 𝑏𝑛 𝑐𝑛 𝑎𝑛−1 + 𝑏 𝑛−1 + 𝑐 𝑛−1 ⟹ + + ≥ 𝑏+𝑐 𝑐+𝑎 𝑎+𝑏 2

225. តាមលកខណៈសុី យមទ្ទី យយើ ងោចសនមតថា 𝑥 ≤ 𝑦 ≤ 𝑧 ។ ដូយចនេះ 1 តាមវ ិសមភាព

1

1+𝑦 1+𝑧 1+𝑧 1+𝑥 យយើ ងទញបាន

1 1+𝑥 1+𝑦

𝑥3 𝑦3 𝑧3 + + 1+𝑦 1+𝑧 1+𝑧 1+𝑥 1+𝑥 1+𝑦 𝑥3 + 𝑦3 + 𝑧3 1 1 1 ≥ + + 3 1+𝑦 1+𝑧 1+𝑧 1+𝑥 1+𝑥 1+𝑦 𝑥3 + 𝑦3 + 𝑧3 3+𝑥+𝑦+𝑧 = 3 1+𝑦 1+𝑧 1+𝑥 តាង (𝑥 + 𝑦 + 𝑧) 3 = 𝑎។ តាមវ ិសមភាព យយើ ងមាន 𝑥3 + 𝑦3 + 𝑧3 𝑥2 + 𝑦2 + 𝑧2 𝑥 + 𝑦 + 𝑧 𝑥 + 𝑦 + 𝑧 𝑥 + 𝑦 + 𝑧 𝑥 + 𝑦 + 𝑧 ≥ ≥ . . 3 3 3 3 3 3 3 𝑥+𝑦+𝑧 = = 𝑎3 3 និង តាមវ ិសមភាពកូសុី 3𝑎 = 𝑥 + 𝑦 + 𝑧 ≥ 3 3 𝑥𝑦𝑧 = 3 លឹម សុ វណ្ណវិចិត្រ | V. វិសមភាព

121


1+𝑦 + 1+𝑧 + 1+𝑥 3

1+𝑦 1+𝑧 1+𝑥 ≤

3

= 1+𝑎

ដូយចនេះ 𝑥3 𝑦3 𝑧3 6 + + ≥ 𝑎3 1+𝑦 1+𝑧 1+𝑧 1+𝑥 1+𝑥 1+𝑦 1+𝑎 ដូយចនេះ យយើ ងទ្តូវបង្ហាញថា 6𝑎3 1+𝑎

3

3

3 4

យោយ 𝑎 ≥ 1 យ េះ 3

6𝑎3 𝑓 𝑎 = 1+𝑎 ជាអនុគមន៍យកើនោច់ ខាតយលើ ℝ+ ។ យយើ ងមាន

3

1 =6 1− 1+𝑎

𝑓 𝑎3 ≥ 𝑓 1 = ដូយចនេះវ ិសមភាពព ិត។

3

3 4

226.  រនរៀរទីមួយ ចាំ យ េះទ្គប់ ចាំ នន ួ ព ិត មិនអវ ិជាមាន យគមាន

𝑎1 + 𝑎2 ≥ 𝑎1 𝑎2 2 ដូយចនេះ វ ិសមភាពខាងយលើព ិត ចាំ យ េះ 𝑛 = 2 ។ សនមតថា វ ិសមភាពខាងយលើព ិត រហូ តដល់ 2

𝑎1 − 𝑎2

≥0 ⟹

𝑛 = 2𝑘−1 , 𝑘 > 2 ។ ដូយចនេះ 2 𝑘−1

តាង

𝑎1 𝑎2 … 𝑎2𝑘−1 ≤

𝑎1 + 𝑎2 + ⋯ + 𝑎2𝑘−1 2𝑘−1

𝑎1 + 𝑎2 + ⋯ + 𝑎2𝑘−1 2𝑘−1 𝑎2𝑘−1 +1 + ⋯ + 𝑎2𝑘 𝑥2 = 2𝑘−1 𝑥1 =

យយើ ងមាន

122

𝑥1 + 𝑥2 ≥ 2

១. អនុគមន៍ងាយ | លឹម សុ វណ្ណវិចិត្រ

𝑥1 𝑥2

3


𝑎1 + 𝑎2 + ⋯ + 𝑎2𝑘−1 𝑎 𝑘−1 + ⋯ + 𝑎2𝑘 + 2 +1 𝑘−1 𝑘−1 2 2 ⟹ 2 ≥

𝑎1 + 𝑎2 + ⋯ + 𝑎2𝑘−1 𝑎2𝑘−1 +1 + ⋯ + 𝑎2𝑘 . 2𝑘−1 2𝑘−1 𝑘−1

𝑘

𝑘−1

≥ 2 𝑎1 𝑎2 … 𝑎2𝑘−1 . 2 𝑎2𝑘−1 +1 … 𝑎2𝑘 = 2 𝑎1 𝑎2 … 𝑎2𝑘 𝑎1 + 𝑎2 + ⋯ + 𝑎2𝑘 𝑘 ⟹ ≥ 2 𝑎1 𝑎2 … 𝑎2𝑘 𝑘 2 ដូយចនេះ វ ិសមភាពព ិតចាំ យ េះ ទ្គប់ 𝑛 = 2𝑘 , 𝑘 ≥ 1 ។ ឥលូ វសនមតថា 2𝑘−1 < 𝑛 < 2𝑘 ។ តាង 𝑦1 = 𝑎1 , 𝑦2 = 𝑎2 , … , 𝑦𝑛 = 𝑎𝑛 𝑦𝑛+1 = 𝑦𝑛+2 = ⋯ = 𝑦2𝑘 = 𝐴 𝑎1 + 𝑎2 + ⋯ + 𝑎𝑛 𝐴= 𝑛 𝐺 = 𝑛 𝑎1 𝑎2 … 𝑎𝑛 យយើ ងមាន 𝑦1 + 𝑦2 + ⋯ + 𝑦2𝑘 2𝑘 ≥ 𝑦1 𝑦2 … 𝑦2𝑘 2𝑘 𝑎1 + 𝑎2 + ⋯ + 𝑎𝑛 + 2𝑘 − 𝑛 𝐴 ⟺ ≥ 2𝑘 𝑛𝐴 + 2𝑘 − 𝑛 𝐴 2𝑘 𝑛 2𝑘 −𝑛 ⟺ ≥ 𝐺 𝐴 2𝑘 𝑛

2𝑘

𝑎1 𝑎2 … 𝑎𝑛 . 𝐴2

𝑘 −𝑛

𝑛 1− 𝑘 2

⟺ 𝐴 ≥ 𝐺 2𝑘 𝐴 𝑛

𝑛

⟺ 𝐴2 𝑘 ≥ 𝐺 2 𝑘 ⟺𝐴≥𝐺 𝑎1 + 𝑎2 + ⋯ + 𝑎𝑛 𝑛 ⟺ ≥ 𝑎1 𝑎2 … 𝑎𝑛 𝑛 ដូយចនេះវ ិសមភាពព ិតចាំ យ េះទ្គប់ 𝑛 ≥ 2 ។

 រនរៀរទីពីរ អនុគមន៍ 𝑓 𝑥 = ln 𝑥 ជាអនុគមន៍យបា៉ាងយលើ ℝ+∗ ។ តាមវ ិសមភាពយិ នសិ ន យយើ ងទញបាន 1 1 1 1 1 1 ln 𝑎1 + 𝑎2 + ⋯ + 𝑎𝑛 ≥ ln 𝑎1 + ln 𝑎2 + ⋯ + ln 𝑎𝑛 = ln 𝑛 𝑎1 𝑎2 … 𝑎𝑛 𝑛 𝑛 𝑛 𝑛 𝑛 𝑛 𝑎1 + 𝑎2 + ⋯ + 𝑎𝑛 𝑛 ⟹ ≥ 𝑎1 𝑎2 … 𝑎𝑛 𝑛

227. តាង 𝑎 = 𝑥 + 𝑦, 𝑏 = 𝑦 + 𝑧, 𝑐 = 𝑧 + 𝑥 (𝑥, 𝑦, 𝑧 > 0) វ ិសមភាពដដលយោយសមមូលនឹង លឹម សុ វណ្ណវិចិត្រ | V. វិសមភាព

123


6𝑥𝑦𝑧 ≤ 𝑥 2 𝑦 + 𝑥𝑦 2 + 𝑦 2 𝑧 + 𝑦𝑧 2 + 𝑧 2 𝑥 + 𝑧𝑥 2 វ ិសមភាពយនេះព ិត យោយយទ្បើវ ិសមភាពមធែមនព េនតមធែមធរណីមាទ្ត សាំ រាប់ 𝑥 2 𝑦, 𝑥𝑦 2 , 𝑥 2 𝑧, 𝑥𝑧 2 , 𝑦 2 𝑧, 𝑦𝑧 2 ។

228. យយើ ងមាន 𝑛!

2/𝑛

=

1.2. … 𝑛

1 2 𝑛

1+ 2+ ⋯+𝑛 2

2

=

𝑛+1 2

2

1 1 1 ≤ 𝑛2 + 𝑛 + 3 2 6

229. តាមវ ិសមភាពកូសុី 8𝑎2 𝑏 3 𝑐 3 ≤ 2𝑎8 + 3𝑏 8 + 3𝑐 8 8𝑎3 𝑏 2 𝑐 3 ≤ 3𝑎8 + 2𝑏 8 + 3𝑐 8 8𝑎3 𝑏 3 𝑐 2 ≤ 3𝑎8 + 3𝑏 8 + 2𝑐 8 បូកវ ិសមភាពអស់ យនេះបញ្ចូ លគ្នន យហើ យដចកនឹង 3𝑎3 𝑏 3 𝑐 3 យយើ ងទញបានវ ិសមភាពដដលចង់ បាន។

230. តាមវ ិសមភាពកូសុី 𝑛 + 𝑘 − 1 𝑥1𝑛 𝑥2 … 𝑥𝑘 ≤ 𝑛𝑥1𝑛+𝑘−1 + 𝑥2𝑛+𝑘−1 + ⋯ + 𝑥𝑘𝑛+𝑘−1 𝑛 + 𝑘 − 1 𝑥1 𝑥2𝑛 … 𝑥𝑘 ≤ 𝑥1𝑛+𝑘−1 + 𝑛𝑥2𝑛+𝑘−1 + ⋯ + 𝑥𝑘𝑛 +𝑘−1 … … 𝑛 + 𝑘 − 1 𝑥1 𝑥2 … 𝑥𝑘𝑛 ≤ 𝑥1𝑛+𝑘−1 + 𝑥2𝑛+𝑘−1 + ⋯ + 𝑛𝑥𝑘𝑛+𝑘−1 បូកវ ិសមភាពទាំងអស់ បញ្ចូ លគ្នន ប ទ ប់ មកដចកនឹង (𝑛 + 𝑘 − 1) យយើ ងទញបានវ ិសមភាពដដលទ្តូវ ទ្ាយបញ្ជាក់ ។

231. តាមវ ិសមភាពកូសុី 𝑥1 + 𝑥2 + ⋯ + 𝑥𝑛 ≥ 𝑛 𝑛 𝑥1 𝑥2 … 𝑥𝑛

𝑛 1 1 1 1 1 1 + +⋯+ ≥𝑛 … 𝑥1 𝑥2 𝑥𝑛 𝑥1 𝑥2 𝑥𝑛 1 1 1 𝑥1 + 𝑥2 + ⋯ + 𝑥𝑛 + +⋯+ ≥ 𝑛2 𝑥1 𝑥2 𝑥𝑛

232. តាមលកខណៈសុី យមទ្ទី យយើ ងោចសនមតថា 𝑥 ≤ 𝑦 ≤ 𝑧 ។ តាង 𝑓 𝑥, 𝑦, 𝑧 = 𝑥 2 + 𝑦 2 + 𝑧 2 + 𝑥 + 𝑦 + 𝑧 − 2 𝑥𝑦 + 𝑦𝑧 + 𝑧𝑥 យយើ ងមាន 𝑓 𝑥, 𝑦, 𝑧 − 𝑓 𝑥, 𝑦𝑧, 𝑦𝑧 = 𝑦 2 + 𝑧 2 + 𝑦 + 𝑧 − 2 𝑥𝑦 + 𝑦𝑧 + 𝑧𝑥 − 2 𝑦𝑧 + 4𝑥 𝑦𝑧 = 𝑦−𝑧 124

2

+

𝑦− 𝑧

១. អនុគមន៍ងាយ | លឹម សុ វណ្ណវិចិត្រ

2

− 2𝑥

𝑦− 𝑧

2


= =

𝑦− 𝑧 𝑦− 𝑧

2 2

𝑦+ 𝑧

2

+ 1 − 2𝑥

𝑦 + 𝑧 − 2𝑥 + 1 + 2 𝑦𝑧

យោយ 𝑥 ≤ 𝑦 ≤ 𝑧 យ េះ 𝑦 + 𝑧 − 2𝑥 ≥ 0 ⟹ 𝑓 𝑥, 𝑦, 𝑧 − 𝑓 𝑥, 𝑦𝑧, 𝑦𝑧 ≥ 0 យហើ យអងគទង ាំ ២ យសមើ គ្នន លុ េះទ្តាដត 𝑦 = 𝑧 ។ ប ទ ប់ មកយទៀត យយើ ងនឹងបង្ហាញថា 𝑓 𝑥, 𝑦𝑧, 𝑦𝑧 ≥ 0 ។ តាង 𝑎 = 𝑥 និង 𝑏 =

𝑦𝑧 ។ ដូយចនេះ 𝑎, 𝑏 > 0 និង 𝑎𝑏 2 = 1 ។ យយើ ងមាន

𝑓 𝑎, 𝑏, 𝑏 = 𝑎2 + 𝑎 + 2𝑏 − 4𝑎𝑏 1 1 4 = 4 + 2 + 2𝑏 − 𝑏 𝑏 𝑏 1 5 3 = 4 2𝑏 − 4𝑏 + 𝑏 2 + 1 𝑏 1 = 4 𝑏 − 1 2 2𝑏3 + 4𝑏 2 + 2𝑏 + 1 ≥ 0 𝑏 អងគទង ាំ ព ីរយសមើ គ្នន លុ េះទ្តាដត 𝑏 = 1។ ដូយចនេះ 𝑓 𝑥, 𝑦, 𝑧 ≥ 𝑓 𝑎, 𝑏, 𝑏 ≥ 0 យហើ យយសមើ គ្នន លុ េះទ្តាដត 𝑦 = 𝑧, 𝑏 = 1, 𝑥𝑦𝑧 = 1 ⟹ 𝑥 = 𝑦 = 𝑧 = 1។

233. តាមវ ិសមភាពកូសុី 𝑎2 + 𝑏 2 + 𝑐 2 + 𝑑2 + 𝑎𝑏 + 𝑎𝑐 + 𝑎𝑑 + 𝑏𝑐 + 𝑏𝑑 + 𝑐𝑑 ≥ 10 𝑎2 𝑏2 𝑐 2 𝑑 2 𝑎𝑏𝑎𝑐𝑎𝑑𝑏𝑐𝑏𝑑𝑐𝑑 1

= 10 𝑎5 𝑏 5 𝑐 5 𝑑5 10 = 10 អងគទង ាំ ២យសមើ គ្នន យពល 𝑎 = 𝑏 = 𝑐 = 𝑑 = 1 ។

234. យយើ ងមាន 𝑎+𝑏+𝑐

3

= 𝑎3 + 𝑏 3 + 𝑐 3 + 6𝑎𝑏𝑐 + 3 𝑎2 𝑏 + 𝑎2 𝑐 + 𝑏 2 𝑎 + 𝑏 2 𝑐 + 𝑐 2 𝑎 + 𝑐 2 𝑏 6 ≥ 𝑎3 + 𝑏 3 + 𝑐 3 + 6𝑎𝑏𝑐 + 3.6 𝑎6 𝑏 6 𝑐 6 = 𝑎3 + 𝑏 3 + 𝑐 3 + 24𝑎𝑏𝑐

(តាមវ ិសមភាពកូសុី)

235. ករណីអងគខាងាតាំននវ ិសមភាព អវ ិជាមាន រ ឺ សូ នែ យ េះវ ិសមភាពជាវ ិសមភាពោច់ ខាត។ ករណីអងគខាងាដាំវ ិជាមាន យយើ ងសនមតថា 𝑎 = max 𝑎, 𝑏, 𝑐 ។ ដូយចនេះ 𝑎 + 𝑏 − 𝑐 > 0 និង 𝑐 + 𝑎 − 𝑏 > 0 ⟹ 𝑏 + 𝑐 − 𝑎 > 0 យទ្ េះអងគខាងាដាំវ ិជាមាន។ ដូយចនេះ 𝑎, 𝑏, 𝑐 ជារង្ហេស់ ទ្ជុងនន ទ្តីយោណ។ ដូយចនេះ យយើ ងតាង 𝑎 + 𝑏 − 𝑐 = 𝑥, 𝑏 + 𝑐 − 𝑎 = 𝑦, 𝑐 + 𝑎 − 𝑏 = 𝑧 𝑥, 𝑦, 𝑧 > 0 𝑥+𝑧 𝑦+𝑥 𝑧+𝑦 ⟹ 𝑎= ,𝑏 = ,𝑐 = 2 2 2 លឹម សុ វណ្ណវិចិត្រ | V. វិសមភាព

125

1 10


វ ិសមភាពយៅជា 𝑥 + 𝑧 𝑦 + 𝑥 𝑧 + 𝑦 ≥ 8𝑥𝑦𝑧 តាមវ ិសមភាពកូសុី យយើ ងទញបាន 𝑥 + 𝑧 𝑦 + 𝑥 𝑧 + 𝑦 ≥ 2 𝑥𝑧. 2 𝑦𝑥. 2 𝑧𝑦 = 8𝑥𝑦𝑧 ព ិត។ អងគទង ាំ ព ីរយសមើ គ្ននទល់ ដតនិងមានដត 𝑥 = 𝑦 = 𝑧 មានន័យថា 𝑎 = 𝑏 = 𝑐 ។

236. តាមវ ិសមភាពកូសុី យយើ ងមាន 𝑛

𝑛

𝑎𝑘 1 − 𝑎𝑘 = 𝑘=1

𝑛

𝑎𝑘 𝑘=1

𝑛−1 = 𝑛2𝑛

1 − 𝑎𝑘 𝑛

𝑘=1

1 ≤ 𝑛

𝑛

𝑛

𝑎𝑘 𝑘=1

1 𝑛

𝑛

𝑛

1 − 𝑎𝑘

=

𝑘=1

1 𝑛−1 . 𝑛𝑛 𝑛𝑛

237. យយើ ងមាន 𝑎2 + 𝑏 2 +

1 𝑏 1 2 3 + = 𝑏 + + 𝑎2 + 2 2 𝑎 𝑎 2𝑎 4𝑎 3 1 ≥ 𝑎2 + 2 យសមើ គ្ននទល់ ដត b = − 4𝑎 2a ≥2

3 = 3 4

តាមវ ិសមភាពកូសុី

1 𝑏 + ≥ 3 𝑎2 𝑎 យសមើ គ្នន លុ េះទ្តាដត 𝑎4 = 3 4 និង 𝑏 = −1/2𝑎 ។ ⟹ 𝑎2 + 𝑏 2 +

238. តាមវ ិសមភាពកូសុី ចាំ យ េះ 𝑛 ≥ 2 1 1+ 𝑛−1

⟹ មយ៉ាងវ ិញយទៀត 1 1− 𝑛 126

1 𝑛 −1 𝑛

<

1 1 1 1 + 1 +𝑛 − 1 + 1 + 𝑛 − 1 + ⋯+ 1 +𝑛 − 1

𝑛 1 1+ 𝑛−1 1+𝑛−1 1 = =1+ 𝑛 𝑛 1 𝑛−1 1 𝑛 1+ < 1+ ⟹ 𝑈𝑛−1 < 𝑈𝑛 𝑛−1 𝑛

1 𝑛 𝑛+1

<

1 1 1 1 + 1 − 𝑛 + 1 −𝑛 + ⋯+ 1 − 𝑛

១. អនុគមន៍ងាយ | លឹម សុ វណ្ណវិចិត្រ

𝑛+1

=

1 1+𝑛 1−𝑛 𝑛+1

=

𝑛 𝑛+1

𝑛


⟹ ⟺

𝑛−1 𝑛

𝑛

𝑛 𝑛+1

<

1 1 1+𝑛−1

𝑛

<

𝑛+1

1 1 𝑛 < 𝑛−1+1 𝑛+1 𝑛 𝑛−1 𝑛 1 1 ⟺ < ⟹ 𝑉𝑛 < 𝑉𝑛−1 𝑉𝑛−1 𝑉𝑛

⟺ 1

1 1+𝑛

𝑛

239. យយើ ងតាង

𝑎1 𝑎2 𝑎𝑛−1 𝑎𝑛 + + ⋯+ + 𝑎2 + 𝑎3 𝑎3 + 𝑎4 𝑎𝑛 + 𝑎1 𝑎1 + 𝑎2 កនង ុ សាំ យនរប ទ ប់ មកយទៀតយនេះ យយើ ងសនមតថា 𝑎𝑛+1 = 𝑎1 ; 𝑎𝑛+2 = 𝑎2 ; 𝑎𝑛+3 = 𝑎3 ; … ។ 𝑎1 𝑎2 𝑎𝑛−1 𝑎𝑛 𝑓 𝑎1 , 𝑎2 , … , 𝑎𝑛 = + +⋯+ + 𝑎2 + 𝑎3 𝑎3 + 𝑎4 𝑎𝑛 + 𝑎𝑛+1 𝑎𝑛+1 + 𝑎𝑛+2 យយើ ងសនមតថា 𝑎1 = max 𝑎1 , 𝑎2 , … , 𝑎𝑛 ។ តាង 𝑖1 = 1 ។ តាង 𝑖2 ជាសនទសេន៍របស់ ចាំ នន ួ ធាំ បាំផត ុ 𝑓 𝑎1 , 𝑎2 , … , 𝑎𝑛 =

រវាង 𝑎2 និង 𝑎3 គឺ យបើ 𝑎2 > 𝑎3 យក 𝑖2 = 2 និងយបើ 𝑎3 > 𝑎2 យក 𝑖2 = 3 យហើ យយបើ 𝑎2 = 𝑎3 យ េះយយើ ងយក 𝑖2 = 2 ។ ដូយចនេះ 𝑖2 ≤ 𝑖1 + 2 ។ យយើ ងបយងេត ើ សេុី ត 𝑖𝑘 មួយ យោយកាំយនើនដូចតយៅយនេះ៖ យបើ បយងេត ើ បាន 𝑖𝑘 រួចយហើ យ តាង 𝑖𝑘+1 ជាសនទសេន៍ននចាំ នន ួ ធាំ ជាងយគរវាង 𝑎𝑖𝑘 +1 និង 𝑎𝑖𝑘 +2 យោយ យបើ 𝑎𝑖𝑘 +1 = 𝑎𝑖𝑘 +2 យក 𝑖𝑘+1 = 𝑖𝑘 + 1។ កនង ុ លកខខណឌយនេះ 𝑖𝑘+1 ≤ 𝑖𝑘 + 2។ យោយ 𝑖1 = 1 យ េះ 𝑖𝑘+1 ≤ 1 + 2𝑘 ចាំ យ េះទ្គប់ 𝑘 ។ យោយ 𝑎1 = 𝑎𝑛+1 = max 𝑎1 , 𝑎2 , … , 𝑎𝑛 = max 𝑎𝑛 , 𝑎𝑛+1 យ េះ យយើ ងមាន 𝑟 ដដល 𝑖𝑟+1 = 𝑛 + 1 ជាសនទសេន៍របស់ ចាំ នន ួ ធាំ បាំផត ុ រវាង 𝑎𝑛 និង 𝑎𝑛+1 ដដលទ្តូវនឹងចាំ នន ួ ធាំ បាំផត ុ រវាង 𝑎𝑖𝑟 +1 និង 𝑎𝑖𝑟 +2 ។ ដូយចនេះ 𝑖𝑟 = 𝑛 − 1 រ ឺ 𝑖𝑟 = 𝑛 ។ ដូយចនេះ 𝑛 − 1 ≤ 𝑖𝑟 ≤ 1 + 2 𝑟 − 1 ⟹ 𝑟 ≥ 𝑛/2

[ដ ម្ ើ បីពន្យល់ អំន្ះអំនាងខាងដលើ ខ្ំស ញ ូម្ដាក់ ឧទាហរណ៍ ម្ួយ សន្មតថា 𝑛 = 5; 𝑎1 ធំជាងដគន្ិង 𝑎3 > 𝑎2 ; 𝑎4 > 𝑎5 ។ ដយើ ងមាន្ 𝑖1 = 1; 𝑖2 ជាសន្ទសសន្៍ របស់ ចន្ ំ ន្ ួ ធំបំផត ុ រវាង 𝑎𝑖1 +1 = 𝑎2 ន្ិង 𝑎𝑖1 +2 = 𝑎3 ។ ដដាយ 𝑎3 > 𝑎2 ⟹ 𝑖2 = 3; 𝑖3 ជាសន្ទសសន្៍ របស់ ចន្ ំ ន្ ួ ធំបំផត ុ រវាង 𝑎𝑖2 +1 = 𝑎4 ន្ិង 𝑎𝑖2 +2 = 𝑎5 ។ ដដាយ 𝑎4 > 𝑎5 ដនាះ 𝑖3 = 4។ 𝑖4 ជាសន្ទសសន្៍ របស់ ចន្ ំ ន្ ួ ធំបំផត ុ រវាង 𝑎𝑖3 +1 = 𝑎5 ន្ិង 𝑎𝑖3 +2 = 𝑎6 = 𝑎1 ។ ដដាយ 𝑎1 > 𝑎5 ដនាះ 𝑖4 = 6។ ករណី ដន្ះដយើ ងទាញបាន្ 𝑟 = 4 ។ ដយើ ងមាន្ 𝑎1 𝑎2 𝑎3 𝑎4 𝑎5 𝑓 𝑎1 , 𝑎2 , 𝑎3 , 𝑎4 , 𝑎5 = + + + + 𝑎2 + 𝑎3 𝑎3 + 𝑎4 𝑎4 + 𝑎5 𝑎5 + 𝑎1 𝑎1 + 𝑎2 𝑎𝑖 𝑎𝑖 𝑎𝑖 𝑎1 𝑎3 𝑎4 𝑎1 𝑎3 𝑎4 > +0+ + +0 ≥ + + = 1 + 2 + 3 𝑎2 + 𝑎3 𝑎4 + 𝑎5 𝑎5 + 𝑎1 2𝑎2 2𝑎4 2𝑎1 2𝑎𝑖2 2𝑎𝑖3 2𝑎𝑖4 លឹម សុ វណ្ណវិចិត្រ | V. វិសមភាព

127


]ď Š

ដសយចនá&#x; á&#x;‡áž™áž™ážž ងទញបវន đ?‘“ đ?‘Ž1 , đ?‘Ž2 , ‌ , đ?‘Žđ?‘› ≼

đ?‘Žđ?‘–1 đ?‘Žđ?‘– đ?‘Žđ?‘–đ?‘&#x; + 2 + â‹Ż+ 2đ?‘Žđ?‘–2 2đ?‘Žđ?‘–3 2đ?‘Žđ?‘–đ?‘&#x;+1

2

đ?‘&#x;

យដឞមបីយá&#x;„áž™ đ?‘“ đ?‘Ž1 , đ?‘Ž2 , ‌ , đ?‘Žđ?‘›

đ?‘&#x; đ?‘Žđ?‘–1 đ?‘Žđ?‘–2 ‌ đ?‘Žđ?‘–đ?‘&#x; 2 ≼ ážœ ឡáž&#x;áž˜áž—ážśáž–áž€ážźáž&#x;ឝី 2 đ?‘Žđ?‘–2 đ?‘Žđ?‘–3 ‌ đ?‘Žđ?‘–đ?‘&#x;+1 đ?‘&#x; đ?‘› = ≼ 2 4 = đ?‘›/4 យគទá&#x;’áž?ážźážœážŠáž?áž˜ážśáž“ នឡង ជវáž&#x;áž˜áž—ážśáž–á&#x;” ដáž?áž&#x;áž˜áž—ážśáž– (2)

áž™á&#x;„áž™ đ?‘&#x; = đ?‘› ដáž?áž&#x;áž˜áž—ážśáž–áž™áž“á&#x; á&#x;‡ áž˜ážˇáž“áž™áž•áž‘á&#x;€áž„áž•á&#x;’áž‘áž‘áž?á&#x;‹ áž&#x;áž˜áž—ážśáž–

ážśá&#x;†

á&#x;” ដសយចនá&#x; á&#x;‡ đ?‘“ đ?‘Ž1 , đ?‘Ž2 , ‌ , đ?‘Žđ?‘› > đ?‘›/4 á&#x;”

240.ď ° áž?ážśáž˜ážœ ឡáž&#x;áž˜áž—ážśáž–áž€ážźáž&#x;ឝី áž…ážśá&#x;† áž™ á&#x; á&#x;‡áž‘á&#x;’áž‚áž”á&#x;‹ đ?‘– យយឞ áž„áž˜ážśáž“ 2 + đ?‘Žđ?‘– = 1 + 1 + đ?‘Žđ?‘– ≼ đ?‘›

â&#x;š

đ?‘›

2 + �� ≼ 3 �=1

đ?‘›

1 3 đ?‘Žđ?‘–3 1 3

đ?‘Žđ?‘–

= 3đ?‘›

đ?‘–=1

241.ď ° ď‚Œ áž™á&#x;„áž™ đ?‘Ž, đ?‘? ជវចវá&#x;† áž“áž“ áž˝ áž– ឡáž?ážœ ážˇáž‡ážśáž˜ážśáž“ áž™ á&#x; á&#x;‡áž™áž‚áž˜ážśáž“ đ?‘Ľ, đ?‘Ś ដដល 0 < đ?‘Ľ, đ?‘Ś < 90° យហឞ យដដល tan đ?‘Ľ = đ?‘Ž; tan đ?‘Ś = đ?‘? á&#x;” ážœ ឡáž&#x;áž˜áž—ážśáž–áž™áž“á&#x; á&#x;‡áž– ឡáž? យបឞ đ?‘Ž = đ?‘?á&#x;” ដសយចនá&#x; á&#x;‡ យយឞ áž„áž&#x;នមáž?áž?ážś đ?‘Ž ≠đ?‘? ážš ážş áž&#x;áž˜áž˜ážźáž›áž“ážšáž„ đ?‘Ľ ≠đ?‘Ś á&#x;” ដសយចនá&#x; á&#x;‡ 1

1 + đ?‘Ž2 = cos đ?‘Ľ ; 1 1 + đ?‘? 2 = cos đ?‘Ś á&#x;” យយឞ áž„áž˜ážśáž“ cos đ?‘Ľ cos đ?‘Ś + sin đ?‘Ľ sin đ?‘Ś cos đ?‘Ľ − đ?‘Ś 1 + đ?‘Žđ?‘? = = cos đ?‘Ľ cos đ?‘Ś cos đ?‘Ľ cos đ?‘Ś ដសយចនá&#x; á&#x;‡ážœ ឡáž&#x;áž˜áž—ážśáž–áž&#x;áž˜áž˜ážźáž›áž“ážšáž„ cos đ?‘Ľ + cos đ?‘Ś ≼ 2 â&#x;š

cos2 � + cos 2 � + 2 cos � cos � ≤

cos đ?‘Ľ cos đ?‘Ś cos đ?‘Ľ − đ?‘Ś

(∗)

4 cos đ?‘Ľ cos đ?‘Ś cos đ?‘Ľ − đ?‘Ś

áž™á&#x;„áž™ 0 < đ?‘Ľ − đ?‘Ś < 90° áž™ á&#x; á&#x;‡ 0 < cos đ?‘Ľ − đ?‘Ś < 1á&#x;” ដសយចនá&#x; á&#x;‡ 2 cos đ?‘Ľ cos đ?‘Ś ≤ 2 cos đ?‘Ľ cos đ?‘Ś cos đ?‘Ľ − đ?‘Ś á&#x;” ដសយចនá&#x; á&#x;‡áž™áž™ážž áž„áž‘á&#x;’áž‚á&#x;’áž“áž“á&#x;‹ ដáž?បងá&#x;’ហវញáž?ážś 2 cos đ?‘Ľ cos đ?‘Ś cos2 đ?‘Ľ + cos 2 đ?‘Ś ≤ cos đ?‘Ľ − đ?‘Ś 2 2 â&#x;ş cos đ?‘Ľ − đ?‘Ś cos đ?‘Ľ + cos đ?‘Ś ≤ 2 cos đ?‘Ľ cos đ?‘Ś â&#x;ş cos đ?‘Ľ − đ?‘Ś cos2 đ?‘Ľ + cos2 đ?‘Ś + 2 ≤ 4 cos đ?‘Ľ cos đ?‘Ś 128

á&#x;Ą. áž˘áž“ážťáž‚áž˜áž“á&#x;?áž„ážśáž™ | លរម áž&#x;ážť វណá&#x;’ážŽážœážˇáž…ážˇáž?á&#x;’ážš


â&#x;ş cos đ?‘Ľ − đ?‘Ś 2 cos đ?‘Ľ − đ?‘Ś cos đ?‘Ľ + đ?‘Ś + 2 ≤ 2 cos đ?‘Ľ − đ?‘Ś + cos đ?‘Ľ + đ?‘Ś â&#x;ş cos2 đ?‘Ľ − đ?‘Ś cos đ?‘Ľ + đ?‘Ś ≤ cos đ?‘Ľ + đ?‘Ś đ?œ‹ đ?œ‹ áž– ឡáž? យទá&#x;’ á&#x; á&#x;‡ 0 < đ?‘Ž, đ?‘? ≤ 1 យយឞ áž„áž˜ážśáž“ 0 < đ?‘Ľ, đ?‘Ś < ដសយចនá&#x; á&#x;‡ 0 < đ?‘Ľ + đ?‘Ś < នឡង cos đ?‘Ľ + đ?‘Ś > 0 á&#x;”

ď‚?

4

2

យយឞ ងបវá&#x;† ដលងវ ឡáž&#x;áž˜áž—ážśáž–(*) ជវ

đ?‘Ľ+đ?‘Ś đ?‘Ľâˆ’đ?‘Ś 1/2 cos đ?‘Ľ + đ?‘Ś + cos đ?‘Ľ − đ?‘Ś cos ≼2 2 2 cos đ?‘Ľ − đ?‘Ś đ?‘Ľ+đ?‘Ś đ?‘Ľâˆ’đ?‘Ś â&#x;ş 4 cos2 cos2 cos đ?‘Ľ − đ?‘Ś ≼ 2 cos đ?‘Ľ + đ?‘Ś + cos đ?‘Ľ − đ?‘Ś 2 2 â&#x;ş 1 + cos đ?‘Ľ + đ?‘Ś 1 + cos đ?‘Ľ − đ?‘Ś cos đ?‘Ľ − đ?‘Ś ≼ 2 cos đ?‘Ľ + đ?‘Ś + cos đ?‘Ľ − đ?‘Ś áž?ážśáž„ đ?‘ = cos đ?‘Ľ + đ?‘Ś នឡង đ?‘Ą = cos đ?‘Ľ − đ?‘Ś á&#x;” ដសយចនá&#x; á&#x;‡áž™áž™ážž áž„áž‘á&#x;’áž‚á&#x;’áž“áž“á&#x;‹ ដáž?បងá&#x;’ហវញáž?ážś 2 cos

1+đ?‘ 1+đ?‘Ą đ?‘Ą ≼2 đ?‘ +đ?‘Ą â&#x;ş 0 ≤ 1 + đ?‘ đ?‘Ą 2 + đ?‘ − 1 đ?‘Ą − 2đ?‘ = đ?‘Ą − 1 1 + đ?‘ đ?‘Ą + 2đ?‘ áž™á&#x;„áž™ đ?‘Ą ≤ 1 ដសយចនá&#x; á&#x;‡áž™áž™ážž áž„áž‘á&#x;’áž‚á&#x;’áž“áž“á&#x;‹ ដáž?បងá&#x;’ហវញáž?ážś 1 + đ?‘ đ?‘Ą + 2đ?‘ ≤ 0 áž™á&#x;„áž™ đ?‘Žđ?‘? ≼ 3 áž™ á&#x; á&#x;‡ tan đ?‘Ľ tan đ?‘Ś ≼ 3 ážš ážşáž&#x;áž˜áž˜ážźáž›áž“ážšáž„ sin đ?‘Ľ sin đ?‘Ś ≼ 3 cos đ?‘Ľ cos đ?‘Ś á&#x;” យវá&#x;† á&#x;„áž™ 1 3 cos đ?‘Ľ − đ?‘Ś − cos đ?‘Ľ + đ?‘Ś ≼ cos đ?‘Ľ − đ?‘Ś + cos đ?‘Ľ + đ?‘Ś â&#x;š đ?‘Ą ≤ −2đ?‘ 2 2 áž™á&#x;„áž™ 1 + đ?‘ ≼ 0 áž™ á&#x; á&#x;‡ 1 + đ?‘ đ?‘Ą ≤ − 1 + đ?‘ 2đ?‘ á&#x;” យយឞ ងទញបវន 1 + đ?‘ đ?‘Ą + 2đ?‘ ≤ − 1 + đ?‘ 2đ?‘ + 2đ?‘ = −2đ?‘ 2 ≤ 0 ដសចដដលចងá&#x;‹ បវនá&#x;”

242.ď ° áž…ážśá&#x;† áž™ á&#x; á&#x;‡áž‘á&#x;’áž‚áž”á&#x;‹ đ?‘– áž?ážśáž„ đ?‘Žđ?‘– = 1/(1 + đ?‘Ľđ?‘– ) á&#x;” ដសយចនá&#x; á&#x;‡ 0 < đ?‘Žđ?‘– < 1, đ?‘Ľđ?‘– = 1 − đ?‘Žđ?‘– /đ?‘Žđ?‘– នឡង đ?‘Ž1 + đ?‘Ž2 + â‹Ż + đ?‘Žđ?‘›+1 = 1

đ?‘›+1

â&#x;š

đ?‘Ľđ?‘– = đ?‘–=1

áž?ážśáž˜ážœ ឡáž&#x;áž˜áž—ážśáž–áž€ážźáž&#x;ឝី

đ?‘›+1 đ?‘–=1 1 − đ?‘Žđ?‘– đ?‘›+1 đ?‘–=1 đ?‘Žđ?‘–

1 đ?‘›

1 − đ?‘Žđ?‘– = đ?‘Ž1 + đ?‘Ž2 + â‹Ż + đ?‘Žđ?‘–−1 + đ?‘Žđ?‘–+1 +. . +đ?‘Žđ?‘›+1 ≼ đ?‘› đ?‘Ž1 đ?‘Ž2 ‌ đ?‘Žđ?‘–−1 đ?‘Žđ?‘–+1 ‌ đ?‘Žđ?‘›+1 1 đ?‘›

â&#x;ş 1 − đ?‘Žđ?‘– ≼ đ?‘›

đ?‘Žđ?‘˜ đ?‘˜â‰ đ?‘–

áž™á&#x;„យអងគទង ážśá&#x;† áž– ីយáž&#x;មឞ áž‚á&#x;’ននយបឞ đ?‘Žđ?‘˜ , đ?‘˜ ≠đ?‘– áž™áž&#x;មឞ áž‚á&#x;’áž“áž“áž‘ážśá&#x;†áž„អáž&#x;á&#x;‹ á&#x;” ដសយចនá&#x; á&#x;‡ 1 đ?‘›

đ?‘›+1

1 − đ?‘Žđ?‘– ≼ đ?‘›

đ?‘›+1

đ?‘–=1

លរម áž&#x;ážť វណá&#x;’ážŽážœážˇáž…ážˇáž?á&#x;’ážš | V. ážœážˇáž&#x;áž˜áž—ážśáž–

đ?‘Žđ?‘˜ đ?‘˜â‰ 1

1 đ?‘›

‌

đ?‘Žđ?‘˜ đ?‘˜â‰ đ?‘›+1

1 đ?‘›

đ?‘›+1

=đ?‘›

đ?‘Žđ?‘–đ?‘›

đ?‘›+1 đ?‘–=1

đ?‘›+1

=đ?‘›

đ?‘›+1

đ?‘Žđ?‘– đ?‘–=1

129


𝑛+1

𝑥𝑖 = 𝑖=1

𝑛+1 𝑖=1 1 − 𝑎𝑖 𝑛+1 𝑖=1 𝑎𝑖

≥ 𝑛𝑛+1

យោយអងគទង ាំ ព ីរយសមើ គ្នន ទល់ ដត 𝑎1 = 𝑎2 = ⋯ = 𝑎𝑛+1 = 1 (𝑛 + 1) ⟹ 𝑥1 = 𝑥2 = ⋯ = 𝑥𝑛+1 = 𝑛 ។

243. ចាំ យ េះទ្គប់ ចាំ នន ួ គត់ 𝑘, 𝑗 តាង 𝛼𝑘,𝑗 = 𝑥𝑗 +𝑘 /𝑥𝑗 ដដលកនង ុ ប្ ត សនទសេន៍ទាំងអស់ យនេះ យបើ សនទសេន៍្មួយ ធាំ ជាង 𝑛 យយើ ងដក 𝑛 យចញ។ ឧទហរណ៍ 𝑛 + 1 ជាំនស ួ យោយ 1; 𝑛 + 2 ជាំនស ួ យោយ 2; 2𝑛 + 1 → 𝑛 + 1 → 1 ។ល។ ដូយចនេះ ចាំ យ េះទ្គប់ 𝑘 យយើ ងមាន

𝑥1+𝑘 𝑥2+𝑘 𝑥𝑛+𝑘 … =1 𝑥1 𝑥2 𝑥𝑛 𝑥1 + 𝑥2 + ⋯ + 𝑥𝑛 = 𝑥1+𝑗 + 𝑥2+𝑗 + ⋯ + 𝑥𝑛+𝑗 𝑥𝑛+𝑗 = 𝑥𝑗 𝛼𝑘,1 𝛼𝑘,2 … 𝛼𝑘,𝑛 =

តាមវ ិសមភាពកូសុី 𝑛

𝑗 =1

𝑎𝑗 𝑠 − 𝑥𝑗 = 𝑥𝑗

𝑛

𝑗 =1 𝑛

𝑎𝑗 𝑥1 + 𝑥2 + ⋯ + 𝑥𝑛 − 𝑥𝑗 𝑥𝑗

= 𝑗 =1 𝑛

= 𝑗 =1 𝑛

=

𝑎𝑗 𝑥1+𝑗 + 𝑥2+𝑗 + ⋯ + 𝑥𝑛−1+𝑗 + 𝑥𝑛+𝑗 − 𝑥𝑗 𝑥𝑗 𝑎𝑗 𝑥1+𝑗 + 𝑥2+𝑗 + ⋯ + 𝑥𝑛−1+𝑗 𝑥𝑗 𝑎𝑗

𝑗 =1 𝑛

=

𝑥𝑗 +1 𝑥𝑗 +2 𝑥𝑗 +𝑛−1 + + ⋯+ = 𝑥𝑗 𝑥𝑗 𝑥𝑗 𝑛

𝑎𝑗 𝛼1,𝑗 + 𝑗 =1

𝑎𝑗 𝛼2,𝑗 + ⋯ +

𝑗 =1

𝑎𝑗 𝛼𝑘,𝑗 𝑗 =1

𝑛−1

𝑛 −1

=

𝑎𝑗 𝛼𝑘,𝑗 𝑘=1 𝑗 =1

1 𝑛

𝑛

𝑛 𝑘=1

𝑎𝑗

130

១. អនុគមន៍ងាយ | លឹម សុ វណ្ណវិចិត្រ

1 𝑛

𝑛

= 𝑛(𝑛 − 1)

𝑗 =1

សមភាពយកើតមាន យពល 𝑥1 = 𝑥2 = ⋯ = 𝑥𝑛 ។ ដូយចនេះ 𝐶 𝑛 = 𝑛 𝑛 − 1 ។

244. តាង

𝑛

𝑎𝑗 𝛼𝑛−1,𝑗 = 𝑗 =1

1 𝑛

𝑛

𝑛 𝑘=1

𝑎𝑗 𝛼1,𝑗 + 𝛼2,𝑗 + ⋯ + 𝛼𝑛−1,𝑗

𝑛

𝑗 =1

𝑛−1

𝑛

𝑎𝑗 𝑗 =1


1 1 1 1 1+ 1+ ; 𝑄 = 𝑥 𝑦 𝑧 𝑥𝑦𝑧 1 1 1 1 1 1 1 ⟹ 𝑃=1+ + + + + + + 𝑥 𝑦 𝑧 𝑥𝑦 𝑦𝑧 𝑧𝑥 𝑥𝑦𝑧 តាមវ ិសមភាពកូសុី 1 1 1 + + ≥ 3𝑄 𝑥 𝑦 𝑧 1 1 1 + + ≥ 3𝑄 2 𝑥𝑦 𝑦𝑧 𝑧𝑥 3 𝑄≥ =3 𝑥+𝑦+𝑧 1 = 𝑄3 𝑥𝑦𝑧 ⟹ 𝑃 ≥ 1 + 3𝑄 + 3𝑄 2 + 𝑄 3 = 1 + 𝑄 3 ≥ 1 + 3 3 = 64 អងគទង ាំ ២យសមើ គ្នន យពល 𝑥 = 𝑦 = 𝑧 = 1 3 ។ 𝑃 = 1+

1/3

245. ជាដំបូងយ ើ ងនឹងបង្ហាញថា ចំយ ោះគ្គប់ 𝑎, 𝑏 > 0 យគមាន យ ើ

𝑎2 − 𝑎𝑏 + 𝑏 2 1 ≥ 𝑎2 + 𝑎𝑏 + 𝑏 2 3 អងគ ទំង២យ្មគ្ន ើ ា ទល់ តែ 𝑎 = 𝑏 ។

យ ើ ងមាន 𝑎2 − 𝑎𝑏 + 𝑏 2 1 ≥ ⟺ 3 𝑎2 − 𝑎𝑏 + 𝑏 2 ≥ 𝑎2 + 𝑎𝑏 + 𝑏 2 ⟺ 𝑎2 + 𝑏 2 ≥ 2𝑎𝑏 𝑎2 + 𝑎𝑏 + 𝑏 2 3 ពិែ។ អងគ ទំង២យ្មគ្ន ើ ា ទល់ តែ 𝑎 = 𝑏។  តាង 𝑥𝑖9 + 𝑥𝑗9

𝑓 𝑥1 , 𝑥2 , … , 𝑥𝑛 = ∀𝑖, 𝑎𝑖 = 𝑥𝑖3

1≤𝑖<𝑗 ≤𝑛

𝑥𝑖6 + 𝑥𝑖3 𝑥𝑗3 + 𝑥𝑗6

យយើ ងមាន 𝑎1 𝑎2 … 𝑎𝑛 = 1 យហើ យ

លឹម សុ វណ្ណវិចិត្រ | V. វិសមភាព

131


𝑥𝑖9 + 𝑥𝑗9

𝑓 𝑥1 , 𝑥2 , … , 𝑥𝑛 = 1≤𝑖<𝑗 ≤𝑛

𝑥𝑖6 + 𝑥𝑖3 𝑥𝑗3 + 𝑥𝑗6

=

𝑎𝑖 + 𝑎𝑗 1≤𝑖<𝑗 ≤𝑛

1 = 3

𝑎𝑖2 𝑎𝑖2

𝑎𝑖3 + 𝑎𝑗3

=

1≤𝑖<𝑗 ≤𝑛 − 𝑎𝑖 𝑎𝑗 + 𝑎𝑗2 + 𝑎𝑖 𝑎𝑗 + 𝑎𝑗2

𝑎𝑖2 + 𝑎𝑖 𝑎𝑗 + 𝑎𝑗2 ≥

𝑎𝑖 + 𝑎𝑗 1≤𝑖<𝑗 ≤𝑛

1 3

𝑎𝑖 + 𝑎𝑗 1≤𝑖<𝑗 ≤𝑛

1 𝑎 + 𝑎2 + 𝑎1 + 𝑎3 + ⋯ + 𝑎1 + 𝑎𝑛 + 𝑎2 + 𝑎3 + 𝑎2 + 𝑎4 3 1 𝑛−1 + ⋯ + 𝑎2 + 𝑎𝑛 + ⋯ . = 𝑎1 + 𝑎2 + ⋯ + 𝑎𝑛 3 𝑛 𝑛−1 𝑛 𝑛−1 ≥ 𝑎1 𝑎2 … 𝑎𝑛 1/𝑛 = 3 3 𝑛 𝑛−1 ⟹ 𝑓 𝑥1 , 𝑥2 , … , 𝑥𝑛 ≥ 3 អងគទង ាំ ២យសមើ គ្នន ទល់ ដត 𝑥1 = 𝑥2 = ⋯ = 𝑥𝑛 = 1 ។ =

246. តាង 176 176 𝑎𝑏𝑐𝑑 = 𝑏𝑐 𝑎 + 𝑑 + 𝑎𝑑 𝑏 + 𝑐 − 𝑏𝑐 27 27 យយើ ងយឃើ ញថា 𝑓 𝑎, 𝑏, 𝑐, 𝑑 ជាទាំ ក់ ទាំនងឆាេះុ (យបើយគបតូរ 𝑎 យៅ 𝑏 និង 𝑏 យៅ 𝑎 ។ល។ យ េះ 𝑓 យៅ 𝑓 𝑎, 𝑏, 𝑐, 𝑑 = 𝑎𝑏𝑐 + 𝑏𝑐𝑑 + 𝑐𝑑𝑎 + 𝑑𝑎𝑏 −

ដដដល)។ 1) ករណី 𝑏 + 𝑐 −

176 𝑏𝑐 27

≤0

តាមវ ិសមភាពកូសុី 𝑏+𝑐+ 𝑎+𝑑 𝑓 𝑎, 𝑏, 𝑐, 𝑑 ≤ 𝑏𝑐 𝑎 + 𝑑 ≤ 3

3

=

1 27

⟹ វ ិសមភាពព ិត។ 2) ករណី 𝑏 + 𝑐 −

176 𝑏𝑐 27

>0

តាមវ ិសមភាពកូសុី

ដូយចនេះ

𝑎+𝑑 𝑓 𝑎, 𝑏, 𝑐, 𝑑 ≤ 𝑏𝑐 𝑎 + 𝑑 + 2

2

𝑏+𝑐−

176 𝑎+𝑑 𝑎+𝑑 𝑏𝑐 = 𝑓 , 𝑏, 𝑐, 27 2 2

𝑎+𝑑 𝑎+𝑑 𝑎+𝑑 𝑎+𝑑 , 𝑏, 𝑐, = 𝑓 𝑏, , , 𝑐 យទ្ េះ f ជាអនុគមន៍សុី យមទ្ទី 2 2 2 2 𝑏+𝑐 𝑎+𝑑 𝑎+𝑑 𝑏+𝑐 𝑎+𝑑 𝑏+𝑐 𝑎+𝑑 𝑏+𝑐 ≤𝑓 , , , =𝑓 , , , 2 2 2 2 2 2 2 2 𝑓 𝑎, 𝑏, 𝑐, 𝑑 ≤ 𝑓

132

១. អនុគមន៍ងាយ | លឹម សុ វណ្ណវិចិត្រ


𝑎+𝑑+𝑏+𝑐 𝑏+𝑐 𝑎+𝑑 𝑎+𝑑+𝑏+𝑐 1 𝑏+𝑐 𝑎+𝑑 1 , , , =𝑓 , , , 4 2 2 4 4 2 2 4 𝑏+𝑐 1 1 𝑎+𝑑 1 1 1 1 1 =𝑓 , , , ≤𝑓 , , , = 2 4 4 2 4 4 4 4 27 ⟹ វ ិសមភាពព ិត។ ≤𝑓

247. តាង 𝐸 = 𝑎1 , 𝑎2 , … , 𝑎𝑛 ∈ ℝ+∗ |𝑎1 + 𝑎2 + ⋯ + 𝑎𝑛 = 1 និង 𝑓 ជាអនុគមន៍មួយ កាំនត់ យលើ 𝐸 យោយ

𝑛

𝑛

2

𝑓 𝑎1 , 𝑎2 , … , 𝑎𝑛 = 𝑛 𝑛 − 1

𝑎𝑘 + 𝑘=1

ដដល

𝑘=1

1 = 𝑎𝑘

𝑃𝑘 𝑎1 , 𝑎2 , … , 𝑎𝑛

𝑃𝑘 𝑎1 , 𝑎2 , … , 𝑎𝑛

𝑛

𝑎𝑖 𝑖=1

យបើ សិ នជាទ្គប់ 𝑎𝑖 សុ ទដធ តខុសគ្ននទាំងអស់ យ េះយគមានព ីរកនង ុ ចាំ យ មយ េះ ឧទហរណ៍ 𝑎1 , 𝑎2 ដដល 𝑎1 < 𝑚 < 𝑎2 ដដល 𝑚 = 𝑎1 + 𝑎2 + ⋯ + 𝑎𝑛

𝑛=1 𝑛។

យយើ ងមាន 𝑓 𝑎1 , 𝑎2 , … , 𝑎𝑛 = 𝑎1 + 𝑎2 𝐴 + 𝑎1 𝑎2 𝐵

ដដល

𝑛

𝑛

𝑛

2

𝐴=

𝑎𝑖 ; 𝐵 = 𝑛 𝑛 − 1 𝑖=3

𝑎𝑖 + 𝑖=3

𝑖=3

𝑃𝑖 𝑎1 , 𝑎2 , … , 𝑎𝑛 𝑎1 𝑎2

(យបើ 𝑛 = 2 យយើ ងតាង 𝐴 = 1; 𝐵 = 𝑛2 𝑛 − 1 )។ យយើ ងទញបាន 𝑓 𝑚, 𝑎1 + 𝑎2 − 𝑚, … , 𝑎𝑛 − 𝑓 𝑎1 , 𝑎2 , … , 𝑎𝑛 = 𝐵 𝑚 𝑎1 + 𝑎2 − 𝑚 − 𝑎1 𝑎2 = 𝐵 𝑚 − 𝑎1 𝑎2 − 𝑚 > 0 ដូចគ្នន យយើ ងទញបាន 𝑓 𝑚, 𝑎1 + 𝑎2 − 𝑚, … , 𝑎𝑛 < 𝑓 𝑚, 𝑚, … , 𝑎𝑛 < ⋯ < 𝑓 𝑚, 𝑚, … , 𝑚 ដូយចនេះ 𝑓 𝑎1 , 𝑎2 , … , 𝑎𝑛 ≤ 𝑓 𝑚, 𝑚, … , 𝑚 យោយអងគទង ាំ ព ីរយសមើ គ្ននយពល 𝑎1 = 𝑎2 = ⋯ = 𝑎𝑛 = 𝑚 ⟹ 𝑚 = 1 𝑛 ។ ចាំ យ េះ 𝑚 = 1 𝑛 𝑓 𝑚, 𝑚, … , 𝑚 = 1 ⟹ 𝑓 𝑎1 , 𝑎2 , … , 𝑎𝑛 ≤ 𝑛−3 𝑛 𝑛

2

⟺ 𝑛 𝑛−1

𝑛

𝑎𝑘 + 𝑘=1

𝑛2 𝑛 − 1 1 𝑛3 1 + 𝑛 = = 𝑛−3 𝑛 𝑛−1 𝑛 𝑛 𝑛 𝑛 𝑛

𝑃𝑘 𝑎1 , 𝑎2 , … , 𝑎𝑛 ≤ 𝑘=1

លឹម សុ វណ្ណវិចិត្រ | V. វិសមភាព

1 𝑛𝑛−3 133


𝑛 2

⟺ 𝑛 𝑛 − 1 𝑎1 𝑎2 … 𝑎𝑛 + 𝑎1 𝑎2 … 𝑎𝑛 𝑛

⟺ 𝑛2 𝑛 − 1 + 𝑘=1

𝑘=1

1 1 ≤ 𝑛−3 𝑎𝑘 𝑛

1 1 1 ≤ 𝑛−3 𝑎𝑘 𝑛 𝑎1 𝑎2 … 𝑎𝑛

យោយអងគទង ាំ ព ីរយសមើ គ្នន យពល 𝑎1 = 𝑎2 = ⋯ = 𝑎𝑛 = 1/𝑛 ។

248. តាង 𝑎0 = 1 − 𝑎1 + 𝑎2 + ⋯ + 𝑎𝑛 ។ យយើ ងមាន 𝑎0 > 0, 𝑛

𝑎𝑘 = 1, 𝑘=0

𝑛 𝑘=0 𝑎𝑘

𝑎1 𝑎2 … 𝑎𝑛 1 − 𝑎1 − 𝑎2 − ⋯ − 𝑎𝑛 = 𝑎1 + 𝑎2 + ⋯ + 𝑎𝑛 1 − 𝑎1 1 − 𝑎2 … 1 − 𝑎𝑛

𝑛 𝑘=0

1 − 𝑎𝑘

តាមវ ិសមភាពកូសុីយយើ ងមាន 1 𝑛

𝑛

1 − 𝑎𝑖 = 𝑛

𝑎𝑘 − 𝑎𝑖 = 𝑘=0 1 𝑛

1 − 𝑎𝑘 ≥ 𝑛 𝑘=0

𝑛+1

1 𝑛

𝑎𝑘 𝑘≠0

𝑎𝑘

𝑎𝑘 ≥ 𝑛 𝑘≠𝑖

𝑘≠1

យទ្ េះ 𝑎𝑖 និមួយៗមាន 𝑛 ដង។

1 𝑛

𝑎𝑘

𝑎𝑘 𝑘≠𝑖 𝑛

=𝑛

𝑘≠𝑛 𝑛 𝑘=0 𝑎𝑘

𝑛 𝑘=0

1 − 𝑎𝑘

𝑛+1

𝑎𝑘 𝑘=0

1 𝑛𝑛+1

ព ិត។ សមភាពយកើតមាន យពល 𝑎0 = 𝑎1 = ⋯ = 𝑎𝑛 = 1 (𝑛 + 1)។

249. តាមសមមតក ិ មម 𝑥1 ; 𝑥2 និង 𝑦1 ; 𝑦2 ជាកូអរយោយនននចាំ នច ុ សថិ តយលើរងេង់ោាំរង្ហេស់ 𝑐 មានផចត ិ យៅទ្តង់ គល់ តាំ រយ ុ ។ ដូយចនេះ 𝑥1 = 𝑐 cos 𝜃 ; 𝑥2 = 𝑐 sin 𝜃 ; 𝑦1 = 𝑐 cos 𝜙 ; 𝑦2 = 𝑐 sin 𝜙 ។ដូយចនេះ 𝑆 = 2 − 𝑐 cos 𝜃 + sin 𝜃 + cos 𝜙 + sin 𝜙 + 𝑐 2 cos 𝜃 + cos 𝜙 + sin 𝜃 + sin 𝜙 𝜋 𝜋 = 2 − 2𝑐 sin 𝜃 + + sin 𝜙 + + 𝑐 2 cos 𝜃 − 𝜙 ≤ 2 + 2 2𝑐 + 𝑐 2 = 4 4 អងគទង ាំ ព ីរយសមើ គ្ននយពល 𝜃 = 𝜙 = 5𝜋 4 មានន័យថា 𝑥1 = 𝑥2 = 𝑦1 = 𝑦2 = −𝑐 2 2 ។

250.  រនរៀរទី១ 134

១. អនុគមន៍ងាយ | លឹម សុ វណ្ណវិចិត្រ

2+𝑐

2


តាង 𝑎 = 1 𝑥 , 𝑏 = 1 𝑦 , 𝑐 = 1 𝑧 យយើ ងទញបាន 𝑥𝑦𝑧 = 1 ។ វ ិសមភាពសមមូលនឹង 𝑥2 𝑦2 𝑧2 3 + + ≥ 𝑦+𝑧 𝑧+𝑥 𝑥+𝑦 2 តាមវ ិសមភាពកូសុីាេត 𝑥2 𝑦2 𝑧2 𝑦+𝑧 + 𝑧+𝑥 + 𝑥+𝑦 + + ≥ 𝑥+𝑦+𝑧 𝑦+𝑧 𝑧+𝑥 𝑥+𝑦 𝑥2 𝑦2 𝑧2 𝑥+𝑦+𝑧 ⟹ + + ≥ 𝑦+𝑧 𝑧+𝑥 𝑥+𝑦 2 តាមវ ិសមភាពកូសុី 𝑥 + 𝑦 + 𝑧 3 𝑥𝑦𝑧 ≥ 2 2

ដូយចនេះវ ិសមភាពព ិត។

1 3

=

2

3 2

 រនរៀរទី២ វ ិសមភាពសមមូលនឹង 1 1 1 + 3 + 3 ≥ + 𝑐) 𝑏 (𝑐 + 𝑎) 𝑐 𝑎 + 𝑏

𝑎3 (𝑏

3 2 𝑎𝑏𝑐

4 3

តាង 𝑎 = 𝑥 3 , 𝑏 = 𝑦 3 , 𝑐 = 𝑧 3 ដដល 𝑥, 𝑦, 𝑧 > 0 យ េះ 1 1 1 3 + 9 3 + 9 3 ≥ 𝑥9 𝑦3 + 𝑧3 𝑦 𝑧 + 𝑥3 𝑧 (𝑥 + 𝑦 3 ) 2𝑥 4 𝑦 4 𝑧 4 ⟺ 2𝑦 9 𝑧 9 𝑧 3 + 𝑥 3 𝑥 3 + 𝑦 3 + 2𝑥 9 𝑧 9 𝑦 3 + 𝑧 3 𝑥 3 + 𝑦 3 + 2𝑥 9 𝑦 9 𝑦 3 + 𝑧 3 𝑧 3 + 𝑥 3 ≥ 3𝑥 5 𝑦 5 𝑧 5 𝑦 3 + 𝑧 3 𝑧 3 + 𝑥 3 𝑥 3 + 𝑦 3 9 9 3 3 ⟺ 2𝑦 𝑧 𝑥 𝑧 + 𝑧 3 𝑦 3 + 𝑥 6 + 𝑥 3 𝑦 3 + 2𝑥 9 𝑧 9 𝑥 3 𝑦 3 + 𝑦 6 + 𝑥 3 𝑧 3 + 𝑦 3 𝑧 3 + 2𝑥 9 𝑦 9 𝑦 3 𝑧 3 + 𝑥 3 𝑦 3 + 𝑧 6 + 𝑥 3 𝑧 3 ≥ 3𝑥 5 𝑦 5 𝑧 5 𝑦 3 𝑧 3 + 𝑥 3 𝑦 3 + 𝑧 6 + 𝑥 3 𝑧 3 𝑥 3 + 𝑦 3 3 9 12 ⟺ 2𝑥 𝑦 𝑧 + 2𝑦12 𝑧12 + 2𝑥 6 𝑦 9 𝑧 9 + 2𝑥 3 𝑦12 𝑧 9 + 2𝑥 12 𝑦 3 𝑧 9 + 2𝑥 9 𝑦 6 𝑧 9 + 2𝑥 12 𝑧12 + 2𝑥 9 𝑦 3 𝑧12 + 2𝑥 9 𝑦12 𝑧 3 + 2𝑥 12 𝑦12 + 2𝑥 9 𝑦 9 𝑧 6 + 2𝑥 12 𝑦 9 𝑧 3 ≥ 3𝑥 5 𝑦 5 𝑧 5 𝑥 3 𝑦 3 𝑧 3 + 𝑦 6 𝑧 3 + 𝑥 6 𝑦 3 + 𝑥 3 𝑦 6 + 𝑥 3 𝑧 6 + 𝑦 3 𝑧 6 + 𝑥 6 𝑧 3 + 𝑥3 𝑦3𝑧3 12 12 ⟺ 2 𝑥 𝑦 + 𝑦12 𝑧12 + 𝑧12 𝑥12 + 2 𝑥 12 𝑦 9 𝑧 3 + 𝑥 12 𝑦 3 𝑧 9 + 𝑥 9 𝑦12 𝑧 3 + 𝑥 9 𝑦 3 𝑧12 + 𝑥 3 𝑦12 𝑧 9 + 𝑥 3 𝑦 9 𝑧12 + 2 𝑥 6 𝑦 9𝑧 9 + 𝑥 9 𝑦 6 𝑧 9 + 𝑥 9 𝑦 9𝑧 6 ≥ 3𝑥 8 𝑦 8 𝑧 8 + 3𝑥 5 𝑦11 𝑧 8 + 3𝑥 11 𝑦 8 𝑧 5 + 3𝑥 8 𝑦11 𝑧 5 + 3𝑥 8 𝑦 5 𝑧11 + 3𝑥 5 𝑦 8 𝑧11 + 3𝑥 11 𝑦 5 𝑧 8 + 3𝑥 8 𝑦 8 𝑧 8

លឹម សុ វណ្ណវិចិត្រ | V. វិសមភាព

135


𝑥 12 𝑦12 + 2

⟺ 𝑠𝑦𝑚

𝑥 12 𝑦 9 𝑧 3 + 𝑠𝑦𝑚

𝑥 9 𝑦 9𝑧 6 𝑠𝑦𝑚

≥ 3(𝑥 11 𝑦 8 𝑧 5 + 𝑥 11 𝑦 5 𝑧 8 + 𝑥 8 𝑦11 𝑧 5 + 𝑥 8 𝑦 5 𝑧11 + 𝑥 5 𝑦11 𝑧 8 + 𝑥 5 𝑦 8 𝑧11 + 6𝑥 8 𝑦 8 𝑧 8 𝑥 12 𝑦12 + 2

⟺ 𝑠𝑦𝑚

𝑥 12 𝑦 9 𝑧 3 + 𝑠𝑦𝑚

𝑥 12 𝑦12 −

⟺ 𝑠𝑦𝑚

𝑥 9 𝑦 9𝑧 6 ≥ 3 𝑠𝑦𝑚

𝑥 11 𝑦 8 𝑧 5 + 2 𝑠𝑦𝑚

𝑠𝑦𝑚

𝑥 12 𝑦 9 𝑧 3 − 𝑠𝑦𝑚

𝑥 9 𝑦 9𝑧 6 −

+ 𝑠𝑦𝑚

𝑥 11 𝑦 8 𝑧 5 + 𝑠𝑦𝑚

𝑥 11 𝑦 8 𝑧 5 𝑠𝑦𝑚

𝑥8 𝑦8 𝑧8 ≥ 0 𝑠𝑦𝑚

-យយើ ងមាន (12,12,0) លុ បយលើ (11,8,5) យទ្ េះ

តាមវ ិសមភាព

12 ≥ 12 ≥ 0; 11 ≥ 8 ≥ 5 12 ≥ 11; 12 + 12 ≥ 11 + 8 12 + 12 + 0 = 11 + 8 + 5 យយើ ងទញបាន 𝑥 12 𝑦12 − 𝑠𝑦𝑚

𝑥 11 𝑦 8 𝑧 5 ≥ 0 𝑠𝑦𝑚

-យយើ ងមាន (12,9,3) លុ បយលើ (11,8,5) យទ្ េះ

តាមវ ិសមភាព

12 ≥ 9 ≥ 3; 11 ≥ 8 ≥ 5 12 ≥ 11; 12 + 9 ≥ 11 + 8 12 + 9 + 3 = 11 + 8 + 5 យយើ ងទញបាន 𝑥 12 𝑦 9 𝑧 3 − 𝑠𝑦𝑚

𝑥 11 𝑦 8 𝑧 5 ≥ 0 𝑠𝑦𝑚

-យយើ ងមាន (9,9,6) លុ បយលើ (8,8,8) យទ្ េះ

តាមវ ិសមភាព

9 ≥ 9 ≥ 6; 8 ≥ 8 ≥ 8 9 ≥ 8; 9 + 9 ≥ 8 + 8 9+9+6 = 8+8+8 យយើ ងទញបាន 𝑥 9𝑦9𝑧6 − sym

ដូយចនេះវ ិសមភាពព ិត។

136

១. អនុគមន៍ងាយ | លឹម សុ វណ្ណវិចិត្រ

𝑥8 𝑦8𝑧8 ≥ 0 sym

𝑥8 𝑦8 𝑧8


251.  រនរៀរទី១ យយើ ងតាង 𝑥=

𝑎 𝑎2

,𝑦 =

𝑏

,𝑧 =

𝑏2

𝑐 𝑐2

+ 8𝑏𝑐 + 8𝑐𝑎 + 8𝑎𝑏 យយើ ងយឃើ ញថា 𝑥, 𝑦, 𝑧 ∈ 0; 1 ។ យយើ ងទ្តូវបង្ហាញថា 𝑥 + 𝑦 + 𝑧 ≥ 1 ។ យយើ ងមាន 𝑎2 𝑥2 𝑏2 𝑦2 𝑐2 𝑧2 = , = , = 8𝑏𝑐 1 − 𝑥 2 8𝑎𝑐 1 − 𝑦 2 8𝑎𝑐 1 − 𝑎2 1 𝑥2 𝑦2 𝑧2 ⟹ = 512 1 − 𝑥 2 1 − 𝑦 2 1 − 𝑧 2 ⟹ 1 − 𝑥 2 1 − 𝑦 2 1 − 𝑧 2 = 512 𝑥𝑦𝑧 2 ឧបមាថា 1 > 𝑥 + 𝑦 + 𝑧 យ េះ 1 − 𝑥2 1 − 𝑦2 1 − 𝑧2 > 𝑥 + 𝑦 + 𝑧 2 − 𝑥2 𝑥 + 𝑦 + 𝑧 2 − 𝑦2 𝑥 + 𝑦 + 𝑧 = 𝑥+𝑥+𝑦+𝑧 𝑦+𝑧 𝑥+𝑦+𝑧+𝑦 𝑥+𝑧 𝑥+𝑦+𝑧+𝑧 𝑥+𝑦 1

1

1

1

1

≥ 4 𝑥 2 𝑦𝑧 4 . 2 𝑦𝑧 2 . 4 𝑥𝑦 2 𝑧 4 . 2 𝑥𝑧 2 . 4 𝑥𝑦𝑧 2 4 . 2 𝑥𝑦 = 512 𝑥𝑦𝑧 2 ផទយ ុ ព ីលកខខណឌ។ ដូយចនេះ 𝑥 + 𝑦 + 𝑧 ≥ 1 ។

1 2

2

− 𝑧2

តាមវ ិសមភាពកូសុី

 រនរៀរទី២ យយើ ងជាំនស ួ 𝑥= វ ិសមភាពយៅជា

𝑎 𝑏 𝑐 ,𝑦 = ,𝑧 = 𝑎+𝑏+𝑐 𝑎+𝑏+𝑐 𝑎+𝑏+𝑐

𝑥𝑓 𝑥 2 + 8𝑦𝑧 + 𝑦𝑓 𝑦 2 + 8𝑧𝑥 + 𝑧𝑓 𝑧 2 + 8𝑥𝑦 ≥ 1 ដដល 𝑓 𝑡 = 1/ 𝑡។ យោយ 𝑓 ជាអនុគមន៍ផតយលើ ℝ+ យហើ យ 𝑥 + 𝑦 + 𝑧 = 1 យ េះយោយយទ្បើវ ិសមភាពយិ នសិ ន យយើ ង ទញបាន 𝑥𝑓 𝑥 2 + 8𝑦𝑧 + 𝑦𝑓 𝑦 2 + 8𝑧𝑥 + 𝑧𝑓 𝑧 2 + 8𝑥𝑦 ≥ 𝑓 𝑥 𝑥 2 + 8𝑦𝑧 + 𝑦 𝑦 2 + 8𝑧𝑥 + 𝑧 𝑧 2 + 8𝑥𝑦 យយើ ងមាន 𝑓 1 = 1 ។ យោយអនុគមន៍ 𝑓 ចុេះោច់ ខាត យ េះយយើ ងទ្គ្នន់ ដតបង្ហាញថា 1 ≥ 𝑥 𝑥 2 + 8𝑦𝑧 + 𝑦 𝑦 2 + 8𝑧𝑥 + 𝑧 𝑧 2 + 8𝑥𝑦 ជាោរយស្សច។ យោយ 𝑥 + 𝑦 + 𝑧 = 1 យ េះ វ ិសមភាពយនេះសមមូលនឹង 𝑥 + 𝑦 + 𝑧 3 ≥ 𝑥 𝑥 2 + 8𝑦𝑧 + 𝑦 𝑦 2 + 8𝑧𝑥 + 𝑧 𝑧 2 + 8𝑥𝑦 ⟺ 3 𝑥 𝑦 − 𝑧 2 + 𝑦 𝑧 − 𝑥 2 + 𝑧 𝑥 − 𝑦 2 ≥ 0 ព ិត។ លឹម សុ វណ្ណវិចិត្រ | V. វិសមភាព

137


 រនរៀរទី៣ តាង 𝑥 = 𝑏𝑐 𝑎2 , 𝑦 = 𝑐𝑎 𝑏 2 , 𝑧 = 𝑎𝑏 𝑐 2 យ េះ 𝑥𝑦𝑧 = 1។ វ ិសមភាពសមមូលនឹង 1 1 1 + + ≥1 1 + 8𝑥 1 + 8𝑧 1 + 8𝑦 ⟺

1 + 8𝑥 (1 + 8𝑦) + 1 + 8𝑦 (1 + 8𝑧) + 1 + 8𝑧 (1 + 8𝑥) ≥ 1 + 8𝑥 1 + 8𝑦 1 + 8𝑧 ⟺ 1 + 8𝑥 1 + 8𝑦 + 1 + 8𝑦 1 + 8𝑧 + 1 + 8𝑧 1 + 8𝑥 + 2 1 + 8𝑥 1 + 8𝑦 1 + 8𝑧 1 + 8𝑥 + 1 + 8𝑦 + ≥ 1 + 8𝑥 1 + 8𝑦 1 + 8𝑧 ⟺ 1 + 8𝑥 + 8𝑦 + 64𝑥𝑦 + 1 + 8𝑦 + 8𝑧 + 64𝑦𝑧 + 1 + 8𝑥 + 8𝑧 + 64𝑥𝑧 + 2 1 + 8𝑥 1 + 8𝑦 1 + 8𝑧 1 + 8𝑥 + 1 + 8𝑦 + ≥ 1 + 8𝑥 + 8𝑦 + 64𝑥𝑦 1 + 8𝑧 ⟺ 3 + 16𝑥 + 16𝑦 + 16𝑧 + 64𝑥𝑦 + 64𝑦𝑧 + 64𝑥𝑧 + 2 1 + 8𝑥 1 + 8𝑦 1 + 8𝑧 1 + 8𝑥 + 1 + 8𝑦 + ≥ 1 + 8𝑧 + 8𝑥 + 64𝑥𝑧 + 8𝑦 + 64𝑦𝑧 + 64𝑥𝑦 + 512𝑥𝑦𝑧 ⟺ 8 𝑥 + 𝑦 + 𝑧 + 2 1 + 8𝑥 1 + 8𝑦 1 + 8𝑧 1 + 8𝑥 + 1 + 8𝑦 +

1 + 8𝑧

1 + 8𝑧

1 + 8𝑧 1 + 8𝑧 ≥ 510

យោយ xyz = 1 យ េះតាមវ ិសមភាពកូសុី 𝑥+𝑦+𝑧 ≥3 1 8 9 = 9𝑥 9 1 8 8𝑦 = 1 + 𝑦 + 𝑦 + ⋯ + 𝑦 ≥ 9 𝑦 8 9 = 9𝑦 9 1 8 8𝑧 = 1 + 𝑧 + 𝑧 + ⋯ + 𝑧 ≥ 9 𝑧 8 9 = 9𝑧 9 8 8 8 1 + 8𝑥 1 + 8𝑦 1 + 8𝑧 ≥ 9𝑥 9 . 9𝑦 9 . 9𝑧 9

1 + 8𝑥 = 1 + 𝑥 + 𝑥 + ⋯ + 𝑥 ≥ 9 𝑥 8 1+ 1+ ⟹

1 + 8𝑥 + 1 + 8𝑦 + 1 + 8𝑧 ≥ ≥9

8 9𝑥 9

+

= 729

8 9𝑦 9

1 4 4 4 3 𝑥9𝑦9𝑧9

8

4

4

4

+ 9𝑧 9 = 3 𝑥 9 + 𝑦 9 + 𝑧 9

=9

⟹ 8 𝑥 + 𝑦 + 𝑧 + 2 1 + 8𝑥 1 + 8𝑦 1 + 8𝑧 ≥ 8.3 + 2. 729. 9 = 510 ⟹ វ ិសមភាពព ិត។

1 + 8𝑥 + 1 + 8𝑦 + 1 + 8𝑧

252. តាង 𝑎 = tan 𝑥 , 𝑏 = tan 𝑦 , 𝑐 = tan 𝑧 ដដល − 2 < 𝑥, 𝑦, 𝑧 < 2 ។ យ េះ 𝑎2 + 1 = 𝜋

𝜋

1 cos2 𝑥 , 𝑏 2 + 1 = 1 cos2 𝑦 , 𝑐 2 + 1 = 1 cos 2 𝑧 ។ គុណអងគទង ាំ ព ីរននវ ិសមភាពយោយ cos2 𝑥 cos2 𝑦 cos2 𝑧 យយើ ងទញបាន 𝑎𝑏 + 𝑏𝑐 + 𝑐𝑎 − 1 cos 𝑥 cos 𝑦 cos 𝑧 138

១. អនុគមន៍ងាយ | លឹម សុ វណ្ណវិចិត្រ

2

≤1


យយឞ áž„áž˜ážśáž“ đ?‘Žđ?‘? + đ?‘?đ?‘? cos đ?‘Ľ cos đ?‘Ś cos đ?‘§ = sin đ?‘Ľ sin đ?‘Ś cos đ?‘§ + sin đ?‘Ś sin đ?‘§ cos đ?‘Ľ = sin đ?‘Ś sin đ?‘Ľ + đ?‘§ នឡង đ?‘?đ?‘Ž − 1 cos đ?‘Ľ cos đ?‘Ś cos đ?‘§ = sin đ?‘§ sin đ?‘Ľ cos đ?‘Ś − cos đ?‘Ľ cos đ?‘Ś cos đ?‘§ = − cos đ?‘Ś cos đ?‘Ľ + đ?‘§ ដសយចនá&#x; á&#x;‡áž™áž™ážž ងទញបវន đ?‘Žđ?‘? + đ?‘?đ?‘? + đ?‘?đ?‘Ž − 1 cos đ?‘Ľ cos đ?‘Ś cos đ?‘§

2

= sin đ?‘Ś sin đ?‘Ľ + đ?‘§ − cos đ?‘Ś cos đ?‘Ľ + đ?‘§ = cos 2 đ?‘Ľ + đ?‘Ś + đ?‘§ ≤ 1

2

253.ď ° ď‚Œ ážšáž“ážšá&#x;€ážšáž‘ីá&#x;Ą áž?ážśáž„ đ?‘“ đ?‘Ľ, đ?‘Ś, đ?‘§ = đ?‘Ľđ?‘Ś + đ?‘Śđ?‘§ + đ?‘§đ?‘Ľ − 2đ?‘Ľđ?‘Śđ?‘§ á&#x;” áž?ážśáž˜áž›áž€áž ážŽá&#x;ˆáž†ážśá&#x; á&#x;‡ážť យយឞ áž„á&#x;„áž…áž&#x;នមáž?áž?ážś 0 ≤ đ?‘Ľ ≤ đ?‘Ś ≤ đ?‘§ ≤ 1 á&#x;” áž™á&#x;„áž™ đ?‘Ľ + đ?‘Ś + đ?‘§ = 1 áž™ á&#x; á&#x;‡ đ?‘Ľ ≤ 1/3 á&#x;” ដសយចនá&#x; á&#x;‡ đ?‘“ đ?‘Ľ, đ?‘Ś, đ?‘§ = 1 − 3đ?‘Ľ đ?‘Śđ?‘§ + đ?‘Ľđ?‘Śđ?‘§ + đ?‘§đ?‘Ľ + đ?‘Ľđ?‘Ś ≼ 0 áž?ážśáž˜ážœ ឡáž&#x;áž˜áž—ážśáž–áž€ážźáž&#x;ឝី យយឞ ងទញបវន

áž™á&#x;„áž™ 1 − 2đ?‘Ľ > 0 áž™ á&#x; á&#x;‡

�+� �� ≤ 2

2

1−đ?‘Ľ = 2

đ?‘“ đ?‘Ľ, đ?‘Ś, đ?‘§ = đ?‘Ľ đ?‘Ś + đ?‘§ + đ?‘Śđ?‘§ 1 − 2đ?‘Ľ ≤ đ?‘Ľ 1 − đ?‘Ľ + áž” áž‘ áž”á&#x;‹ មកយទá&#x;€áž? យយឞ áž„áž‘á&#x;’áž?ážźážœážšáž€áž?ážśá&#x;† នលធវá&#x;† បវá&#x;†áž•áž? ážť ážšáž”áž&#x;á&#x;‹ áž˘áž“ážťáž‚áž˜áž“á&#x;? đ??š đ?‘Ľ = ដដល đ?‘Ľ ∈ 0, 1 3 á&#x;” យយឞ áž„áž˜ážśáž“

2

1−đ?‘Ľ 2

2

1 − 2đ?‘Ľ =

−2đ?‘Ľ 3 + đ?‘Ľ 2 + 1 4

−2đ?‘Ľ 3 + đ?‘Ľ 2 + 1 4

3 1 1 đ??š ′ đ?‘Ľ = đ?‘Ľ − đ?‘Ľ ≼ 0; ∀đ?‘Ľ ∈ 0; 2 3 3 យយឞ ងទញបវន đ??š đ?‘Ľ ≤ đ??š 1 3 = 7 27 ; ∀đ?‘Ľ ∈ 0; 1 3 á&#x;”

ď‚? ážšáž“ážšá&#x;€ážšáž‘ីá&#x;˘ យយឞ ងបវá&#x;† ដលងវ ឡáž&#x;áž˜áž—ážśáž–áž™á&#x;…ជវ 0 ≤ đ?‘Ľđ?‘Ś + đ?‘Śđ?‘§ + đ?‘§đ?‘Ľ đ?‘Ľ + đ?‘Ś + đ?‘§ − 2đ?‘Ľđ?‘Śđ?‘§ ≤ យយឞ áž„áž˜ážśáž“

7 �+�+� 27

3

đ?‘Ľ2đ?‘Ś

0 ≤ đ?‘Ľđ?‘Ś + đ?‘Śđ?‘§ + đ?‘§đ?‘Ľ đ?‘Ľ + đ?‘Ś + đ?‘§ − 2đ?‘Ľđ?‘Śđ?‘§ â&#x;ş 0 ≤ đ?‘Ľđ?‘Śđ?‘§ + sym

áž– ឡáž?á&#x;” យយឞ áž„áž˜ážśáž“ លរម áž&#x;ážť វណá&#x;’ážŽážœážˇáž…ážˇáž?á&#x;’ážš | V. ážœážˇáž&#x;áž˜áž—ážśáž–

139


7 𝑥+𝑦+𝑧 27 𝑥2𝑦 ≥ 0

𝑥𝑦 + 𝑦𝑧 + 𝑧𝑥 𝑥 + 𝑦 + 𝑧 − 2𝑥𝑦𝑧 ≤ 𝑥 3 + 15𝑥𝑦𝑧 − 6

⟺7 cyclic

3

sym

𝑥3 −

⟺ 2 cyclic

𝑥 2 𝑦 + 5 3𝑥𝑦𝑧 + sym

𝑥3 − cyclic

𝑥2𝑦 ≥ 0 sym

ដូយចនេះ យយើ ងទ្គ្នន់ ដតបង្ហាញថា 𝑥 2 𝑦 ≥ 0; និង 3𝑥𝑦𝑧 +

𝑥3 −

2 cyclic

sym

𝑥3 − cyclic

𝑥2𝑦 ≥ 0 sym

ជាោរទ្គប់ ទ្គ្នន់ ។ យយើ ងមាន 𝑥3 −

2 cyclic

𝑥2𝑦 = sym

𝑥3 + 𝑦3 − cyclic

𝑥 2 𝑦 + 𝑥𝑦 2 cyclic

3

𝑥 + 𝑦 − 𝑥 2 𝑦 − 𝑥𝑦 2

=

3

cyclic

𝑥2 𝑥 − 𝑦 − 𝑦2 𝑥 − 𝑦

= cyclic

=

𝑥−𝑦

2

𝑥+𝑦 ≥0

cyclic

វ ិសមភាព 𝑥3 −

3𝑥𝑦𝑧 + cyclic

ព ិត តាមវ ិសមភាព

254.  យយើ ងមាន

sym

𝑥 𝑥−𝑦 𝑥−𝑧 ≥0 cyclic

ករណី 𝑟 = 1 ។

𝑧 𝑥 𝑥 𝑦 𝑦 𝑧 𝑧 =2+ + + + + + 𝑥 𝑦 𝑧 𝑥 𝑧 𝑥 𝑦 𝑥 𝑥 𝑦 𝑦 𝑧 𝑧 = + +1 + + +1 + + +1 −1 𝑦 𝑧 𝑥 𝑧 𝑥 𝑦 1+

≥3 =3 =2 140

𝑥2𝑦 ≥ 0 ⟺

𝑥 𝑦

3

1+

𝑦 𝑧

1+

𝑥2 3 𝑦2 3 𝑧2 + + −1 𝑦𝑧 𝑥𝑧 𝑥𝑦

𝑥+𝑦+𝑧 3

−1

3

+

𝑥𝑦𝑧 𝑥+𝑦+𝑧 𝑥𝑦𝑧

១. អនុគមន៍ងាយ | លឹម សុ វណ្ណវិចិត្រ

𝑥+𝑦+𝑧 3

𝑥𝑦𝑧

−1


≥2 =2

𝑥+𝑦+𝑧 3

𝑥𝑦𝑧 𝑥+𝑦+𝑧 3

=2 1+

+3

3

𝑥𝑦𝑧

3

𝑥𝑦𝑧

−1

+2 𝑥𝑦𝑧 𝑥+𝑦+𝑧 3

𝑥𝑦𝑧

255.  តាង 𝑎 𝑏 = 𝑝; 𝑏 𝑐 = 𝑞 ។ ដូយចនេះ 𝑑 𝑐 = 𝑎 𝑏 = 𝑝; 𝑎 𝑑 = 𝑏 𝑐 = 𝑞 ។ យយើ ងមាន 1 1 + =4 1 𝑝 𝑞 𝑎 𝑏 𝑐 𝑑 𝑝 𝑞 1 1 1 + + + = 𝑝𝑞 + + + = 𝑝+ 𝑞+ 𝑐 𝑑 𝑎 𝑏 𝑞 𝑝 𝑝𝑞 𝑝 𝑞 លកខខណឌ 𝑎, 𝑏, 𝑐, 𝑑 ជាចាំ នន ួ ព ិតខុសគ្នន យាំ ោយ 𝑝 ≠ 1; 𝑞 ≠; 𝑝𝑞 ≠ 1 ។ 𝑝+𝑞+

យបើ 𝑝 > 0; 𝑞 > 0 យ េះ 1 ≥ 2; 𝑝 1 𝑞+ ≥2 𝑞 យាំ ោយ 𝑝 + 1 𝑝 = 2; 𝑞 + 1 𝑞 = 2 ⟹ 𝑝 = 𝑞 = 1 ផទយ ុ ព ីសមម តក ិ មម ។ ដូយចនេះទ្តូវមានមួយ 𝑝+

(1)

កនង ុ ចាំ យ ម 𝑝; 𝑞 ជាចាំ នន ួ អវ ិជាមាន សនមតថា 𝑝 < 0 ។ ដូយចនេះ 1 𝑝 + ≤ −2 𝑝 1 1 1 1 ⟹ 𝑞 + ≥ 6; 𝑝 + 𝑞+ ≤ −12 𝑞 𝑝 𝑞 សញ្ជាយសមើ យកើតមានយពល 𝑝 = −1; 𝑞 + 1 𝑞 = 6 ⟹ 𝑞 = 3 ± 2 2 ។ 𝑑

យក 𝑐 = 1; 𝑐 = 𝑝 = −1 ⟹ 𝑑 = −1 ។ យក 𝑞 = 3 − 2 2 យយើ ងទញបាន 𝑏 = 𝑞 =3−2 2 ⟹𝑏 = 3−2 2 𝑐 𝑎 = −1 ⟹ 𝑎 = −3 + 2 2 𝑏 ដូយចនេះតាំ នលធាំ បាំផត ុ យសមើ −12 ។ តាំ នលធាំ បាំផត ុ យនេះយកើតមាន ឧទហរណ៍យៅទ្តង់ ចាំ នច ុ 𝑎 = −3 + 2 2; 𝑏 = 3 − 2 2; 𝑐 = 1; 𝑑 = −1

256.  សនមតថា 𝑎 ≤ 𝑏 ≤ 𝑐 ។ យយើ ងមាន 𝑐 < 𝑎 + 𝑏 ⟹ 2𝑐 < 𝑎 + 𝑏 + 𝑐 = 2 ⟹ 𝑐 < 1 ។ 𝑎2 + 𝑏 2 + 𝑐 2 + 2𝑎𝑏𝑐 = 𝑎 + 𝑏 + 𝑐 2 − 2 𝑎𝑏 + 𝑏𝑐 + 𝑐𝑎 + 2𝑎𝑏𝑐 = 4 − 2 𝑎𝑏 + 𝑏𝑐 + 𝑐𝑎 + 2𝑎𝑏𝑐 2 − 2 1 − 𝑎 1 − 𝑏 1 − 𝑐 = 4 − 2 𝑎𝑏 + 𝑏𝑐 + 𝑐𝑎 + 2𝑎𝑏𝑐 លឹម សុ វណ្ណវិចិត្រ | V. វិសមភាព

141


ដូយចនេះ 𝑎2 + 𝑏 2 + 𝑐 2 + 2𝑎𝑏𝑐 = 2 − 2 1 − 𝑎 1 − 𝑏 1 − 𝑐 < 2 យទ្ េះ 𝑎 < 1; 𝑏 < 1; 𝑐 < 1 ។ យោយ 𝑎 ≤ 𝑏 ≤ 𝑐 < 1 យ េះ 1 − 𝑎, 1 − 𝑏, 1 − 𝑐 ជាចាំ នន ួ វ ិជាមាន។ តាមវ ិសមភាពកូសុី យយើ ងទញ បាន 1−𝑎 + 1−𝑏 + 1−𝑐 ≥ 3 1 ⟹ 1−𝑎 1−𝑏 1−𝑐 ≤ 27 2 52 2 2 2 ⟹ 𝑎 + 𝑏 + 𝑐 + 2𝑎𝑏𝑐 ≥ 2 − = 27 27 សញ្ជាយសមើ យកើតមានយពល 𝑎 = 𝑏 = 𝑐 ។

3

1 − 𝑎 1 − 𝑏 (1 − 𝑐)

257. ដចកអងគទង ាំ ព ីរនឹង 𝑎𝑏𝑐 យយើ ងទញបាន 𝑎 𝑏 𝑐 + + 𝑐 𝑎 𝑏

𝑎2 +1 𝑏𝑐

3 𝑐 𝑎 𝑏 + + ≥1+ 𝑎 𝑐 𝑐

𝑏2 +1 𝑐𝑎

𝑐2 +1 𝑎𝑏

តាង 𝑥 = 𝑎 𝑏 , 𝑦 = 𝑏 𝑐 , 𝑧 = 𝑐 𝑎 ⟹ 𝑥𝑦𝑧 = 1 ។ ដូយចនេះវ ិសមភាពយៅជា 𝑥 + 𝑦 + 𝑧 𝑥𝑦 + 𝑦𝑧 + 𝑧𝑥 ≥ 1 +

តាង 𝑝 =

3

3

𝑥 +1 𝑧

𝑥 + 𝑦 𝑦 + 𝑧 𝑧 + 𝑥 + 𝑥𝑦𝑧 ≥ 1 +

𝑥+𝑦 𝑦+𝑧 𝑧+𝑥 +1≥1+

3

3

𝑦 +1 𝑥

𝑧 +1 𝑦

𝑥+𝑧 𝑦+𝑥 𝑧+𝑦 𝑥𝑦𝑧

𝑥+𝑧 𝑦+𝑥 𝑧+𝑦

𝑥 + 𝑧 𝑦 + 𝑥 𝑧 + 𝑦 ។ វ ិសមភាពយៅជា

𝑝3 + 1 ≥ 1 + 𝑝 ⟺ 𝑝3 + 1 − 1 + 𝑝

2

≥ 0 ⟺ 𝑝 𝑝+1 𝑝−2 ≥ 0

យយើ ងមាន 𝑝≥

3

2 𝑥𝑧. 2 𝑦𝑥. 2 𝑧𝑦 = 2

ដូយចនេះ (*) ព ិត។

258.  យក 𝑝 = 𝑚 +𝑛 ; 𝑞 = 𝑚 +𝑛 ដដល 𝑚, 𝑛 ជាចាំ នន ួ គត់ វ ិជាមាន។ វ ិសមភាព 𝑚

𝑛

𝑝𝑎 + 𝑞𝑏 ≥ 𝑎𝑝 𝑏 𝑞 142

១. អនុគមន៍ងាយ | លឹម សុ វណ្ណវិចិត្រ

𝑚𝑎 + 𝑛𝑏 ≥ 𝑚+𝑛

𝑚 +𝑛

𝑎𝑚 𝑏 𝑛


ព ិត តាមវ ិសមភាពកូសុី។

 យយើ ងយទ្ជើសយរ ើសសេុី តននចាំ នន ួ សនិទន 𝑎1 , 𝑎2 , … , 𝑎𝑛 , .. ដដល lim𝑛⟶∞ 𝑎𝑛 = 𝑝 ។ តាង 𝑏𝑖 = 1 − 𝑎𝑖 យយើ ងទញបាន lim𝑛⟶∞ 𝑏𝑛 = 𝑞 ។ តាមសាំ នួរទីមួយ យយើ ងទញបាន 𝑎𝑛 𝑥 + 𝑏𝑛 𝑦 ≥ 𝑥 𝑎 𝑛 𝑦 𝑏𝑛 ⟹ lim 𝑎𝑛 𝑥 + 𝑏𝑛 𝑦 ≥ lim 𝑥 𝑎 𝑛 𝑦 𝑏𝑛 ⟺ 𝑝𝑥 + 𝑞𝑦 ≥ 𝑥 𝑝 𝑦 𝑞 𝑛⟶∞

𝑛⟶∞

 តាមវ ិសមភាពយិ នសិ ន យយើ ងមាន

𝑎

𝑎

𝑎

ln 𝑎1 𝑥1 + 𝑎2 𝑥2 + ⋯ + 𝑎𝑛 𝑥𝑛 ≥ 𝑎1 ln 𝑥1 + 𝑎2 ln 𝑥2 + ⋯ + 𝑎𝑛 ln 𝑥𝑛 = ln 𝑥1 1 𝑥2 2 … 𝑥𝑛 𝑛 𝑎 𝑎 𝑎 ⟹ 𝑎1 𝑥1 + 𝑎2 𝑥2 + ⋯ + 𝑎𝑛 𝑥𝑛 ≥ 𝑥1 1 𝑥2 2 … 𝑥𝑛 𝑛

 តាង

1 𝑝

𝑛

𝑢=

𝑎𝑖

1 𝑝

𝑝

𝑎𝑖 𝑢

𝑝

𝑎𝑖

𝑝 ∑𝑛𝑖=1 𝑎𝑖 𝑎𝑖 𝑝 ⟹ 𝑢𝑣 𝑝 ∑𝑛𝑖=1 𝑎𝑖 𝑎𝑖 𝑝 ⟹ 𝑢𝑣 𝑝 ∑𝑛𝑖=1 𝑎𝑖 𝑛

⟹ 𝑢𝑣 𝑖=1

+

1 + 𝑞 𝑞 𝑏𝑖

𝑝

𝑝 𝑝

𝑝 ∑𝑛𝑖=1 𝑎𝑖

𝑞

1 𝑝

𝑛

𝑎𝑖 𝑖=1

𝑎𝑖 𝑏𝑖

≥ 𝑢𝑣

𝑞

𝑝

𝑏𝑖 𝑖=1

អងគទង ាំ ព ីរយសមើ គ្ននទល់ ដត លឹម សុ វណ្ណវិចិត្រ | V. វិសមភាព

𝑎𝑖 𝑝 𝑢

𝑏𝑖 𝑣

𝑞

1 𝑞

=

𝑎𝑖 𝑏𝑖 𝑢 𝑣

𝑞

𝑎𝑖

𝑝 ∑𝑛𝑖=1 𝑎𝑖

𝑝

+

𝑏𝑖

𝑞 ∑𝑛𝑖=1 𝑏𝑖

𝑞

≥ 𝑎𝑖 𝑏𝑖

𝑛

𝑞 𝑞

≥ 𝑛

≥ 𝑛

⟺ 𝑢𝑣 ≥

𝑞

1 𝑝

≥ 𝑎𝑖 𝑏𝑖

𝑞

𝑞 ∑𝑛𝑖=1 𝑏𝑖

𝑎𝑖 𝑏𝑖 𝑖=1

𝑎𝑖 𝑏𝑖 𝑖=1

𝑎𝑖 𝑏𝑖 𝑖=1

1 𝑞

𝑛

𝑝

𝑝

𝑛

𝑖=1

𝑞

≥ 𝑎𝑖 𝑏𝑖

∑𝑛𝑖=1 𝑎𝑖 𝑝 ∑𝑛𝑖=1 𝑏𝑖 𝑞 ⟺ 𝑢𝑣 + 𝑝 ∑𝑛𝑖=1 𝑎𝑖 𝑝 𝑞 ∑𝑛𝑖=1 𝑏𝑖 𝑞 1 1 ⟺ 𝑢𝑣 + ≥ 𝑝 𝑞

𝑎𝑖 𝑢

𝑞

𝑏𝑖

+

𝑝

𝑏𝑖 𝑖=1

𝑏𝑖 𝑣

𝑞 ∑𝑛𝑖=1 𝑏𝑖 𝑏𝑖 𝑞 + 𝑞 ∑𝑛𝑖=1 𝑏𝑖 𝑏𝑖 𝑞 + 𝑞 ∑𝑛𝑖=1 𝑏𝑖

𝑝

𝑎𝑖

;𝑣 =

𝑖=1

ចាំ យ េះទ្គប់ 𝑖 យយើ ងមាន

⟺ 𝑢𝑣

𝑝

1 𝑞

𝑛

𝑛

≥ =

𝑎𝑖 𝑏𝑖 𝑖=1 𝑏𝑖 𝑞 𝑣

និង 𝑎𝑖 𝑏𝑖 មានសញ្ជាដូចគ្នន ចាំ យ េះទ្គប់ 𝑖 143


𝑝

𝑞

𝑎𝑖

∑𝑛𝑖=1 𝑎𝑖

𝑝

𝑏𝑖

=

𝑝 𝑎𝑖

∑𝑛𝑖=1 𝑎𝑖 = 𝑛 ∑𝑖=1 𝑏𝑖

⟹ ∑𝑛𝑖=1 𝑏𝑖 𝑞 𝑞 𝑞 𝑞 = 𝑐𝑣 𝑏1 , 𝑏2 , … , 𝑏𝑛

𝑝 𝑞

𝑞

𝑞

𝑝

𝑝

𝑝

𝑏𝑖 = 𝑐𝑏𝑖 ⟹ 𝑢 𝑎1 , 𝑎2 , … , 𝑎𝑛

ដូយចនេះមានន័យថា វុចទ័ ិ រ 𝑢 កូលយី នដអ៊ែ រនឹង 𝑣 យហើ យ 𝑎𝑖 𝑏𝑖 មានសញ្ជាដូចគ្នន ចាំ យ េះទ្គប់ 𝑖 ។ ករណីយនេះ យយើ ងទញបានសមភាព។

សំគាល់ យបើ 𝑝 = 𝑞 = 2 វ ិសមភាពយនេះ ោាយជាវ ិសមភាពកូសុី-ាេត។

259. តាមវ ិសមភាពមន-មធទូយៅ យយើ ងទញបាន 1 𝑝

1 𝑝 1 𝑞 𝑎 + 𝑏 ≥ 𝑎𝑝 𝑝 𝑞

𝑏𝑞

1 𝑞

= 𝑎𝑏

260. តាង 𝐻𝑛 = 1 + 2 + ⋯ + 𝑛 និង ចាំ យ េះ 𝑖 = 1,2, … , 𝑛 តាង 𝑤𝑖 = 1 (𝑖𝐻𝑛 ) ។ 1

1

យយើ ងមាន 𝑤𝑖 > 0 យហើ យ

តាង 𝑆 = 𝑥1 +

𝑥 22 2

𝑛

+ ⋯+

𝑆 = 𝐻𝑛

𝑛

𝑥 𝑛2 𝑛

𝑖=1

𝑥𝑖𝑖

𝑖=1

𝑤𝑖

តាមវ ិសមភាពមន − មធទូយៅ

𝑖=1 𝑛

𝑖=1

𝑖=1

1 1 = 𝐻 =1 𝑖 𝐻𝑛 𝑛

𝑛

𝑤𝑖 𝑥𝑖𝑖 𝑖𝑤 𝑥𝑖 𝑖

𝑛

។ យយើ ងមាន

𝑛

=

1 𝑤𝑖 = 𝐻𝑛

=

1/𝐻𝑛

𝑛 1/𝐻 𝑥𝑖 𝑛

=

𝑖=1

𝑥𝑖 𝑖=1

តាមវ ិសមភាពកូសុី

𝑛

𝑛

𝑥𝑖 ≥

=1 1 𝑥𝑖 ≥ 1 យហើ យយសមើ គ្នន យពល 𝑥1 = 𝑥2 = ⋯ = 𝑥𝑛 = 1។ 𝑖=1

𝑆

ដូយចនេះ 𝐻

𝑛

𝑛

∑𝑛𝑖=1

261. តាង 𝑞 > 0 ដដល 1 1 + =1 ⟹ 𝑝−1 𝑞 =𝑝 𝑝 𝑞 144

១. អនុគមន៍ងាយ | លឹម សុ វណ្ណវិចិត្រ


ចាំ យ េះទ្គប់ 𝑖 យយើ ងមាន ⟹

𝑝

𝑎𝑖 + 𝑏𝑖

𝑛

𝑛

𝑎𝑖 𝑎𝑖 + 𝑏𝑖

𝑖=1

+

1 𝑝

𝑛 𝑝−1

𝑖=1

𝑝−1

𝑝

𝑎𝑖

𝑖=1

𝑏𝑖

𝑎𝑖 + 𝑏𝑖

1 𝑝

+

𝑏𝑖 1 𝑝

𝑝

𝑎𝑖

𝑛

𝑎𝑖 + 𝑏𝑖

𝑝

យយើ ងទញបានសមភាពយបើ 𝑝

𝑝

; ∀𝑖 ។

𝑖=1

𝑝

𝑖=1

𝑝

1 𝑞

𝑎𝑖 + 𝑏𝑖

𝑝

1 𝑝 𝑝

𝑏𝑖 𝑖=1

𝑝

𝑎𝑖

𝑎𝑖 + 𝑏𝑖

𝑖=1

+

1 𝑝

𝑛

𝑖=1

𝑏𝑖 = 𝑑 𝑎𝑖 + 𝑏𝑖

𝑎𝑖 + 𝑏𝑖

𝑝

𝑛

𝑖=1 1 𝑝

𝑝

1 𝑞

𝑛

𝑖=1

𝑖=1

𝑝

𝑛

𝑝

𝑝

1 𝑞

𝑛

𝑖=1

𝑛

𝑖=1 1 1− 𝑞

𝑛

1 𝑝

𝑎𝑖

𝑛

𝑖=1

𝑛

𝑎𝑖 + 𝑏𝑖

1 𝑝

𝑖=1

𝑛

= 𝑖=1

𝑛

𝑏𝑖 𝑎𝑖 + 𝑏𝑖

1 𝑝

𝑛

𝑖=1

𝑛

ដូយចនេះ

(𝑝−1)𝑞

𝑎𝑖 + 𝑏𝑖

𝑖=1

ដូចគ្នន

𝑝−1

𝑝−1

1 𝑞

𝑛

𝑝

𝑎𝑖

𝑏𝑖 𝑎𝑖 + 𝑏𝑖

+ 𝑏𝑖 𝑎𝑖 + 𝑏𝑖

𝑖=1

យយើ ងមាន

𝑛

𝑎𝑖 𝑎𝑖 + 𝑏𝑖

𝑝−1

𝑖=1

តាមវ ិសមភាព

𝑝−1

𝑎𝑖 + 𝑏𝑖 ≤ 𝑎𝑖 𝑎𝑖 + 𝑏𝑖

𝑛

𝑝

𝑎𝑖 + 𝑏𝑖

𝑝−1

= 𝑎𝑖 + 𝑏𝑖

1 𝑝

𝑛

+

𝑏𝑖

𝑝

𝑖=1

𝑎𝑖 មានសញ្ជាដូច 𝑏𝑖 និង

និង

𝑝

𝑎𝑖 = 𝑐 𝑎𝑖 + 𝑏𝑖

𝑝

; ∀𝑖 និង

⟹ 𝑎𝑖 = 𝛼𝑏𝑖 និង 1 ⟹ 𝑎𝑖 = 𝛽𝑏𝑖 , 𝛽 > 0 ។ ដូយចនេះ 𝑝

𝑝

សមភាពយកើតមានយពល 𝑢 𝑎1 , 𝑎2 , … , 𝑎𝑛 = 𝛽 𝑣 𝑏1 , 𝑏2 , … , 𝑏𝑛 ដដល 𝛽 > 0។

262. តាង 4

𝑆=

4

𝑥3 4

1

2

𝑥3 + 𝑥2 + 𝑦2 3 𝑥 + 𝑧 3 𝑥 = 𝑎3 ; 𝑦 = 𝑏 3 ; 𝑧 = 𝑐 3 តាមវ ិសមភាព យយើ ងមាន

លឹម សុ វណ្ណវិចិត្រ | V. វិសមភាព

+

4

𝑦3 4

𝑦3 + 𝑦2 + 𝑧2

1 3

𝑦+𝑥

2 3

+

𝑧3 4

𝑧3 + 𝑧2 + 𝑥2

1 3

𝑧+𝑦

145

2 3


đ?‘Ľ2 +

1 đ?‘Ś2 3

�+�

2 3

đ?‘Ž2

= 2 3

= đ?‘Ľđ?‘Ś

3

+

+ ��

1 đ?‘?2 3 3

3 đ?‘?2 2

≼ đ?‘Ž2 đ?‘? 2 + đ?‘? 2 đ?‘Ž2

2 3

4

4

đ?‘Ľ3

â&#x;š

+

2 3 3 đ?‘Ž2 2

4

1 3

đ?‘Ľ3 + đ?‘Ľ2 + đ?‘Ś2

�+�

2 3

2

đ?‘Ľ3

≤

4

2

� 3 + �� 3 + ��

2 3

đ?‘Ľ3

=

2

2

2

�3 + �3 + �3

áž?ážśáž˜ážšáž™áž”á&#x;€áž”ដសចគá&#x;’áž“áž“ 4

2

đ?‘Ś3 4

�3 + �2 + �2

1 3

đ?‘Ś+đ?‘Ľ

2 3

≤

đ?‘Ś3 2

4

�2

2

2

�3 4 �3

2

�3 + �3 + �3

1 đ?‘Ľ2 3

2 3

≤

�3 2

2

2

+ + đ?‘§+đ?‘Ś đ?‘Ľ3 + đ?‘Ś3 + đ?‘§3 បសកអងគនង ážš áž˘áž„áž‚áž“áž“ážœ ឡáž&#x;áž˜áž—ážśáž–áž”áž‰á&#x;’áž…ážź លគá&#x;’áž“áž“ យយឞ ងទញបវន đ?‘† ≤ 1á&#x;”

263.ď ° យយឞ áž„áž˜ážśáž“ đ?‘Ž, đ?‘?, đ?‘?, đ?‘‘ ∈ â„? á&#x;” áž?ážśáž„ đ?‘ = đ?‘Ž + đ?‘? + đ?‘? + đ?‘‘ á&#x;” áž?ážśáž˜ážœ ឡáž&#x;áž˜áž—ážśáž– đ?‘†1 =

đ?‘Ž+1

2

+2 ≼ 2 đ?‘?+1 +2 ≼

�2 =

đ?‘?−2 2 + đ?‘?+3 2+ đ?‘?+1 2 +2 đ?‘?−2 2 + đ?‘‘+3 2 đ?‘Ž+đ?‘?+2 2+2 đ?‘?+đ?‘?−4 2+ đ?‘?+đ?‘‘+6 2 đ?‘‘−2 2+ đ?‘Ž+3 2+ đ?‘‘+1 2 +2 đ?‘Žâˆ’2 2 + đ?‘?+3 2 đ?‘?+đ?‘‘+2 2+2 đ?‘‘+đ?‘Žâˆ’4 2+ đ?‘Ž+đ?‘?+6 2

đ?‘† = đ?‘†1 + đ?‘†2 ≼ đ?‘ + 4 2 + 2 đ?‘ − 8 2 + đ?‘ + 12 2 = 4đ?‘ 2 + 288 ≼ 288 = 12 2 áž…ážśá&#x;† áž™ á&#x; á&#x;‡ đ?‘Ž = đ?‘? = đ?‘? = đ?‘‘ = 0 យយឞ áž„áž˜ážśáž“ đ?‘† = 12 2 á&#x;” ដសយចនá&#x; á&#x;‡áž?ážśá&#x;† áž“áž›áž?សចបវá&#x;† áž•áž? ážť ážšáž”áž&#x;á&#x;‹ đ?‘† áž‚ážş 12 2 á&#x;”

264.ď ° ď‚Œ áž?ážśáž„ đ?‘Ľ = tan đ??´ , đ?‘Ś = tan đ??ľ , đ?‘§ = tan đ??ś ដដល 0 < đ??´, đ??ľ, đ??ś < đ?œ‹ 2á&#x;” លកហហណឌ áž&#x;áž˜áž˜ážźáž›áž“ážšáž„ tan đ??´ + tan đ??ľ + tan đ??ś = tan đ??´ tan đ??ľ tan đ??ś â&#x;š đ??´ + đ??ľ + đ??ś = đ?œ‹ នឡងយá&#x;„áž™ tan đ??´ ážœ ឡáž&#x;áž˜áž—ážśáž–ážŠážŠáž›áž™á&#x;„áž™áž&#x;áž˜áž˜ážźáž›áž“ážšáž„

1 + tan2 đ??´

= sin đ??´

3 3 2 á&#x;” ដសយចនá&#x; á&#x;‡ áž?ážśáž˜ážœ ឡáž&#x;áž˜áž—ážśáž–áž™ážˇ áž“áž&#x;ឡ áž“ យយឞ ងទញបវន

sin đ??´ + sin đ??ľ + sin đ??ś ≤ đ?œ‹

áž˘áž“ážťáž‚áž˜áž“á&#x;? đ?‘Ś = sin đ?‘Ľ áž‡ážśáž˘áž“ážťáž‚áž˜áž“á&#x;?យបវá&#x;‰ážśáž„យលឞ 0; 2

sin đ??´ + sin đ??ľ + sin đ??ś đ??´+đ??ľ+đ??ś 3 ≤ sin = 3 3 2 146

á&#x;Ą. áž˘áž“ážťáž‚áž˜áž“á&#x;?áž„ážśáž™ | លរម áž&#x;ážť វណá&#x;’ážŽážœážˇáž…ážˇáž?á&#x;’ážš


យវá&#x;† á&#x;„យវ ឡáž&#x;áž˜áž—ážśáž–áž– ឡáž?á&#x;”

ď‚? áž?ážśáž„ đ?‘Ľ = tan đ??´2 ; đ?‘Ś = tan đ??ľ2 ; đ?‘§ = tan đ??ś2 ដដល 0 < đ??´, đ??ľ, đ??ś < đ?œ‹2 á&#x;” លកហហណឌáž&#x;áž˜áž˜ážźáž›áž“ážšáž„ đ??´ đ??ľ đ??ľ đ??ś đ??´ đ??ś tan tan + tan tan + tan tan = 1 â&#x;š đ??´ + đ??ľ + đ??ś = đ?œ‹ 2 2 2 2 2 2 ážœ ឡáž&#x;áž˜áž—ážśáž–áž&#x;áž˜áž˜ážźáž›áž“ážšáž„ tan đ??´ + tan đ??ľ + tan đ??ś ≼ 3 3 áž– ឡáž?áž?ážśáž˜ážœ ឡáž&#x;áž˜áž—ážśáž–áž™ážˇ áž“áž&#x;ឡ áž“á&#x;”

265.ď ° áž?ážśáž„ đ?‘Ľđ?‘› = tan đ?‘Žđ?‘› ដដល 0 < đ?‘Žđ?‘› < đ?œ‹ 2 á&#x;” យយឞ áž„áž˜ážśáž“

đ?œ‹ đ?‘Žđ?‘› + 4 2 đ?œ‹ á&#x;” ដសយចនá&#x; á&#x;‡ đ?‘Ľđ?‘› = cot đ?œƒđ?‘› ដដល đ?œƒđ?‘› = 6.2đ?‘› −1 á&#x;” ដសចគá&#x;’ននយយឞ ងទញបវន

đ?‘Ľđ?‘›+1 = tan đ?‘Žđ?‘› + 1 + tan2 đ?‘Žđ?‘› = tan đ?œ‹

đ?œ‹

យយឞ ងទញបវន đ?‘Žđ?‘› = 2 − 6.2đ?‘› −1

2

đ?‘Śđ?‘› = tan 2đ?œƒđ?‘› á&#x;” ដសយចនá&#x; á&#x;‡ đ?‘Ľđ?‘› đ?‘Śđ?‘› = 1−tan 2 đ?œƒ

đ?‘›

á&#x;” áž™á&#x;„áž™ 0 < đ?œƒđ?‘› <

đ?œ‹ 4

áž™ á&#x; á&#x;‡áž™áž™ážž áž„áž˜ážśáž“ 0 < tan2 đ?œƒđ?‘› < 1

នឡង đ?‘Ľđ?‘› đ?‘Śđ?‘› > 2á&#x;” áž…ážśá&#x;† áž™ á&#x; á&#x;‡ đ?‘› > 1 យយឞ áž„áž˜ážśáž“ đ?œƒđ?‘› < đ?œ‹ 6 ដសយចនá&#x; á&#x;‡ tan2 đ?œƒđ?‘› < 1 3 នឡង đ?‘Ľđ?‘› đ?‘Śđ?‘› < 3 á&#x;”

266.ď ° áž?ážśáž„ đ?‘‡đ?‘› ជវពហឝ áž’ážś

ទី đ?‘› á&#x;” យយឞ ងដរងáž?ážś đ?‘‡đ?‘› cos đ?‘Ľ = cos đ?‘›đ?‘Ľ នឡង đ?‘‡đ?‘› កវá&#x;†áž“áž?á&#x;‹

áž™á&#x;„áž™ đ?‘‡đ?‘›+1 đ?‘Ľ = 2đ?‘Ľđ?‘‡đ?‘› đ?‘Ľ − đ?‘‡đ?‘›âˆ’1 đ?‘Ľ , đ?‘‡0 đ?‘Ľ = 1 នឡង đ?‘‡1 đ?‘Ľ = đ?‘Ľ á&#x;” ដសយចនá&#x; á&#x;‡áž™áž˜áž‚ឝណធវá&#x;† ážšáž”áž&#x;á&#x;‹ đ?‘‡đ?‘› áž™áž&#x;មឞ áž“áž„ ážš 2đ?‘›âˆ’1 áž…ážśá&#x;† áž™ á&#x; á&#x;‡ đ?‘› ≼ 1 á&#x;” យទá&#x;’បឞរប ážź មនáž?á&#x;„áž„ ážśá&#x;† ដទរបá&#x;‰ážś សឥវáž&#x;á&#x;‚ážťáž„ážšáž”áž&#x;á&#x;‹

áž™á&#x;…យលឞពហឝ ធវដឺយទá&#x;’កទី đ?‘‡đ?‘›âˆ’1 đ?‘Ľ នឡងចវá&#x;† áž™ á&#x; á&#x;‡áž…ážśá&#x;† áž“áž… ážť

đ?‘Ľ1 , đ?‘Ľ2 , â‹Ż , đ?‘Ľđ?‘› យយឞ áž„áž˜ážśáž“

đ?‘›

đ?‘‡đ?‘›âˆ’1 đ?‘Ľ = đ?‘˜=1

đ?‘‡đ?‘›âˆ’1 đ?‘Ľđ?‘˜ đ?‘Ľ − đ?‘Ľ1 â‹Ż đ?‘Ľ − đ?‘Ľđ?‘˜âˆ’1 đ?‘Ľ − đ?‘Ľđ?‘˜+1 â‹Ż đ?‘Ľ − đ?‘Ľđ?‘› đ?‘Ľđ?‘˜ − đ?‘Ľ1 â‹Ż đ?‘Ľđ?‘˜ − đ?‘Ľđ?‘˜âˆ’1 đ?‘Ľđ?‘˜ − đ?‘Ľđ?‘˜+1 â‹Ż đ?‘Ľđ?‘˜ − đ?‘Ľđ?‘›

ផទម ážš áž™áž˜áž‚ážťážŽáž’ážśá&#x;† យយឞ ងទញបវន đ?‘›

2

đ?‘›âˆ’2

= đ?‘˜=1

đ?‘Ľđ?‘˜ − đ?‘Ľ1

đ?‘‡đ?‘›âˆ’1 đ?‘Ľđ?‘˜ â‹Ż đ?‘Ľđ?‘˜ − đ?‘Ľđ?‘˜âˆ’1 đ?‘Ľđ?‘˜ − đ?‘Ľđ?‘˜+1 â‹Ż đ?‘Ľđ?‘˜ − đ?‘Ľđ?‘›

យយឞ ងយក đ?œƒđ?‘˜ ដដល cos đ?œƒđ?‘˜ = đ?‘Ľđ?‘˜ á&#x;” ដសយចនá&#x; á&#x;‡ đ?‘‡đ?‘›âˆ’1 đ?‘Ľđ?‘˜ đ?‘›

2đ?‘›âˆ’2 ≤ đ?‘˜=1

đ?‘Ľđ?‘˜ − đ?‘Ľ1

|đ?‘‡đ?‘›âˆ’1 đ?‘Ľđ?‘˜ | â‹Ż đ?‘Ľđ?‘˜ − đ?‘Ľđ?‘˜âˆ’1 đ?‘Ľđ?‘˜ − đ?‘Ľđ?‘˜+1 â‹Ż đ?‘Ľđ?‘˜ − đ?‘Ľđ?‘› đ?‘›

= យវá&#x;† á&#x;„យវ ឡáž&#x;áž˜áž—ážśáž–áž– ឡáž?á&#x;” លរម áž&#x;ážť វណá&#x;’ážŽážœážˇáž…ážˇáž?á&#x;’ážš | V. ážœážˇáž&#x;áž˜áž—ážśáž–

= | cos đ?‘› − 1 đ?œƒđ?‘˜ | ≤ 1 á&#x;” ដសយចនá&#x; á&#x;‡

đ?‘˜=1

1 đ?‘Ąđ?‘˜

147


267.ď ° ដចកអងគទង ážśá&#x;† áž– ីរននវ ឡáž&#x;áž˜áž—ážśáž–áž“ážšáž„ 4 យយឞ ងទញបវន đ?‘Ž đ?‘? 1− 2 2 យយឞ ងជវá&#x;†áž“áž&#x; áž˝

đ?‘Ž 2

= sin � នឡង

đ?‘? 2

2

−

2

≤

1 2

= sin đ?‘Ś á&#x;” ážœ ឡáž&#x;áž˜áž—ážśáž–áž™áž‘á&#x;’á&#x;„យយនá&#x; á&#x;‡áž™á&#x;…ជវ

sin đ?‘Ľ − đ?‘Ś

= sin đ?‘Ľ cos đ?‘Ś − sin đ?‘Ś cos đ?‘Ľ ≤ sin

យយឞ áž„áž…áž„á&#x;‹ បវន đ?‘Ą1 នឡង đ?‘Ą2 ដដល sin đ?‘Ą1 =

2− 6 4

នឡង sin �2 =

យយឞ ងទញបវន cos 2đ?‘Ą1 = 1 − 2 sin2 đ?‘Ą1 = 1 − 5đ?œ‹ cos á&#x;” យទá&#x;’ 6 5đ?œ‹ đ?‘Ą2 = 12 á&#x;”

đ?‘? đ?‘Ž 1− 2 2

8−4 3 8

á&#x; á&#x;‡ đ?‘Ś = sin đ?‘Ľ áž‡ážśáž˘áž“ážťáž‚áž˜áž“á&#x;?áž˜áž˝áž™áž‘áž›á&#x;‹ áž˜áž˝áž™áž– ី

2+ 6 4

3 = = cos 2 đ?œ‹ đ?œ‹ − ; áž™á&#x;… 2 2

đ?œ‹ 6

(∗)

á&#x;” Âą

đ?œ‹ 6

នឡង cos 2đ?‘Ą2 = −

−1; 1 áž™ á&#x; á&#x;‡ đ?‘Ą1 =

đ?œ‹ − 12

3 2

=

នឡង

đ?œ‹ 5đ?œ‹ đ?œ‹ đ?œ‹ đ?œ‹ ; ជវចយ ážś á&#x; á&#x;‡á&#x;„áž…á&#x;‹ បី ážŠážŠáž›áž“ážˇáž˜áž˝áž™á&#x;—áž˜ážśáž“áž‘á&#x;’បដវង áž‚ážş đ??ź1 = − ; ,đ??ź2 = 12 12 6 12 12 đ?œ‹ đ?œ‹ đ?œ‹ 5đ?œ‹ ; នឡង đ??ź3 = 4 ; 12 á&#x;” áž˘áž“ážťáž‚áž˜áž“á&#x;? đ?‘Ś = 2 sin đ?‘Ľ áž˘áž“ážťážœáž?áž?áž…ážśá&#x;†áž™ á&#x; á&#x;‡áž…áž™ ážś á&#x; á&#x;‡ đ??ź1 , đ??ź2 , đ??ź3 áž˜ážśáž“ážšážźáž”áž—ážśáž–áž˜áž˝áž™áž‚áž?á&#x;‹ 12 4 2− 6 đ?œ‹ đ?œ‹ 2+ 6 យលឞចយ ážś á&#x; á&#x;‡ đ??ź1′ = ; 2 sin 12 , đ??ź2′ = 2 sin 12 ; 2 , đ??ź3′ = 2; 2 យរá&#x;€áž„áž‚á&#x;’áž“áž“á&#x;” áž?ážśáž˜ 2 យគá&#x;’áž“áž›á&#x;„រណá&#x;?ážšáž“áž’áž‘á&#x;’ áž” áž…áž™ ážś á&#x; á&#x;‡áž˜áž˝áž™áž“áž“ đ??ź1′ , đ??ź2′ , đ??ź3′ áž˜ážśáž“áž…ážśá&#x;† áž“áž“ áž˝ áž– ីរននចវá&#x;† áž“áž“ áž˝ ដដលយá&#x;„យចវá&#x;† áž“áž“ áž˝ បួន áž?ážśáž„áž™á&#x;„áž™ đ?‘Ž

យយឞ ងដចកចយ ážś á&#x; á&#x;‡ −

នឡង đ?‘?á&#x;” យយឞ ងទញបវន áž…áž™ ážś á&#x; á&#x;‡áž˜áž˝áž™áž€áž“áž„ ážť áž…ážśá&#x;† áž™ ម đ??ź1 , đ??ź2 , đ??ź3 áž˜ážśáž“áž…ážśá&#x;† áž“áž“ áž˝ áž– ឡáž? đ?‘Ľ នឡង đ?‘Ś ដដល đ?‘Ž = 2 sin đ?‘Ľ នឡង đ?‘? = 2 sin đ?‘Ś á&#x;” áž™á&#x;„យចយ ážś á&#x; á&#x;‡ đ??ź1 , đ??ź2 , đ??ź3 áž˜ážśáž“ážšáž„á&#x;’áž á&#x; áž&#x;á&#x;‹ áž™áž&#x;មឞ đ?œ‹ 6 ដសចគá&#x;’áž“áž“ áž™ á&#x; á&#x;‡ đ?‘Ľ − đ?‘Ś ≤ đ?œ‹ 6 á&#x;” យយឞ áž„ áž‘áž‰áž”ážśáž“ážœ ឡáž&#x;áž˜áž—ážśáž–(*) áž– ឡáž?á&#x;”

268.ď ° áž…ážśá&#x;† áž™ á&#x; á&#x;‡ đ?‘› = 1 : đ?‘“ đ?‘™1 đ?‘Ľ1 ≤ đ?‘™1 đ?‘“ đ?‘Ľ1 áž&#x;ážśá&#x;† យនឞព ឡáž?á&#x;” áž…ážśá&#x;† áž™ á&#x; á&#x;‡ đ?‘› = 2 ážœážśáž‡ážśáž“ážˇáž™áž˜áž“á&#x;?យរបáž&#x;á&#x;‹ áž—ážśáž–áž•áž?á&#x;” áž&#x;នមáž?áž?ážśáž– ឡáž?ážšáž ážź áž?ដលá&#x;‹ đ?‘› á&#x;” យយឞ áž„áž˜ážśáž“ đ?‘›+1

đ?‘›

đ?‘™đ?‘˜ đ?‘Ľđ?‘˜ = ដដល

đ?‘˜=1

đ?‘™đ?‘˜ đ?‘Ľđ?‘˜ + đ?‘™đ?‘›+1 đ?‘Ľđ?‘›+1 = 1 − đ?‘™đ?‘›+1 đ?‘Ś + đ?‘™đ?‘›+1 đ?‘Ľđ?‘›+1 đ?‘˜=1 đ?‘›

đ?‘Ś= ដសយចនá&#x; á&#x;‡

148

đ?‘˜=1

đ?‘™đ?‘˜ đ?‘Ľ ; 1 − đ?‘™đ?‘›+1 đ?‘˜

á&#x;Ą. áž˘áž“ážťáž‚áž˜áž“á&#x;?áž„ážśáž™ | លរម áž&#x;ážť វណá&#x;’ážŽážœážˇáž…ážˇáž?á&#x;’ážš

đ?‘›

đ?‘˜=1

đ?‘™đ?‘˜ =1 1 − đ?‘™đ?‘›+1


đ?‘›+1

đ?‘“

đ?‘™đ?‘˜ đ?‘Ľđ?‘˜ = đ?‘“ 1 − đ?‘™đ?‘›+1 đ?‘Ś + đ?‘™đ?‘›+1 đ?‘Ľđ?‘›+1 ≤ 1 − đ?‘™đ?‘›+1 đ?‘“ đ?‘Ś + đ?‘™đ?‘› +1 đ?‘“ đ?‘Ľđ?‘›+1 đ?‘˜=1

đ?‘›

đ?‘™đ?‘˜ đ?‘Ľ + đ?‘™đ?‘›+1 đ?‘“ đ?‘Ľđ?‘›+1 1 − đ?‘™đ?‘›+1 đ?‘˜ đ?‘˜=1 đ?‘™1 đ?‘™2 đ?‘™đ?‘› đ?‘“ đ?‘Ľ1 + đ?‘“ đ?‘Ľ2 + â‹Ż + đ?‘“ đ?‘Ľđ?‘› 1 − đ?‘™đ?‘›+1 1 − đ?‘™đ?‘›+1 1 − đ?‘™đ?‘›+1

= 1 − đ?‘™đ?‘›+1 đ?‘“ ≤ 1 − đ?‘™đ?‘›+1

đ?‘›+1

+ ��+1 � ��+1 =

đ?‘™đ?‘˜ đ?‘“ đ?‘Ľđ?‘˜ đ?‘˜=1

269.ď ° áž?ážśáž„ đ?‘Ž = tan đ?‘Ľ , đ?‘? = tan đ?‘Ś , đ?‘? = tan đ?‘§ á&#x;” ដសយចនá&#x; á&#x;‡ đ?‘Ľ, đ?‘Ś, đ?‘§ áž‡ážśáž˜ážťážśá&#x;†áž€áž“áž„ ážť áž“áž“áž‘á&#x;’áž?ីយá&#x;„ណáž&#x;á&#x;’áž&#x;áž˝ áž…áž˜áž˝áž™á&#x;” យយឞ áž„áž˜ážśáž“ 3 2 ážœ ឡáž&#x;áž˜áž—ážśáž–áž™áž“á&#x; á&#x;‡áž– ឡáž? áž?ážśáž˜ážœ ឡáž&#x;áž˜áž—ážśáž–áž™ážˇ áž“áž&#x;ឡ áž“á&#x;” យយឞ áž„áž˜ážśáž“áž&#x;áž˜áž—ážśáž–áž™áž–áž› đ?‘Ž = đ?‘? = đ?‘? = 3 á&#x;” cos đ?‘Ľ + cos đ?‘Ś + cos đ?‘§ ≤

270.ď ° áž˘áž“ážťáž‚áž˜áž“á&#x;? đ?‘“ đ?‘Ľ = đ?‘Ľ 1

3

áž‡ážśáž˘áž“ážťáž‚áž˜áž“á&#x;?យបវá&#x;‰ážśáž„យលឞ 0; +∞ á&#x;” áž?ážśáž˜ážœ ឡáž&#x;áž˜áž—ážśáž–áž™ážˇ áž“áž&#x;ឡ áž“ យយឞ áž„

áž˜ážśáž“ 1

1

đ?‘Ž3 + đ?‘? 3 đ?‘Ž+đ?‘? ≤ 2 2

1 3

1

1

â&#x;š đ?‘Ž3 + đ?‘? 3 ≤

2

đ?‘Ž+đ?‘?

1 23

1 3

អងគទង ážśá&#x;† áž– ីរយáž&#x;មឞ áž‚á&#x;’áž“áž“ áž‘áž›á&#x;‹ ដáž? នឡង យវá&#x;† á&#x;„áž™ đ?‘Ž = đ?‘? á&#x;” ដសយចនá&#x; á&#x;‡áž…ážśá&#x;† áž“áž“ áž˝ áž™áž?ážš đ?‘€ áž?សចបវá&#x;† áž•áž? ážť áž‚ážş đ?‘€ =

271.ď ° áž˘áž“ážťáž‚áž˜áž“á&#x;? đ?‘“ đ?‘Ľ =

1 1−đ?‘Ľ

2 1 23

1

= 43 á&#x;”

áž•áž?យលឞ 1;+∞ á&#x;” áž™á&#x;„áž™ đ?‘Ľ1 + đ?‘Ľ2 + â‹Ż + đ?‘Ľđ?‘› = 1 áž™ á&#x; á&#x;‡áž?ážśáž˜

ážœ ឡáž&#x;áž˜áž—ážśáž–áž™ážˇ áž“áž&#x;ឡ áž“ đ?‘Ľ1 đ?‘“ đ?‘Ľ1 + đ?‘Ľ2 đ?‘“ đ?‘Ľ2 + â‹Ż + đ?‘Ľđ?‘› đ?‘“ đ?‘Ľđ?‘› ≼ đ?‘“ đ?‘Ľ1 . đ?‘Ľ1 + đ?‘Ľ2 . đ?‘Ľ2 + â‹Ż + đ?‘Ľđ?‘› . đ?‘Ľđ?‘› 1 1 1 1 â&#x;ş đ?‘Ľ1 + đ?‘Ľ2 + â‹Ż + đ?‘Ľđ?‘› ≼ 2 1 − đ?‘Ľ1 1 − đ?‘Ľ2 1 − đ?‘Ľđ?‘› 1 − đ?‘Ľ1 + đ?‘Ľ22 + â‹Ż + đ?‘Ľđ?‘›2

áž?ážśáž˜ážœ ឡáž&#x;áž˜áž—ážśáž–áž€ážźáž&#x;ឝីវá&#x; áž? យយឞ áž„áž˜ážśáž“

1 đ?‘Ľ + đ?‘Ľ2 + â‹Ż + đ?‘Ľđ?‘› đ?‘› 1 đ?‘Ľ1 đ?‘Ľ2 đ?‘Ľđ?‘› + + â‹Ż+ ≼ 1 − đ?‘Ľ1 1 − đ?‘Ľ2 1 − đ?‘Ľđ?‘›

đ?‘Ľ12 + đ?‘Ľ22 + â‹Ż + đ?‘Ľđ?‘›2 ≼ â&#x;š

2

= 1

1 đ?‘›

1 1−đ?‘›

=

đ?‘› đ?‘›âˆ’1

អងគទង ážśá&#x;† áž– ីរយáž&#x;មឞគá&#x;’áž“áž“ យពល đ?‘Ľ1 = đ?‘Ľ2 = â‹Ż = đ?‘Ľđ?‘› = 1 đ?‘› á&#x;”

272.ď ° យបឞ đ?‘› ≼ 0 áž˘áž“ážťáž‚áž˜áž“á&#x;? đ?‘“ đ?‘Ľ = đ?‘Ľ đ?‘› áž‡ážśáž˘áž“ážťáž‚áž˜áž“á&#x;?áž•áž?យលឞ â„?+∗ á&#x;” ដសយចនá&#x; á&#x;‡áž?ážśáž˜ážœ ឡáž&#x;áž˜áž—ážśáž–áž™ážˇ áž“áž&#x;ឡ áž“ លរម áž&#x;ážť វណá&#x;’ážŽážœážˇáž…ážˇáž?á&#x;’ážš | V. ážœážˇáž&#x;áž˜áž—ážśáž–

149


𝑎 1+ 𝑏 𝑎 𝑏

𝑛

𝑏 + 1+ 𝑎

𝑛

≥2

1+

𝑎 𝑏 +1+ 𝑎 𝑏 2

𝑏 𝑎

ដតថា + ≥ 2 យ េះ 𝑎 1+ 𝑏 យបើ 𝑛 < −1 តាង 𝑝 = −𝑛 > 1

𝑛

𝑏 + 1+ 𝑎

𝑛

1 𝑎 𝑏 =2 1+ + 2 𝑏 𝑎

𝑛

𝑛

≥2 2

𝑛

= 2𝑛+1

𝑏 𝑛 𝑏𝑝 𝑎𝑝 1 𝑎𝑝 + 𝑏 𝑝 ≥ 2𝑛+1 ⟺ + ≥ ⟺ 𝑎 𝑎+𝑏 𝑝 𝑎 + 𝑏 𝑝 2𝑝−1 2 𝑝 𝑎+𝑏 ≥ 2 𝑝 អនុគមន៍ 𝑓 𝑥 = 𝑥 ជាអនុគមន៍ផតយលើ ℝ+∗ ។ ដូយចនេះតាមវ ិសមភាពយិ នសិ ន ⟹

1+

𝑎 𝑏

𝑛

+ 1+

𝑎𝑝 + 𝑏 𝑝 𝑎+𝑏 𝑝 ≥ 2 2 ដូយចនេះវ ិសមភាពព ិត។ យយើ ងយឃើ ញថា អងគទង ាំ ព ីរននវ ិសមភាពយសមើគ្នន ទល់ ដត 𝑛 ∈ 0; −1 និង 𝑎, 𝑏 យ៉ាងយម៉ា ចក៏បាន រ ឺ 𝑛 ∉ 0; −1 និង 𝑎 = 𝑏 ។

273. យយើ ងយឃើ ញថាវ ិសមភាពយៅដដដល យបើយយើ ងជាំនស ួ 𝑎, 𝑏, 𝑐 យោយ 𝛼𝑎, 𝛼𝑏, 𝛼𝑐 (វ ិសមភាពអូ ម៉ាូដសន)។ ដូយចនេះ យយើ ងោចសនមតថា 𝑎 + 𝑏 + 𝑐 = 1 (*)។ អនុគមន៍ 𝑓 𝑥 = 1

𝑥 ជាអនុគមន៍ផត ដូយចនេះតាមវ ិសមភាពយិ នសិ ន និង (*) យយើ ងទញបាន

2

𝑎𝑓 𝑎 + 𝜆𝑏𝑐 + 𝑏𝑓 𝑏 2 + 𝜆𝑐𝑎 + 𝑐𝑓 𝑐 2 + 𝜆𝑎𝑏 ≥ 𝑓 𝑎 𝑎2 + 𝜆𝑏𝑐 + 𝑏 𝑏 2 + 𝜆𝑐𝑎 + 𝑐 𝑐 2 + 𝜆𝑎𝑏 𝑎 𝑏 𝑐 1 ⟹ + + ≥ 𝑎2 + 𝜆𝑏𝑐 𝑏 2 + 𝜆𝑐𝑎 𝑐 2 + 𝜆𝑎𝑏 𝑎 𝑎2 + 𝜆𝑏𝑐 + 𝑏 𝑏 2 + 𝜆𝑐𝑎 + 𝑐 𝑐 2 + 𝜆𝑎𝑏 ដូយចនេះ យយើ ងទ្គ្នន់ ដតបង្ហាញថា 1+𝜆 𝑎 𝑎2 + 𝜆𝑏𝑐 + 𝑏 𝑏 2 + 𝜆𝑐𝑎 + 𝑐 𝑐 2 + 𝜆𝑎𝑏 ≤ 9 1+𝜆 3 3 3 ⟺ 𝑎 + 𝑏 + 𝑐 + 3𝜆𝑎𝑏𝑐 ≤ 9 យយើ ងមាន 𝑎3 + 𝑏 3 + 𝑐 3 = 𝑎 + 𝑏 + 𝑐 តាម(*) យយើ ងទញបាន

3

− 3 𝑎2 𝑏 + 𝑎2 𝑐 + 𝑎𝑏 2 + 𝑎𝑐 2 + 𝑏 2 𝑐 + 𝑏𝑐 2 + 2𝑎𝑏𝑐

𝑎3 + 𝑏 3 + 𝑐 3 + 3𝜆𝑎𝑏𝑐 = 1 − 3 𝑎2 𝑏 + 𝑎2 𝑐 + 𝑎𝑏 2 + 𝑎𝑐 2 + 𝑏 2 𝑐 + 𝑏𝑐 2 + 3 𝜆 − 2 𝑎𝑏𝑐 ≤ 1 − 3.6𝑎𝑏𝑐 + 3 𝜆 − 2 𝑎𝑏𝑐 វ ិសមភាពកូសុី = 1 + 3 𝜆 − 8 𝑎𝑏𝑐 150

១. អនុគមន៍ងាយ | លឹម សុ វណ្ណវិចិត្រ


𝑎+𝑏+𝑐 3 ≤1+3 𝜆−8 វ ិសមភាពកូសុី និង យោយារ λ ≥ 8 3 𝜆−8 1+𝜆 =1+ = 9 9 ព ិត ។ សមភាពយកើតមានយពលវ ិសមភាពកូសុីយៅជាសមភាព មានន័យថា a = b = c ។

274. យោយអនុគមន៍ 𝑓 𝑥 = 𝑥 𝑡 ផតយលើ ℝ+យ េះតាមវ ិសមភាពយិ នសិ ន យយើ ងទញបាន 𝑛

𝑎𝑖𝛼 ដត

+

𝑖=1

1 𝛽

𝑎𝑖

𝑡

1 ≥𝑛 𝑛

𝑛

𝑎𝑖𝛼

+

𝑖=1

𝑡

1 𝛽

𝑎𝑖

1 =𝑛 𝑛

𝑛

𝑎𝑖𝛼 𝑖=1

1 + 𝑛

𝑛

𝑡

1

(∗)

𝛽

𝑖=1 𝑎𝑖

ក) យោយ 𝛼 ≥ 1 យ េះអនុគមន៍ 𝑓 𝑥 = 𝑥 𝛼 ផតយលើ ℝ+ ដូយចនេះ យយើ ងទញបាន 1 𝑛

𝑛

𝑖=1

1 𝑎𝑖𝛼 ≥ 𝑛

𝛼

𝑛

𝑎𝑖

=

𝑖=1

1 𝑛𝛼

ខ) យោយ 𝛽 > 0 យ េះអនុគមន៍ 𝑓 𝑥 = 1 𝑥 ផតយលើ ℝ+ ដូយចនេះ យយើ ងទញបាន 𝛽

1 𝑛 តាម (*), ក) និង ខ) យយើ ងទញបាន

𝑛

1 𝛽

𝑖=1 𝑎𝑖

𝑛

𝑎𝑖𝛼 𝑖=1

+

1 𝛽 𝑎𝑖

𝛽

𝑛

= 𝑛𝛽

∑𝑛𝑖=1 𝑎𝑖 𝑡

≥𝑛

1 + 𝑛𝛽 𝑛𝛼

អងគទង ាំ ព ីរយសមើគ្នន ទល់ ដត 𝑎1 = 𝑎2 = ⋯ = 𝑎𝑛 ។ យោយ 𝑎1 + 𝑎2 + ⋯ + 𝑎𝑛 = 1 យ េះ 𝑎1 = 𝑎2 = ⋯ = 𝑎𝑛 = 1/𝑛។

275. អនុគមន៍ 𝑓 𝑥 = 1+𝑒 𝑥 ផតយលើ ℝ+ យទ្ េះ 1

𝑒 2𝑥 − 𝑒 𝑥 ≥0 1 + 𝑒𝑥 3 ។ យោយ 𝑥𝑖 ≥ 1 យ េះ 𝑦𝑖 ≥ 0 ។ 𝑓 ′′ 𝑥 =

ចាំ យ េះ 𝑖 = 1,2, … , 𝑛 តាង 𝑥𝑖 = 𝑒 𝑦 𝑖 តាមវ ិសមភាពយិ នសិ ន 1 𝑛

𝑛

𝑖=1

1 1 = 𝑥𝑖 + 1 𝑛

𝑛

𝑖=1

1 ≥ 𝑒𝑦𝑖 + 1

1 1 𝑛 ∑ 𝑒 𝑛 𝑖=1 𝑦 𝑖

យោយអងគទង ាំ ព ីរយសមើ គ្នន យពល x1 = x 2 = L = x n ។

= +1

1 𝑥1 𝑥2 … 𝑥𝑛

1 𝑛

+1

276. តាង 𝑓 𝑎 =

𝑎 𝑏 𝑐 + + + 1−𝑎 1−𝑏 1−𝑐 𝑏+𝑐+1 𝑐+𝑎+1 𝑎+𝑏+1

លឹម សុ វណ្ណវិចិត្រ | V. វិសមភាព

151


áž™á&#x;„យចវáž?á&#x;‹ ទក ážť áž?ážś đ?‘?, đ?‘? ជវបវá&#x;‰ážśážšážśá&#x;‰ážśážŠáž˜á&#x;‰ážś áž‘á&#x;’áž?á&#x;” យយឞ ងយឃឞ ញáž?ážś đ?‘“ áž‡ážśáž˘áž“ážťáž‚áž˜áž“á&#x;?áž•áž?យធá&#x;€áž”áž“ážšáž„ đ?‘Ž ដសយចនá&#x; á&#x;‡ áž?ážśá&#x;† នលធវá&#x;† បវá&#x;† áž•áž? ážť ážšáž”áž&#x;á&#x;‹ ážœážś áž‚ážşáž™á&#x;…áž‘á&#x;’áž?áž„á&#x;‹ đ?‘Ž = 0 ážš ážş đ?‘Ž = 1á&#x;” áž?ážśáž˜ážœ ឡចវរដសចគá&#x;’áž“áž“ យយឞ ងទញបវនáž?ážś đ?‘“ áž˜ážśáž“áž?ážśá&#x;† នលធវá&#x;† បវá&#x;†áž•áž? ážť áž™á&#x;…áž‘á&#x;’áž?áž„á&#x;‹ áž…ážśá&#x;† áž“áž… ážť áž˜áž˝áž™ (ážš ឺយទá&#x;’áž…ážžáž“áž‡ážśáž„áž˜áž˝áž™) កនង ážť áž…ážśá&#x;† áž™ áž˜áž…ážśá&#x;† áž“áž… ážť áž‘ážśá&#x;†áž„áž‘á&#x;’បវវá&#x;† បី ážŠážŠáž›áž˜ážśáž“ đ?‘Ž, đ?‘?, đ?‘? = 0; 0; 0 , 0; 0; 1 , 0; 1; 0 , 0; 1; 1 , 1; 0; 0 , 1; 1; 0 , 1; 0; 1 , 1; 1; 1 )á&#x;” យយឞ áž„áž– ឡនឡáž?á&#x;‚យឃឞ ញ áž?ážś áž…ážśá&#x;† áž™ á&#x; á&#x;‡áž‘á&#x;’áž?ីធវáž?ážťáž“áž˜ ឡ ួយá&#x;— áž?ážśá&#x;† នលរបáž&#x;á&#x;‹ áž˘áž“ážťáž‚áž˜áž“á&#x;? đ?‘“ áž™áž&#x;មឞ 1 á&#x;” ដសយចនá&#x; á&#x;‡ážœ ឡáž&#x;áž˜áž—ážśáž–áž– ឡáž?á&#x;”

277.ď ° អងគហវងយឆá&#x; áž„áž‡ážśáž˘áž“ážťáž‚áž˜áž“á&#x;?áž•áž? áž…ážśá&#x;† áž™ á&#x; á&#x;‡áž˘áž™áž?ážšáž“ážˇáž˜áž˝áž™á&#x;— áž™á&#x;„យទបá&#x;‹ អយáž?រយផá&#x; ងយទá&#x;€áž?á&#x;” ážœážśáž˜ážśáž“áž?ážśá&#x;†áž“áž› áž’ážśá&#x;† បវá&#x;†áž•áž? ážť áž™á&#x;…áž‘á&#x;’áž?áž„á&#x;‹ áž…ážśá&#x;† áž“áž… ážť áž˜áž˝áž™áž“áž“áž”áž‰á&#x;’áž… áž’ážśáž?ážť đ?‘Ž, đ?‘?, đ?‘?, đ?‘‘, đ?‘’ áž‘ážśá&#x;†áž„ 32 ដដល đ?‘Ž, đ?‘?, đ?‘?, đ?‘‘, đ?‘’ ∈ đ?‘?; đ?‘ž á&#x;” áž?ážśáž„ đ?‘› ជវចវá&#x;† áž“áž“ áž˝ អយáž?ážš ដដលយáž&#x;មឞ đ?‘? យហឞ យដសយចនá&#x; á&#x;‡áž˘áž™áž?ážšáž…ážśá&#x;† áž“áž“ áž˝ 5 − đ?‘› យផá&#x; ងយទá&#x;€áž?áž™áž&#x;មឞ đ?‘ž ដដល đ?‘› ∈ 0; 1; 2; 3; 4; 5 á&#x;” ដសយចនá&#x; á&#x;‡ យយឞ áž„áž‘á&#x;’áž?ážźážœážšáž€áž?ážśá&#x;† នលធវá&#x;† បវá&#x;†áž•áž? ážť ážšáž”áž&#x;á&#x;‹ đ?‘› 5−đ?‘› đ?‘“ đ?‘› = đ?‘›đ?‘? + 5 − đ?‘› đ?‘ž + đ?‘? đ?‘ž យយឞ áž„áž˜ážśáž“ đ?‘? đ?‘ž đ?‘“ đ?‘› = đ?‘›2 + 5 − đ?‘› 2 + đ?‘› 5 − đ?‘› + đ?‘ž đ?‘? đ?‘? đ?‘ž = đ?‘›2 + 5 − đ?‘› 2 + đ?‘› 5 − đ?‘› + −2+2 đ?‘ž đ?‘? 2

= đ?‘›2 + 5 − đ?‘›

2

đ?‘? đ?‘ž − đ?‘ž đ?‘?

+ 2đ?‘› 5 − đ?‘› + đ?‘› 5 − đ?‘› 2

= 25 + đ?‘› 5 − đ?‘›

đ?‘? đ?‘ž − đ?‘ž đ?‘?

ដសយចនá&#x; á&#x;‡ đ?‘“ đ?‘› áž’ážśá&#x;† បវá&#x;†áž•áž? ážť យបឞ 25 5 đ?‘› 5−đ?‘› = − đ?‘›âˆ’ 4 2 áž˜ážśáž“áž?ážśá&#x;† នលធវá&#x;† បវá&#x;†áž•áž? ážť áž˜ážśáž“áž“á&#x;?áž™áž?ážś đ?‘› = 2 ážš ážş đ?‘› = 3 á&#x;” ដសយចនá&#x; á&#x;‡

2

2

đ?‘“max = 25 + 6

đ?‘? đ?‘ž − đ?‘ž đ?‘?

278.ď ° áž?ážśáž„ đ??´, đ??ľ, đ??ś ∈ 0, 2 នឡង đ?‘Ž = 2 tan đ??´ , đ?‘? = 2 tan đ??ľ នឡង đ?‘? = 2 tan đ??ś á&#x;” áž?ážśáž˜ đ?œ‹

áž‘ážśá&#x;† កá&#x;‹ áž‘ážśá&#x;†áž“áž„ 1 + tan2 đ?œƒ = 1/ cos 2 đ?œƒ ážœ ឡáž&#x;áž˜áž—ážśáž–á&#x;„áž…áž&#x;ážšáž™áž&#x;រជវ đ?‘Ž2 + 2 đ?‘? 2 + 2 đ?‘? 2 + 2 ≼ 9 đ?‘Žđ?‘? + đ?‘?đ?‘? + đ?‘?đ?‘Ž 152

á&#x;Ą. áž˘áž“ážťáž‚áž˜áž“á&#x;?áž„ážśáž™ | លរម áž&#x;ážť វណá&#x;’ážŽážœážˇáž…ážˇáž?á&#x;’ážš


â&#x;ş 2 tan2 đ??´ + 2 2 tan2 đ??ľ + 2 2 tan2 đ??ś + 2 ≼ 9 2 tan đ??´ tan đ??ľ + 2 tan đ??ľ tan đ??ś + 2 tan đ??ś tan đ??´ 4 â&#x;ş ≼ cos đ??´ cos đ??ľ cos đ??ś sin đ??´ sin đ??ľ cos đ??ś + sin đ??ľ sin đ??ś cos đ??´ + sin đ??ś sin đ??´ cos đ??ľ 9 យយឞ áž„áž˜ážśáž“ cos đ??´ + đ??ľ + đ??ś = cos đ??´ cos đ??ľ cos đ??ś − cos đ??´ sin đ??ľ sin đ??ś − sin đ??´ cos đ??ľ sin đ??ś − sin đ??´ sin đ??ľ cos đ??ś ដសយចនá&#x; á&#x;‡ážœ ឡáž&#x;áž˜áž—ážśáž–áž™á&#x;…ជវ 4 ≼ cos đ??´ cos đ??ľ cos đ??ś (cos đ??´ cos đ??ľ cos đ??ś − cos(đ??´ + đ??ľ + đ??ś)) 9 đ??´+đ??ľ+đ??ś áž?ážśáž„ đ?‘ž = á&#x;” áž?ážśáž˜ážœ ឡáž&#x;áž˜áž—ážśáž–áž€ážźáž&#x;ឝី នឡង ážœ ឡáž&#x;áž˜áž—ážśáž–áž™ážˇ áž“áž&#x;ឡ áž“ យយឞ ងទញបវន 3

cos đ??´ + cos đ??ľ + cos đ??ś 3

cos đ??´ cos đ??ľ cos đ??ś ≤ យយឞ áž„áž‘á&#x;’áž?ážźážœáž”áž„á&#x;’ហវញáž?ážś

3

≤ cos 3 �

4 ≼ cos 3 đ?‘ž (cos 3 đ?‘ž − cos 3đ?‘ž) 9

យយឞ áž„áž˜ážśáž“

cos 3đ?‘ž = 4 cos 3 đ?‘ž − 3 cos đ?‘ž â&#x;š cos 3 đ?‘ž − cos 3đ?‘ž = 3 cos đ?‘ž − 3 cos3 đ?‘ž ដសយចនá&#x; á&#x;‡ážœ ឡáž&#x;áž˜áž—ážśáž–áž™á&#x;…ជវ 4 ≼ cos4 đ?‘ž 1 − cos 2 đ?‘ž ∗ 27 áž?ážśáž˜ážœ ឡáž&#x;áž˜áž—ážśáž–áž€ážźáž&#x;ឝី cos2 đ?‘ž cos2 đ?‘ž . . 1 − cos2 đ?‘ž 2 2

1 3

≤

1 cos 2 đ?‘ž cos2 đ?‘ž + + 1 − cos 2 đ?‘ž 3 2 2

=

1 3

â&#x;š (*) áž– ឡáž?á&#x;” áž&#x;áž˜áž—ážśáž–áž™áž€ážžáž?áž˜ážśáž“ áž‘áž›á&#x;‹ ដáž? tan đ??´ = tan đ??ľ = tan đ??ś =

279.ď ° ď‚Œ ដá&#x;†áž“áž“ážśá&#x;„á&#x;‡áž?á&#x;’វយទីá&#x;Ą

1 2

â&#x;šđ?‘Ž=đ?‘?=đ?‘?=1

áž?ážśáž„ đ?‘Ž = đ?‘Ś + đ?‘§, đ?‘? = đ?‘§ + đ?‘Ľ, đ?‘? = đ?‘Ľ + đ?‘Ś á&#x;” đ?‘Ž+đ?‘? >đ?‘? â&#x;šđ?‘Ś+đ?‘§+đ?‘§+đ?‘Ľ >đ?‘Ľ+đ?‘Ś â&#x;šđ?‘§ >0 ដសចគá&#x;’áž“áž“ đ?‘Ľ, đ?‘Ś, đ?‘§ > 0 á&#x;” ážœ ឡáž&#x;áž˜áž—ážśáž–áž ážśáž„áž™áž›ážžáž&#x;áž˜áž˜ážźáž›áž“ážšáž„ đ?‘Ľ 3 đ?‘§ + đ?‘Ś 3 đ?‘Ľ + đ?‘§ 3 đ?‘Ś ≼ đ?‘Ľ 2 đ?‘Śđ?‘§ + đ?‘Ľđ?‘Ś 2 đ?‘§ + đ?‘Ľđ?‘Śđ?‘§ 2

លរម áž&#x;ážť វណá&#x;’ážŽážœážˇáž…ážˇáž?á&#x;’ážš | V. ážœážˇáž&#x;áž˜áž—ážśáž–

153


đ?‘Ľ2 đ?‘Ś2 đ?‘§2 + + ≼đ?‘Ľ+đ?‘Ś+đ?‘§ đ?‘Ś đ?‘§ đ?‘Ľ áž™á&#x;„យវ ឡáž&#x;áž˜áž—ážśáž–áž ážśáž„áž™áž›ážžáž˘ážźáž˜á&#x;‰ážśážźážŠáž&#x;áž“ (យបឞយយឞ ងជវá&#x;†áž“áž&#x; áž˝ đ?‘Ľ, đ?‘Ś, đ?‘§ áž™á&#x;„áž™ đ?‘Ąđ?‘Ľ, đ?‘Ąđ?‘Ś, đ?‘Ąđ?‘§) áž™ á&#x; á&#x;‡ážœ ឡáž&#x;áž˜áž—ážśáž–áž™á&#x;… â&#x;ş

ដដដល )áž™ á&#x; á&#x;‡áž™áž™ážž áž„á&#x;„áž…áž&#x;នមáž?áž?ážś đ?‘Ľ + đ?‘Ś + đ?‘§ = 1 á&#x;” ដសយចនá&#x; á&#x;‡ážœ ឡáž&#x;áž˜áž—ážśáž–áž&#x;áž˜áž˜ážźáž›áž“ážšáž„ đ?‘Ľ đ?‘Ś đ?‘§ đ?‘Śđ?‘“ + đ?‘§đ?‘“ + đ?‘Ľđ?‘“ ≼1 đ?‘Ś đ?‘§ đ?‘Ľ ដដល đ?‘“ đ?‘Ą = đ?‘Ą 2 á&#x;” áž™á&#x;„áž™ đ?‘“ áž•áž?យលឞ â„? áž?ážśáž˜ážœ ឡáž&#x;áž˜áž—ážśáž–áž™ážˇ áž“áž&#x;ឡ áž“ យយឞ ងទញបវន đ?‘Ľ đ?‘Ś đ?‘§ đ?‘Ľ đ?‘Ś đ?‘§ đ?‘Śđ?‘“ + đ?‘§đ?‘“ + đ?‘Ľđ?‘“ ≼ đ?‘“ đ?‘Ś. + đ?‘§. + đ?‘Ľ. =đ?‘“ 1 =1 đ?‘Ś đ?‘§ đ?‘Ľ đ?‘Ś đ?‘§ đ?‘Ľ

ď‚? ដá&#x;†áž“áž“ážśá&#x;„á&#x;‡áž?á&#x;’វយទីá&#x;˘

áž?ážśáž„ đ?‘“ đ?‘Ž, đ?‘?, đ?‘? = đ?‘Ž2 đ?‘? đ?‘Ž − đ?‘? + đ?‘? 2 đ?‘? đ?‘? − đ?‘? + đ?‘? 2 đ?‘Ž đ?‘? − đ?‘Ž á&#x;” យយឞ ងយឃឞ ញáž?ážś đ?‘“ áž˜ážˇáž“ážŠáž‘á&#x;’បទá&#x;’បួលយទ យពលយយឞ ងយធá&#x; áž… ážž ážśá&#x;† លវáž&#x;á&#x;‹ áž&#x;ឝី áž‚áž… ážś ឡ áž“áž“ đ?‘Ž, đ?‘?, đ?‘? áž˜ážśáž“áž“á&#x;?áž™áž?ážś ដសរ đ?‘Ž, đ?‘?, đ?‘? ជវ đ?‘?, đ?‘?, đ?‘Ž ážš ážş đ?‘?, đ?‘Ž, đ?‘? á&#x;” ដសយចនá&#x; á&#x;‡ យយឞ áž„á&#x;„áž…áž&#x;នមáž?áž?ážś đ?‘Ž = max đ?‘Ž, đ?‘?, đ?‘? (ដáž?áž˜ážˇáž“á&#x;„áž…áž&#x;នមáž?áž?ážś đ?‘Ž ≼ đ?‘? ≼ đ?‘? បវនយទ យទá&#x;’ á&#x; á&#x;‡ đ?‘“ áž˜ážˇáž“ áž&#x;ឝី យមទá&#x;’ទី)á&#x;” យយឞ ងទញបវន đ?‘“ đ?‘Ž, đ?‘?, đ?‘? = đ?‘Ž đ?‘? − đ?‘? 2 đ?‘? + đ?‘? − đ?‘Ž + đ?‘? đ?‘Ž − đ?‘? đ?‘Ž − đ?‘? đ?‘Ž + đ?‘? − đ?‘? ≼ 0 áž&#x;áž˜áž—ážśáž–áž™áž€ážžáž?áž˜ážśáž“áž™áž–áž› đ?‘Ž = đ?‘? = đ?‘? á&#x;”

280.ď ° យបឞទá&#x;’áž‚áž”á&#x;‹ đ?‘Žđ?‘– áž™áž&#x;មឞ áž‚á&#x;’áž“áž“áž‘ážśá&#x;†áž„អáž&#x;á&#x;‹ áž™ á&#x; á&#x;‡áž˘áž“ážťáž‚áž˜áž“á&#x;? đ?‘€ áž™áž?រយលឞ â„?∗ á&#x;” ូលស វយនá&#x; á&#x;‡ យយឞ áž„áž&#x;នមáž?áž?ážś áž”á&#x;’áž? đ?‘Žđ?‘– áž˜ážˇáž“áž™áž&#x;មឞ áž‚á&#x;’áž“áž“áž‘ážśá&#x;†áž„អáž&#x;á&#x;‹ យទá&#x;” áž?ážśáž„ đ?‘“ đ?›ź = ln đ?‘™1 đ?‘Ž1đ?›ź + đ?‘™2 đ?‘Ž2đ?›ź + â‹Ż + đ?‘™đ?‘› đ?‘Žđ?‘›đ?›ź á&#x;” យយឞ áž„áž˜ážśáž“ 1 đ?‘“ đ?›ź ln đ?‘™1 đ?‘Ž1đ?›ź + đ?‘™2 đ?‘Ž2đ?›ź + â‹Ż + đ?‘™đ?‘› đ?‘Žđ?‘›đ?›ź = đ?›ź đ?›ź យយឞ áž„áž˜ážśáž“ limđ?›ź â&#x;ś0 đ?‘Žđ?‘–đ?›ź = 1 áž…ážśá&#x;† áž™ á&#x; á&#x;‡áž‘á&#x;’áž‚áž”á&#x;‹ đ?‘Žđ?‘– > 0 ដសយចនá&#x; á&#x;‡ limđ?›ź â&#x;ś0 đ?‘“(đ?›ź) = 0 á&#x;” មá&#x;‰ážś យងវ ឡញយទá&#x;€áž? áž™á&#x;„áž™ đ?‘™1 đ?‘Ž1đ?›ź ln đ?‘Ž1 + đ?‘™2 đ?‘Ž2đ?›ź ln đ?‘Ž2 + â‹Ż + đ?‘™đ?‘› đ?‘Žđ?‘›đ?›ź ln đ?‘Žđ?‘› đ?‘“′ đ?›ź = đ?‘™1 đ?‘Ž1đ?›ź + đ?‘™2 đ?‘Ž2đ?›ź + â‹Ż + đ?‘™đ?‘› đ?‘Žđ?‘›đ?›ź đ?‘™1 ln đ?‘Ž1 + đ?‘™2 ln đ?‘Ž2 + â‹Ż + đ?‘™đ?‘› ln đ?‘Žđ?‘› â&#x;š lim đ?‘“ ′ đ?›ź = = đ?‘™1 ln đ?‘Ž1 + đ?‘™2 ln đ?‘Ž2 + â‹Ż + đ?‘™đ?‘› ln đ?‘Žđ?‘› đ?›źâ&#x;ś0 đ?‘™1 + đ?‘™2 + â‹Ż + đ?‘™đ?‘› đ?‘™ đ?‘™ đ?‘™ = ln đ?‘Ž11 đ?‘Ž22 ‌ đ?‘Žđ?‘›đ?‘› ln đ?‘€ đ?›ź =

áž?ážśáž˜áž€áž”áž˝áž“ អស áž– ីáž?ážśáž›á&#x;‹ យយឞ ងទញបវន lim ln đ?‘€(đ?›ź) = ln đ?‘Ž1đ?‘™1 đ?‘Ž2đ?‘™2 ‌ đ?‘Žđ?‘›đ?‘™đ?‘› đ?›źâ&#x;ś0

đ?‘™

đ?›ź â&#x;ś0

ដសយចនá&#x; á&#x;‡ áž™á&#x;„យយក đ?‘€ 0 = đ?‘Ž11 đ?‘Ž22 ‌ đ?‘Žđ?‘›đ?‘› យយឞ ងទញបវន đ?‘€ ជវបá&#x;‹ áž‘á&#x;’áž?áž„á&#x;‹ 0 á&#x;” đ?‘™

đ?‘™

đ?‘™

ក) ករណី đ?›ź > 1 áž˘áž“ážťáž‚áž˜áž“á&#x;? đ?‘“ đ?‘Ľ = đ?‘Ľ đ?›ź áž•áž?á&#x;„áž…á&#x;‹ áž ážśáž?យលឞ â„?+∗ á&#x;” ដសយចនá&#x; á&#x;‡ 154

á&#x;Ą. áž˘áž“ážťáž‚áž˜áž“á&#x;?áž„ážśáž™ | លរម áž&#x;ážť វណá&#x;’ážŽážœážˇáž…ážˇáž?á&#x;’ážš

đ?‘™

đ?‘™

â&#x;š lim đ?‘€(đ?›ź) = đ?‘Ž11 đ?‘Ž22 ‌ đ?‘Žđ?‘›đ?‘›


𝑙1 𝑎1𝛼 + 𝑙2 𝑎2𝛼 + ⋯ + 𝑙𝑛 𝑎𝑛𝛼 > 𝑙1 𝑎1 + 𝑙2 𝑎2 + ⋯ + 𝑙𝑛 𝑎𝑛 វ ិសមភាពយនេះ ោច់ ខាត យទ្ េះ ប្ ត 𝑎𝑖 មិនយសមើ គ្ននទាំងអស់ យទ។

𝛼

ដូយចនេះ 1 𝛼

𝑀 𝛼 = 𝑙1 𝑎1𝛼 + 𝑙2 𝑎2𝛼 + ⋯ + 𝑙𝑛 𝑎𝑛𝛼 ខ) ករណី 𝛼, 𝛽 ∈ ℝ

> 𝑙1 𝑎1 + 𝑙2 𝑎2 + ⋯ + 𝑙𝑛 𝑎𝑛 = 𝑀 1

ករណីទី១: 0 < 𝛼 < 𝛽 តាង 𝑡 = 𝛽 𝛼 > 1 ។ ចាំ យ េះទ្គប់ 𝑖 ∈ {1, … , 𝑛} យយើ ងតាង 𝑏𝑖 = 𝑎𝑖𝛼 ។ តាម ក) យយើ ងមាន 1 𝑡

𝑛

𝑙𝑖 𝑏𝑖𝑡

𝑛

𝑛

>

𝑖=1

𝑙𝑖 𝑏𝑖

𝛽 𝑙𝑖 𝑎𝑖

𝑖=1

ករណីទី២: 𝛼 < 𝛽 < 0

𝛼 𝛽

𝑛

𝑙𝑖 𝑎𝑖𝛼 ⟹ 𝑀 𝛽 > 𝑀(𝛼)

>

𝑖=1

𝑖=1

តាង 𝑡 = 𝛼/𝛽 > 1។ ចាំ យ េះទ្គប់ 𝑖 ∈ {1, … , 𝑛} យយើ ងតាង 𝑏𝑖 = 𝑎𝑖 ។ តាម ក) យយើ ងមាន 𝛽

1 𝑡

𝑛

𝑙𝑖 𝑏𝑖𝑡

𝑛

>

𝑖=1

𝛽 𝛼

𝑛

𝑙𝑖 𝑏𝑖

𝑙𝑖 𝑎𝑖𝛼

𝑖=1

𝑖=1 −∗

ដូយចនេះ អនុគមន៍ 𝑀 យកើនោច់ ខាតយលើ ℝ+∗ ; ℝ

𝑛 𝛽

>

𝑙𝑖 𝑎𝑖

⟹ 𝑀 𝛽 > 𝑀(𝛼)

𝑖=1

យហើ យជាប់ ទ្តង់ 0 ។ ដូយចនេះ វាយកើនោច់ ខាតយលើ ℝ។

281.  ចាំ យ េះ 𝛼 > 0 : តាមលកខណៈសុី យមទ្ទី យយើ ងោចសនមតថា 𝑎1 = max{𝑎1 , 𝑎2 , … , 𝑎𝑛 } ។ ដូយចនេះ 1

1

ln 𝑀 𝛼 = 𝛼 ln 𝑙1 𝑎1𝛼 + 𝑙2 𝑎2𝛼 + ⋯ + 𝑙𝑛 𝑎𝑛𝛼 = 𝛼 ln 𝑎1𝛼 ∑𝑛𝑖=1 𝑙𝑖 1 𝛼

ln 𝑎1 + ln ∑𝑛𝑖=1 𝑙𝑖

𝑎𝑖 𝛼 𝑎1

𝑎

ចាំ យ េះទ្គប់ 𝑖 យយើ ងមាន 0 < 𝑎 𝑖 ≤ 1 ។ ដូយចនេះ 0 < 1

𝑛

𝑙𝑖 𝑖=1

𝑎𝑖 𝑎1

𝑛

𝛼

𝑎𝑖 𝛼 𝑎1

𝑎𝑖 𝛼 𝑎1

=

≤1

𝑙𝑖 = 1 𝑖=1

1 ln 1 = ln 𝑎1 𝛼 lim ln 𝑀 𝛼 = ln 𝑎1

ln 𝑀 𝛼 ≤ ln 𝑎1 + ⟹ ⟹  ចាំ យ េះ 𝛼 < 0

𝛼⟶+∞

lim 𝑀(𝛼) = 𝑎1 = max{𝑎1 , 𝑎2 , … , 𝑎𝑛 }

𝛼⟶+∞

យលើកយនេះយយើ ងសនមតថា 𝑎1 = min{𝑎1 , 𝑎2 , … , 𝑎𝑛 }។ ដូចខាងយលើ យយើ ងមាន លឹម សុ វណ្ណវិចិត្រ | V. វិសមភាព

155


𝑛

1 ln 𝑀 𝛼 = ln 𝑎1 + ln 𝛼 ដដល

𝑎𝑖 𝑎1

≥ 1 ចាំ យ េះទ្គប់ 𝑖 ។ ដូយចនេះ 0 <

តាមវ ិចារដូចករណីខាងយលើ យយើ ងទញបាន

𝑎𝑖 𝛼 𝑎1

𝑙𝑖 𝑖=1

𝑎𝑖 𝑎1

𝛼

≤1 ។

lim 𝑀(𝛼) = 𝑎1 = min{𝑎1 , 𝑎2 , … , 𝑎𝑛 }

𝛼⟶−∞

ដូយចនេះ

lim 𝑀(𝛼) = 𝑎1 = max{𝑎1 , 𝑎2 , … , 𝑎𝑛 } lim 𝑀(𝛼) = 𝑎1 = min{𝑎1 , 𝑎2 , … , 𝑎𝑛 }

𝛼 ⟶+∞

𝛼 ⟶−∞

យហើ យយោយអនុគមន៍ 𝑀 យកើនោច់ ខាតយលើ ℝ ដូយចនេះ ចាំ យ េះទ្គប់ 𝛼 ∈ ℝ យគមាន min{𝑎1 , 𝑎2 , … , 𝑎𝑛 } < 𝑀 𝛼 < max{𝑎1 , 𝑎2 , … , 𝑎𝑛 } 1  យក 𝑙1 = 𝑙2 = ⋯ = 𝑙𝑛 = 𝑛 យយើ ងទញបាន 𝑀 −1 =

1 −1 𝑎 + 𝑎2−1 + ⋯ + 𝑎𝑛−1 𝑛 1

−1

=

1

𝑀 0 = 𝑎1 𝑎2 … 𝑎𝑛 𝑛 1 1 1 𝑀 1 = 𝑎1 + 𝑎2 + ⋯ + 𝑎𝑛 𝑛 𝑛 𝑛 1 1 1 𝑀 2 = 𝑎12 + 𝑎22 + ⋯ + 𝑎𝑛2 𝑛 𝑛 𝑛

1

= 1 2

=

𝑛 1 1 1 + + ⋯+ 𝑎1 𝑎2 𝑎𝑛

𝑎1 + 𝑎2 + ⋯ + 𝑎𝑛 𝑛 𝑎12 + 𝑎22 + ⋯ + 𝑎𝑛2 𝑛

282. តាង 𝐴 = 𝑥13 + 𝑥23 + 𝑥33 + 𝑥43 និង 𝐴𝑖 = 𝐴 − 𝑥𝑖3 ⟹ 𝐴 = 3 ∑4𝑖=1 𝐴𝑖 ។ 1

តាមវ ិសមភាពកូសុី យយើ ងទញបាន 1 𝐴 ≥ 𝑥23 𝑥33 𝑥43 1/3 = 𝑥2 𝑥3 𝑥4 = 1/𝑥1 3 1 តាមរយបៀបដូចគ្នន យយើ ងទញបាន 1 1 𝐴𝑖 ≥ ; ∀𝑖 = 1,2,3,4 3 𝑥𝑖 4

⟹𝐴≥ ម៉ា យងវ ិញយទៀត 1 1 𝐴= 4 4 156

1/𝑥𝑖

(1)

𝑖=1

4

𝑖=1

1 𝑥𝑖3 ≥ 4

3

4

𝑥𝑖 𝑖=1

១. អនុគមន៍ងាយ | លឹម សុ វណ្ណវិចិត្រ

តាមវ ិសមភាពមធែមលាំ ោប់ 3 និង 1


1 = 4 យទ្ េះ

4

𝑥𝑖 𝑖=1

1 4

2

4

1 4

1 ≥ 4

𝑥𝑖 𝑖=1

4

𝑥𝑖 𝑖=1

4

𝑥𝑖 ≥ 𝑥1 𝑥2 𝑥3 𝑥4 𝑖=1

1 4

តាមវ ិសមភាពកូសុី

=1

4

⟹𝐴≥

𝑥𝑖

(2)

𝑖=1

និង

⟹ ∑4𝑖=1 𝑥𝑖3 ≥ max ∑4𝑖=1 𝑥𝑖 ; ∑4𝑖=1 1/𝑥𝑖 ។

283. យបើមានចាំ នន ួ ្មួយយសមើ សូនែ ឧទហរណ៍ 𝑧 = 0 យ េះ វ ិសមភាពសមមូលនឹង 8 𝑥 6 + 𝑦 6 + 2𝑥 3 𝑦 3 ≥ 9𝑥 3 𝑦 3 ព ិត យហើ យយសមើ គ្ននយពល 𝑥 = 𝑦 = 𝑧 = 0 ។ យបើ 𝑥, 𝑦, 𝑧 > 0 យយើ ងមាន 9 2𝑥 2 + 𝑦 2 + 𝑧 2 𝑥 2 + 2𝑦 2 + 𝑧 2 𝑥 2 + 𝑦 2 + 2𝑧 2 8 3 9 4 𝑥2 + 𝑦2 + 𝑧2 ≤ តាមវ ិសមភាពកូសុី 8 3 3 2 𝑥2 + 𝑦2 + 𝑧2 𝑥3 + 𝑦3 + 𝑧3 = 9.8 ≤ 9.8 តាម មធែមលាំ ោប់ ៣និង២ 3 3 = 8 𝑥3 + 𝑦3 + 𝑧3 2 អងគទង ាំ ព ីរយសមើ គ្នន ទល់ ដតនិង យាំ ោយ 𝑥 = 𝑦 = 𝑧 ។ 9 𝑥 2 + 𝑦𝑧 𝑦 2 + 𝑧𝑥 𝑧 2 + 𝑥𝑦 ≤

284. អនុគមន៍ 𝑓 𝑥 = 1 𝑛

𝑛

𝑥 1−𝑥 𝑛

1 ≥𝑓 𝑛

𝑓 𝑥𝑖 𝑖=1

ជាអនុគមន៍ផត។ តាមវ ិសមភាពយិ នសិ ន យយើ ងទញបាន 𝑥𝑖

𝑖=1

1 1 𝑛 =𝑓 = ⟹ 𝑛 𝑛 𝑛−1

𝑛

𝑖=1

𝑥𝑖 1 − 𝑥𝑖

𝑛 𝑛−1

ម៉ា យងវ ិញយទៀត តាមវ ិសមភាពមធែមលាំ ោប់ 1 និង 1 2 យយើ ងទញបាន 1 1 = 𝑛 𝑛 𝑛

𝑛

𝑖=1

⟹ ព ិត។

𝑖=1

1 𝑥𝑖 ≥ 𝑛 𝑥𝑖

1 − 𝑥𝑖

លឹម សុ វណ្ណវិចិត្រ | V. វិសមភាព

2

𝑛

𝑥𝑖 𝑖=1

𝑛

𝑥𝑖 ≤ 𝑛 ⟹ 𝑖=1

∑𝑛𝑖=1 𝑥𝑖 𝑛−1

𝑛 𝑛−1

∑𝑛𝑖=1 𝑥𝑖 𝑛 ≥ 𝑛−1 𝑛−1

157


285. តាង 𝑆 = 𝑥1 + 𝑥2 + ⋯ + 𝑥𝑛 ។ យយើ ងមាន 𝑛

𝑖=1

= 𝑛2 =

𝑛

𝑥𝑖5 𝑆 − 𝑥𝑖 1 𝑛

𝑛2

𝑆 − 𝑥𝑖

𝑛

5

𝑥𝑖2

2

𝑛

𝑖=1 2

≥ 𝑛2

𝑖=1

≥ 1 𝑛

𝑛

តាមវ ិសមភាពកូសុីាេត

𝑥𝑖5 𝑖=1 5 2 𝑥𝑖2

តាមវ ិសមភាពមធែមលាំ ោប់

𝑖=1

(∗)

5 𝑛2

យោយអងគទង ាំ ព ីរយសមើ គ្នន យពល 𝑥1 = 𝑥2 = ⋯ = 𝑥𝑛 = 1 ម៉ា យងវ ិញយទៀត

𝑛

≤𝑛 𝑛−1

⟹0<

𝑛

𝑆 − 𝑥𝑖 = 𝑛 − 1 𝑖=1

𝑛

𝑛។

𝑛

0<

=

1 𝑛

𝑛 𝑛−1

1 2

𝑛

𝑥𝑖2

𝑥𝑖 = 𝑛 𝑛 − 1 𝑖=1

𝑖=1

𝑥𝑖 𝑛

តាមវ ិសមភាពមធែមលាំ ោប់ 1 និង 2

𝑖=1

𝑛

𝑆 − 𝑥𝑖 ≤ 𝑖=1

𝑛 𝑛−1 𝑛

(∗∗)

យោយអងគទង ាំ ព ីរយសមើ គ្នន យពល 𝑥1 = 𝑥2 = ⋯ = 𝑥𝑛 = 1 (*) និង (**)

5 និង 2 2

𝑛

𝑥𝑖5 𝑛 𝑛−1 𝑛2 ≥ 5 ⟹ 𝑆 − 𝑥𝑖 𝑛 𝑛2 𝑖=1 យោយអងគទង ាំ ព ីរយសមើ គ្នន យពល 𝑥1 = 𝑥2 = ⋯ = 𝑥𝑛 = 1

𝑛។ 𝑛

𝑖=1

𝑥𝑖5 1 ≥ 𝑆 − 𝑥𝑖 𝑛 𝑛 − 1

𝑛។

286. យោយវ ិសមភាពសុី យមទ្ទីយធៀបនឹងអយថរទាំងបី យយើ ងោចសនមតថា 𝑥 ≥ 𝑦 ≥ 𝑧 យោយមិន បាត់ បង់ លកខណៈទូយៅអេី ទង ាំ អស់ ។ វ ិសមភាពដដលយោយ ោចសរយសរជា 𝑥𝑟 𝑥 − 𝑦 𝑥 − 𝑧 + 𝑦𝑟 𝑦 − 𝑧 𝑦 − 𝑥 + 𝑧𝑟 𝑧 − 𝑥 𝑧 − 𝑦 ≥ 0 ⟺ 𝑥 − 𝑦 𝑥𝑟 𝑥 − 𝑧 − 𝑦𝑟 𝑦 − 𝑧 + 𝑧𝑟 𝑥 − 𝑧 𝑦 − 𝑧 ≥ 0 ព ិត យទ្ េះតួនម ិ ួយៗ សុ ទដធ តវ ិជាមានទាំងអស់ ។ អងគទង ាំ ព ីរយសមើ គ្នន ទល់ ដត 𝑥 = 𝑦 = 𝑧 ។

287.  វ ិសមភាពខាងយឆេងដដលយោយមក សមមូលនឹង 𝑎3 + 𝑏 3 + 𝑐 3 + 3𝑎𝑏𝑐 ≥ 𝑎𝑏 𝑎 + 𝑏 + 𝑏𝑐 𝑏 + 𝑐 + 𝑐𝑎 𝑐 + 𝑎 158

១. អនុគមន៍ងាយ | លឹម សុ វណ្ណវិចិត្រ


⟺𝑎 𝑎−𝑏 𝑎−𝑐 +𝑏 𝑏−𝑐 𝑏−𝑎 +𝑐 𝑐−𝑎 𝑐−𝑏 ≥0 ព ិត តាមវ ិសមភាព ករណី 𝑟 = 1 ។ អងគទង ាំ ព ីរយសមើ គ្នន ទល់ ដតនិងមានដត 𝑎 = 𝑏 = 𝑐 ។ វ ិសមភាពខាងាតាំសមមូលនឹង 𝑎𝑏 𝑎 + 𝑏 + 𝑏𝑐 𝑏 + 𝑐 + 𝑐𝑎 𝑐 + 𝑎 ≥ 2 𝑎𝑏 𝑎2 𝑏 + 𝑎𝑏 2 ≥

⟺ cyclic

2 𝑎𝑏

3 2

+ 𝑏𝑐

3 2

+ 𝑐𝑎

3 2

3 2

cyclic

ព ិតតាមវ ិសមភាពកូសុី។  តាង 𝑥 = 𝑎2 3 , 𝑦 = 𝑏 2 3 , 𝑧 = 𝑐 2 3 − 𝑡 + 𝑡 𝑎𝑏𝑐

2 𝑡

3

វ ិសមភាពសមមូលនឹង

+ 𝑎2 + 𝑏 2 + 𝑐 2 ≥ 2 𝑎𝑏 + 𝑏𝑐 + 𝑐𝑎

⟺ 3 − 𝑡 + 𝑡 𝑥𝑦𝑧

3/𝑡

+ 𝑥 3 + 𝑦 3 + 𝑧 3 ≥ 2 𝑥𝑦

3 2

+ 𝑦𝑧

3 2

+ 𝑧𝑥

3 2

+ 𝑧𝑥

3 2

តាមសាំ នួរទី យយើ ងមាន 𝑥 3 + 𝑦 3 + 𝑧 3 + 3𝑥𝑦𝑧 ≥ 2 𝑥𝑦

3 2

+ 𝑦𝑧

3 2

ដូយចនេះ យពលយនេះ យយើ ងទ្គ្នន់ ដតបង្ហាញថា 3 − 𝑡 + 𝑡 𝑥𝑦𝑧

3 𝑡

≥ 3𝑥𝑦𝑧 𝑡

3 3−𝑡 3 3 3−𝑡 𝑡 ⟺ . 1 + 𝑥𝑦𝑧 𝑡 ≥ 1 3 . 𝑥𝑦𝑧 𝑡 = 𝑥𝑦𝑧 3 3 ព ិត តាមវ ិសមភាពមន-មធទូយៅ។ យយើ ងយឃើ ញថាអងគទង ាំ ព ីរយសមើ គ្នន យពល 𝑎 = 𝑏 = 𝑐 = 1 ។ 1

 ចាំ យ េះ 𝑡 = 2 ; 1; 2 យយើ ងមាន 5 1 + 𝑎𝑏𝑐 4 + 𝑎2 + 𝑏 2 + 𝑐 2 ≥ 2 𝑎𝑏 + 𝑏𝑐 + 𝑐𝑎 2 2 2 + 𝑎𝑏𝑐 2 + 𝑎2 + 𝑏 2 + 𝑐 2 ≥ 2 𝑎𝑏 + 𝑏𝑐 + 𝑐𝑎 1 + 2𝑎𝑏𝑐 + 𝑎2 + 𝑏 2 + 𝑐 2 ≥ 2 𝑎𝑏 + 𝑏𝑐 + 𝑐𝑎

288.  យយើ ងមាន 1 𝑏+𝑐 𝑎−𝑏 𝑎−𝑐 − 2 = 𝑎 𝑎 + 𝑏𝑐 𝑎 𝑎2 + 𝑏𝑐 1 𝑐+𝑎 𝑏−𝑐 𝑏−𝑎 − = 𝑏 𝑏 2 + 𝑐𝑎 𝑏 𝑏 2 + 𝑐𝑎 1 𝑎+𝑏 𝑐−𝑎 𝑐−𝑏 − = 𝑐 𝑐 2 + 𝑎𝑏 𝑐 𝑐 2 + 𝑎𝑏 ដូយចនេះវ ិសមភាពដដលយោយសមមូលនឹង

លឹម សុ វណ្ណវិចិត្រ | V. វិសមភាព

159


𝑎−𝑏 𝑎−𝑐 𝑏−𝑐 𝑏−𝑎 𝑐−𝑎 𝑐−𝑏 + + ≥0 ∗ 2 2 𝑎 𝑎 + 𝑏𝑐 𝑏 𝑏 + 𝑐𝑎 𝑐 𝑐 2 + 𝑎𝑏 វ ិសមភាពខាងយលើសុីយមទ្ទីយធៀបនឹង 𝑎, 𝑏, 𝑐 ដូយចនេះ យយើ ងោចសនមតថា 𝑎 ≤ 𝑏 ≤ 𝑐។ ដូយចនេះ 𝑐−𝑎 𝑐−𝑏 ≥0 𝑐 𝑐 2 + 𝑎𝑏 យោយអងគទង ាំ ព ីរយសមើ គ្នន យពល 𝑐 = 𝑎 រ ឺ 𝑐 = 𝑏 ។ ប ទ ប់ មកយទៀត 𝑎−𝑏 𝑎−𝑐 𝑏−𝑐 𝑏−𝑎 𝑏−𝑎 2 𝑎+𝑏 + = 𝑎 𝑐 − 𝑏 + 𝑐𝑏 ≥ 0 𝑎 𝑎2 + 𝑏𝑐 𝑏 𝑏 2 + 𝑐𝑎 𝑎𝑏 𝑎2 + 𝑏𝑐 𝑏 2 + 𝑐𝑎 អងគទង ាំ ព ីរយសមើ គ្នន យពល 𝑎 = 𝑏 ។ ដូយចនេះវ ិសមភាព(*) ព ិត យោយ អងគទង ាំ ព ីរយសមើ គ្នន យពល 𝑎 = 𝑏 = 𝑐 ។

 តាមវ ិសមភាព

ចាំ យ េះ 𝑟 = −1 និង តាង 𝑥 = 𝑎 + 𝑏, 𝑦 = 𝑏 + 𝑐 និង 𝑧 = 𝑐 + 𝑎 យយើ ង

ទញបាន 1 1 1 𝑥−𝑦 𝑥−𝑧 + 𝑦−𝑧 𝑦−𝑥 + 𝑧−𝑥 𝑧−𝑦 𝑥 𝑦 𝑧 𝑎−𝑐 𝑏−𝑐 𝑏−𝑎 𝑐−𝑎 𝑐−𝑏 𝑎−𝑏 = + + 𝑎+𝑏 𝑏+𝑐 𝑐+𝑎 𝑐 2 + 𝑎𝑏 𝑎2 + 𝑏𝑐 𝑏 2 + 𝑐𝑎 = −𝑐+ −𝑎+ −𝑏 𝑎+𝑏 𝑏+𝑐 𝑐+𝑎 2 2 2 𝑐 + 𝑎𝑏 𝑎 + 𝑏𝑐 𝑏 + 𝑐𝑎 ⟹ + + ≥𝑎+𝑏+𝑐 𝑎+𝑏 𝑏+𝑐 𝑐+𝑎 អងគទង ាំ ព ីរយសមើ គ្នន យពល 𝑥 = 𝑦 = 𝑧 រ ឺ 𝑎 = 𝑏 = 𝑐 ។ 0≤

289.  យយើ ងោចសនមតថា 𝑎1 ≥ 𝑎2 ; 𝑏1 ≥ 𝑏2 ; 𝑎1 ≥ 𝑏1 យោយមិនយធេយើ ោយបាត់ បង់ លកខណៈទូ យៅយទ។ យបើ 𝑥 រ ឺ 𝑦 យសមើ សូនែ យ េះវ ិសមភាពព ិត។ ដូយចនេះ សនមតថា 𝑥 និង 𝑦 មិនសូ នែទាំងព ីរ។ ប ទ ប់ មកយទៀត វ ិសមភាពសុី យមទ្ទីយធៀបនឹង 𝑥, 𝑦 ដូយចនេះ សនមតថា 𝑥 ≥ 𝑦 ។យោយ 𝑎1 + 𝑎2 = 𝑏1 + 𝑏2 យ េះ 𝑎1 − 𝑎2 = 𝑏1 − 𝑎2 + 𝑏2 − 𝑎2 ⟹ 𝑥 𝑎 1 𝑦 𝑎 2 + 𝑥 𝑎 2 𝑦 𝑎 1 − 𝑥 𝑏1 𝑦 𝑏2 − 𝑥 𝑏2 𝑦 𝑏1 = 𝑥 𝑎 2 𝑦 𝑎 2 𝑥 𝑎 1 −𝑎 2 + 𝑦 𝑎 1 −𝑎 2 − 𝑥 𝑏1 −𝑎 2 𝑦 𝑏2 −𝑎 2 − 𝑥 𝑏2 −𝑎 2 𝑦 𝑏1 −𝑎 2 = 𝑥 𝑎 2 𝑦 𝑎 2 𝑥 𝑏1 −𝑎 2 − 𝑦 𝑏1 −𝑎 2 𝑥 𝑏2 −𝑎 2 − 𝑦 𝑏2 −𝑎 2 = 𝑥 𝑎 2 𝑦 𝑎 2 𝑥 𝑎 1 −𝑏2 − 𝑦 𝑎 1 −𝑏2 𝑥 𝑎 1 −𝑏1 − 𝑦 𝑎 1 −𝑏1 𝑥 𝑎 1 −𝑏2 𝑥 𝑎 1 −𝑏1 = 𝑥 𝑎 2 𝑦 𝑎 2 𝑦 𝑎 1 −𝑎 2 −1 −1 ≥0 𝑦 𝑦 ព ិត។ 160

១. អនុគមន៍ងាយ | លឹម សុ វណ្ណវិចិត្រ


 ក) ករណី 𝑏1 ≥ 𝑎2 យោយ 𝑎1 ≥ 𝑎1 + 𝑎2 − 𝑏1 និង 𝑎1 ≥ 𝑏1 យ េះ 𝑎1 ≥ max 𝑎1 + 𝑎2 − 𝑏1 , 𝑏1 ⟹ max(𝑎1 , 𝑎2 ) = 𝑎1 ≥ max 𝑎1 + 𝑎2 − 𝑏, 𝑏1 ។ យយើ ងមាន 𝑎1 + 𝑎2 − 𝑏1 ≥ 𝑏2 ≥ 𝑏3 និង 𝑎1 + 𝑎2 − 𝑏1 ≥ 𝑏1 + 𝑎3 − 𝑏1 = 𝑎3 ។ ⟹ max 𝑎1 + 𝑎2 − 𝑏1 , 𝑎3 ≥ max 𝑏2 , 𝑏3 តាមសាំ នួរទី១ យយើ ងទញបាន 𝑥 𝑎1 𝑦𝑎2 𝑧𝑎3 = sym

𝑧 𝑎 3 𝑥 𝑎 1 𝑦𝑎 2 + 𝑥 𝑎 2 𝑦𝑎 1 cyclic

𝑧

𝑎3

𝑥 𝑎 1 +𝑎 2 −𝑏1 𝑦 𝑏1

+ 𝑥 𝑏1 𝑦 𝑎 1 +𝑎 2 −𝑏1

cyclic

𝑥 𝑏1 𝑦 𝑎 1 +𝑎 2 −𝑏1 𝑧 𝑎 3

= sym

𝑥 𝑏1 𝑦 𝑎 1 +𝑎 2 −𝑏1 𝑧 𝑎 3 + 𝑦 𝑎 3 𝑧 𝑎 1 +𝑎 2 −𝑏1

= cyclic

𝑥 𝑏1 𝑦 𝑏2 𝑧 𝑏3 + 𝑦 𝑏3 𝑧 𝑏2

≥ cyclic

𝑥 𝑏1 𝑦 𝑏2 𝑧 𝑏3

= sym

ខ) ករណី 𝑏1 ≤ 𝑎2 យោយ 3𝑏1 ≥ 𝑏1 + 𝑏2 + 𝑏3 = 𝑎1 + 𝑎2 + 𝑎3 ≥ 𝑏1 + 𝑎2 + 𝑎3 យ េះ 𝑏1 ≥ 𝑎2 + 𝑎3 − 𝑏1 𝑎1 ≥ 𝑎2 ≥ 𝑏1 ≥ 𝑎2 + 𝑎3 − 𝑏1

ដូយចនេះ តាមសាំ នួរទី១ យយើ ងទញបាន

max 𝑎2 , 𝑎3 ≥ max 𝑏1 , 𝑎2 + 𝑎3 − 𝑏1 max 𝑎1 , 𝑎2 + 𝑎3 − 𝑏1 ≥ max 𝑏2 , 𝑏3 𝑥𝑎1 𝑦𝑎2 𝑧𝑎3 =

sym

𝑥 𝑎1 𝑦𝑎2 𝑧𝑎3 + 𝑦𝑎3 𝑧𝑎2 cyclic

𝑥 𝑎 1 𝑦 𝑏1 𝑧 𝑎 2 +𝑎 3 −𝑏1 + 𝑦 𝑎 2 +𝑎 3 −𝑏1 𝑧 𝑏1

≥ cyclic

𝑦 𝑏1 𝑥 𝑎 1 𝑧 𝑎 2 +𝑎 3 −𝑏1 + 𝑥 𝑎 2 +𝑎 3 −𝑏1 𝑧 𝑎 1

= cyclic

𝑦 𝑏1 𝑥 𝑏2 𝑧 𝑏3 + 𝑥 𝑏3 𝑧 𝑏2

≥ cyclic

លឹម សុ វណ្ណវិចិត្រ | V. វិសមភាព

161


𝑥 𝑏1 𝑦 𝑏2 𝑧 𝑏3

= sym

សមភាពយកើតមាន ទល់ ដត និង មានដត 𝑥 = 𝑦 = 𝑧 ។

290. វ ិសមភាពដដលយោយសមមូលនឹង 1 1 3

1

+

1 3

1

+

𝑎 + 𝑏 + 𝑎𝑏𝑐 𝑏 + 𝑐 + 𝑎𝑏𝑐 𝑐 + 𝑎 + 𝑎𝑏𝑐 តាង 𝑎 = 𝑥 3 , 𝑏 = 𝑦 3 , 𝑐 = 𝑧 3 ដដល 𝑥, 𝑦, 𝑧 > 0 ។ វ ិសមភាពយៅជា 1 1 1 1 + 3 + 3 ≤ 3 3 3 3 𝑥 + 𝑦 + 𝑥𝑦𝑧 𝑦 + 𝑧 + 𝑥𝑦𝑧 𝑧 + 𝑥 + 𝑥𝑦𝑧 𝑥𝑦𝑧

1 3

1

𝑎𝑏𝑐

1 3

𝑥 3 + 𝑦 3 + 𝑥𝑦𝑧 𝑦 3 + 𝑧 3 + 𝑥𝑦𝑧

⟺ 𝑥𝑦𝑧 cyclic

≤ 𝑥 3 + 𝑦 3 + 𝑥𝑦𝑧 𝑦 3 + 𝑧 3 + 𝑥𝑦𝑧 𝑧 3 + 𝑥 3 + 𝑥𝑦𝑧 𝑥6 𝑦3 ≥

𝑥5 𝑦2𝑧2

sym

sym

ព ិតតាមវ ិសមភាព

យទ្ េះសេុី ត (6,3,0) លប់ យលើសេុីត 5,2,2 ។

291. យយើ ងមាន 𝑎3 = 6 𝑎3 + 𝑏 3 + 𝑐 3 + 𝑑 3 sym

𝑎𝑏 𝑐𝑑 = 4 𝑎𝑏 𝑐𝑑 + 𝑎𝑐 𝑏𝑑 + 𝑎𝑑 𝑏𝑐 + 𝑏𝑐 𝑑𝑎 + 𝑏𝑑 𝑐𝑎 + 𝑐𝑑 𝑎𝑏 sym

ដូយចនេះវ ិសមភាព សមមូលនឹង 𝑎3 ≥ sym

sym

ព ិត តាមវ ិសមភាព

1 1

𝑎3 𝑏 0 𝑐 0 𝑑 0 ≥

𝑎𝑏 𝑐𝑑 ⟺ sym

យទ្ េះ 3,0,0,0 លប់ យលើ

𝑎1 𝑏1 𝑐 2 𝑑2

sym 1 1 1,1, 2 , 2 ។

292. វ ិសមភាពដដលយោយសមមូលនឹង 𝑥5𝑦 + 2

4 sym

𝑥 4 𝑦𝑧 + 6𝑥 2 𝑦 2 𝑧 2 − cyclic

𝑥5𝑦 −

⟺ sym

sym

𝑥4 𝑦2 + 3 sym

𝑥5𝑦 − sym

≥0 តាមវ ិសមភាព

162

𝑥 4 𝑦2 − 6

១. អនុគមន៍ងាយ | លឹម សុ វណ្ណវិចិត្រ

𝑥3 𝑦3 − 2 cyclic

𝑥3 𝑦2 𝑧 ≥ 0 sym

𝑥 3 𝑦 3 + 2𝑥𝑦𝑧 3𝑥𝑦𝑧 + sym

𝑥3 − cyclic

𝑥2𝑦 sym


𝑥5𝑦 − sym

𝑥4 𝑦2 ≥ 0 sym

5

𝑥3 𝑦3 ≥ 0

𝑥 𝑦− sym

តាមវ ិសមភាព

sym

ចាំ យ េះ 𝑟 = 1 𝑥 𝑥−𝑦 𝑥−𝑧 ≥0 cyclic

𝑥 𝑥 2 − 𝑥𝑧 − 𝑥𝑦 + 𝑦𝑧 ≥ 0

⟺ cyclic

x 3 − x 2 z − x 2 y + xyz ≥ 0

⟺ cyclic

𝑥3 −

⟺ cyclic

𝑥2𝑧 + 𝑥2𝑦 + cyclic

3

cyclic 2

𝑥 − cyclic

𝑥𝑦𝑧 ≥ 0

𝑥 𝑦 + 3xyz ≥ 0 sym

ដូយចនេះវ ិសមភាពព ិត យទ្ េះអងគខាងយឆេងជាផលបូកននតួមិនអវ ិជាមាន។

293. យោយ 𝑥𝑦 + 𝑦𝑧 + 𝑧𝑥 = 1 យយើ ងបាំ ដលងវ ិសមភាពយោយយៅជាអូម៉ាូដសន 1 1 1 5 + + ≥ 𝑥+𝑦 𝑦+𝑧 𝑧+𝑥 2 1 1 1 ⟺ 𝑥𝑦 + 𝑦𝑧 + 𝑧𝑥 + + 𝑥+𝑦 𝑦+𝑧 𝑧+𝑥 𝑥5𝑦 +

⟺4 sym

𝑥 4 𝑦𝑧 + 14 sym

𝑥5𝑦 −

⟺ sym

2

5 2

2

𝑥 3 𝑦 2 𝑧 + 38𝑥 2 𝑦 2 𝑧 2 ≥ sym

𝑥4𝑦2 + 3 sym

sym

𝑥5𝑦 − sym

𝑥4 𝑦2 + 3

𝑥 3 𝑦 3 + 𝑥𝑦𝑧 sym

𝑥3 𝑦3 sym

𝑥 3 + 14 sym

𝑥 2 𝑦 + 38𝑥𝑦𝑧 sym

≥0 តាមវ ិសមភាព

វ ិសមភាពខាងយលើព ិត។ អងគទង ាំ ព ីរយសមើ គ្នន ទល់ ដត 𝑥 = 𝑦, 𝑧 = 0 រ ឺ

𝑦 = 𝑧, 𝑥 = 0 រ ឺ 𝑧 = 𝑥, 𝑦 = 0 ។ ដតយោយ 𝑥𝑦 + 𝑦𝑧 + 𝑧𝑥 = 1 យ េះ វាយសមើ គ្ននយពល 𝑥, 𝑦, 𝑧 = 1,1,0 ; 1,0,1 , 0,1,1

294. យយើ ងមាន

លឹម សុ វណ្ណវិចិត្រ | V. វិសមភាព

163


𝑎2 + 𝑎𝑏 + 𝑏 2 𝑏2 + 𝑏𝑐 + 𝑐 2 𝑐 2 + 𝑐𝑎 + 𝑎2 1 3 3 1 2 2 2 1 4 = 𝑎 𝑏 + 𝑎 𝑏 𝑐 + 𝑎 𝑏𝑐 + 2𝑎3 𝑏 2 𝑐 + 𝑎4 𝑏 2 2 2 2 sym

យហើ យ 𝑎𝑏 + 𝑏𝑐 + 𝑐𝑎

3

= sym

1 3 3 𝑎 𝑏 + 𝑎2 𝑏 2 𝑐 2 + 3𝑎3 𝑏 2 𝑐 2

ដូយចនេះ 𝑎2 + 𝑎𝑏 + 𝑏 2 𝑏2 + 𝑏𝑐 + 𝑐 2 𝑐 2 + 𝑐𝑎 + 𝑎2 − 𝑎𝑏 + 𝑏𝑐 + 𝑐𝑎 1 4 1 = 𝑎 𝑏𝑐 + 𝑎4 𝑏 2 − 𝑎2 𝑏 2 𝑐 2 − 𝑎3 𝑏 2 𝑐 2 2

3

sym

ដតថា សេុី ត 4,2,0 និង 4,1,1 លប់ សេុី ត 2,2,2 និង 3,2,1 ។ ដូយចនេះតាមវ ិសមភាព 1 4 1 𝑎 𝑏𝑐 + 𝑎4 𝑏 2 − 𝑎2 𝑏 2 𝑐 2 − 𝑎3 𝑏 2 𝑐 ≥ 0 2 2 sym

295. យយើ ងមាន 𝑥𝑦 + 𝑦𝑧 + 𝑧𝑥

𝑥+𝑦 = sym

2

+ 𝑦+𝑧 2 𝑧+𝑥 2 + 𝑧+𝑥 2 𝑥+𝑦 2 5 3 𝑥 5 𝑦 + 2𝑥 4 𝑦 2 + 𝑥 4 𝑦𝑧 + 𝑥 3 𝑦 3 + 13𝑥 3 𝑦 2 𝑧 + 4𝑥 2 𝑦 2 𝑧 2 2 2 𝑦+𝑧

2

និង 𝑥+𝑦

2

𝑦+𝑧

2

𝑧+𝑥

2

= sym

5 𝑥 4 𝑦 2 + 𝑥 4 𝑦𝑧 + 𝑥 3 𝑦 3 + 6𝑥 3 𝑦 2 𝑧 + 𝑥 2 𝑦 2 𝑧 2 3

វ ិសមភាពដដលយោយសមមូលនឹង 4𝑥 5 𝑦 + 𝑥 4 𝑦𝑧 + 𝑥 2 𝑦 2 𝑧 2 − 𝑥 4 𝑦 2 − 3𝑥 3 𝑦 3 − 2𝑥 3 𝑦 2 𝑧 ≥ 0 ∗ sym

ដតសេុី ត 5,1,0 លប់ យលើសេុីត 4,2,0 និង សេុី ត 3,3,0 ដូយចនេះតាមវ ិសមភាព 4𝑥 5 𝑦 − 𝑥 4 𝑦 2 − 3𝑥 3 𝑦 3 ≥ 0

(∗∗)

sym

ម៉ា យងវ ិញយទៀត តាមវ ិសមភាព sym

ចាំ យ េះ 𝑟 = 1 យយើ ងមាន 1 3 1 𝑥 + 𝑥𝑦𝑧 − 𝑥 2 𝑦 ≥ 0 2 2

គុណអងគទង ាំ ព ីរនឹង 2𝑥𝑦𝑧 វ ិសមភាពយនេះយៅជា 𝑥 4 𝑦𝑧 + 𝑥 2 𝑦 2 𝑧 2 − 2𝑥 3 𝑦 2 𝑧 ≥ 0 sym

164

១. អនុគមន៍ងាយ | លឹម សុ វណ្ណវិចិត្រ

(∗∗∗)


បូក(**) និង (***) យយើ ងទញបាន (*)ព ិត។ សមភាពយកើតមានយពល 𝑥 = 𝑦 = 𝑧 ។

296. យយើ ងមាន 𝑏+𝑐−𝑎

2

𝑐 + 𝑎 2 + 𝑏 2 𝑎 + 𝑏 2 + 𝑐 2 + 𝑐 + 𝑎 − 𝑏 2 𝑏 + 𝑐 + 𝑎2 + 𝑎 + 𝑏 − 𝑐 2 𝑏 + 𝑐 2 + 𝑎2 𝑐 + 𝑎 2 + 𝑏 2 3 6 = 𝑎 + 2𝑎5 𝑏 + 𝑎4 𝑏 2 + 3𝑎4 𝑏𝑐 + 2𝑎3 𝑏 3 + 3𝑎2 𝑏 2 𝑐 2 2

𝑎+𝑏

2

+ 𝑐2

sym

𝑏+𝑐

2

+𝑎

2

𝑐+𝑎 = sym

2 + 𝑏2 𝑎 + 𝑏 2 + 𝑐 2 1 6 7 𝑎 + 2𝑎5 𝑏 + 3𝑎4 𝑏2 + 3𝑎4 𝑏𝑐 + 2𝑎3 𝑏 3 + 8𝑎3 𝑏2 𝑐 + 𝑎2 𝑏 2 𝑐 2 2 3

វ ិសមភាពសមមូលនឹង 3𝑎6 + 2𝑎5 𝑏 − 2𝑎4 𝑏 2 + 3𝑎4 𝑏𝑐 + 2𝑎3 𝑏 3 − 12𝑎3 𝑏2 𝑐 + 4𝑎2 𝑏 2 𝑐 2 ≥ 0

(1)

sym

ដតតាមវ ិសមភាព sym

1 3 1 𝑎 + 𝑎𝑏𝑐 − 𝑎2 𝑏 ≥ 0 2 2

យោយគុណអងគនម ិ ួយៗនន

(2)

នឹង 4𝑎𝑏𝑐 យយើ ងទញបាន

4𝑎4 𝑏𝑐 − 8𝑎3 𝑏 2 𝑎 + 4𝑎2 𝑏2 𝑐 2 ≥ 0

(3)

sym

ដូយចនេះយយើ ងទ្តូវបង្ហាញថា 3𝑎6 + 2𝑎5 𝑏 − 2𝑎4 𝑏 2 − 𝑎4 𝑏 2 + 2𝑎3 𝑏 3 − 4𝑎3 𝑏2 𝑐 ≥ 0

(4)

sym

ដតសេុី ត 6,0,0 លប់ យលើសេុីត 4,1,1 និង សេុី ត 3,2,1 ដូយចនេះ 3𝑎6 − 𝑎4 𝑏𝑐 − 2𝑎3 𝑏 2 𝑐 ≥ 0

(5)

sym

ប ទ ប់ មកយទៀតសេុី ត 5,1,0 លប់ យលើសេុីត 4,2,0 ដូយចនេះ 2𝑎5 𝑏 − 2𝑎4 𝑏 2 ≥ 0

6

sym

យហើ យជាចុងយទ្ោយ សេុី ត 3,3,0 លប់ យលើសេុីត 3,2,1 ដូយចនេះ 2𝑎3 𝑏 3 − 2𝑎3 𝑏 2 𝑐 ≥ 0

7

sym

យោយបូក

,

និង

លឹម សុ វណ្ណវិចិត្រ | V. វិសមភាព

យយើ ងទញបាន

។ អងគទង ាំ ព ីរយសមើ គ្នន ទល់ ដត 𝑎 = 𝑏 = 𝑐 ។

165


VI. វិសមីការ 297. −3; 2 ∪ 4; ∞ . 298. −∞; −2 ∪ −1 . 299. 301. −5; −1 . 302. −∞; − 3 ∪ − 3; 2 + 4/ 3 . 304. −∞; ∞ . 305. −

1

;

1

;

2

; ∞ . 300. −∞; −1 ∪ 2; ∞ .

3; ∞ . 303. 2 − 4/ 3; 1 ∪

3 3 3+ 33 −3+ 33 2

1

2

.

306. −1 + 3 − 3 + 2 3; 1 + 3 + 3 + 2 3 . វ ិសមីោរយនេះសមមូលនឹង 𝑝 𝑥 = 𝑥 4 − 4𝑥 3 − 6𝑥 2 − 4𝑥 + 1 < 0 ។ យយើ ងកាំនត់ 𝑥 ដដល 𝑝 𝑥 = 0 ។ យោយ 𝑥 = 0 មិន ដមនជារ ឺសននពហុ ធាយនេះ ដូយចនេះយយើ ងដចក𝑝 𝑥 = 0 នឹង 𝑥 យយើ ងទញបាន 𝑥 2 + 1 𝑥 2 − 4 𝑥 + 1/𝑥 − 6 = 0 ។ តាង 𝑦 = 𝑥 + 1 𝑥 យយើ ងទញបាន 𝑦 2 − 4𝑦 − 8 = 0 ។ ដូយចនេះ 𝑦1 = 2 1 + 3 និង 𝑦1 = 2 1 − 3 ។ ដូយចនេះពហុ ធា𝑝 𝑥 ោចសរយសរជា 𝑝 𝑥 = 𝑥 2 − 2 1 − 3 𝑥 + 1 ។ យោយ 𝑥 2 − 2 1 − 3 𝑥 + 1 > 0 ចាំ យ េះ

𝑥2 − 2 1 + 3 𝑥 + 1

ទ្គប់ 𝑥 ∈ ℝ ដូយចនេះវ ិសមីោរសមមូលនឹង 𝑥 2 − 2 1 + 3 𝑥 + 1 < 0។

. 307. −1 − 3;

3− 17 2

3 − 1;

3+ 17 2

. តាង 𝑦 = 𝑥 − 2/𝑥. 308. −2; 0 ∪

2; ∞ . 309. −1; 2 . 310. −∞; −1 ∪ 0; 1/2 ∪ 1; 2 . 311. −1; 5/8 ∪ 2; 7/3 ∪ 3; ∞ . 312. −2; −1 ∪ 0; 1 ∪ 2; ∞ . 313. −7; − 37 ∪ −5; 0 ∪ 5; 37 ∪ 7; ∞ . 314. −1; 0 . 315. −∞; −2 ∪ −1; 1 ∪ 2; 3 ∪ 4; 6 ∪ 7; ∞ . 316. −∞; 1 ∪ 3/2; 5/2 ∪ 7/2; 4 . 317. −1/2; 1 . 318. −2; 1 . 319. 0; 1/3 ∪ 0; ∞ . 320. −

1+ 5

6−6 26 5

2

; −1 ∪

1− 5 2

;0 ∪

; −4 ∪ −4; 0 ∪ 6;

−1+ 5

2 6+6 26 5

;1 ∪

1+ 5 2

; ∞ . 321. −∞; −6 ∪

. 322. 0; 2 . 323. −1 − 2 2; −3 ∪ 1; 3 .

324. −1 ∪ 2; ∞ . 325. 0,5; 2 . 326. − 3; 0 ∪ 0; 2 . 327. −0,5; ∞ ចំយ ោះ 𝑎 ∈ 1; ∞ ; −0,5; −0,5 1 −

1 1−𝑎 2

ចំយ ោះ 𝑎 ∈ −∞; 1 . 328. 1; 3 . 329. −2; ∞ 5

ចំយ ោះ 𝑎 ∈ −2; ∞ ; ∅ ចំយ ោះ 𝑎 ∈ −∞; −2 . 330. − 8 ; 2,4 . 331. 332. 3; 4,8 . 333. 1; 2/ 3 . 334.

166

១. អនុគមន៍ងាយ | លឹម សុ វណ្ណវិចិត្រ

13−5 2

5−1 2

; 1 . 335. 3; 12 . 336. −2; ∞ .

;1 .


337.

5− 13 6

; ∞ . 338.

8 3

; ∞ . 339. 2;

(1; ∞). 342. 0; 1/2 . 343.

1 2

2 3

21 . 340. 3/2; 2 ∪ (2; 26). 341. −2; 1 ∪

;∞ .

344. −4; 0 ∪ 4; 6 . លកខខណឌរបស់ វ ិសមីោរ 24 + 2𝑥 − 𝑥 2 ≥ 0 និង 𝑥 ≠ 0 ាំ

យោយយយើ ងទញបានចយ ា េះព ីរ −4; 0 និង 0; 6 ។ យយើ ងយឃើ ញថា 𝑥 ∈ −4; 0 ជាចាំ យលើយរបស់ វ ិសមីោរ យទ្ េះអងគខាងយឆេងអវ ិជាមាន ដដលតូចជាង១ជានិច។ ច ចាំ យ េះចយ ា េះទីព ីរ យយើ ងទញបាន 24 + 2𝑥 − 𝑥 2 < 𝑥។ . 3

345. −1; − 4 . តាង 𝑦 = 𝑥 + 1 + 𝑥 + 3 ។ យយើ ងទញបាន 𝑦 2 − 3𝑦 + 2 < 0។ ដូយចនេះ 1 < 𝑦 < 2 សមមូលនឹង 𝑥+1+ 𝑥+3>1 𝑥+1+ 𝑥+3<2 𝑥 ≥ −1 យយើ ងព ិនិតែវ ិសមីោរទីមួយ។ វាសមមូលនឹង 2 𝑥 2 + 4𝑥 + 3 > −3 − 2𝑥 ។ អងគខាងាតាំ អវ ិជាមានយបើ 𝑥 ≥ −1 ។ ដូយចនេះ 𝑥 ∈ −1; ∞ ជាចាំ យលើយរបស់ វ ិសមីោរ។ យលើកអងគទង ាំ ព ីរនន វ ិសមីោរទីព ីរជាោយរ សាំ រួលរួចយហើ យ យយើ ងទញបាន 𝑥 2 + 4𝑥 + 3 < −𝑥 ។ វ ិសមីោរយនេះមាន ចាំ យលើយទល់ ដត −1 ≤ 𝑥 ≤ 0 (យោយគិតលកខខណឌវ ិសមីោរទីមួយចូល)។ យលើអងគទង ាំ ព ីរជាោយរ យយើ ងទញបាន 4𝑥 + 3 < 0។.

346. យយើ ងមាន រឺ

𝑃 𝑥 = 𝑥 − 1 𝑥 𝑥4 𝑥2 + 𝑥 + 1 + 1 + 1

(1)

𝑃 𝑥 = 1 − 𝑥 + 𝑥2 1 − 𝑥3 + 𝑥8

(2)

តាម(១) យយើ ងមាន 𝑃 𝑥 > 0 ចាំ យ េះទ្គប់ 𝑥 ∈ −∞; 0 ∪ 1; ∞ យហើ យតាម(២) យយើ ងមាន 𝑃 𝑥 ចាំ យ េះទ្គប់ 𝑥 ∈ 0; 1 ។ ដូយចនេះ 𝑃 𝑥 > 0 ចាំ យ េះទ្គប់ 𝑥 ∈ ℝ ។

347. អនុគមន៍ 𝑓 𝑥 = 𝑃 𝑥 𝑄 𝑥 កាំនត់ ចាំយ េះទ្គប់ 𝑥 យលើកដលងដតចាំ ចាំនច ុ 𝑥𝑘 , 𝑘 ∈ ℕ, 𝑘 ≤ 𝑛 ដដលជាចាំ នច ុ សូ នែរបស់ ពហុ ធា 𝑄 𝑥 ។ សនមតថា 𝑥0 ជាចាំ យលើយមួយរបស់ វ ិសមីោរ 𝑓 𝑥 > 0 ។ ដូយចនេះ 𝑃 𝑥0 និង 𝑄(𝑥0 ) មានសញ្ជាដូចគ្នន ដូយចនេះ 𝜙 𝑥0 = 𝑃 𝑥0 𝑄 𝑥0 > 0 មានន័យថា 𝑥0 ក៏ជា រ ឺសនន𝜙 𝑥 = 𝑃 𝑥 𝑄 𝑥 > 0 ។ ចាំ យ េះ 𝑥 = 𝑥𝑘 យយើ ងមាន 𝜙 𝑥𝑘 = 0។ ដូយចនេះ 𝑥𝑘 មិនដមនជារ ឺស នន 𝜙 𝑥 យទ។ ដូយចនេះរ ឺសទាំងអស់ របស់ 𝑓 𝑥 > 0 ក៏ជារ ឺសរបស់ 𝜙 𝑥 > 0 ដដរ ។ យយើ ងទ្ាយ បញ្ជាក់ ដច ូ គ្ននថា រ ឺសរបស់ 𝜙 𝑥 > 0 ក៏ជារ ឺសរបស់ 𝑓 𝑥 > 0 ដដរ។ យបើ𝑃 𝑥 និង 𝑄 𝑥 មានតាំ នល លឹម សុ វណ្ណវិចិត្រ | VI. វិសមីការ

167


មានសញ្ជាយផេងគ្នន ចាំ យ េះតាំ នល𝑥 សថិ តកនង ុ ដដនកាំនត់ របស់ 𝑓 𝑥 យ េះវ ិសមភាព 𝑓 𝑥 > 0 និង 𝜙 𝑥 > 0 គ្នមនរ ឺស។ ដូយចនេះវ ិសមីោរទាំងព ីរសមមូលគ្នន។.

348. 𝑎 < 0. 1 45

349. – 2 ;

8

4𝑥 2

1

∖ 0  អងគខាងយឆេងកាំនត់ យបើ 𝑥 ≥ − 2 និង 𝑥 ≠ 0 ។ យយើ ងមាន 2

1 + 1 + 2𝑥 ។ យោយអនុគមន៍ 𝑓 𝑥 = 1 + 1 + 2𝑥 យហើ យយោយ 𝑓 45/8 = 0 យ េះ វ ិសមីោរមានរ ឺស 31

350. – 1; 1 −

8

1 −2

2

1− 1+2𝑥

2

=

− 2𝑥 − 9 = 2 1 + 2𝑥 − 7 យកើន

≤ 𝑥 < 45/8 និង 𝑥 ≠ 0 ។

. យយើ ងមាន 𝑓 𝑥 = 3 − 𝑥 − 𝑥 + 1 មានន័យចាំ យ េះ −1 ≤ 𝑥 ≤ 3 1 2

យហើ យជាអនុគមន៍ថយ យហើ យជាប់ យៅយលើចយ ា េះយនេះ។ យយើ ងមាន 𝑓 −1 = 2 > និង 𝑓 1−

31 8

1

= 2 ។ ដូយចនេះវ ិសមីោរយផទៀងផ្ទទត់ ចាំយ េះ −1 ≤ 𝑥 < 1 −

31 8

351.  លកខខណឌ 2𝑥 + 4 ≥ ⟺ −2 ≤ 𝑥 ≤ 2 2−𝑥 ≥0 យោយអងគទង ាំ ព ីរននវ ិសមីោរវ ិជាមាន យ េះយលើកអងគទង ាំ ព ីរននវ ិសមីោរជាោយរ យយើ ងទញបាន 9𝑥 2 + 16 ≥ 4 2𝑥 + 4 + 16 2 − 𝑥 + 16

2𝑥 + 4 2 − 𝑥

2

9𝑥 + 16 ≥ −8𝑥 + 48 + 16 −2𝑥 2 + 8 9𝑥 2 + 8𝑥 − 32 − 8 −8𝑥 2 + 32 ≥ 0 (1) 2 2 តាង 𝑡 = −8𝑥 2 + 32 ≥ 0; 𝑡 = −8𝑥 + 32 ។ វ ិសមីោរសមមូលនឹង −𝑡 2 − 8𝑡 + 𝑥 2 + 8𝑥 ≥ 0 (2) 2 2 2 យយើ ងមាន 𝑔 𝑡 = −𝑡 − 8𝑡 + 𝑥 + 8𝑥 មាន Δ = 16 + 𝑥 + 8𝑥 = 𝑥 + 4

2

។ ដូយចនេះ 𝑔 𝑡

មានរ ឺស 𝑡1 = 𝑥; 𝑡2 = −𝑥 − 8 វ ិសមីោរ(២) សមមូលនឹង min 𝑡1 ; 𝑡2 ≤ 𝑡 ≤ max 𝑡1 ; 𝑡2 ។ យយើ ងមាន 𝑡1 − 𝑡2 = 2𝑥 + 8 យោយ −2 ≤ 𝑥 ≤ 2 យ េះ 2𝑥 + 8 > 0 ។ ដូយចនេះ −𝑥 − 8 ≤ 𝑡 ≤ 𝑥 ⟺ 𝑡 ≤ 𝑥 𝑥≥0 −8𝑥 2 + 32 ≤ 𝑥 ⟹ 2 32 𝑥 ≥ 9 4 2 ⟹ ≤𝑥≤2 3 168

១. អនុគមន៍ងាយ | លឹម សុ វណ្ណវិចិត្រ


VII. ត្រព័នវធ ស ិ មីការ 352. 0; 2  ចាំ យលើយរបស់ ទ្បព័នស ធ ថិ តយៅកនង ុ នផទឆត ូ ។ យយើ ងសង់ ប ទ ត់ 2𝑥 + 3𝑦 = 0។ យយើ ងចង់ បានតាំ នលធាំ បាំផត ុ នន 𝑧 = 2𝑥 + 3𝑦 ។ ចាំ យ េះតាំ នលយផេងគ្នននន z យយើ ងគូសបានប ទ ត់ ស្សបនឹងប ទ ត់ 2𝑥 + 3𝑦 = 0។ យៅយពលដដល 𝑧 ោន់ ដតធាំ យៅៗ ប ទ ត់ ទ្តូវនឹងតាំ នល𝑧 ទាំងយ េះឃ្លាតោន់ ដតឆ្លាយព ី គល់ តាំរយ ុ ។ ដូយចនេះ z ធាំ បាំផត ុ ដដលយផទៀងផ្ទទត់ ទ្បព័នជា ធ តាំ នល 𝑧 យៅយពលដដលប ទ ត់ 𝑧 = 2𝑥 + 3𝑦 ោត់ តាមA។

353.

3; 1 . 354. 𝑎 = −1. ចាំ នច ុ ចង់ បានគឺ 0; −1 ។ 355.

357.  សមីោរទី៣

7

7

3; 2 ; −3; − 2 . 356.{1}.

យាំ ោយ 4 ≤ 𝑥 ≤ 7 ។ តាង 𝑢 = 𝑦 + 𝑧; 𝑣 = 𝑦𝑧 ។ សមីោរ(១)និង(២) យៅ

ជា 𝑥 3 + 𝑥 2 13 − 𝑢 + 𝑥 2𝑢 − 2𝑣 − 26 + 5𝑣 − 7𝑢 + 30 = 0 𝑥 3 + 𝑥 2 17 − 𝑢 − 𝑥 2𝑢 + 2𝑣 − 26 − 3𝑣 + 𝑢 − 2 = 0 𝑢 −𝑥 2 + 2𝑥 − 7 + 𝑣 −2𝑥 + 5 + 𝑥 3 + 13𝑥 2 − 26𝑥 + 30 = 0 (4) 𝑢 −𝑥 2 − 2𝑥 + 1 + 𝑣 −2𝑥 − 3 + 𝑥 3 + 17𝑥 2 + 26𝑥 − 2 = 0 (5) យកសមីោរទី(៤)ដកទី(៥) យយើ ងទញបាន 𝑢 4𝑥 − 8 + 8𝑣 − 4𝑥 2 − 52𝑥 + 32 = 0 1 ⟹ 𝑣 = 𝑢 2 − 𝑥 + 𝑥 2 + 13𝑥 − 8 2 ជាំនស ួ ចូលសមីោរទី(៤) យយើ ងទញបាន 2𝑢 −𝑥 2 + 2𝑥 − 7 + −2𝑥 + 5 𝑢 2 − 𝑥 + 𝑥 2 + 13𝑥 − 8 + 2𝑥 3 + 26𝑥 2 − 52𝑥 + 60 =0 ⟹ 𝑢 −5𝑥 − 4 + 29𝑥 + 5𝑥 2 + 20 = 0 ⟹ 𝑢 5𝑥 + 4 = 5𝑥 + 4 𝑥 + 5 លឹម សុ វណ្ណវិចិត្រ | VII. ត្រព័នវធ ស ិ មីការ

169


យោយ 4 ≤ 𝑥 ≤ 7 យ េះ 5𝑥 + 4 ≠ 0 ដូយចនេះ 𝑢 = 𝑥 + 5 ។ យយើ ងទញបាន 𝑣 = 5𝑥 + 1 ។ ដូយចនេះ 𝑦 + 𝑧 = 𝑥 + 5; 𝑦𝑧 = 5𝑥 + 1 ដូយចនេះ 𝑦, 𝑧 ជារ ឺសននសមីោរ 𝑡 − 𝑥 + 5 𝑡 + 5𝑥 + 1 = 0 ។ សមីោរយនេះមានរ ឺសយបើ 2

ឌ ីសទ្គីសមីណង់

Δ= 𝑥−3 𝑥−7 ≥0 ⟹ 𝑥 ≤ 3 រឺ 𝑥≥7 ដត 4 ≤ 𝑥 ≤ 7 ដូយចនេះ 𝑥 = 7 ។ យយើ ងទញបាន 𝑦 = 𝑧 = 6 ។

358.  គុណវ ិសមីោរទីមួយនឹង (−2) រួចបូកចូលវ ិសមីោរទីព ីរ យយើ ងទញបាន 𝑥 + 3𝑦

2

≤−

4 𝑎+1

(1)

4 ព ី(១) យយើ ងទញបាន − > 0 ⟹ 𝑎 < −1 ។ 𝑎+1 1−𝑎 2 1−𝑎 យយើ ងមាន 𝑎+1 + 1 = 𝑎+1 < 0 ⟹ 𝑎+1 < −1 ។ ព ិនិតែទ្បព័នស ធ មីោរ 2 2

𝑥 + 2𝑥𝑦 − 7𝑦 = −1 (4) 3𝑥 2 + 10𝑥𝑦 − 5𝑦 2 = −2 (5) 1−𝑎 យោយ 𝑎+1 < −1 យ េះ យបើ (4)និង(៥) មានរ ឺស យ េះទ្បព័នវធ ិសមីោរក៏មានរ ឺសដដរ។ គុណសមីោរទី៤ នឹង(−2) បូកចូលសមីោរទី៥ យយើ ងទញបាន 𝑥 2 + 2𝑥𝑦 − 7𝑦 2 = −1 4 2 𝑥 + 3𝑦 = 0 5 𝑥 = −3𝑦 1 ⟺ 𝑦2 = 𝑦 ទ្បព័នយធ នេះមានរ ឺស។ ដូយចនេះ 𝑎 < −1 ជាលកខខណឌចា​ាំបាច់ និងទ្គប់ ទ្គ្នន់ យដើមបីយោយទ្បព័នមា ធ នរ ឺស។

170

១. អនុគមន៍ងាយ | លឹម សុ វណ្ណវិចិត្រ


á&#x;˘. áž˘áž“ážťáž‚áž˜áž“á&#x;? អឝá&#x;Šážˇ áž…áž&#x;á&#x;’áž”á&#x;‰ážźážŽáž„á&#x;‹áž&#x;á&#x;’áž&#x;á&#x;‚លលá&#x;„កវរីáž?

គណá&#x;’áž“ážś 359. 9.

360. ln 3.

361. ln đ?‘Ž .

362. 4 log đ?‘Ž đ?‘? .

363. 3.

364. 2.

365. 1

366. 2.

367. 0 ď ° đ?‘Ž

log đ?‘Ž đ?‘?

= đ?‘Ž

log đ?‘Ž đ?‘?

368. 3 1 − đ?‘? − đ?‘‘ ; 369. 2

1/ log đ?‘Ž đ?‘?

5−đ?‘‘ đ?‘?−2đ?‘‘+đ?‘?đ?‘‘ +1

=đ?‘?

log đ?‘? đ?‘Ž

, ដសចá&#x;’áž“á&#x; á&#x;‡áž•áž›áž&#x;áž„áž…áž&#x;មឞ áž&#x;ážźáž“áž™á&#x;”

;

370. 5đ?‘? − 6đ?‘‘ − 4 .ď ° áž… ážž áž„áž˜ážśáž“ 0,175 = 1000 = 40 = 7/(22 . 10).á&#x;” ដសចá&#x;’áž“á&#x; á&#x;‡ log 0,175 = 175

7

log 7 − 2 log 2 − 1 á&#x;” áž… ážž áž„áž˜ážśáž“ log 196 = log 22 . 72 = 2 log 2 + 2 log 7 = đ?‘?; នឡង log 56 = log 23 . 7 = 3 log 2 + log 7 = đ?‘‘ á&#x;” áž… ážž ងទវញបវន log 2 = ដសចá&#x;’áž“á&#x; á&#x;‡ log 0,175

4

2đ?‘‘−đ?‘? 4

; log 7 =

3đ?‘?−2đ?‘‘ 4

á&#x;”

= 4 log 7 − 2 log 2 − 1 = 5đ?‘? − 6đ?‘‘ − 4 .

371. 3 ď ° áž?ážśáž„ log 2 12 = đ?‘Ž áž… ážž áž„áž˜ážśáž“ 1 log 96 2 = log 2 23 . 12 = đ?‘Ž + 3; log 2 24 = 1 + đ?‘Ž; log 2 196 = đ?‘Ž + 4 នឡង đ?‘Ž đ?‘Ž+4 =3 á&#x;”

លរម áž&#x;ážť វណá&#x;’ážŽážœážˇáž…ážˇáž?á&#x;’ážš | I. គណá&#x;’áž“ážś

1 log 12 2

= đ?‘Ž á&#x;” កចនá&#x;„មដដលចá&#x;„ áž&#x;áž˜áž˜ážźáž›áž“ážšáž„ đ?‘Ž + 1 đ?‘Ž + 3 −

171


II. áž&#x;áž˜áž—ážśáž– 372. 1. ď ° áž… ážž áž„áž˜ážśáž“ log 2 18 1 + 2 log 2 3 = log 2 12 2 + log 2 3 log 2 54 1 + 3 log 2 3 log 24 54 = = log 2 24 3 + log 2 3 áž?ážśáž„ log 2 3 = đ?‘Ľ áž… ážž ងទវញបវន log12 18 =

1 + 2đ?‘Ľ 1 + 3đ?‘Ľ 1 + 2đ?‘Ľ 1 + 3đ?‘Ľ 6đ?‘Ľ 2 + 5đ?‘Ľ + 1 + 5 −đ?‘Ľ 2 + 1 +5 − = 2+đ?‘Ľ 3+đ?‘Ľ 2+đ?‘Ľ 3+đ?‘Ľ đ?‘Ľ+2 đ?‘Ľ+3 đ?‘Ľ 2 + 5đ?‘Ľ + 6 = =1 đ?‘Ľ+2 đ?‘Ľ+3

đ?‘Žđ?‘? + 5 đ?‘Ž − đ?‘? =

III. áž&#x;áž˜áž¸áž€ážśážš 373. log 3/2 2 ; 2 log 3/2 2 . 374. 1 − 3; 0; 2; 1 + 3 . 375. 1 − 3; 1 + 3 . 376. log1

5 2−7

6;0

377. ď ° −2; 3 . áž?ážśáž„ 3đ?‘Ľ = đ?‘˘, 3đ?‘Ľ+6 = đ?‘Ł á&#x;” áž&#x;áž˜áž¸áž€ážśážšážŠážŠáž›áž…á&#x;„ áž˜ážśáž“ážšážśáž„ đ?‘˘2 − 2đ?‘˘đ?‘Ł + đ?‘Ł 2 = 0 ážš ážş 2

đ?‘˘âˆ’đ?‘Ł

2

= 0 á&#x;” ដសចá&#x;’áž“á&#x; á&#x;‡ 3đ?‘Ľ = 3đ?‘Ľ+6 á&#x;” ...á&#x;” 2

378. ď ° −2; 3 . áž?ážśáž„ 2 đ?‘Ľ2 4

−1

2đ?‘Ľ+1

= �, 2� = � ច ឞ ងទទួលបវន � 2 � + � = 2� + � 2 �/4 រ ឺ

2đ?‘Ś − đ?‘§ = 0 á&#x;” áž… ážž ងទវញបវន đ?‘Ľ1 = 2 ážš ážş 2đ?‘Ś = đ?‘§ áž“ážśá&#x;†áž…á&#x;„

379. 11 .ď ° áž… ážž ងបវá&#x;† ážŠáž›áž„áž˘áž„áž‚áž ážśáž„áž…ážœá&#x; áž„áž“áž“áž&#x;áž˜áž¸áž€ážśážš4log 64 52 4log 4

đ?‘Ľâˆ’3

1/3

= 25 đ?‘Ľ − 3

1/3

đ?‘Ľâˆ’3 +log 2 5

á&#x;” áž… ážž áž„áž˜ážśáž“ 25 đ?‘Ľ − 3

1/3

đ?‘Ľ4 = 4 á&#x;” = 4log 4 3

đ?‘Ľâˆ’3

2log 2 5

= 50, đ?‘Ľ − 3 = 23 , đ?‘Ľ = 11 á&#x;”

380. 4 . 381. −3; −1 . 382. 27 ; 383. −1 ď ° áž… ážž áž„áž”ážšáž„áž˝áž˜áž&#x;áž˜áž¸áž€ážśážšáž…á&#x;…ជវរវង log 2 3 − đ?‘Ľ 1 − đ?‘Ľ = log 2 23 áž“ážśá&#x;†áž…á&#x;„

đ?‘Ľ 2 − 4đ?‘Ľ − 5 = 0 áž“ážśá&#x;†áž…á&#x;„

đ?‘Ľ = −1 á&#x;”

384. 4 ; 385. 8 ; 386. 2 ; 387. 3 ; 388. 3; 3 + 2 ; 389. −11; −6 − 7; −6 +

7; −1; 390. 3 ; 391. −17 ; 392. 1 ; 393. 2 ; 394. 395. −21/ log đ?‘Ž

4đ?‘Ž 4

; 21/ log đ?‘Ž

4đ?‘Ž 4

2; 6 .

á&#x;’ážśá&#x;† áž… á&#x; á&#x;‡ đ?‘Ž ∈ 0; 1/ 2 âˆŞ 1

áž&#x;áž˜áž¸áž€ážśážšáž‚á&#x;’មមនរ ážşáž&#x;á&#x;” 172

á&#x;˘. áž˘áž“ážťáž‚áž˜áž“á&#x;?អឝឡចáž&#x;á&#x;’áž”á&#x;‰ážźážŽá&#x;’áž„á&#x;‹áž&#x;á&#x;’áž&#x;á&#x;‚áž›-áž›á&#x;„កវរីរ | លរម áž&#x;ážť វណá&#x;’ážŽážœážˇáž…ážˇáž?á&#x;’ážš

2

2 ; 1 âˆŞ 1; ∞ áž…ážšá&#x;…áž– ីចនá&#x; á&#x;‡

=


396. 397.

3đ?‘Ž+3 7−đ?‘Ž 2đ?‘Žâˆ’1 6

á&#x;’ážśá&#x;† áž… á&#x; á&#x;‡ đ?‘Ž ∈ 3; 7 áž…ážšá&#x;…áž– ីចនá&#x; á&#x;‡áž&#x;áž˜áž¸áž€ážśážšáž‚á&#x;’មមនរ ážşáž&#x;á&#x;” á&#x;’ážśá&#x;† áž… á&#x; á&#x;‡ đ?‘Ž ∈ −∞; −12 âˆŞ 1 2 ; ∞ áž…ážšá&#x;…áž– ីចនá&#x; á&#x;‡áž&#x;áž˜áž¸áž€ážśážšáž‚á&#x;’មមនរ ážşáž&#x;á&#x;”

398. 1; 60 . á&#x;’ážśá&#x;† áž… á&#x; á&#x;‡ đ?‘Ľ = 1 អងគទវង ážśá&#x;† áž– ីរននáž&#x;áž˜áž¸áž€ážśážšáž…áž&#x;មឞ áž&#x;ážźáž“áž™ ដសចá&#x;’áž“á&#x; á&#x;‡ đ?‘Ľ = 1 ជវរ ážşáž&#x;áž˜áž˝ áž“áž“áž&#x;áž˜áž¸áž€ážśážšáž…áž“á&#x; á&#x;‡á&#x;” ូលស វចនá&#x; á&#x;‡áž… ážž ងរករ ážşáž&#x;áž…áž•á&#x; áž„áž…áž‘á&#x;€áž?ដដលចផá&#x; áž„áž– áž¸áž˜áž˝ á&#x;” គឝណអងគទវង ážśá&#x;† áž– ីរននáž&#x;áž˜áž¸áž€ážśážšáž“ážšáž„ log áž… ážž ងទវញបវន

1 đ?‘Ľ log 3 4 đ?‘Ľ log 5 đ?‘Ľ

1 1 1 + + = log đ?‘Ľ 3 + log đ?‘Ľ 4 + log đ?‘Ľ 5 log 5 đ?‘Ľ log 3 đ?‘Ľ log 4 đ?‘Ľ â&#x;š log đ?‘Ľ 3.4.5 = 1; â&#x;š đ?‘Ľ = 60 1 1 399. 1; 3/8 . 400. 10 ; 10 . 401. 2 ; 4 . 402. 3; 39 . 403. 1/625; 5 .404. 1=

5

5; 5 .

405. 10 . 7 3+ 24 2

406. −1/4 . 407. 0; 4 ; 1 ; 1; 3 9

. 408. 2 . 409.

1 3

4

; 8 . 410.

1 ; 1; 4 2

. 411.

1 ; 1; 4 8

. 412.

.

413. 5 . 414. đ?‘Ž − 1; đ?‘Ž + 1 á&#x;’ážśá&#x;† áž… á&#x; á&#x;‡ đ?‘Ž ∈ 1; 2 âˆŞ

2; 2 âˆŞ 2; ∞ ; 3 á&#x;’ážśá&#x;† áž… á&#x; á&#x;‡ đ?‘Ž = 2 á&#x;”

415. đ?‘Ž2 á&#x;’ážśá&#x;† áž… á&#x; á&#x;‡ đ?‘Ž ∈ 0; 1/ 2 âˆŞ 1/ 2; 1 âˆŞ 1; ∞ á&#x;” 416. 25

417. 1/9

421. 1/3

422. 3; 10

1+ 5 2

418.

419. 2

420. 3 3

423. 2; 4

2

424. log 2 5 ; log 2 5

425. 2

426. 2

427. 0

429. 2

430. – log 2 3

431. – 10 ; 99 432. – 10 5 ; 103

433. 1000

9

434. 0

437. 2 . ď‚˜ áž?ážśáž„ 2

log đ?‘Ľ 2 3đ?‘Ľâˆ’2

435. = �, 3

428. −1; 2

log đ?‘Ľ 2 3đ?‘Ľâˆ’2

1

1 ; 2; 1000 10

= đ?‘Ł ážšáž˝á&#x;’áž… ážž

436. 0,2; 6 áž… á&#x; á&#x;‡ážšážś áž&#x;áž˜áž¸áž€ážśážš

3đ?‘˘2 − 5đ?‘˘đ?‘Ł + 2đ?‘Ł 2 = 0 áž‡ážśáž˘áž“ážťáž‚áž˜áž“á&#x;?áž“áž“ đ?‘˘ ážš ážş đ?‘Ł á&#x;” 438.

1 ;∞ 5

439. 1/16 âˆŞ 4; ∞ .ď ° áž?ážśáž„ đ?‘Ľ −

đ?‘Ľ − 2 = đ?‘Ś áž&#x;áž˜áž¸áž€ážśážšážŠážŠáž›áž…á&#x;„ áž…á&#x;…ជវ log 2 đ?‘Ś + 6 =

log 2 2 đ?‘Ś ážš ážş 2đ?‘Ś 2 − đ?‘Ś − 6 = 0 ážŠážŠáž›áž˜ážśáž“ážš ážşáž&#x; đ?‘Ś1 = 2 នឡង đ?‘Ś2 = −3/2 á&#x;” ដសចá&#x;’áž“á&#x; á&#x;‡ đ?‘Ľ − đ?‘Ľ − 2 = đ?‘Ś1 = 2 â&#x;š đ?‘Ľ − 2 = 3

đ?‘Ľ − 2 â&#x;š đ?‘Ľ ≼ 4 á&#x;” ជវá&#x;†áž“áž&#x; áž˝ đ?‘Ľâˆ’

đ?‘Ľ − 2 = đ?‘Ś2 =

1

− 2 â&#x;š 2 đ?‘Ľ = 2 â&#x;š đ?‘Ľ = 1/16 á&#x;” 440. 2 .ď ° ដá&#x;’កអងគទវង ážśá&#x;† áž– ីរននáž&#x;áž˜áž¸áž€ážśážšáž“ážšáž„ 13đ?‘Ľ áž… ážž ងទវញបវន 5 13 លរម áž&#x;ážť វណá&#x;’ážŽážœážˇáž…ážˇáž?á&#x;’ážš | III. áž&#x;áž˜áž¸áž€ážśážš

đ?‘Ľ

12 + 13

đ?‘Ľ

=1 173


5 2 13

áž… ážž áž„áž˜ážśáž“ -

ចបឞ â&#x;š

-

ចបឞ

5 đ?‘Ľ នឡង 13 đ?‘Ľ 2 đ?‘Ľ 5 5 12 12 2 đ?‘Ľ < 2 áž…áž“á&#x; á&#x;‡ 13 < 13 ; 13 < 13 5 đ?‘Ľ 12 đ?‘Ľ 5 2 12 2 + < + =1 13 13 13 13 đ?‘Ľ đ?‘Ľ 5 12 5 2 12 2 đ?‘Ľ > 2 áž…áž“á&#x; á&#x;‡ 13 + 13 > 13 + 13

+

12 2 13

= 1 á&#x;” áž˘áž“ážťáž‚áž˜áž“á&#x;? đ?‘Ś1 =

đ?‘Ś2 =

12 đ?‘Ľ 13

áž‡ážśáž˘áž“ážťáž‚áž˜áž“á&#x;?á&#x;’ážťá&#x; á&#x;‡á&#x;” ដសចá&#x;’áž“á&#x; á&#x;‡

=1

ដសចá&#x;’áž“á&#x; á&#x;‡áž&#x;áž˜áž¸áž€ážśážšáž˜ážśáž“ážš ážşáž&#x;ដáž?áž˜áž˝ áž‚áž?á&#x;‹ áž‚ ážş đ?‘Ľ = 2 á&#x;” 441. 3

442. 0

443. 3

444. 15 áž… ážž

a = 3á&#x;”

IV. áž?á&#x;’ážšáž–á&#x;?áž“áž&#x; áž’ áž˜áž¸áž€ážśážš 445. 448.

đ?‘Ž

66/5

8; 1

; đ?‘Ž

á&#x;’ážśá&#x;† áž… á&#x; á&#x;‡ đ?‘Ž ≠−1; 0; 1 á&#x;” 446.

72/5

449.

450.

3/2; 1/2

4; −1/2 452. 0,1; 2 ; 100; −1 454.

2; 1/6

455.

456.

2; 4 ; 4; 2

457.

đ?‘Ž2 ; đ?‘Ž ; đ?‘Ž; đ?‘Ž2

453.

3; 9 ; 9; 3

462.

5; 1/2

3; −1 ;

4

451.

3; 1

2; 10 ; 10; 2

á&#x;’ážśá&#x;† áž… á&#x; á&#x;‡ đ?‘Ž ∈ −∞; −2 âˆŞ −2; −1 á&#x;”

2

460.

3; 2 464.

64; 1/4

−2; −2 ; 2; 2

461.

−2; 4

V. ážœážˇáž&#x;áž˜áž—ážśáž– 465.ď ° áž… ážž áž„áž˜ážśáž“ 2300 = 8100 < 3200 = 9100 á&#x;” 466.ď ° áž… ážž áž„áž˜ážśáž“

đ?‘›+1 đ?‘›

1

đ?‘›+2

1

= 1 + đ?‘› > đ?‘›+1 = 1 + đ?‘›+1á&#x;” á&#x;’ážśá&#x;† áž… á&#x; á&#x;‡ đ?‘› > 1 áž… ážž áž„áž˜ážśáž“ đ?‘›+1 đ?‘›+1 đ?‘›+2 log đ?‘› > log đ?‘›+1 > log đ?‘›+1 đ?‘› đ?‘› đ?‘›+1 â&#x;š log đ?‘› đ?‘› + 1 − log đ?‘› đ?‘› > log đ?‘›+1 (đ?‘› + 2) − log đ?‘›+1 (đ?‘› + 1) áž… ážž ងទវញបវន log đ?‘› đ?‘› + 1 > log đ?‘›+1 (đ?‘› + 1) á&#x;”

467.ď ° áž… 174

9/2; 1/2

á&#x;’ážśá&#x;† áž… á&#x; á&#x;‡ đ?‘Ž ∈ 0; 1 âˆŞ (1; ∞); 459.

463.

4

á&#x;’ážśá&#x;† áž… á&#x; á&#x;‡ đ?‘Ž ∈ 0; 1 âˆŞ 1; ∞ á&#x;”

9đ?‘Ž/2; đ?‘Ž/2 ; đ?‘Ž 2 ; 9đ?‘Ž/2

đ?‘Ž + 1 2 ; −(đ?‘Ž + 1) ; − đ?‘Ž + 1 ; đ?‘Ž + 1 458.

447.

1/2; 1/2

27 > 25 áž…áž“á&#x; á&#x;‡ log 8 27 = log 4 9 > log 9 25á&#x;” á&#x;˘. áž˘áž“ážťáž‚áž˜áž“á&#x;?អឝឡចáž&#x;á&#x;’áž”á&#x;‰ážźážŽá&#x;’áž„á&#x;‹áž&#x;á&#x;’áž&#x;á&#x;‚áž›-áž›á&#x;„កវរីរ | លរម áž&#x;ážť វណá&#x;’ážŽážœážˇáž…ážˇáž?á&#x;’ážš

12; 4


468.ď ° áž?ážśáž˜áž›áž€áž ážŽá&#x;ˆáž&#x;ឝី ចមរទី áž… ážž áž„á&#x;„á&#x;’áž&#x;នមáž?áž?ážś áž&#x;á&#x; វី áž? đ?‘Ľđ?‘– ជវáž&#x;á&#x; វី áž?ចកឞនá&#x;” ដសចá&#x;’áž“á&#x; á&#x;‡ áž&#x;á&#x; វី áž? ln đ?‘Ľđ?‘– កá&#x;?ជវ áž&#x;á&#x; វី áž?ចកឞនដដរá&#x;” áž?ážśáž˜ážœ ឡáž&#x;áž˜áž—ážśáž– đ?‘›

đ?‘–=1

đ?‘›

1 �� ln �� ≼ �

đ?‘›

đ?‘Ľđ?‘– đ?‘–=1

đ?‘›

đ?‘› đ?‘Ľ

�� � ≼

â&#x;ş

ច ឞ ងទវញបវន

đ?‘–=1

đ?‘›

ln đ?‘Ľđ?‘– 1 đ?‘›

đ?‘Ľ đ?‘Ľđ?‘– đ?‘–

â&#x;š ln

đ?‘–=1

đ?‘–=1

đ?‘›

1 ≼ �

đ?‘›

đ?‘Ľđ?‘– ln đ?‘–=1

đ?‘Ľđ?‘– đ?‘–=1

đ?‘› đ?‘–=1 đ?‘Ľ đ?‘–

đ?‘Ľđ?‘– đ?‘–=1

469.ď ° ážœ ឡáž&#x;áž˜áž—ážśáž–áž&#x;áž˜áž˜ážźáž›áž“ážšáž„ đ?‘Ľ 2 + 2đ?‘Śđ?‘§ ln đ?‘Ľ + đ?‘Ś 2 + 2đ?‘§đ?‘Ľ ln đ?‘Ś + đ?‘§ 2 + 2đ?‘Ľđ?‘Ś ln đ?‘§ ≼ đ?‘Ľđ?‘Ś + đ?‘Śđ?‘§ + đ?‘§đ?‘Ľ ln đ?‘Ľ + ln đ?‘Ś + ln đ?‘§ â&#x;ş đ?‘Ľ − đ?‘Ś đ?‘Ľ − đ?‘§ ln đ?‘Ľ + đ?‘Ś − đ?‘§ đ?‘Ś − đ?‘Ľ ln đ?‘Ś + đ?‘§ − đ?‘Ľ đ?‘§ − đ?‘Ś ln đ?‘§ ≼ 0 áž… ážž áž„áž˜ážśáž“ ln đ?‘Ľ , ln đ?‘Ś , ln đ?‘§ > 0 áž…ážš á&#x; á&#x;‡ đ?‘Ľ, đ?‘Ś, đ?‘§ > 1 á&#x;” ážœ ឡáž&#x;áž˜áž—ážśáž–áž ážśáž„áž…áž›ážžáž˜ážśáž“áž›áž€áž ážŽá&#x;ˆáž&#x;ឝី ចមរទី (ជវá&#x;†áž“áž&#x; áž˝ đ?‘Ľáž…

đ?‘Ś ážšážşáž…

đ?‘§ áž‚á&#x;’áž˜áž˜áž“áž˘á&#x; ី ដរបរបួល)á&#x;” ដសចá&#x;’áž“á&#x; á&#x;‡ áž… ážž áž„

á&#x;„á&#x;’áž&#x;នមáž?áž?ážś đ?‘Ľ ≼ đ?‘Ś ≼ đ?‘§ á&#x;” ដសចá&#x;’áž“á&#x; á&#x;‡ đ?‘§ − đ?‘Ľ đ?‘§ − đ?‘Ś ln đ?‘§ ≼ 0 á&#x;” បនវបá&#x;‹ មកចទá&#x;€áž? áž˘áž“ážťáž‚áž˜áž“á&#x;? ln

ជវ

áž˘áž“ážťáž‚áž˜áž“á&#x;?ចកឞន ចលឞ â„?+∗ áž… ážž ងទវញបវន đ?‘Ľ − đ?‘Ś đ?‘Ľ − đ?‘§ ln đ?‘Ľ ≼ đ?‘Ś − đ?‘§ đ?‘Ľ − đ?‘Ś ln đ?‘Ś áž…ážš á&#x; á&#x;‡ កáž?ážśážśáž“ážˇáž˜áž˝ á&#x;— áž&#x;ážť ទដធ áž? ážœ ážˇáž‡áž‡áž˜ážśáž“ážš ážşáž&#x;ážź áž“áž™ áž… ážž កáž?ážśážśáž“ážˇáž˜áž˝ á&#x;—áž…á&#x;…áž˘áž„áž‚áž ážśáž„áž…ážœá&#x; áž„ áž’ážśá&#x;† ជវងកáž?ážśážśáž“ážˇáž˜áž˝ á&#x;—áž“áž“ អងគហវងវវវá&#x;†á&#x;”

VI. ážœážˇáž&#x;áž˜áž¸áž€ážśážš 470. −∞; −2,5 âˆŞ 0; ∞

471. −1 − 2; −2 âˆŞ 0; 2 − 1

472. −∞; 2 − 2 âˆŞ 2 + 2; ∞

473. −∞; −2 âˆŞ 5/8; ∞

474. −7; − 35 âˆŞ 5; 35

475. 2; 7

476. − 5; −2 âˆŞ 1; 5

477. −1; 1 âˆŞ 3; ∞

478.

1− 5 1+ 5 ; 2 2

479. −1; 1 + 2 2

480. 2; 7 âˆŞ (22; 27) 482. 1; 11/10

481. 2; 4 483. −1; 4

484. −4; −1 − 3 âˆŞ 0; 3 − 1

485. 3; 5

487. −∞; −1 âˆŞ (2; ∞)

488. 0; 10−4 âˆŞ [10; ∞)

1

489. 0; 2 âˆŞ [ 2; ∞) លរម áž&#x;ážť វណá&#x;’ážŽážœážˇáž…ážˇáž?á&#x;’ážš | VI. ážœážˇáž&#x;áž˜áž¸áž€ážśážš

490. 0; 1 âˆŞ

486. 2; 5 3; 9 175


1 ; 3

491. 1/16; 1/8 ∪ 8; 16

492. 0; 1/ 27 ∪

493. 1/16; 1/4 ∪ 1/2; 2

494. 0; 1/2 ∪ 32; ∞

3 ;∞ 2 3− 5 3+ 5 log 2 2 ; log 2 2

495. log 2

496. −1; ∞

497.

498. −∞; 1 − log 3 5 3

499. −∞; 0 ∪ log 2 3 ; ∞ 502. log 2 (5 + 33) − 1; ∞ 504. log 5

243 ∪ 27; ∞

500. 0,01; ∞

501. −∞; −1 ∪ −0,1; 0

503. −∞; 0 ∪ log 6 5 ; 1

2 + 1 ; log 5 3

505. −∞; 0 ∪ 0; ∞ 83

506. 2; ∞

507. 28/3; ∞ 508. log 3 19 ; ∞

509. −3; − 6 ∪

510. −1/2; 2

6; 3

1+ 17 2

511. −3; 1

512. −4;

513. 0; 2 ∪ 4; ∞

514. 1000; ∞ 515. 0;

516.

1+4𝑎 2

1+

517. 0; 𝑎

2 5

; ∞ ្ាំ ច េះ 𝑎 ∈ (1; ∞); 1;

1+

1+4𝑎 2

1 4

∪ 4; ∞

្ាំ ច េះ 𝑎 ∈ (0; 1)

2

1

∪ 𝑎3 ; 𝑎2 ∪ (𝑎 ; ∞)

518. 1/𝑎; 𝑎4 ្ាំ ច េះ 𝑎 ∈ (1; ∞); 𝑎4 ; 1/𝑎 ្ាំ ច េះ 𝑎 ∈ (0; 1) 519. log 𝑎 4 + 16 + 𝑎2 ; 3 log 𝑎 2 ្ាំ ច េះ 𝑎 ∈ (0; 1); log 𝑎 4 + 16 + 𝑎2 ; ∞ ្ាំ ច េះ 𝑎 ∈ (1; ∞) 520. 3; 4 ∪ 5; ∞

521. 1; 2

522. 0; 1/2 ∪ 1; 2 ∪ (3; 6) 523. −3; −2 ∪ −1; 0 ∪ (1; 3) 4

23

3

524. − 3 ; − 22 526.

525. −2; − 2 ∪ −1; 3

1 1 ; 5 2

528. 0;

527. −3; −1 13−3 2

∪ 1; ∞

530. 3; 5 − 3 ∪ 7; ∞

529. −1; 1/2 ∪ 1; 2 ∪ 2;

7 2

531. 1; 2

532. 0; 3 ∪ 4; 6 ∪ 6; 12 ∪ (14; ∞) 1 ;4 2

533. 0; 4

534. 2; 4

535.

536. 1; 2

537. 0; 2

538. 1/ 5; 1 ∪ 3; ∞

539. −1; ∞

176

២. អនុគមន៍អុិចស្ប៉ូណ្ង់ស្សែល-លោការីរ | លឹម សុ វណ្ណវិចិត្រ


VII. ត្រព័នវធ ស ិ មីការ 540. 8

541. 4

542. 0; 1/𝑎4 ្ាំ ច េះ 𝑎 ∈ 1; ∞ , 0; 𝑎8

្ាំ ច េះ 𝑎 ∈ 0; 1 ។

លឹម សុ វណ្ណវិចិត្រ | VII. ត្រព័នវធ ស ិ មីការ

177


178

២. អនុគមន៍អុិចស្ប៉ូណ្ង់ស្សែល-លោការីរ | លឹម សុ វណ្ណវិចិត្រ


á&#x;Ł. áž?á&#x;’រី កá&#x;„ážŽáž˜ážśáž?á&#x;’ážš

គណá&#x;’áž“ážś 543. ក) 2 − đ?‘Ž2 ; áž ) 1 −

đ?‘Ž 2 −1

2

2

sin � + cos �

ď‚˜ sin4 đ?›ź + cos 4 đ?›ź = sin2 đ?›ź + cos2 đ?›ź 2

2

− 2 sin đ?›ź cos đ?›ź 2 ; đ?‘Ž2 =

= 1 + 2 sin � cos �.

544. ក) đ?‘?2 − 2; áž ) đ?‘?3 − 3đ?‘? 545. 546.

4 3+3 ; 10 3 1−đ?‘? 2 −đ?‘? 2

1 sin đ?›˝ 2

ď ° áž?ážśáž„ 40° + đ?›ź = đ?›˝ á&#x;” cos 70° + đ?›ź = cos 30° + đ?›˝ =

á&#x;” ដá&#x;„áž™ 0 < đ?›ź < 45° ដ á&#x;„á&#x;‡ cos đ?›˝ > 0 ដសដá&#x;’áž“á&#x;„á&#x;‡ cos đ?›˝ = 1 − đ?‘? 2 á&#x;” 117

3 cos đ?›˝ 2

−

44

547. 1073/1105; 548. sin 3đ?›ź = − 125 , cos 3đ?›ź = 125 , tan 3đ?›ź = −117/44. 549. tan 2đ?›ź; 550. − tan đ?›ź ; 551. tan 2đ?›ź; 552. cosec đ?›ź 553. −1/2. ď °

đ?œ‹ 2đ?œ‹ 4đ?œ‹ 6đ?œ‹ 2 sin 7 2đ?œ‹ 4đ?œ‹ 6đ?œ‹ cos + cos + cos = Ă— cos + cos + cos đ?œ‹ 7 7 7 7 7 7 2 sin 7 3đ?œ‹ đ?œ‹ 5đ?œ‹ 3đ?œ‹ 5đ?œ‹ đ?œ‹ sin − sin + sin − sin + sin đ?œ‹ − sin −sin 7 7 7 7 7 7 = = đ?œ‹ đ?œ‹ 2 sin 7 2 sin 7 1 =− 2

554. 1. ď‚˜ áž”á&#x;† លលងជវ

sin

13 đ?œ‹âˆ’sin 14 đ?œ‹ 2 sin 14

đ?œ‹ 14

−

2 tan �/2 �/2

555. 1 3 ď ° sin đ?›ź + cos đ?›ź = 1+tan 2

1−tan 2 đ?›ź/2 đ?›ź/2

+ 1+tan 2

= 1,4 á&#x;” ដសដá&#x;’áž“á&#x;„á&#x;‡ 2,4 tan2 đ?›ź/2 −

2 tan đ?›ź 2 + 0,4 = 0 â&#x;š tan đ?›ź/2 = 1 3 ; tan đ?›ź/2 = 1 2 á&#x;” ដយឞ áž„áž˜ážśáž“ 0 < đ?›ź 2 < đ?œ‹ 8 ; tan đ?œ‹ 8 = 2 − 1 < 1/2 á&#x;” លរម áž&#x;ážť វណá&#x;’ážŽážœážˇáž…ážˇáž?á&#x;’ážš | I. គណá&#x;’áž“ážś

179


556. đ?œ‹ ď ° áž?ážśáž˜áž&#x;មម áž?ក ឡ មម ដយឞ ងទវញបវន 0 < đ?›ź + đ?›˝ + đ?›ž <

3đ?œ‹ 2

,0 <

�+� 2

đ?œ‹ 2

đ?›˝ 2

� 2

< , tan tan < 1

á&#x;” ដសដá&#x;’áž“á&#x;„á&#x;‡ 1 đ?›ź 2 cot 2 + đ?›˝ đ?›ž đ?›ź đ?›ź 3 tan 2 + tan 2 3 tan 2 + cot 2 đ?›˝+đ?›ž đ?›ź tan = = = cot 1 đ?›ź 2 đ?›˝ đ?›ž 2 2 1 − tan 2 tan 2 1 − 3 cot 2 đ?›ź đ?›ź 3 tan 2 + cot 2 đ?›˝+đ?›ž đ?›ź â&#x;š tan − cot = 0 2 2 đ?›ź+đ?›˝+đ?›ž â&#x;š −cos =0 2 đ?›ź+đ?›˝+đ?›ž đ?œ‹ â&#x;š = 2 2 557.

2− 2 2

ď‚˜ cos 292°30′ = sin 22°30′ =

558. 4 ď ° cosec 10° − 3 sec 10° =

2

1 cos 2

1−cos 45° 2

10塉ˆ’

3 sin 2

10°

sin 10° cos 10°

; =

4 sin 30°âˆ’10° sin 20°

=4

559. 3 ď ° 2 cos 40° − cos 20° cos 40° − 2 sin 30° sin 10° sin 50° − sin 10° 2 cos 30° sin 20° = = = sin 20° sin 20° sin 20° sin 20° = 3 560. 4 ď ° ដយឞ áž„áž˜ážśáž“ sec 5° cos 40° −2 2 sin 10° 2 sin 35° − − 2 sin 5° 1 sin 10° cos 40° sin 10° = −2 2 2 sin 35° sin 10° − 2 − cos 5° sin 5° = −2 2 2 sin 35° sin 10° − sin 5° − 2 cos 40° cos 5° = −2 2 − 2cos 45° − sin 5° + cos 25° − cos 35° = −2 2 − 2 − sin 5° + 2 sin 30° Ă— sin 5° = 4 561. 3/4 ď ° ដយឞ áž„áž˜ážśáž“

180

á&#x;Ł. áž?á&#x;’រីកá&#x;„ណá&#x;’áž˜ážśáž?á&#x;’ážš | លរម áž&#x;ážť វណá&#x;’ážŽážœážˇáž…ážˇáž?á&#x;’ážš


cos2 73° + cos 2 47° + cos 73° cos 47° 1 + cos 146° 1 + cos 94° 1 = + + cos 120° + cos 26° 2 2 2 1 3 = + cos 146° + cos 94° + cos 26° 2 2 1 3 120° 68° = − cos 180° − 146° + 2 cos cos 2 2 2 2 1 3 3 = − cos 34° + cos 34° = 2 2 4 1 562. − 2 ď ° ដយឞ áž„áž˜ážśáž“ sin 6° − sin 66° + sin 78° − sin 42° = 2 cos 60° sin 18° − 2 sin 30° cos 36° 2 cos 36° sin 18° 2 cos 36° sin 36° = sin 18° − sin 54° = − cos 18° = − cos 18° 2 cos 18° sin 72° 1 =− =− 2 cos 18° 2 1−cos 40° 563. − 1 2 ; 564. 1 ; 565. 0 ď‚˜ tan2 20° = 1+cos 40° 566. 1/5 ď ° ដយឞ áž„áž˜ážśáž“ cos 108° cos 36° sin2 36° sin2 72° 1 = 1 − cot 36° cot 72° 2 cos 36° cos 72° 2.2 sin 36° 2 2 = 1 − cot 36° cot 72° sin 144° =1 − 4 cot 2 36° . cot 2 72°; â&#x;š 5cot 2 36° cot 2 72° = 1 â&#x;š cot 2 36° cot 2 72° 1 = 5 567. 433ď °ážŠáž™ážž áž„áž˜ážśáž“ đ?œ‹ đ?œ‹ 1 tan2 3 = tan2 = 18 6 3 5đ?œ‹ 5đ?œ‹ 1 2 2 tan 3 = tan = 18 6 3 7đ?œ‹ 7đ?œ‹ 1 tan2 3 = tan2 = 18 6 3 đ?œ‹ 5đ?œ‹ 7đ?œ‹ 1 2 ដសដá&#x;’áž“á&#x;„á&#x;‡ đ?‘Ľ1 = 18 ; đ?‘Ľ2 = 18 ; đ?‘Ľ3 = 18 ជវរ ážşáž&#x;បីននáž&#x;áž˜áž¸áž€ážśážš tan 3đ?‘Ľ = 3 á&#x;” áž&#x;áž˜áž¸áž€ážśážšážŠáž“á&#x;„á&#x;‡áž&#x;áž˜áž˜ážźáž›áž“ážšáž„ cot 2 36° cot 2 72° − 1 + 1 = 1 +

3 tan đ?‘Ľ − tan3 đ?‘Ľ 1 − 3 tan2 đ?‘Ľ

2

tan6 đ?‘Ľ − 6 tan4 đ?‘Ľ + 9 tan2 đ?‘Ľ 1 = 9 tan4 đ?‘Ľ − 6 tan2 đ?‘Ľ + 1 3 â&#x;ş 3 tan6 đ?‘Ľ − 25 tan4 đ?‘Ľ + 33 tan2 đ?‘Ľ − 1 = 0 đ?œ‹ 5đ?œ‹ 7đ?œ‹ áž?ážśáž„ đ?‘Ą = tan2 đ?‘Ľ ≼ 0 ដ á&#x;„á&#x;‡ đ?‘Ą1 = tan2 18 ; đ?‘Ą2 = tan2 18 ; đ?‘Ą3 = tan2 18 ជវá&#x;’á&#x;† ដលឞយននáž&#x;áž˜áž¸áž€ážśážš =

1 3

â&#x;ş

3đ?‘Ą 3 − 27đ?‘Ą 2 + 33đ?‘Ą − 1 = 0 á&#x;” áž?ážśáž˜áž‘á&#x;’ážšážšáž&#x;áž?ីបរលវá&#x;‚áž? ដយឞ ងទវញបវន លរម áž&#x;ážť វណá&#x;’ážŽážœážˇáž…ážˇáž?á&#x;’ážš | I. គណá&#x;’áž“ážś

181


27 = 9; 3 33 đ?‘Ą1 đ?‘Ą2 + đ?‘Ą2 đ?‘Ą3 + đ?‘Ą3 đ?‘Ą1 = = 11 3 1 đ?‘Ą1 đ?‘Ą2 đ?‘Ą3 = 3 đ?‘Ą1 + đ?‘Ą2 + đ?‘Ą3 =

ដសដá&#x;’áž“á&#x;„á&#x;‡ đ??´ = đ?‘Ą13 + đ?‘Ą23 + đ?‘Ą33 = đ?‘Ą1 + đ?‘Ą2 + đ?‘Ą3

3

− 3 đ?‘Ą1 + đ?‘Ą2 + đ?‘Ą3 đ?‘Ą1 đ?‘Ą2 + đ?‘Ą2 đ?‘Ą3 + đ?‘Ą3 đ?‘Ą1 + 3đ?‘Ą1 đ?‘Ą2 đ?‘Ą3 1 = 93 − 3.9.11 + 3. = 433 3 568. ď °ážŠáž™ážž áž„áž˜ážśáž“ đ?‘˜đ?œ‹ sin 1999đ?‘Ľ = 0 â&#x;şđ?‘Ľ= ; đ?‘˜ = 1,2, ‌ ,1998 1 0<đ?‘Ľ<đ?œ‹ 1999 1 2 sin đ?‘Ľ + cos 2đ?‘Ľ + cos 4đ?‘Ľ + â‹Ż + cos 1998đ?‘Ľ 2 = sin đ?‘Ľ + sin −đ?‘Ľ + sin 3đ?‘Ľ + sin −3đ?‘Ľ + sin 5đ?‘Ľ + â‹Ż + sin −1997đ?‘Ľ + sin 1999đ?‘Ľ = sin 1999đ?‘Ľ sin 1999đ?‘Ľ 1 â&#x;š = 2 + cos 2đ?‘Ľ + cos 4đ?‘Ľ + â‹Ż + cos 1998đ?‘Ľ , ∀đ?‘Ľ ∈ 0, đ?œ‹ 2 sin đ?‘Ľ 2 ដយឞ ងដរងáž?ážś á&#x;’á&#x;† ដ á&#x;„á&#x;‡áž‘á&#x;’ážšáž”á&#x;‹ á&#x;’á&#x;† áž“áž“ áž˝ ážšáž?á&#x;‹ ážœ ឡជá&#x;’áž‡áž˜ážśáž“ đ?‘š ដយឞ áž„áž˜ážśáž“ cos đ?‘šđ?‘Ľ ជវពហឝ ធវដឺដទá&#x;’ករីđ?‘š áž“áž“ cos đ?‘Ľ ដហឞ áž™ áž›ážŠáž›áž˜ážśáž“ážŠáž˜ážšážťážŽážšáž”áž&#x;á&#x;‹ áž?ួដដឺ áž‘á&#x;’ករីđ?‘š ដáž&#x;មឞ 2đ?‘š −1 (áž–áž ážť áž’ážś

)á&#x;” ដសដá&#x;’áž“á&#x;„á&#x;‡

1 + cos 2đ?‘Ľ + cos 4đ?‘Ľ + â‹Ż + cos 1998đ?‘Ľ 2 ជវពហឝ ធវដឺដទá&#x;’ករី 1998 áž“áž“ cos đ?‘Ľ ដហឞ áž™áž›ážŠáž›áž˜ážśáž“ážŠáž˜ážšážťážŽážšáž”áž&#x;á&#x;‹ áž?ួដដឺ áž‘á&#x;’ក1998 ដáž&#x;មឞ 2.21998−1 = 2

21998 á&#x;”

(3)

áž– ី(á&#x;Ą)(á&#x;˘)នឡង(á&#x;Ł) ដយឞ ងទវញបវន sin 1999đ?‘Ľ 1 = 2 + cos 2đ?‘Ľ + cos 4đ?‘Ľ + â‹Ż + cos 1998đ?‘Ľ sin đ?‘Ľ 2 1998 đ?‘˜đ?œ‹ 1998 =2 cos đ?‘Ľ − cos 1999 áž›áž?ដយឞ áž„áž˜ážśáž“ 999đ?œ‹ − cos 1999 ;

1998đ?œ‹ cos 1999

=

đ?‘˜=1 đ?œ‹ 1997đ?œ‹ − cos 1999 ; cos 1999

ដ á&#x;„á&#x;‡ sin 1999đ?‘Ľ = 21998 sin đ?‘Ľ

182

2đ?œ‹

= − cos 1999 ; ‌ ; cos

á&#x;Ł. áž?á&#x;’រីកá&#x;„ណá&#x;’áž˜ážśáž?á&#x;’ážš | លរម áž&#x;ážť វណá&#x;’ážŽážœážˇáž…ážˇáž?á&#x;’ážš

1998

cos đ?‘Ľ − cos đ?‘˜=1

đ?‘˜đ?œ‹ 1999

1000 đ?œ‹ 1999

=


999 1998

cos 2 đ?‘Ľ − cos2

=2

đ?‘˜=1

đ?‘˜đ?œ‹ 1999

(4)

កនង ážť (á&#x;¤) ដយឞ ងយក đ?‘Ľ = đ?œ‹/3 ដយឞ ងទវញបវន 1999đ?œ‹ 999 sin 3 1 đ?‘˜đ?œ‹ 1998 − cos2 đ?œ‹ =2 4 1999 sin đ?‘˜=1 3 đ?œ‹ đ?‘˜đ?œ‹ 999 sin 3 + 666đ?œ‹ 1 − 4 cos 2 1999 1998 =2 đ?œ‹ 22 sin đ?‘˜=1 3 999 đ?‘˜đ?œ‹ 1 − 4cos 2 =1 1999 đ?‘˜=1

ដសដá&#x;’áž“á&#x;„á&#x;‡ đ?‘ƒ = 1 á&#x;”

II. áž&#x;áž˜áž—ážśáž– 569. 570. 571. ď‚˜ đ?›ź + đ?›˝ + đ?›ž = đ?œ‹ â&#x;š đ?›ź + đ?›˝ = đ?œ‹ − đ?›ž á&#x;” 572. 573. 574. 575. 576. 577. 578. 579. 580. 581. 582. 583. 584. 585. 586. 587. 588. 589. 590. 591. ď ° ដយឞ áž„áž˜ážśáž“ 16 sin 10° sin 30° sin 50° sin 70° = 8 cos 80° cos 40° cos 20° 2sin 20° cos 20° 2 sin 40° cos 40° = 4 cos 80° cos 40° = 2 cos 80° sin 20° sin 20° 2 cos 80° sin 80° sin 160° sin 180° − 160° = = = =1 sin 20° sin 20° sin 20° 592. ď‚˜ ដទá&#x;’បឞរប ážź មនáž? 2 cos 2 đ?›ź = 1 + cos 2đ?›ź , 2 sin2 đ?›ź = 1 − cos 2đ?›ź 593.594. đ?›ź

� 2

595. ď‚˜ ដទá&#x;’បឞរប ážź មនáž? 1 Âą sin đ?›ź = sin 2 Âą cos 2 596. ď ° áž?ážśáž„ đ?‘…đ?‘› =

2 + 2 + â‹Ż + 2 á&#x;” ដយឞ áž„áž˜ážśáž“ đ?‘›

đ?‘…1 = 2 = 2 cos លរម áž&#x;ážť វណá&#x;’ážŽážœážˇáž…ážˇáž?á&#x;’ážš | II. áž&#x;áž˜áž—ážśáž–

đ?œ‹ 22 183


đ?‘…2 =

2+ 2=

‌ đ?‘…đ?‘› = 2 cos 597. ď ° ដយឞ áž„áž˜ážśáž“

2 + 2 cos

đ?œ‹ đ?œ‹ = 2 cos 3 2 2 2

đ?œ‹ 2đ?‘›+1

cos 2đ?‘Ľ = 2 cos 2 đ?‘Ľ − 1 cos 3đ?‘Ľ = cos 2đ?‘Ľ + đ?‘Ľ = cos 2đ?‘Ľ cos đ?‘Ľ − sin 2đ?‘Ľ sin đ?‘Ľ = 2 cos2 đ?‘Ľ − 1 cos đ?‘Ľ − 2 sin2 đ?‘Ľ cos đ?‘Ľ = 4 cos3 đ?‘Ľ − 3 cos đ?‘Ľ ‌ cos đ?‘›đ?‘Ľ = đ?‘‡đ?‘› cos đ?‘Ľ ដយឞ ងនរងបងá&#x;’ហវញáž?ážś cos đ?‘› + 1 đ?‘Ľ = đ?‘‡đ?‘›+1 cos đ?‘Ľ á&#x;” ដយឞ áž„áž˜ážśáž“ cos đ?‘› + 1 đ?‘Ľ = cos đ?‘›đ?‘Ľ cos đ?‘Ľ − sin đ?‘›đ?‘Ľ sin đ?‘Ľ cos đ?‘›đ?‘Ľ = cos đ?‘› − 1 đ?‘Ľ cos đ?‘Ľ + sin đ?‘› − 1 đ?‘Ľ sin đ?‘Ľ â&#x;š sin đ?‘› − 1 đ?‘Ľ sin đ?‘Ľ = cos đ?‘›đ?‘Ľ − cos đ?‘› − 1 đ?‘Ľ cos đ?‘Ľ sin đ?‘›đ?‘Ľ = sin đ?‘› − 1 đ?‘Ľ cos đ?‘Ľ − sin đ?‘Ľ cos đ?‘› − 1 đ?‘Ľ â&#x;š sin đ?‘›đ?‘Ľ sin đ?‘Ľ = sin đ?‘› − 1 đ?‘Ľ cos đ?‘Ľ sin đ?‘Ľ − sin đ?‘Ľ cos đ?‘› − 1 đ?‘Ľ sin đ?‘Ľ = cos đ?‘›đ?‘Ľ − cos đ?‘› − 1 đ?‘Ľ cos đ?‘Ľ − 1 − cos2 đ?‘Ľ cos đ?‘› − 1 đ?‘Ľ = cos đ?‘›đ?‘Ľ cos đ?‘Ľ − cos đ?‘› − 1 đ?‘Ľ cos đ?‘Ľ − cos đ?‘› − 1 đ?‘Ľ + cos 2 đ?‘Ľ cos đ?‘› − 1 đ?‘Ľ â&#x;š cos đ?‘› + 1 đ?‘Ľ = cos 2 đ?‘Ľ + 1 − cos 2 đ?‘Ľ cos đ?‘› − 1 đ?‘Ľ = 1 + cos đ?‘Ľ − cos2 đ?‘Ľ đ?‘‡đ?‘›âˆ’1 cos đ?‘Ľ = đ?‘‡đ?‘›+1 cos đ?‘Ľ 598. ď ° ដយឞ áž„áž˜ážśáž“ 1 đ?‘Ľ đ?‘Ľ 2đ?‘Ľ 3+2 2 = 2−1 +3 â&#x;ş 2+1 = đ?‘Ľ + 3 (1) 2+1 áž?ážśáž„ 2đ?‘Ą =

đ?‘Ľ

> 0 á&#x;” áž&#x;áž˜áž¸áž€ážśážš(1) áž&#x;áž˜áž˜ážźáž›áž“ážšáž„ 1 1 4đ?‘Ą 2 = + 3 â&#x;ş 4đ?‘Ą 3 − 3đ?‘Ą = (2) 2đ?‘Ą 2 1 áž˘áž“ážťážšáž˜áž“á&#x;? đ?‘“ đ?‘Ą = 4đ?‘Ą 3 − 3đ?‘Ą − áž˜ážśáž“ áž›áž&#x;á&#x;‚ដកវងកវáž?á&#x;‹ អá&#x;?កá&#x;‚អវបá&#x;‹ áž&#x;ឝី áž&#x;áž‘á&#x;’áž?áž„á&#x;‹ á&#x;’á&#x;† áž“á&#x;’ ážť បីáž&#x;áž?ឡáž?ដá&#x;…កនង ážť á&#x;’ដ áž› á&#x;„á&#x;‡ 2+1

2

−1; 1 á&#x;” áž˜ážśáž“áž“á&#x;?áž™áž?ážśáž&#x;áž˜áž¸áž€ážśážš(á&#x;˘)áž˜ážśáž“á&#x;’á&#x;† ដលឞយ đ?‘Ą ∈ (−1; 1) á&#x;” áž?ážśáž„ đ?‘Ą = cos đ?›ź លដលđ?›ź ∈ 0; đ?œ‹ á&#x;”

y 1 0 -1.5

-1

-0.5

-1 -2

184

á&#x;Ł. áž?á&#x;’រីកá&#x;„ណá&#x;’áž˜ážśáž?á&#x;’ážš | លរម áž&#x;ážť វណá&#x;’ážŽážœážˇáž…ážˇáž?á&#x;’ážš

0

0.5

1

1.5


ដយឞ ងទវញបវន 1 1 4 cos3 đ?›ź − 3 cos đ?›ź = â&#x;š cos 3đ?›ź = 2 2 đ?œ‹ 2đ?‘˜đ?œ‹ â&#x;š đ?›ź=Âą + đ?‘˜âˆˆâ„¤ 9 3 đ?œ‹ 5đ?œ‹ 7đ?œ‹ đ?œ‹ ដá&#x;„áž™ đ?›ź ∈ 0; đ?œ‹ ដ á&#x;„á&#x;‡ đ?›ź = 9 ; 9 ; 9 á&#x;” ដសដá&#x;’áž“á&#x;„á&#x;‡áž&#x;áž˜áž¸áž€ážśážšáž˜ážśáž“ážš ážşáž&#x;បីរ ážş đ?‘Ą1 = cos 9 ; đ?‘Ą2 = cos

5đ?œ‹ 9

; đ?‘Ą3 = cos

7đ?œ‹ 9

đ?œ‹

á&#x;” ដយឞ áž„áž˜ážśáž“ đ?‘Ą2 ; đ?‘Ą3 < 0 á&#x;” ដសដá&#x;’áž“á&#x;„á&#x;‡áž˜ážśáž“áž›áž? đ?‘Ą1 = cos 9 áž›áž?áž˜áž˝áž™ážšáž?á&#x;‹ លដល

ដផទá&#x;€áž„áž•á&#x;’áž‘áž‘áž?á&#x;‹ đ?‘Ą > 0 á&#x;”áž˜ážśáž“áž“á&#x;?áž™áž?ážś

2+1

đ?‘Ľ

= 2đ?‘Ą = 2 cos

đ?œ‹ 9

á&#x;”

599. ď ° ដយឞ áž„áž˜ážśáž“ đ?œ‹ 2đ?œ‹ 3đ?œ‹ 1 2đ?œ‹ 4đ?œ‹ 6đ?œ‹ sin2 + sin2 + sin2 = 1 − cos + 1 − cos + 1 − cos 7 7 7 2 7 7 7 3 1 2đ?œ‹ 4đ?œ‹ 6đ?œ‹ = − cos + cos + cos 2 2 7 7 7 2đ?œ‹ 4đ?œ‹ 6đ?œ‹ áž?ážśáž„ đ??´ = cos 7 + cos 7 + cos 7 ដ á&#x;„á&#x;‡ đ?œ‹ đ?œ‹ 2đ?œ‹ đ?œ‹ 4đ?œ‹ đ?œ‹ 6đ?œ‹ 2 sin . đ??´ = 2 sin cos + 2 sin cos + 2 sin cos 7 7 7 7 7 7 7 đ?œ‹ 3đ?œ‹ đ?œ‹ 5đ?œ‹ 3đ?œ‹ 5đ?œ‹ 2 sin . đ??´ = sin − sin + sin − sin + sin đ?œ‹ − sin 7 7 7 7 7 7 đ?œ‹ đ?œ‹ 1 2 sin . đ??´ = − sin â&#x;š đ??´=− 7 7 2 đ?œ‹ 2đ?œ‹ 3đ?œ‹ 3 1 1 7 â&#x;š sin2 + sin2 + sin2 = − − = 7 7 7 2 2 2 4

III. áž&#x;មីá&#x;„ážš 600. đ?œ‹đ?‘›; 2đ?œ‹đ?‘› Âą 3đ?œ‹/4 đ?‘› ∈ ℤ 601. 2đ?œ‹đ?‘› Âą đ?œ‹ 6 ; 2đ?œ‹đ?‘› + đ?œ‹/2 đ?‘› ∈ ℤ 602. đ?œ‹đ?‘› 2 + −1 603.

đ?œ‹đ?‘› 2đ?œ‹đ?‘› ; 3 3

Âą

đ?‘›

đ?œ‹ 12 đ?‘› ∈ ℤ

1 1 arccos 3 3 2

đ?‘›âˆˆâ„¤

604. đ?œ‹đ?‘› + đ?œ‹/4; 2đ?œ‹đ?‘› Âą đ?œ‹ 3 đ?‘› ∈ ℤ 605. đ?œ‹đ?‘› + −1

đ?‘›+1

đ?œ‹/6 đ?‘› ∈ ℤ

606. 2đ?œ‹đ?‘› Âą đ?œ‹/3 đ?‘› ∈ ℤ 607. đ?œ‹đ?‘› + −1

đ?‘›

arcsin

2 3

đ?‘›âˆˆâ„¤

608. 2đ?œ‹đ?‘› Âą 2đ?œ‹/3 đ?‘› ∈ ℤ លរម áž&#x;ážť វណá&#x;’ážŽážœážˇáž…ážˇáž?á&#x;’ážš | III. áž&#x;មីá&#x;„ážš

185


609. 𝜋 5𝑛 + 2 + 𝜋 2 ; 5𝜋𝑛 𝑛 ∈ ℤ 610. 𝜋𝑛; 2𝜋𝑛 + 𝜋/6 𝑛 ∈ ℤ 611. 𝜋𝑛 + 𝜋/4; 𝜋𝑛 + arctan 3 𝑛 ∈ ℤ 612. 𝜋𝑛 − arctan 1/2 ; 𝜋𝑛 + arctan 2 𝑛 ∈ ℤ 613. 𝜋𝑛 + 𝜋/6; 𝜋𝑛 + 𝜋/3 𝑛 ∈ ℤ 614. 𝜋𝑛 + 3𝜋/4; 𝜋𝑛 + arccot 2 𝑛 ∈ ℤ 615. 𝜋𝑛 + −1

𝑛+1

𝜋/4; 𝜋𝑛 + arccot 2 𝑛 ∈ ℤ

616. 𝜋𝑛 + −1 𝑛 𝜋/4 𝑛 ∈ ℤ 617. 2𝜋𝑛 + arctan 1/2 𝑛 ∈ ℤ 618. 𝜋𝑛/2 + 𝜋/8 𝑛 ∈ ℤ 619. 620. 621. 622. 623.

𝜋𝑛 −1 𝑛 + 2 arcsin 2(2 − 3) 𝑛 ∈ ℤ 2 𝜋𝑛 −1 𝑛 + arcsin(1 − 3 + 2𝑎) 𝑛 ∈ 2 2 𝑛 +1 𝜋𝑛 −1 𝜋 + 2 12 𝑛 ∈ ℤ 2 𝜋𝑛 2𝜋𝑛 1 1 ; ± arccos 𝑛 ∈ ℤ 5 5 5 4 33−3 𝑛 𝜋𝑛 + −1 arcsin 4 𝑛∈ℤ 𝑛

3 1

ℤ ្ំ ដ ោះ 𝑎 ∈ − 2 ; 2

624. 𝜋𝑛 5 + −1 𝜋 20 − 6/5 𝑛 ∈ ℤ 625. 𝜋𝑛 2 + 𝜋/4 𝑛 ∈ ℤ 626. 𝜋 2𝑛 + 1 𝜋 2

𝑛∈ℤ 𝜋

627. 𝜋𝑛 + ; 2𝜋𝑛 ± 3 ; 2𝜋𝑛 ± 𝜋 − arccos 2/3

𝑛∈ℤ

628. 𝜋𝑛 − 𝜋 4 ; 𝜋𝑛 ± 𝜋 3 𝑛 ∈ ℤ 629. 2𝜋𝑛; 2𝜋𝑛 ± 2 𝜋 3 ; 2𝜋𝑛 ± arccos

−1± 5 4

630. 𝜋𝑛 + arctan 5 𝑛 ∈ ℤ 631. 𝜋𝑛 + 𝜋/4 𝑛 ∈ ℤ 632. 𝜋𝑛 − 𝜋/4 𝑛 ∈ ℤ 633. 2𝜋𝑛 − 𝜋 4 ; 2𝜋𝑛 + 𝜋 2 𝑛 ∈ ℤ 634. 𝜋𝑛 + 𝜋 2 ; 𝜋𝑛 + arctan 1/4 𝑛 ∈ ℤ 635. 𝜋𝑛 + 𝜋 4 ; 2𝜋𝑛 ± 2𝜋 3 𝑛 ∈ ℤ 186

៣. ត្រីកោណ្មាត្រ | លឹម សុ វណ្ណវិចិត្រ

𝑛∈ℤ


636. 𝜋𝑛 + arctan 2 𝑛 ∈ ℤ 637. 2𝜋𝑛; 𝜋𝑛 + 𝜋/4 𝑛 ∈ ℤ 638. 𝜋𝑛 − 𝜋/4; 𝜋𝑛 𝑛 ∈ ℤ 639. 𝜋𝑛 + 𝜋/4 𝑛 ∈ ℤ 𝜋

640. 𝜋𝑛 − 4 ; 𝜋𝑛 − arctan 2 𝑛 ∈ ℤ 641. 𝜋𝑛 + 𝜋 4 ; 𝜋𝑛 − arctan 1/4 𝑛 ∈ ℤ 𝜋

642. 𝜋𝑛 + 4 ; 𝜋𝑛 + arctan 3/5

𝑛∈ℤ

643. 𝜋𝑛 + arctan 2 ; 𝜋𝑛 − arctan 3/4 𝑛 ∈ ℤ  សរដសរអងគខាងស្តំជា −2 sin2 𝑥 + cos2 𝑥 រួ្ល្កអងគទាង ំ ព ីរននសមីការនឹង cos 2 𝑥 ។ 𝜋

644. 𝜋𝑛 − 4 ; 𝜋𝑛 + arctan 5 𝑛 ∈ ℤ 645. 𝜋𝑛 − arctan 4 𝑛 ∈ ℤ 3

646. 𝜋𝑛 − 𝜋 4 ; 𝜋𝑛 ± 𝜋/3 𝑛 ∈ ℤ  បំ លលងអងគខាងស្តំននសមីការជា 3 sin 𝑥 cos 𝑥 − sin 𝑥 + 3 = 3 sin 𝑥 cos 𝑥 − 3 sin2 𝑥 + 3 sin2 𝑥 + cos 2 𝑥 = 3 cos 𝑥 sin 𝑥 + cos 𝑥 = 3 cos 2 𝑥 tan 𝑥 + 1 647. 𝜋𝑛/2 𝑛 ∈ ℤ  បំ លលងអងគខាងស្តំននសមីការជា 1 = sin2 𝑥 + cos 2 𝑥 2 = sin4 𝑥 + 2 sin2 𝑥 cos2 𝑥 + cos 4 𝑥 648. 2𝜋𝑛; 2𝜋𝑛 + 𝜋/3 𝑛 ∈ ℤ 649. 2𝜋𝑛 + 𝜋 12 ; 2𝜋𝑛 + 7𝜋/12 𝑛 ∈ ℤ 650. 2𝜋𝑛 5 − 𝜋 5 ; 2𝜋𝑛/5 + 2𝜋/15 𝑛 ∈ ℤ 651. 2𝜋𝑛; 2𝜋𝑛 + 𝜋/2 𝑛 ∈ ℤ 652. 2𝜋𝑛 + 5𝜋/3 𝑛 ∈ ℤ 653.

𝜋 𝜋 ; 12 12

654. 𝜋𝑛 +

2𝑛 + 1 ; 7𝜋 12

𝜋 8

1 − 2𝑛

𝑛∈ℕ

𝑛∈ℤ

655. 𝜋𝑛/4 𝑛 ∈ ℤ ∖ 4𝑘 + 2 𝑘 ∈ ℤ 656.

𝜋𝑛 3

𝜋

− 12 𝑛 ∈ ℤ ∖ 1 + 3𝑚 𝑚 ∈ ℤ

657. ∅ 658. 𝜋𝑛 𝑛 ∈ ℤ  បំ លលងសមីការជា 2 tan 3𝑥 − tan 2𝑥 = tan 2𝑥 1 + tan 3𝑥 tan 2𝑥 (១)។ ដបើ 1 + tan 3𝑥 tan 2𝑥 = 0 ដ ោះ លឹម សុ វណ្ណវិចិត្រ | III. សមីោរ

187


sin 3x sin 2đ?‘Ľ + cos 3đ?‘Ľ cos 2đ?‘Ľ cos đ?‘Ľ 1 =0 â&#x;š =0 â&#x;š =0 cos 2đ?‘Ľ cos 3đ?‘Ľ cos 2đ?‘Ľ cos 3đ?‘Ľ cos 2đ?‘Ľ 4 cos 2 đ?‘Ľ − 3 áž˜ážˇáž“áž˜ážśáž“ážš ážşáž&#x;á&#x;” ដសដá&#x;’áž“á&#x;„á&#x;‡ážŠáž™ážž ងអវá&#x;’áž›á&#x;’កអងគទវង á&#x;† áž– ីរននáž&#x;áž˜áž¸áž€ážśážš(á&#x;Ą) áž“ážšáž„ 1 + tan 3đ?‘Ľ tan 2đ?‘Ľáž”ážśáž“á&#x;” áž&#x;áž˜áž¸áž€ážśážš (á&#x;Ą)ដá&#x;…ជវ tan 3đ?‘Ľ − tan 2đ?‘Ľ = tan 2đ?‘Ľ 1 + tan 3đ?‘Ľ tan 2đ?‘Ľ 2 tan đ?‘Ľ = tan 2đ?‘Ľ 2 tan đ?‘Ľ 2 tan đ?‘Ľ = 1 − tan2 đ?‘Ľ tan đ?‘Ľ = 0 2

659. 90°đ?‘› + 25° đ?‘› ∈ ℤ 660. 2đ?œ‹đ?‘› đ?‘› ∈ ℤ 661. 2đ?œ‹đ?‘› − đ?œ‹/2 đ?‘› ∈ ℤ đ?œ‹ 6

662. đ?œ‹đ?‘› Âą ; đ?œ‹đ?‘› + đ?œ‹/2 đ?‘› ∈ ℤ 663. 2đ?œ‹đ?‘› Âą 2arctan 5 đ?‘› ∈ ℤ 664. 2đ?œ‹đ?‘› Âą 2arctan 3 ; 2đ?œ‹đ?‘› Âą 2 arctan 3/11 đ?‘› ∈ ℤ 665. đ?œ‹đ?‘› Âą

đ?œ‹ 4 đ?œ‹ 3

đ?‘›âˆˆâ„¤

666. đ?œ‹đ?‘› Âą ; đ?œ‹đ?‘› + đ?œ‹/2 đ?‘› ∈ ℤ 667. 2đ?œ‹đ?‘›/3 Âą 2đ?œ‹ 9 ; đ?œ‹đ?‘›/3 + đ?œ‹/6 đ?‘› ∈ ℤ ď‚˜ áž&#x;រដáž&#x;ážšáž&#x;áž˜áž¸áž€ážśážšáž‡ážśážšážśáž„ cos 3(3đ?‘Ľ) − 2 cos 2 3đ?‘Ľ = 2 668. đ?œ‹đ?‘›; đ?œ‹đ?‘› 2 Âą đ?œ‹/12 đ?‘› ∈ ℤ ď‚˜ áž&#x;រដáž&#x;ážšáž&#x;áž˜áž¸áž€ážśážšáž‡ážśážšážśáž„ 2cos 2(2đ?‘Ľ) = 1 + cos 3 2đ?‘Ľ 669. 3đ?œ‹đ?‘› đ?‘› ∈ ℤ 670.

đ?œ‹đ?‘› đ?œ‹đ?‘› ; 2 2

+ −1

đ?‘›+1 đ?œ‹ 12

đ?‘›âˆˆâ„¤

671. 2đ?œ‹đ?‘›; 4đ?œ‹đ?‘› Âą 2đ?œ‹/3 đ?‘› ∈ ℤ 672. đ?œ‹đ?‘› +

−1 đ?‘› đ?œ‹ 6

; 2đ?œ‹đ?‘› + đ?œ‹/2 đ?‘› ∈ ℤ ď‚˜ ដទá&#x;’បឞáž&#x;áž˜áž—ážśáž– 3đ?œ‹ đ?‘Ľ 3đ?œ‹ đ?‘Ľ đ?œ‹ đ?‘Ľ sin + = sin đ?œ‹ − − = sin − 4 2 4 2 4 2 đ?œ‹ 3 đ?œ‹ 3 đ?œ‹ đ?‘Ľ sin + đ?‘Ľ = sin đ?œ‹ − − đ?‘Ľ = sin 3 − 4 2 4 2 4 2 673. đ?œ‹đ?‘› − đ?œ‹/4 đ?‘› ∈ ℤ 674. 2đ?œ‹đ?‘› Âą arccos

đ?‘? đ?‘Ž 2 +đ?‘? 2

đ?‘?

+ arctan đ?‘Ž đ?‘› ∈ ℤ á&#x;’á&#x;† ដ á&#x;„á&#x;‡ đ?‘? 2 ≤ đ?‘Ž2 + đ?‘? 2 ; ∅ á&#x;’á&#x;† ដ á&#x;„á&#x;‡ đ?‘? 2 > đ?‘Ž2 +

đ?‘? 2 ď ° áž›á&#x;’កអងគទវង á&#x;† áž– ីរននáž&#x;áž˜áž¸áž€ážśážšáž“ážšáž„ đ?‘Ž2 + đ?‘? 2 ដយឞ ងទវញបវន 188

á&#x;Ł. áž?á&#x;’រីកá&#x;„ណá&#x;’áž˜ážśáž?á&#x;’ážš | លរម áž&#x;ážť វណá&#x;’ážŽážœážˇáž…ážˇáž?á&#x;’ážš


𝑎 𝑎2 + 𝑏 2

ដយើ ងមាន

cos 𝑥 +

𝑎

𝑏 𝑎2 + 𝑏 2

sin 𝑥 =

2

𝑎2 + 𝑏 2

(1)

2

𝑏

+

𝑎2 + 𝑏 2

𝑐

𝑎2 + 𝑏 2

=1

ដហើ យ −1 ≤

𝑎

;

𝑏

≤1 𝑎2 + 𝑏 2 𝑎2 + 𝑏 2 តាង 𝑎 𝑎2 + 𝑏 2 = cos 𝜙 ; 𝑏 𝑎2 + 𝑏 2 = sin 𝜙 ។ សមីការ(១) ដៅជា 𝑐 cos 𝑥 cos 𝜙 + sin 𝑥 sin 𝜙 = cos 𝑥 − 𝜙 = (2) 2 𝑎 + 𝑏2 សមីការ(២) មានរ ឺសដបើ 𝑐 ≤ 1 ⟹ 𝑐 2 ≤ 𝑎2 + 𝑏 2 𝑎2 + 𝑏 2 ដយើ ងទាញបាន 𝑥 − 𝜙 = 2𝜋𝑘 ± arctan 𝑐/ 𝑎2 + 𝑏 2 , 𝑘 ∈ ℤ (3) 𝑏

លដល 𝜙 = 2𝜋𝑚 + arctan 𝑎 , 𝑚 ∈ ℤ ។ ទ្តង់ ដនោះដយើ ងសនមតថា 𝑎 > 0 ។ ករណី 𝑎 = 0 សមីការដៅ ជា 𝑏 sin 𝑥 = 𝑐 ។ ករណី 𝑎 < 0 រុណសមីការនឹង −1 ។ 675. 𝜋𝑛 − 𝜋/8 𝑛 ∈ ℤ 𝜋 𝑛∈ℤ 6 4𝜋𝑛 2𝜋 4𝜋𝑛 2𝜋 + 5 ; 3 + 3 𝑛∈ℤ 5 𝜋𝑛 𝑛∈ℤ 4 𝜋𝑛 𝜋 3𝜋 + ; 𝜋𝑛 + 𝑛∈ℤ 2 8 4 −1± 4𝑙+3 𝑛; 𝑛 ∈ ℤ; 𝑙 ∈ ℤ0 2

676. 𝜋𝑛; 677. 678. 679. 680.

𝜋𝑛 3

; ℤ0 = 0; 1; 2; …

681. 𝜋𝑛 + 𝜋/4 𝑛 ∈ ℤ 682. 𝜋𝑛/4 𝑛 ∈ ℤ 683.

𝜋𝑛 6

𝜋

+ 12 ; 2𝜋𝑛 ±

2𝜋 3

𝑛∈ℤ

684. 𝜋𝑛; 2𝜋𝑛 ± 𝜋 6 𝑛 ∈ ℤ 685. 𝜋𝑛 − 𝜋 4 ; 𝜋𝑛 + 𝜋/2 𝑛 ∈ ℤ 686. 2𝜋𝑛 3 ; 𝜋𝑛 + 𝜋 4 ; 2𝜋𝑛 − 𝜋/2 𝑛 ∈ ℤ លឹម សុ វណ្ណវិចិត្រ | III. សមីោរ

189


687. 𝜋𝑛/2 𝑛 ∈ ℤ 688. 𝜋𝑛 + 𝜋/2 𝑛 ∈ ℤ 689.

𝜋𝑛 6

𝜋

+ 48 ;

𝜋𝑛 4

3𝜋

+ 32 𝑛 ∈ ℤ

690. ℝ ∖ 𝜋𝑛 𝑛 ∈ ℤ 691. 𝜋𝑛 5 ; 𝜋𝑛 ± 3𝜋/8 𝑛 ∈ ℤ 17 8𝜋𝑛 5 8𝜋𝑛 𝜋; − 𝜋; 6 3 18 3 2𝜋𝑛 𝜋 ; 𝜋𝑛 + 2 ; 2𝜋𝑛 + 𝜋 𝑛 ∈ 5 𝜋𝑛 𝜋𝑛 𝜋 ; 3 +6 𝑛∈ℤ 5

692. 4𝜋𝑛 +

+

693.

694.

𝜋 6

𝑛∈ℤ

695. 𝜋𝑛 + 𝜋 2 ; 𝜋𝑛 6 + 𝜋 24 𝑛 ∈ ℤ 696. 𝜋𝑛 4 + 𝜋 8 ; 𝜋𝑛 3 + −1 697. 𝜋𝑛 5 + −1

𝑛+1

𝑛+1

𝜋 18 𝑛 ∈ ℤ

𝜋 30 ; 𝜋𝑛 4 + 𝜋 16 ; 𝜋𝑛 + 3𝜋/4 𝑛 ∈ ℤ

698. 𝜋𝑛 + 𝜋 2 ; 𝜋𝑛 + −1 𝑛 𝜋 6 ; 2𝜋𝑛 ± 2𝜋/3 𝑛 ∈ ℤ 699. 𝜋𝑛/2; 2𝜋𝑛 ± 2𝜋/3 𝑛 ∈ ℤ 700.

𝜋 7

2𝑛 + 1 𝑛 ∈ ℤ ∖ 7𝑙 − 4| 𝑙 ∈ ℤ

701. 𝜋𝑛 𝑛 ∈ ℤ 702. 703.

𝜋𝑛 3 𝜋𝑛 10

𝑛∈ℤ 𝑛∈ℤ

704. 𝜋𝑛 3 + 𝜋 6 ; 𝜋𝑛 5 + 𝜋 10 𝑛 ∈ ℤ 705. 𝜋𝑛; 𝜋𝑛 5 + 𝜋 10 𝑛 ∈ ℤ 706. 𝜋𝑛; 𝜋𝑛 3 + 𝜋 6 𝑛 ∈ ℤ 707. 𝑛 − 5 12 ; 𝑛 + 1 4 𝑛 ∈ ℤ 708. 𝜋𝑛 2 ; 𝜋𝑛 ± 𝜋 6 𝑛 ∈ ℤ  សមីការសមមូលនឹង 1 − cos 2𝑥 + 2 sin2 2𝑥 = 1 − cos 6𝑥 ; ⟺ 2 sin2 2𝑥 − cos 2𝑥 − cos 6𝑥 = 0 ⟺ 2 sin2 2𝑥 − 2 sin 2𝑥 sin 4𝑥 = 0 𝜋 𝜋𝑛 709. 𝜋𝑛 2 + 4 ; 5 + 𝜋 10 𝑛 ∈ ℤ 710. 𝜋𝑛 + 𝜋 2 ; 2𝜋𝑛 11 + 𝜋 11 ; 2𝜋𝑛 5 𝑛 ∈ ℤ 1

3

711. 𝜋𝑛 ± 2 arccos 4 𝑛 ∈ ℤ 712. 713. 190

2𝜋𝑛 7 2𝜋𝑛 7

𝜋

+7 𝑛∈ℤ 𝑛 ∈ ℤ ∖ 7𝑘|𝑘 ∈ ℤ ៣. ត្រីកោណ្មាត្រ | លឹម សុ វណ្ណវិចិត្រ


714.

𝜋𝑛 4

+ −1

𝑛+1 𝜋 24

𝑛 ∈ ℤ  សមីការសមមូលនឹង sin 𝑥 cos 3𝑥 1 − cos 2𝑥 + 3

3

cos 𝑥 sin 3𝑥 1 + cos 2𝑥 = − 4 ; ⟺ sin 𝑥 cos 3𝑥 + cos 𝑥 sin 3𝑥 + cos 2𝑥 − 4 ⟺ 1

3

1

sin 4𝑥 + 2 sin 4𝑥 = − 4 ; ⟺ sin 4𝑥 = − 2 ។ 𝜋

715. 𝜋𝑛 ± 8 𝑛 ∈ ℤ 𝜋𝑛

𝜋

3 𝜋𝑛 3

−9 𝑛∈ℤ

716. + −1 𝑛 𝑛 ∈ ℤ  បំ លបកអងគខាងស្តំននសមីការ 3 18 1 𝜋 𝜋 1 2𝜋 sin 𝑥 2 sin − 𝑥 sin + 𝑥 = sin 𝑥 cos 2𝑥 − cos 2 3 3 2 3 1 1 1 = 2 sin 𝑥 cos 2𝑥 + sin 𝑥 = sin 3𝑥 − sin 𝑥 + sin 𝑥 = sin 3𝑥 4 4 4 2𝜋𝑛 2𝜋 717. ± 𝑛∈ℤ 718.

9

𝜋

719. 𝜋𝑛; 𝜋𝑛 ± 𝜋/6 𝑛 ∈ ℤ 720. 𝜋𝑛; 𝜋𝑛 ± 𝜋/6 𝑛 ∈ ℤ 1

5

1 2

1 4

721. 𝜋𝑛 ± 2 arccos − 6 722. 𝜋𝑛 ± arccos −

𝑛∈ℤ 1 2

; 𝜋𝑛 ± arccos

1 3

𝑛∈ℤ

723. 𝜋𝑛 + 𝜋/2 𝑛 ∈ ℤ 724. 𝜋𝑛 ± 𝜋/8 𝑛 ∈ ℤ 725. 𝜋𝑛 ± 𝜋/6 𝑛 ∈ ℤ 726. 𝜋𝑛 2 + 𝜋 4 ; 𝜋𝑛 + 𝜋/2 𝑛 ∈ ℤ 1

5−1 2 5−1 𝑛 2

727. 𝜋𝑛 2 + 𝜋 4 ; 𝜋𝑛 ± 2 arccos

730. 731. 732. 733. 734.

𝑛∈ℤ

𝜋 −1 arcsin ∈ℤ 4 2 𝜋 4𝑛+1 ; 𝜋 2𝑛 + 1 𝑛 ∈ ℤ  តាង sin 𝑥 − cos 𝑥 = 𝑦 2 𝜋 𝜋𝑛 𝜋 𝜋𝑛 − 4 ; 2 + −1 𝑛+1 8 𝑛 ∈ ℤ 𝜋 11𝜋 5𝜋 2𝜋𝑛 + ; 2𝜋𝑛 + ; 2𝜋𝑛 − ; 𝑛 ∈ ℤ 4 12 12 𝜋 2𝜋𝑛; 2𝜋𝑛 + 2 ; 𝑛 ∈ ℤ 𝜋 𝜋 1 2𝜋𝑛 − 2 ; 𝜋𝑛 − 4 + −1 𝑛 arcsin 2 − 1 𝑛 ∈ ℤ 4𝜋𝑛 + 𝜋 2 2 ; 4𝜋𝑛 + 11𝜋 6 2 𝑛 ∈ ℤ0 ; 4𝜋𝑚 − 5𝜋

728. 𝜋𝑛 + ; 𝜋𝑛 + 729.

𝑛

⟹ 1 − sin 2𝑥 = 𝑦 2 ។

6

2

𝑚∈ℕ

735. 2𝜋𝑛 + 5𝜋/6 𝑛 ∈ ℤ  ដោ​ោះទ្ស្យសមីការរក tan 𝑥 ជាអនុរមន៍នន sin 𝑥 ដយើ ងទាញបាន

លឹម សុ វណ្ណវិចិត្រ | III. សមីោរ

191


tan 𝑥 = −

2 3

± −2 sin 𝑥 −

1 2

2

/2

សមីការមានរ ឺសទាល់ លត sin 𝑥 = 1/2 និង tan 𝑥 = −1/ 3 ។ 736. 2𝜋𝑛; 2𝜋𝑛 + 𝜋/2 𝑛 ∈ ℤ 737. 2𝜋𝑛 − 𝜋/4 𝑛 ∈ ℤ 738. 2𝜋 1 + 4𝑛 𝑛 ∈ ℤ  សំ រួលសមីការជា 5𝑥 sin + cos 𝑥 = 2 (1) 4 ដោយ sin(5𝑥/4) ≤ 1, cos 𝑥 ≤ 1 ដ ោះសមីការ(១) មានរ ឺសទាល់ លត sin(5𝑥/4) = 1 និង cos 𝑥 = 1 ។ 739. 𝜋𝑛 3 ; 2𝜋𝑛 − 𝜋/2 𝑛 ∈ ℤ 1 6

740. 𝜋𝑛 − arctan ; 𝜋𝑛 − arctan 𝜋

741. 2𝜋𝑛 − 3 ; 2𝜋𝑛 +

2𝜋 3

1 3

𝑛∈ℤ

;𝑛 ∈ ℤ

742. 𝜋𝑛 ± 𝜋/6 𝑛 ∈ ℤ 743. 2𝜋𝑛 + 3𝜋/4 𝑛 ∈ ℤ 744. 2𝜋𝑛 − arccos

5−1 2

𝑛∈ℤ

745. 2𝜋𝑛 + 𝜋 8 ; 2𝜋𝑛 − 3𝜋 8 𝑛 ∈ ℤ 746.

𝜋𝑛 2

; 𝜋𝑛 + 𝜋 6 𝑛 ∈ ℤ

747. 2𝜋𝑛 + 𝜋 2 ; 2𝜋𝑛 − 𝜋 6 𝑛 ∈ ℤ 748. 2𝜋𝑛; 2𝜋𝑛 − 𝜋 2 𝑛 ∈ ℤ 749. 2𝜋𝑛 +

3𝜋 8

; 2𝜋𝑛 + 7𝜋 8 ; 2𝜋𝑛 + 𝜋; 𝜋𝑛 + 𝜋/4 𝑛 ∈ ℤ

750. 2𝜋𝑛; 𝜋𝑛 − 𝜋/4 𝑛 ∈ ℤ 751. 𝜋𝑛 3 + −1 752. 𝜋𝑛 +

𝑛+1

2 arctan 3

𝜋/8 𝑛 ∈ ℤ

𝑛∈ℤ

753. 2𝜋𝑛 + 𝜋 12 ; 2𝜋𝑛 − 7𝜋 12 𝑛 ∈ ℤ 754. 2𝜋𝑛 + 𝜋 6 ; 2𝜋𝑙 3 + 5𝜋/18 𝑛, 𝑙 ∈ ℤ 755. 4𝜋𝑛 + 13𝜋/6 𝑛 ∈ ℤ 756. 2𝜋𝑛 + arccos 1/3 𝑛 ∈ ℤ 192

៣. ត្រីកោណ្មាត្រ | លឹម សុ វណ្ណវិចិត្រ


757. 2đ?œ‹đ?‘› + arccos 1/10 đ?‘› ∈ ℤ 758. đ?œ‹đ?‘› đ?‘› ∈ ℤ 759. log 2 đ?œ‹đ?‘› 4 + đ?œ‹/8 đ?‘› ∈ ℤ0 7

đ?œ‹

7

760. 1; 12 đ?œ‹; đ?œ‹đ?‘› − 12 ; đ?œ‹đ?‘› + 12 đ?œ‹ đ?‘› ∈ â„• 761. áž&#x;áž˜áž¸áž€ážśážšáž‘ážśá&#x;†áž„áž– áž¸ážšáž˜ážˇáž“ážŠážźá&#x;’áž‚á&#x;’ននដរá&#x;” áž&#x;áž˜áž¸áž€ážśážšážšáž¸áž˜áž˝áž™áž˜ážśáž“ážš ážşáž&#x; đ?œ‹đ?‘› − đ?œ‹ 4 ; đ?œ‹đ?‘› + đ?œ‹ 2 |đ?‘› ∈ ℤ áž&#x;áž˜áž¸áž€ážśážšážšáž¸áž– áž¸ážšáž˜ážśáž“ážš ážşáž&#x; đ?œ‹đ?‘› − đ?œ‹ 4 |đ?‘› ∈ ℤ á&#x;” 762.

5 − 1; 2

763.ď ° យក đ?‘Ľ = 0 ដយឞ ងទវញបវន 1 + đ?‘? 2 = cos đ?‘? 2 á&#x;”

ដá&#x;† អវយ đ?‘? = 0 á&#x;” ដយឞ ងទវញបវន

a(cos đ?‘Ľ − 1) = cos đ?‘Žđ?‘Ľ − 1 (∗) ដបឞ đ?‘Ž = 0 ដ á&#x;„á&#x;‡ (*) áž– ឡáž?á&#x;’á&#x;† ដ á&#x;„á&#x;‡áž‘á&#x;’ážšáž”á&#x;‹ đ?‘Ľ á&#x;” ដ á&#x;„á&#x;‡ážŠáž™ážž áž„áž&#x;ឡ កវករណី đ?‘Ž ≠0 មáž? áž„á&#x;” ដá&#x;„áž™(*) áž– ឡáž?ជវនឡá&#x;’áž… ដ á&#x;„á&#x;‡ យក đ?‘Ľ = 2đ?œ‹ ដយឞ ងទវញបវន cos 2đ?‘Žđ?œ‹ = 1

ដá&#x;† អវយ 2đ?‘Žđ?œ‹ = 2đ?‘˜đ?œ‹, đ?‘˜ ∈ ℤ ដសដá&#x;’áž“á&#x;„á&#x;‡

đ?‘Ž = đ?‘˜, đ?‘Ž ≠0 យក đ?‘Ľ =

2đ?œ‹ đ?‘Ž

ជá&#x;’á&#x;†áž“áž&#x; áž˝ á&#x;’ážźáž›(*) ដយឞ ងទវញបវន 2đ?œ‹ −1 =0 đ?‘Ž 1 = 2đ?‘šđ?œ‹; đ?‘š ∈ ℤ á&#x;” ដយឞ ងទវញបវន đ?‘Ž = đ?‘š á&#x;” ដá&#x;„áž™ đ?‘Ž ∈ ℤ ដ á&#x;„á&#x;‡ đ?‘Ž cos

ដá&#x;„áž™ đ?‘Ž ≠0 ដ á&#x;„á&#x;‡ cos

2đ?œ‹ đ?‘Ž

= 1;

2đ?œ‹ đ?‘Ž

đ?‘Ž = Âą1 á&#x;” ដបឞ đ?‘Ž = 1; đ?‘? = 0 ដ á&#x;„á&#x;‡ cos đ?‘Ľ − 1 = cos đ?‘Ľ − 1 áž– ឡáž?áž‘á&#x;’ážšáž”á&#x;‹ đ?‘Ľ á&#x;” ដបឞ đ?‘Ž = −1; đ?‘? = 0 ដ á&#x;„á&#x;‡ −cos đ?‘Ľ + 1 = cos đ?‘Ľ − 1 áž˜ážˇáž“áž– ឡáž?áž‘á&#x;’ážšáž”á&#x;‹ đ?‘Ľ á&#x;” ដសដá&#x;’áž“á&#x;„á&#x;‡áž&#x;áž˜áž¸áž€ážśážšáž– ឡáž?áž‘á&#x;’ážšáž”á&#x;‹ đ?‘Ľ ដបឞ đ?‘Ž = 0; đ?‘? = 0 ážš ážş (đ?‘Ž = 1; đ?‘? = 0) á&#x;”

IV. áž?á&#x;’ážšáž–á&#x;?áž“á&#x;’áž&#x; áž’ មីá&#x;„ážš 764.

2đ?œ‹đ?‘› Âą arccos đ?œ‹ đ?œ‹ ; 2 6

57−6 3

765.

đ?œ‹đ?‘› +

766.

2đ?œ‹đ?‘› Âą arccos 2 cos

13đ?œ‹ 2

đ?‘Ž

Ď€

đ?œ‹ 5

; 2đ?œ‹đ?‘› Âą arccos

57−6 3

đ?‘›âˆˆâ„¤

− đ?œ‹đ?‘› đ?‘› ∈ ℤ đ?œ‹ đ?œ‹

đ?œ‹/8

đ?‘Ž

+ 8 ; 8 − 2đ?œ‹đ?‘› Âą arccos 2 cos

đ?œ‹/8

đ?‘› ∈ ℤ á&#x;’á&#x;† ដ á&#x;„á&#x;‡ đ?‘Ž ∈

Ď€

−2 cos 8 ; 2 cos 8 767.

+

đ?‘› + 4đ?‘˜ Âą

2đ?œ‹ 15

+

−1 đ?‘› 5

arcsin 2đ?‘Ž ;

đ?œ‹ 5

đ?œ‹ 5

đ?‘› − 6đ?‘˜ Âą +

−1 đ?‘› 5

arcsin 2đ?‘Ž

đ?‘›, đ?‘˜ ∈ ℤ

á&#x;’á&#x;† ដ á&#x;„á&#x;‡ đ?‘Ž ∈ −∞; 0 លរម áž&#x;ážť វណá&#x;’ážŽážœážˇáž…ážˇáž?á&#x;’ážš | IV. áž?á&#x;’ážšáž–á&#x;?áž“á&#x;’áž&#x; áž’ មីá&#x;„ážš

193


768.

𝜋 𝑛+

769.

𝜋 4

𝑘 2

+ 𝜋𝑛; 𝜋 2𝑛 − 𝑚 + 𝜋

770.

𝜋 6

+ ; 𝜋 −𝑛 +

𝑘 2

𝜋 4

+

𝜋 3

; 𝜋 𝑛+

𝑘 2

𝜋 3

+ ; 𝜋 −𝑛 +

𝑘 2

+

𝜋 6

𝑛, 𝑘 ∈ ℤ

𝑛, 𝑚 ∈ ℤ

𝜋

2𝜋𝑛 + 6 ; 2𝜋𝑘 + 3 ; 2𝜋𝑛 + 2𝜋𝑛 +

5𝜋 6

7𝜋 6

; 2𝜋𝑘 +

; 2𝜋𝑘 +

5𝜋 3

4𝜋 3

𝜋

; 2𝜋𝑛 − 6 ; 2𝜋𝑘 +

2𝜋 3

;

𝑛, 𝑘 ∈ ℤ

𝜋 4

𝜋 771. 𝜋𝑛 − ; 𝜋𝑚 + −1 𝑚 6 𝑛, 𝑚 ∈ ℤ 3𝜋 𝜋 772. 2𝜋𝑛 ± ; 𝜋𝑚 + −1 𝑚 𝑛, 𝑚 ∈ ℤ 4 6 𝜋 𝑎 773. 2𝜋𝑛 + 2 ; 2𝜋𝑘 ± arccos − 3 𝑛, 𝑘 ∈ ℤ ្ំ ដ ោះ 𝑎 ∈ −3; 3 ; 𝜋 𝜋 2𝜋𝑛 + ; 2𝜋𝑘 ; 2𝜋𝑛 − ; 𝜋 2𝑘 + 1 |𝑛, 𝑘 ∈ ℤ ្ំ ដ ោះ 𝑎 = −3 2 2 1 2𝜋𝑛 ± arccos 𝑎 ; 𝜋𝑘 − arctan 𝑎 + 2 ; 774. 𝑛, 𝑘 ∈ ℤ ្ំ ដ ោះ 𝑎 ∈ −∞; −3 1 2𝜋𝑛 ± arccos 𝑎+2 ; 𝜋𝑘 − arctan 𝑎 1 2𝜋𝑛 ± arccos 𝑎 ; 𝜋𝑘 − arctan 𝑎 + 2 𝑛, 𝑘 ∈ ℤ ្ំ ដ ោះ 𝑎 ∈ −3; −1 ; 1 2𝜋𝑛 ± arccos 𝑎+2 ; 𝜋𝑘 − arctan 𝑎 𝑛, 𝑘 ∈ ℤ ្ំ ដ ោះ 𝑎 ∈ −1; 1 𝜋𝑘 𝜋 𝜋𝑛 1 1 775. 2 + −1 𝑘+1 8 ; 5 − 5 arctan 2 𝑛, 𝑘 ∈ ℤ

776. តាង 𝑎 = tan 𝑥 ; 𝑏 = tan 𝑦 ; 𝑐 = tan 𝑧 ។ ដយើ ងទាញបាន 𝑎2 + 𝑏 2 + 𝑐 2 = 𝑚2 1 𝑎3 + 𝑏 3 + 𝑐 3 = 𝑚3 (2) ក) ដបើ 𝑚 = 0 ដ ោះ 𝑎 = 𝑏 = 𝑐 = 0 ដយើ ងទាញបាន 𝑥; 𝑦; 𝑧 = 𝑝𝜋; ℎ𝜋; 𝑘𝜋 ស) ដបើ 𝑚 ≠ 0 ដលើកសមីការរី(២)ជាកាដរ ដយើ ងទាញបាន 𝑚6 = 𝑎3 + 𝑏 3 + 𝑐 3 តាមវ ិសមភាពកូសុីស្ាត ដយើ ងទាញបាន

សមីការរី(១)

2

𝑚6 = 𝑎3 + 𝑏 3 + 𝑐 3 2 ≤ 𝑎2 + 𝑏 2 + 𝑐 2 𝑎4 + 𝑏 4 + 𝑐 4 ⟹ 𝑚6 ≤ 𝑚2 𝑎4 + 𝑏 4 + 𝑐 4 ⟹ 𝑚4 ≤ 𝑎4 + 𝑏 4 + 𝑐 4 (4) ដំ អាយ

𝑚4 = 𝑎2 + 𝑏 2 + 𝑐 2 ដហើ យតាម(៤) ដយើ ងទាញបាន

2

= 𝑎4 + 𝑏 4 + 𝑐 4 + 2 𝑎2 𝑏 2 + 𝑏 2 𝑐 2 + 𝑐 2 𝑎2

𝑎4 + 𝑏 4 + 𝑐 4 + 2 𝑎2 𝑏 2 + 𝑏 2 𝑐 2 + 𝑐 2 𝑎2 ≤ 𝑎4 + 𝑏 4 + 𝑐 4 ⟹ 𝑎2 𝑏 2 + 𝑏 2 𝑐 2 + 𝑐 2 𝑎2 ≤ 0 𝑎2 𝑏 2 = 0 ⟹ 𝑏2 𝑐2 = 0 𝑐 2 𝑎2 = 0 ⟹ 𝑎 = 𝑚; 𝑏 = 𝑐 = 0 ; 𝑎 = 0; 𝑏 = 𝑚; 𝑐 = 0 ; 𝑎 = 𝑏 = 0; 𝑐 = 𝑚 194

៣. ត្រីកោណ្មាត្រ | លឹម សុ វណ្ណវិចិត្រ

∪ 1; ∞ ;


đ?‘Ľ = arctan đ?‘š + đ?‘?đ?œ‹; đ?‘Ś = â„Žđ?œ‹; đ?‘§ = đ?‘˜đ?œ‹ â&#x;š đ?‘Ľ = đ?‘?đ?œ‹; đ?‘Ś = arctan đ?‘š + â„Žđ?œ‹; đ?‘§ = đ?‘˜đ?œ‹ đ?‘Ľ = đ?‘?đ?œ‹; đ?‘Ś = â„Žđ?œ‹; đ?‘§ = arctan đ?‘š + đ?‘˜đ?œ‹ ដសដá&#x;’áž“á&#x;„á&#x;‡á&#x;’á&#x;† ដ á&#x;„á&#x;‡áž‘á&#x;’ážšáž”á&#x;‹ đ?‘š áž&#x;áž˜áž¸áž€ážśážšáž˜ážśáž“á&#x;’á&#x;† ដលឞយ đ?‘Ľ = arctan đ?‘š + đ?‘?đ?œ‹; đ?‘Ś = â„Žđ?œ‹; đ?‘§ = đ?‘˜đ?œ‹ đ?‘Ľ = đ?‘?đ?œ‹; đ?‘Ś = arctan đ?‘š + â„Žđ?œ‹; đ?‘§ = đ?‘˜đ?œ‹ đ?‘Ľ = đ?‘?đ?œ‹; đ?‘Ś = â„Žđ?œ‹; đ?‘§ = arctan đ?‘š + đ?‘˜đ?œ‹

V. ážœážˇáž&#x;áž˜áž—ážśáž– 777.ď ° ដá&#x;„áž™ đ?œ‹ 6 < đ?›ź < đ?œ‹ 3 ដ á&#x;„á&#x;‡ 1 sin đ?›ź < 2 នឡង 1 cos đ?›ź < 2 á&#x;” ដសដá&#x;’áž“á&#x;„á&#x;‡ đ?œ‹ sin đ?‘Ľ đ?œ‹ đ?œ‹ đ?œ‹ cos đ?‘Ľ đ?œ‹ đ?œ‹ ≤ < ;0 ≤ ≤ < 4 sin đ?›ź 4 sin đ?›ź 2 4 cos đ?›ź 4 cos đ?›ź 2 đ?œ‹ sin đ?‘Ľ đ?œ‹ cos đ?‘Ľ â&#x;š tan ≼ 0; tan ≼0 4 sin đ?›ź 4 cos đ?›ź ករណី đ?‘Ľ < đ?›ź ដ á&#x;„á&#x;‡ cos đ?‘Ľ > cos đ?›ź ដយឞ ងទវញបវន đ?œ‹ sin đ?‘Ľ đ?œ‹ cos đ?‘Ľ đ?œ‹ cos đ?‘Ľ đ?œ‹ cos đ?›ź tan + tan ≼ tan > tan =1 4 sin đ?›ź 4 cos đ?›ź 4 cos đ?›ź 4 cos đ?›ź ករណី đ?‘Ľ = đ?›ź ដយឞ ងទវញបវន đ?œ‹ sin đ?‘Ľ đ?œ‹ cos đ?‘Ľ đ?œ‹ sin đ?›ź đ?œ‹ cos đ?›ź tan + tan = tan + tan =2>1 4 sin đ?›ź 4 cos đ?›ź 4 sin đ?›ź 4 cos đ?›ź ករណី đ?‘Ľ > đ?›ź ដ á&#x;„á&#x;‡ sin đ?‘Ľ > sin đ?›ź ដយឞ ងទវញបវន đ?œ‹ sin đ?‘Ľ đ?œ‹ cos đ?‘Ľ đ?œ‹ sin đ?‘Ľ đ?œ‹ sin đ?›ź tan + tan ≼ tan > tan =1 4 sin đ?›ź 4 cos đ?›ź 4 sin đ?›ź 4 sin đ?›ź 778.ď ° ážœ ឡáž&#x;áž˜áž—ážśáž–áž&#x;áž˜áž˜ážźáž›áž“ážšáž„ 0≤

4 tan2 đ??´ tan2 đ??ľ tan2 đ??ś − 4 tan2 đ??´ + tan2 đ??ľ + tan2 đ??ś − 3 ≤ 1 + tan2 đ??´ 1 + tan2 đ??ľ 1 + tan2 đ??ś 1 1 1 1 1 1 â&#x;ş4 −1 −1 −1 −4 + + −3 −3 2 2 2 2 2 cos đ??´ cos đ??ľ cos đ??ś cos đ??´ cos đ??ľ cos 2 đ??ś 1 1 1 ≤ 2 2 cos đ??´ cos đ??ľ cos2 đ??ś 3 â&#x;ş cos2 đ??´ + cos 2 đ??ľ + cos2 đ??ś ≼ 4 1 + cos 2đ??´ 1 + cos 2đ??ľ 3 â&#x;ş + + cos2 đ??ś ≼ 2 2 4 â&#x;ş 2 cos 2đ??´ + cos 2đ??ľ + 4 cos2 đ??ś + 1 ≼ 0 â&#x;ş 4 cos đ??´ + đ??ľ cos đ??´ − đ??ľ + 4 cos 2 đ??ś + 1 ≼ 0 â&#x;ş 4 cos2 đ??ś − 4 cos đ??ś cos đ??´ − đ??ľ + 1 ≼ 0 â&#x;ş 2 cos đ??ś − cos đ??´ − đ??ľ 2 + sin2 đ??´ − đ??ľ ≼ 0 លរម áž&#x;ážť វណá&#x;’ážŽážœážˇáž…ážˇáž?á&#x;’ážš | V. ážœážˇáž&#x;áž˜áž—ážśáž–

195


áž– ឡáž?á&#x;”

779.ď ° ដយឞ áž„áž˜ážśáž“ đ?œ‹ 3đ?œ‹ đ?œ‹ 5đ?œ‹ 3đ?œ‹ 7đ?œ‹ 5đ?œ‹ = sin − sin + sin − sin + sin − sin 14 14 14 14 14 14 14 đ?œ‹ đ?œ‹ 2đ?œ‹ 3đ?œ‹ = 2 sin cos + cos + cos (1) 14 7 7 7 đ?œ‹ 1 − sin 14 đ?œ‹ 2đ?œ‹ 3đ?œ‹ â&#x;š (2) đ?œ‹ = cos 7 + cos 7 + cos 7 2 sin 14 đ?œ‹ 1 đ?œ‹ 3đ?œ‹ 5đ?œ‹ đ?œ‹ 4đ?œ‹ 2đ?œ‹ cos = cos + cos + cos + cos + cos + cos 7 2 7 7 7 7 7 7 đ?œ‹ 2đ?œ‹ 3đ?œ‹ 2đ?œ‹ 3đ?œ‹ đ?œ‹ = cos cos + cos cos + cos cos (3) 7 7 7 7 7 7 đ?œ‹ 2đ?œ‹ 3đ?œ‹ áž?ážśáž„ đ?‘Ľ = cos ; đ?‘Ś = cos , đ?‘§ = cos á&#x;” áž?ážśáž˜(á&#x;˘) នឡង (á&#x;Ł) ážœ ឡáž&#x;áž˜áž—ážśáž–áž˘ážśá&#x;’áž&#x;រដáž&#x;រជវ 1 − sin

7

7

7

�+�+� >

3(�� + �� + ��)

(4)

ដá&#x;„áž™ đ?‘Ľ, đ?‘Ś, đ?‘§ > 0 ដ á&#x;„á&#x;‡ (á&#x;¤) អវá&#x;’áž&#x;រដáž&#x;រជវ đ?‘Ľ + đ?‘Ś + đ?‘§ 2 > 3 đ?‘Ľđ?‘Ś + đ?‘Śđ?‘§ + đ?‘§đ?‘Ľ â&#x;ş đ?‘Ľâˆ’đ?‘Ś 2 + đ?‘Śâˆ’đ?‘§ 2 + đ?‘§âˆ’đ?‘Ľ ដá&#x;„áž™ đ?‘Ľ, đ?‘Ś, đ?‘§ áž˜ážśáž“áž?á&#x;† áž“áž›áž&#x;ážťáž&#x;áž‚á&#x;’áž“áž“ ដ á&#x;„á&#x;‡ážœ ឡáž&#x;áž˜áž—ážśáž–(á&#x;Ľ)áž– ឡáž?á&#x;”

780.ď ° ដá&#x;„áž™ đ?‘Ľđ?‘– ∈

đ?œ‹ đ?œ‹ ; 6 2

â&#x;š đ?‘–=1

>0

(5)

ដ á&#x;„á&#x;‡

1 ≤ sin đ?‘Ľđ?‘– ≤ 1 â&#x;ş sin đ?‘Ľđ?‘– − 1 2 3 1 â&#x;ş sin2 đ?‘Ľđ?‘– − sin đ?‘Ľđ?‘– + ≤ 0 2 2 1 3 â&#x;ş sin đ?‘Ľđ?‘– + ≤ 2 sin đ?‘Ľđ?‘– 2 2đ?‘›

2

1 sin đ?‘Ľđ?‘– + 2

áž?ážśáž˜ážœ ឡáž&#x;áž˜áž—ážśáž–áž€ážźáž&#x;ឝី ដយឞ áž„áž˜ážśáž“

2đ?‘›

đ?‘–=1

2đ?‘›

sin đ?‘Ľđ?‘– −

1 ≤ 0; 2

� = 1; 2; ‌ 2�

1 3 ≤ 2� = 3� sin �� 2

1 sin đ?‘Ľđ?‘– + 2

2đ?‘›

1 1 ≼2 đ?‘Œ sin đ?‘Ľđ?‘– 2

đ?‘–=1 đ?‘–=1 9 2 ដយឞ ងទវញបវន đ?‘Œ ≤ 2 đ?‘› á&#x;” ដយឞ áž„áž˜ážśáž“ đ?‘Œ = 1800; â&#x;š đ?‘› ≼ 20 á&#x;”áž&#x;ញá&#x;’ញវដáž&#x;មឞ ដកឞáž?áž˜ážśáž“áž‘ážśáž›á&#x;‹ áž›áž? 1 1 2đ?‘› 1 sin đ?‘Ľđ?‘– = 1 ážš ážş sin đ?‘Ľđ?‘– = 2 នឡង 2đ?‘› áž˝ sin đ?‘Ľđ?‘– = 1 នឡង đ?›˝ đ?‘–=1 sin đ?‘Ľđ?‘– = 2 đ?‘–=1 sin đ?‘Ľ á&#x;” áž?ážśáž„ đ?›ź ជវá&#x;’á&#x;† áž“áž“ đ?‘– 1 á&#x;’á&#x;† áž“áž“ áž˝ sin đ?‘Ľđ?‘– = 2 á&#x;” ដសដá&#x;’áž“á&#x;„á&#x;‡

196

á&#x;Ł. áž?á&#x;’រីកá&#x;„ណá&#x;’áž˜ážśáž?á&#x;’ážš | លរម áž&#x;ážť វណá&#x;’ážŽážœážˇáž…ážˇáž?á&#x;’ážš

ជវ


1 1 đ?›ź + đ?›˝ = 2đ?‘›; đ?›ź + đ?›˝ = đ?›ź + đ?›˝ â&#x;š đ?›ź = đ?›˝ = đ?‘› 2 2 ដសដá&#x;’áž“á&#x;„á&#x;‡ đ?‘› áž?ážźá&#x;’áž”á&#x;† áž•áž? ážť ដáž&#x;មឞ đ?‘› = 20 á&#x;”

781.ď ° ដយឞ áž„áž˜ážśáž“ đ??´ đ?œ‹ đ??ľ+đ??ś = tan − 4 4 4 đ??´ đ??ľ đ??ś đ??´ đ??ľ đ??ľ đ??ś đ??ś đ??´ â&#x;š tan + tan + tan + tan tan + tan tan + tan tan 4 4 4 4 4 4 4 4 4 đ??´ đ??ľ đ??ś = 1 + tan tan tan 4 4 4 áž?ážśáž˜ážœ ឡáž&#x;áž˜áž—ážśáž–áž€ážźáž&#x;ឝីដយឞ ងទវញបវន tan

tan

3 đ??´ đ??ľ đ??ś đ??´ đ??ľ đ??ś + tan + tan ≼ 3 tan tan tan 4 4 4 4 4 4

3 đ??´ đ??ľ đ??ľ đ??ś đ??ś đ??´ đ??´ đ??ľ đ??ś tan tan + tan tan + tan tan ≼ 3 tan tan tan 4 4 4 4 4 4 4 4 4

2

ដសដá&#x;’áž“á&#x;„á&#x;‡ 3 3 đ??´ đ??ľ đ??ś đ??´ đ??ľ đ??ś đ??´ đ??ľ đ??ś 1 + tan tan tan ≼ 3 tan tan tan + 3 tan tan tan 4 4 4 4 4 4 4 4 4

áž?ážśáž„ đ?‘Ą =

3

đ??´

đ??ľ

2

đ??ś

tan 4 tan 4 tan 4 > 0 á&#x;” ដសដá&#x;’áž“á&#x;„á&#x;‡

1 + đ?‘Ą 3 ≼ 3đ?‘Ą + 3đ?‘Ą 2 â&#x;š đ?‘Ą ≤ 2 − 3; ážš ážş đ?‘Ą ≼ 2 + 3 ដá&#x;„áž™ đ??´, đ??ľ, đ??ś ជវរងá&#x;’áž ážśáž&#x;á&#x;‹ áž˜ážťá&#x;†áž€áž“áž„ ážť áž“áž“áž‘á&#x;’áž?ីដកវណ ដ á&#x;„á&#x;‡ 0 < đ?‘Ą < 1 ដសដá&#x;’áž“á&#x;„á&#x;‡ đ?‘Ą ≤ 2 − 3 á&#x;” áž˜ážśáž“áž“á&#x;?áž™áž?ážś đ??´ đ??ľ đ??ś 3 tan tan tan ≤ 2 − 3 4 4 4 đ??´ đ??ľ đ??ś đ?œ‹ áž&#x;ញá&#x;’ញវដáž&#x;មឞ ដកឞáž?áž˜ážśáž“ážŠáž–áž› tan 4 = tan 4 = tan 4 = 2 − 3 ážš ឺដពល đ??´ = đ??ľ = đ??ś = 3 á&#x;” ដសដá&#x;’áž“á&#x;„á&#x;‡áž?á&#x;† áž“áž› 3

áž’á&#x;† áž”á&#x;†áž•áž? ážť ážšážş max đ?‘‡ = 2 − 3 á&#x;”

782.ď ° ដយឞ áž„áž˜ážśáž“ 0 < đ?‘› + 1 đ?‘Ľ < đ?œ‹/2 ដសដá&#x;’áž“á&#x;„á&#x;‡ 0 < đ?‘›đ?‘Ľ < 2 ; 0 < đ?‘Ľ < đ?œ‹/2 á&#x;” ážœ ឡáž&#x;áž˜áž—ážśáž–áž˘ážśá&#x;’ đ?œ‹

áž&#x;រដáž&#x;រជវ tan đ?‘›đ?‘Ľ sin đ?‘Ľ + cos2đ?‘› đ?‘Ľ > 1 áž?ážśáž„ đ?‘“ đ?‘› = tan đ?‘›đ?‘Ľ sin đ?‘Ľ + cos2đ?‘› đ?‘Ľ á&#x;” ដយឞ ងនរងបងá&#x;’ហវញáž?ážś ដយឞ áž„áž˜ážśáž“

đ?‘“ đ?‘˜ + 1 > đ?‘“ đ?‘˜ , ∀đ?‘˜ = 0,1,2, ‌ , đ?‘› − 1 đ?‘“ đ?‘˜+1 >đ?‘“ đ?‘˜ â&#x;ş tan đ?‘˜ + 1 đ?‘Ľ sin đ?‘Ľ + cos2

លរម áž&#x;ážť វណá&#x;’ážŽážœážˇáž…ážˇáž?á&#x;’ážš | V. ážœážˇáž&#x;áž˜áž—ážśáž–

đ?‘˜+1

đ?‘Ľ > tan đ?‘˜đ?‘Ľ sin đ?‘Ľ + cos 2đ?‘˜ đ?‘Ľ 197


â&#x;ş cos 2đ?‘˜ đ?‘Ľ − cos 2đ?‘˜+2 đ?‘Ľ < sin đ?‘Ľ tan đ?‘˜ + 1 đ?‘Ľ − tan đ?‘˜đ?‘Ľ sin đ?‘Ľ sin đ?‘Ľ â&#x;ş cos 2đ?‘˜ đ?‘Ľ sin2 đ?‘Ľ < cos đ?‘˜ + 1 đ?‘Ľ cos đ?‘˜đ?‘Ľ 1 â&#x;ş cos 2đ?‘˜ đ?‘Ľ < cos đ?‘˜ + 1 đ?‘Ľ cos đ?‘˜đ?‘Ľ â&#x;ş cos 2đ?‘˜ đ?‘Ľ cos đ?‘˜ + 1 đ?‘Ľ cos đ?‘˜đ?‘Ľ < 1 áž– ឡáž?á&#x;” ដសដá&#x;’áž“á&#x;„á&#x;‡ đ?‘“ đ?‘› > đ?‘“ đ?‘› − 1 > â‹Ż > đ?‘“ 0 = 1 áž– ឡáž?á&#x;”

783.ď ° ដបឞ đ??´, đ??ľ, đ??ś áž‡ážśáž˜ážťá&#x;†áž€áž“áž„ ážť áž“áž“áž‘á&#x;’áž?ីដកវណ ដ á&#x;„á&#x;‡ đ??´ đ??ľ đ??ś đ??´ đ??ľ đ??ś + cot + cot = cot cot cot ; 2 2 2 2 2 2 đ??´ đ??ľ đ??ľ đ??ś đ??ś đ??´ tan tan + tan tan + tan tan = 1 2 2 2 2 2 2 cot

ážœ ឡáž&#x;áž˜áž—ážśáž–áž&#x;áž˜áž˜ážźáž›áž“ážšáž„ đ??´ đ??ľ đ??ľ đ??ś đ??ś đ??´ đ??´ đ??ľ đ??ś 27 tan tan2 + tan tan2 + tan tan2 < 4 cot cot cot 2 2 2 2 2 2 2 2 2 đ??´ đ??ľ đ??ś đ??´ đ??ľ đ??ľ đ??ś đ??ś đ??´ 4 2 2 2 â&#x;ş tan tan tan tan tan + tan tan + tan tan < 2 2 2 2 2 2 2 2 2 27 đ??ľ đ??ś đ??ś đ??´ đ??´ đ??ľ áž?ážśáž„ đ?‘Ľ = tan tan ; đ?‘Ś = tan tan ; đ?‘§ = tan tan á&#x;” ដសដá&#x;’áž“á&#x;„á&#x;‡ đ?‘Ľ, đ?‘Ś, đ?‘§ > 0; đ?‘Ľ + đ?‘Ś + đ?‘§ = 1 2

2

2

2

2

2

ដហឞ យវ ឡáž&#x;áž˜áž—ážśáž–áž&#x;áž˜áž˜ážźáž›áž“ážšáž„ 4 (1) 27 áž?ážśáž„ đ?‘ƒ = đ?‘Ľ 2 đ?‘Ś + đ?‘Ś 2 đ?‘§ + đ?‘§ 2 đ?‘Ľ á&#x;” áž&#x;នមáž?áž?ážś đ?‘Ľ ≼ đ?‘Ś > 0; đ?‘Ľ ≼ đ?‘§ > 0 â&#x;š đ?‘Ś 2 đ?‘§ ≤ đ?‘Ľđ?‘Śđ?‘§; đ?‘§ 2 đ?‘Ľ ≤ đ?‘Ľ 2 đ?‘§ á&#x;” đ?‘Ľ2 đ?‘Ś + đ?‘Ś2 đ?‘§ + đ?‘§2 đ?‘Ľ <

ដសដá&#x;’áž“á&#x;„á&#x;‡ 1 1 đ?‘ƒ = đ?‘Ľ 2 đ?‘Ś + đ?‘Ś 2 đ?‘§ + đ?‘§ 2 đ?‘Ľ ≤ x 2 đ?‘Ś + đ?‘Ľđ?‘Śđ?‘§ + đ?‘§ 2 đ?‘Ľ + đ?‘Ľ 2 đ?‘§ 2 2 1 = đ?‘Ľđ?‘Ś đ?‘Ľ + đ?‘§ + đ?‘Ľđ?‘§ đ?‘Ľ + đ?‘§ 2 đ?‘§ đ?‘Ľ đ?‘Ľ+đ?‘§ đ?‘§ =đ?‘Ľ đ?‘Ľ+đ?‘§ đ?‘Ś+ = 4. đ?‘Ś+ 2 2 2 2 đ?‘Ľ đ?‘Ľ+đ?‘§ đ?‘§ đ?‘Ľ đ?‘Ľ+đ?‘§ ដទá&#x;’បឞវ ឡáž&#x;áž˜áž—ážśáž–áž€ážźáž&#x;ឝី á&#x;’á&#x;† ដ á&#x;„á&#x;‡áž?áž˝ , , đ?‘Ś + ដហឞ យដá&#x;„យដរងáž?ážś ≠ដយឞ ងទវញបវន 2

2

2

đ?‘Ľ đ?‘Ľ+đ?‘§ 1 + 2 +đ?‘Ś +2đ?‘§ 2 đ?‘ƒ<4 3

3

2

�+�+� =4 3

2

3

=

4 27

784.ď ° áž?ážśáž„ đ?‘Ž, đ?‘?, đ?‘? ជវរងá&#x;’áž ážśáž&#x;á&#x;‹ áž‘á&#x;’ជá&#x;’ážťáž„ážˆáž˜áž“ážšáž„áž˜ážťá&#x;† đ??´, đ??ľ, đ??ś á&#x;” áž?ážśáž˜áž‘á&#x;’ážšážšáž&#x;áž?ីបរáž&#x;ឝី áž“áž&#x; ážť ដយឞ áž„áž˜ážśáž“ sin đ??´ =

198

đ?‘Ž đ?‘? đ?‘? ; sin đ??ľ = ; sin đ??ś = 2đ?‘… 2đ?‘… 2đ?‘… đ?‘Ž2 + đ?‘? 2 â&#x;š =đ?‘š đ?‘?2

á&#x;Ł. áž?á&#x;’រីកá&#x;„ណá&#x;’áž˜ážśáž?á&#x;’ážš | លរម áž&#x;ážť វណá&#x;’ážŽážœážˇáž…ážˇáž?á&#x;’ážš


áž?ážśáž˜áž‘á&#x;’ážšážšáž&#x;áž?ីបរកសáž&#x;ឝីនáž&#x; ážť ដយឞ ងទវញបវន đ?‘Ž2 + đ?‘? 2 = đ?‘š đ?‘Ž2 + đ?‘? 2 + 2đ?‘Žđ?‘? cos đ??ś đ?‘š − 1 đ?‘Ž2 + đ?‘? 2 đ?‘šâˆ’1 â&#x;š cos đ??ś = ≼ đ?‘š 2đ?‘Žđ?‘? đ?‘š đ?‘š − 1 2 2đ?‘š − 1 2 2 â&#x;š sin đ??ś = 1 − cos đ??ś ≤ 1 − = đ?‘š2 đ?‘š2 2đ?‘š − 1 â&#x;š sin đ??ś ≤ đ?‘š 2đ?‘šâˆ’1 áž&#x;ញá&#x;’ញវដáž&#x;មឞ ដកឞáž?áž˜ážśáž“ážŠáž–áž› đ?‘Ž = đ?‘? ážš ážş đ??´ = đ??ľá&#x;” ដសដá&#x;’áž“á&#x;„á&#x;‡ max sin đ??ś = đ?‘š á&#x;”

785.ď ° ដយឞ áž„áž˜ážśáž“ đ?‘“ tan 2đ?‘Ľ = tan4 đ?‘Ľ + cot 4 đ?‘Ľ = tan2 đ?‘Ľ + cot 2 đ?‘Ľ 2 − 2 tan2 đ?‘Ľ cot 2 đ?‘Ľ = tan đ?‘Ľ + cot đ?‘Ľ 2 − 2 tan đ?‘Ľ cot đ?‘Ľ 2 − 2 2 1 = −2 −2 sin2 cos2 đ?‘Ľ 2 4 = − 4 + 2 −2 sin2 2đ?‘Ľ 2 cos 2 2đ?‘Ľ = 4 +2 −2 sin2 2đ?‘Ľ 2 4 = + 2 −2 tan2 2đ?‘Ľ 16 16 = + +2 4 tan 2đ?‘Ľ tan2 2đ?‘Ľ áž?ážśáž„ đ?‘Ą = tan 2đ?‘Ľ â&#x;š đ?‘“ đ?‘Ą = 16 đ?‘Ą 4 + 16 đ?‘Ą 2 + 2 á&#x;” ដសដá&#x;’áž“á&#x;„á&#x;‡ 1 1 1 1 đ?‘“ sin đ?‘Ľ + đ?‘“ cos đ?‘Ľ = 16 + + 2 + +4 4 4 sin đ?‘Ľ cos đ?‘Ľ sin đ?‘Ľ cos2 đ?‘Ľ = 16 4 + 3 cot 2 đ?‘Ľ + tan2 đ?‘Ľ + cot 4 đ?‘Ľ + tan4 đ?‘Ľ + 4 áž?ážśáž˜ážœ ឡáž&#x;áž˜áž—ážśáž–áž€ážźáž&#x;ឝី ដយឞ áž„áž˜ážśáž“ cot 2 đ?‘Ľ + tan2 đ?‘Ľ ≼ 2 cot 4 đ?‘Ľ + tan4 đ?‘Ľ ≼ 2 áž&#x;ញá&#x;’ញវដáž&#x;មឞ ដកឞáž?áž˜ážśáž“ážŠáž–áž› tan2 đ?‘Ľ = 1 á&#x;” ដសដá&#x;’áž“á&#x;„á&#x;‡ đ?‘“ sin đ?‘Ľ + đ?‘“ cos đ?‘Ľ ≼ 16 4 + 3.2 + 2 + 4 = 196 đ?œ‹ đ?œ‹ áž&#x;ញá&#x;’ញវដáž&#x;មឞ ដកឞáž?áž˜ážśáž“ážŠáž–áž› đ?‘Ľ = 4 + đ?‘˜ 2 đ?‘˜ ∈ ℤ á&#x;”

786.ď ° ជវដá&#x;†áž”សងដយឞ ងបងá&#x;’ហវញáž?ážśážœ ឡáž&#x;áž˜áž—ážśáž–

đ?‘Ž đ?‘Žâˆ’đ?‘Ą ≼ 1 đ?‘? đ?‘?−đ?‘Ą áž– ឡáž?á&#x;’á&#x;† ដ á&#x;„á&#x;‡áž‘á&#x;’ážšáž”á&#x;‹ đ?‘Ž, đ?‘?, đ?‘Ą ∈ â„? លដល đ?‘Ž < đ?‘?; đ?‘? > đ?‘Ą ≼ 0 á&#x;” ដយឞ áž„áž˜ážśáž“ លរម áž&#x;ážť វណá&#x;’ážŽážœážˇáž…ážˇáž?á&#x;’ážš | V. ážœážˇáž&#x;áž˜áž—ážśáž–

199


đ?‘Ž đ?‘Žâˆ’đ?‘Ą ≼ đ?‘? đ?‘?−đ?‘Ą

â&#x;ş

đ?‘Ž đ?‘?−đ?‘Ą ≼ đ?‘Žâˆ’đ?‘Ą đ?‘?

â&#x;ş đ?‘Žđ?‘? − đ?‘Žđ?‘Ą ≼ đ?‘Žđ?‘? − đ?‘?đ?‘Ą â&#x;ş đ?‘Žđ?‘Ą ≤ đ?‘?đ?‘Ą â&#x;ş 0 ≤ đ?‘Ą(đ?‘? − đ?‘Ž) áž– ឡáž?á&#x;” កនង ážť áž‘á&#x;’áž?ីដកវណđ??´đ??ľđ??ś áž˜áž˝áž™ ដបឞ đ??´ > đ??ľ > đ??ś ដ á&#x;„á&#x;‡ đ?‘Ž > đ?‘? > đ?‘? លដល đ?‘Ž, đ?‘?, đ?‘? ជវរងá&#x;’áž ážśáž&#x;á&#x;‹ áž‘á&#x;’ជá&#x;’ážťáž„ážˆáž˜áž“ážšáž„áž˜ážťá&#x;† đ??´, đ??ľ, đ??ś á&#x;” ដយឞ áž„áž˜ážśáž“ đ?‘Ž = 2đ?‘… sin đ??´ > đ?‘? = 2đ?‘… sin đ??ľ > đ?‘? = 2đ?‘… sin đ??ś ដសដá&#x;’áž“á&#x;„á&#x;‡ sin đ??´ > sin đ??ľ > sin đ??ś á&#x;” លដនកá&#x;†áž“áž?á&#x;‹ ážšáž”áž&#x;á&#x;‹ đ?‘Ś :

á&#x;’á&#x;† ដ á&#x;„á&#x;‡ đ?‘Ľ ≼ sin đ??´ :

đ?‘Ľ − sin đ??´ ≼0 đ?‘Ľ − sin đ??ś â&#x;ş đ?‘Ľ − sin đ??ľ ≼0 đ?‘Ľ − sin đ??ś

đ?‘Ľ ≼ sin đ??´ đ?‘Ľ < sin đ??ś â&#x;ş đ?‘Ľ ≼ sin đ??´ đ?‘Ľ ≼ sin đ??ľ đ?‘Ľ < sin đ??ś đ?‘Ľ < sin đ??ś

ដទá&#x;’បឞវ ឡáž&#x;áž˜áž—ážśáž–(á&#x;Ą)á&#x;’á&#x;† ដ á&#x;„á&#x;‡ đ?‘Ą = đ?‘Ľ − sin đ??´ ≼ 0 đ?‘Ľ − sin đ??´ đ?‘Ľ − sin đ??´ − đ?‘Ľ − sin đ??´ ≼ =0 đ?‘Ľ − sin đ??ś đ?‘Ľ − sin đ??ś − đ?‘Ľ − sin đ??´ đ?‘Ľ − sin đ??ľ đ?‘Ľ − sin đ??ľ − đ?‘Ľ − sin đ??´ sin đ??´ − sin đ??ľ ≼ = đ?‘Ľ − sin đ??ś đ?‘Ľ − sin đ??ś − đ?‘Ľ − sin đ??´ sin đ??´ − sin đ??ś â&#x;š

� ≼ 0+

sin đ??´ − sin đ??ľ −1 sin đ??´ − sin đ??ś

áž&#x;ញá&#x;’ញវដáž&#x;មឞ ដកឞáž?áž˜ážśáž“ážŠáž–áž› đ?‘Ľ = sin đ??´ á&#x;” ដហឞ áž™ sin đ??´ − sin đ??ľ sin đ??´ − sin đ??ľ −1≤1 â&#x;ş ≤4 sin đ??´ − sin đ??ś sin đ??´ − sin đ??ś â&#x;ş sin đ??´ − sin đ??ľ ≤ 4 sin đ??´ − 4 sin đ??ś â&#x;ş 0 ≤ 3 sin đ??´ + sin đ??ľ − 4 sin đ??ś â&#x;ş 0 ≤ 3 sin đ??´ − sin đ??ľ + sin đ??ľ − sin đ??ś áž– ឡáž?á&#x;” á&#x;’á&#x;† ដ á&#x;„á&#x;‡ đ?‘Ľ < sin đ??ś: ដយឞ áž„áž˜ážśáž“ đ?‘Ś > 1+ 1−1=1 ដសដá&#x;’áž“á&#x;„á&#x;‡ min đ?‘Ś =

200

sin đ??´âˆ’sin đ??ľ sin đ??´âˆ’sin đ??ś

− 1 ដពល đ?‘Ľ = sin đ??´ á&#x;”

á&#x;Ł. áž?á&#x;’រីកá&#x;„ណá&#x;’áž˜ážśáž?á&#x;’ážš | លរម áž&#x;ážť វណá&#x;’ážŽážœážˇáž…ážˇáž?á&#x;’ážš


VI. វិសមីោរ 787.

𝜋 3

+ 2𝜋𝑛;

2𝜋 3

+ 2𝜋𝑛 ; 𝑛 ∈ ℤ ;

788. 2𝜋𝑛 − 2𝜋 3 ; 2𝜋𝑛 + 2𝜋/3 ; 𝑛 ∈ ℤ; 789. 𝜋𝑛 − arctan 2 ; 𝜋𝑛 + 𝜋/3 ; 𝑛 ∈ ℤ; 790. 𝜋𝑛; 𝜋𝑛 + 𝜋/2 ∪ 𝜋𝑛 + 3𝜋 4 ; 𝜋(𝑛 + 1) ; 𝑛 ∈ ℤ 791. – 𝜋 4 + 2𝜋𝑛; 𝜋 6 + 2𝜋𝑛 ∪ 5𝜋 6 + 2𝜋𝑛; 5𝜋 4 + 2𝜋𝑛 ; 𝑛 ∈ ℤ 1

792. 2𝑛 − 8 ; 2𝑛 + 7/8 ; 𝑛 ∈ ℤ 𝜋𝑛 5𝜋 𝜋(𝑛+1) 𝜋 3 + 24 ; 2 + 24 ; 𝑛 ∈ ℤ  cos 3 𝑥 sin 3𝑥 + cos 3𝑥 sin3 𝑥 = 4 sin 4𝑥 2 𝜋 𝜋 𝜋 794. 2 + 𝜋𝑛 ∪ 𝜋𝑛 − 4 ; 𝜋𝑛 − 𝜋/6 ∪ 𝜋𝑛 + 6 ; 𝜋𝑛 + 𝜋/4 ; 𝑛 ∈ ℤ  cos 2𝑥 2 cos 2 2𝑥+cos 2𝑥−1 cos 2𝑥+cos 4𝑥 cos 𝑥 cos 2𝑥 cos 3𝑥 = cos 2𝑥 = 2 2 3𝜋 3𝜋𝑛 795. ℝ ∖ + ;𝑛 ∈ ℤ 4 2 𝜋 𝜋𝑛 𝜋 𝜋𝑛 1−cos 2𝑥 2 1+cos 2𝑥 2 5 796. − + ; + ; 𝑛 ∈ ℤ  sin6 𝑥 + cos 6 𝑥 = + = + 8 2 8 2 2 2 8 3 cos 4𝑥 8 1 1 1 1 797. 𝜋𝑛 + arccos ; 𝜋 𝑛 + 1 − arccos ; 𝑛 ∈ ℤ  8 sin6 𝑥 − cos 6 𝑥 = 2 sin2 𝑥 2 3 2 3 2 4 2 2 4

793.

cos 𝑥 4 sin 𝑥 + 2 sin 𝑥 . cos 𝑥 + cos 𝑥 ប ទ ប់ មកដរៀត 4 sin4 𝑥 + 2 sin2 𝑥 . cos 2 𝑥 + cos4 𝑥 > 0, 𝜋 𝜋 𝜋 𝜋 798. 𝜋𝑛 − ; 𝜋𝑛 − ∪ 𝜋𝑛 + ; 𝜋𝑛 + ; 𝑛 ∈ ℤ 4

6

𝜋 4

799. 2𝜋𝑛 − +

6 2 2 3𝜋 arcsin 3 ; 2𝜋𝑛 + 4

∀𝑥 ∈ ℝ

4

− arcsin

2 2 3

∪ 𝜋 2𝑛 + 1 ; 2𝜋𝑛 +

3𝜋 2

;𝑛 ∈ ℤ 

តាង sin 𝑥 + cos 𝑥 = 𝑦 800. arctan

𝜋

2 − 1 ; 4 ∪ 𝜋 + arctan

801. 𝜋𝑛 + arcsin

5−1 ;𝜋 2

𝑛 + 1 − arcsin

2−1 ; 5−1 2

5𝜋 4

;𝑛 ∈ ℤ

802. 2𝜋𝑛 − 7𝜋 6 ; 2𝜋𝑛 + 𝜋 6 ; 𝑛 ∈ ℤ 803. 𝜋 4 + 𝜋𝑛; 𝜋 2 + 𝜋𝑛 ; 𝑛 ∈ ℤ 804. 𝑛 + 1 4 ; 𝑛 + 3/4 ; 𝑛 ∈ ℤ

លឹម សុ វណ្ណវិចិត្រ | VI. វិសមីោរ

201


VII. ត្រព័ន្វធ ស ិ មីោរ 805.  ដោយ 0 ≤ 𝑥 ≤ 𝜋/4 ដ ោះ cos 𝑥 ≠ 0 ។ ល្កសមីការរីមួយនឹង cos 3 𝑥 និងតាង tan 𝑥 = 𝑡 ដយើ ងទាញបាន 4 − 6𝑚 𝑡 3 + 3 2𝑚 − 1 𝑡 𝑡 2 + 1 + 2 𝑚 − 2 𝑡 2 − 4𝑚 − 3 𝑡 2 + 1 = 0 4 − 6𝑚 𝑡 3 + 6𝑚 − 3 𝑡 3 + 6𝑚 − 3 𝑡 + 2𝑚 − 4 𝑡 2 − 4𝑚 − 3 𝑡 2 − 4𝑚 + 3 = 0 𝑡 3 − 1 + 2𝑚 𝑡 2 + 3 2𝑚 − 1 𝑡 − 4𝑚 + 3 = 0 𝑡 − 1 𝑡 2 − 2𝑚𝑡 + 4𝑚 − 3 = 0 𝜋 ដយើ ងទាញបាន 𝑡 = 1 លដលទ្តូវនឹង 𝑥 = 4 ជារ ឺសមួយននទ្បព័ន។ ធ ដូដ្នោះទ្បព័នមា ធ នរ ឺសលតមួយរត់ ដបើ លកខសណឌមួយកនង ុ ្ំ ដ មលកខសណឌខាងដទ្កាមទ្តូវបានបំ ដពញ ១) សមីការ𝑡 2 − 2𝑚𝑡 + 4𝑚 − 3 = 0 មាន Δ = 𝑚2 − 4𝑚 + 3 < 0 ⟹ 1 < 𝑚 < 3 ២) សមីការ𝑡 2 − 2𝑚𝑡 + 4𝑚 − 3 = 0 មានរ ឺសមួយរត់ ដសមើ 1 ។ ដូដ្នោះ 1 − 2𝑚 + 4𝑚 − 3 = 0; 𝑚 = 1។ ដបើ 𝑚 = 1 ដ ោះ Δ = 0 សមីការមានរ ឺសមួយរត់ ។ ៣) សមីការ𝑡 2 − 2𝑚𝑡 + 4𝑚 − 3 = 0 មានរ ឺស 𝑡1 ≤ 𝑡2 < 0 ដូដ្នោះ វ ិសមីការដនោះគ្នមនរ ឺស។

Δ > 0; 𝑡1 𝑡2 = 4𝑚 − 3 > 0; 𝑆 = 𝑡1 + 𝑡2 = 2𝑚 < 0

ដូដ្នោះដដើមបីដអាយទ្បព័នមា ធ នរ ឺសមួយរត់ 1 ≤ 𝑚 < 3 ។

202

៣. ត្រីកោណ្មាត្រ | លឹម សុ វណ្ណវិចិត្រ


៤. ពហុ ធា

គណ្នា 806.  លក្ខខណ្ឌដែលអោយសមមូលនឹង 𝑃 𝑥𝑦 = 𝑃 𝑥 . 𝑃 𝑦 ; ∀𝑥, 𝑦 ∈ ℝ ក្នង ុ (1) អយើ ងយក្ 𝑥 = 𝑦 = 0 អយើ ងទាញបាន

(1)

𝑃 0 = 𝑃2 0 ែូអ្នេះ 𝑃 0 = 1 រ ឺ 𝑃 0 = 0 ។ ១) អ ើ 𝑃 0 = 1 អ េះ ក្នង ុ (1) យក្ 𝑦 = 0 អយើ ងទាញបាន 𝑃 0 = 𝑃 𝑥 . 𝑃(0) អ េះ 𝑃 𝑥 = 1; ∀𝑥 ∈ ℝ ២) អ ើ 𝑃 0 = 0 អ េះ 𝑃 𝑥 = 𝑥𝑄 𝑥 ដែល 𝑄 𝑥 ជាពហុ ធាមានែឺអរក្ទា ជាង 𝑃 𝑥 មួយឯក្តា ។ លក្ខខណ្ឌ(1) អៅជា 𝑥𝑦𝑄 𝑥𝑦 = 𝑥𝑦𝑄 𝑥 . 𝑄 𝑦 , ∀𝑥, 𝑦 ∈ ℝ អ េះ 𝑄 𝑥𝑦 = 𝑄 𝑥 . 𝑄 𝑦 ; ∀𝑥, 𝑦 ∈ ℝ ។ ែូអ្នេះអយើ ងទាញបាន 𝑄 𝑥 = 1 រ ឺ 𝑄 𝑥 = 𝑥 𝑄1 𝑥 ដែល 𝑄1 𝑥 ជាពហុ ធាមានែឺអរក្ទា ជាង 𝑄 𝑥 មួយឯក្តា។ ែូអ្នេះជាសរុ អយើ ងទាញបាន 𝑃 𝑥 = 1; ∀𝑥 ∈ ℝ 𝑃 𝑥 = 𝑥 𝑛 ; ∀𝑥 ∈ ℝ ដែល 𝑛 ជា្ំ នន ួ គត់ វ ិជ្ជមាន។

807.  អោយែឹងថា 2𝑥 + 1 = 𝑥 + 1

2

− 𝑥 2 អ េះលក្ខខណ្ឌដែលអោយសមមូលនឹង

𝑃 𝑥 + 1 − 𝑥 + 1 2 = 𝑃 𝑥 − 𝑥2 តាង 𝑄 𝑥 = 𝑃 𝑥 − 𝑥 2 ។ ែូអ្នេះ 𝑄 𝑥 = 𝑄 𝑥 + 1 = ⋯ = 𝑄 𝑥 + 𝑛 = ⋯ ។ ែូអ្នេះ 𝑄 0 = 𝑄 1 = ⋯ = 𝑄 𝑛 = ⋯ ។ អយើ ងទាញបាន 𝑄 𝑥 − 𝑄 0 ≡ 0 ្ំ អ េះ 𝑥 = 0, 𝑥 = 1, … , 𝑥 = 𝑛, … ។ ែូអ្នេះ 𝑄 𝑥 − 𝑄 0 ≡ 0, ∀𝑥 ∈ ℝ ។ អយើ ងទាញបាន 𝑃 𝑥 = 𝑥2 + 𝐶 ដែល 𝐶 = 𝑄 0 = 𝑃 0 ។ ជ្ំនស ួ ទំ ក្់ ទំនងដែលបាន្ូលលក្ខខណ្ឌអែើម អយើ ងទាញបាន 𝑥+1 លឹម សុ វណ្ណវិចិត្រ | I. គណ្នា

2

+ 𝐶 = 𝑥 2 + 2𝑥 + 1 + 𝐶 203


ព ិត្ំ អ េះរគ ់ 𝐶 ។ ែូអ្នេះ 𝑃 𝑥 = 𝑥2 + 𝐶 ដែល 𝐶 ជា្ំ នន ួ អេរណាមួយ។

808.  អយើ ងមាន 𝑃 𝑥 + 1 2 − 𝑥 + 1 2 = 𝑃 𝑥2 − 𝑥2 តាង 𝑄 𝑡 = 𝑃 𝑡 − 𝑡 ។ អយើ ងទាញបាន 𝑄 0 = 𝑄 1 = ⋯ = 𝑄 𝑛2 = ⋯ ។ ែូអ្នេះ 𝑄 𝑥 = 𝑄(0) ។ អយើ ងទាញបាន 𝑃 𝑥 = 𝑥 + 𝐶 ដែល 𝐶 = 𝑄 0 = 𝑃 0 ។ ជ្ំនស ួ ្ូលលក្ខខណ្ឌ អែើម អយើ ងទាញបាន 𝑃 𝑥 = 𝑥 + 𝐶 ដែល𝐶 ជា្ំ នន ួ អេរណាមួយ។

809.  ពហុ ធា 𝑓 𝑥 − 5 មានរ ឺសជា្ំ នន ួ គត់ 𝑎, 𝑏, 𝑐, 𝑑 ។ ែូអ្នេះ 𝑓 𝑥 −5 = 𝑥−𝑎 𝑥−𝑏 𝑥−𝑐 𝑥−𝑑 𝑔 𝑥 ដែល 𝑔 𝑥 = 𝑥 𝑚 + 𝑏1 𝑥 𝑚 −1 + ⋯ + 𝑏𝑚 និង 𝑏1 , 𝑏2 , … , 𝑏𝑚 ជា្ំ នន ួ គត់ ។ សមីការ 𝑓 𝑥 = 8 សមមូលនឹង 𝑥−𝑎 𝑥−𝑏 𝑥−𝑐 𝑥−𝑑 𝑔 𝑥 =3 ែូអ្នេះ ីក្ង នុ ្ំ អ ម្ំ នន ួ គត់ ួន 𝑥 − 𝑎, 𝑥 − 𝑏, 𝑥 − 𝑐, 𝑥 − 𝑑 អសមើ 1 រ ឺ −1។ ែូអ្នេះរតូវមានព ីរដែល អសមើ គ្នន ដែលក្រណ្ីអនេះមិនោ្ អរ េះ្ំ នន ួ ទាំង ួនខុស។

810. តាង 𝑃 𝑋 = 𝑎0 + 𝑎1 𝑋 + 𝑎2 𝑋 2 + ⋯ + 𝑎𝑛 𝑋 𝑛 អោយ 𝑎𝑖 ≥ 0 ្ំ អ េះ 𝑖 < 𝑛 និង 𝑎𝑛 > 0 ។ អយើ ងមាន 𝑥1 , 𝑥2 , … , 𝑥𝑛 > 0 ។ តាង 𝑥𝑛+1 = 𝑥1 ។ អ ើ 𝑛 = 1 អយើ ងទាញបាន 𝑃 1

2

= 1. 𝑃 1

អ ើ 𝑛 ≥ 2 អយើ ងមាន

2

ព ិត។

𝑛

𝑎𝑖2 𝑋 2𝑖 + 2

2

𝑃 𝑋 = 𝑖=0

𝑖<𝑗

្ំ អ េះ 𝑝 ∈ ℕ∗ តាង 1 𝑆𝑝 = 𝑛 តាមវ ិសមភាពក្ូសុី 𝑛

𝑆𝑝 ≥ 𝑘=1

ដតថា 204

៤. ពហុ ធា | លឹម សុ វណ្ណវិចិត្រ

𝑎𝑖 𝑎𝑗 𝑋 𝑖+𝑗

𝑛

𝑘=1

𝑥𝑘+1 𝑥𝑘

𝑥𝑘+1 𝑥𝑘

𝑝

1 𝑝 𝑛

=1


1 𝑛 𝑛

𝑛

𝑃 𝑘=1

𝑃2

⟹ 𝑘=1

2

𝑥𝑘+1 = 𝑥𝑘

𝑛

𝑛

𝑎𝑖2 𝑆2𝑖

+2

𝑖=0

𝑎𝑖2 + 2

𝑎𝑖 𝑎𝑗 𝑆𝑖+𝑗 ≥ 𝑖<𝑗

𝑖=0

𝑎𝑖 𝑎𝑗 = 𝑃2 1 𝑖<𝑗

𝑥𝑘+1 ≥ 𝑛𝑃2 1 𝑥𝑘

អសមើ គ្នន លុ េះរតាដត 𝑥1 = 𝑥2 = ⋯ = 𝑥𝑛 ។

លឹម សុ វណ្ណវិចិត្រ | I. គណ្នា

205


206

៤. ពហុ ធា | លឹម សុ វណ្ណវិចិត្រ


៥. សមី ការអនុគមន៍

គណ្នា 811.  តាមលក្ខខណ្ឌទី១យ ើ ងទាញបាន 𝑓 𝑓 𝑓 𝑥 +1

=𝑓 1−𝑥

តាមលក្ខខណ្ឌទី២ យ ើ ងទាញបាន 𝑓 𝑥 +1=𝑓 1−𝑥 , ជំនស ួ 𝑡 = 1 − 𝑥 យ ើ ងទាញបាន

∀𝑥 ∈ ℝ

(1)

𝑓 1−𝑡 +1 = 𝑓 𝑡 ដូយចនេះ 𝑓 1 − 𝑥 + 1 = 𝑓 𝑥 , ∀𝑥 ∈ ℝ ជំនស ួ ចូល(1) យ ើ ងទាញបានថា គ្មានអនុគមន៍ណាយផទៀងផ្ទទត់ លក្ខខណ្ឌយទ។

812.  យ ើ ងមាន 𝑓 1 + 𝑓 2 = 4𝑓 2

⟹𝑓 2 =

𝑓 1 3

2

=3

(1) ។ ចំ យ េះ 𝑛 ≥ 3 យ ើ ង

មាន 𝑓 1 + 𝑓 2 + ⋯ + 𝑓 𝑛 − 1 + 𝑓 𝑛 = 𝑛2 𝑓 𝑛 𝑓 1 + 𝑓 2 + ⋯ + 𝑓 𝑛 − 1 = 𝑛 − 1 2𝑓 𝑛 − 1 យ ើ ងទាញបាន 𝑓 𝑛 = 𝑛2 𝑓 𝑛 − 𝑛 − 1 2 𝑓 𝑛 − 1 𝑛−1 ⟹ 𝑓 𝑛 = 𝑓 𝑛−1 𝑛+1 ដូយចនេះចំ យ េះ 𝑛 ≥ 3 យ ើ ងទាញបាន 𝑛 − 1 𝑛 − 2 …2 6 4 𝑓 𝑛 = 𝑓 2 = 𝑓 2 = 𝑛 + 1 𝑛…4 𝑛 𝑛+1 𝑛 𝑛+1 813.  យ ើ ងនឹងបង្ហាញថា 𝑓 𝑥 = 0 ជាចំ យលើ តតមួ គត់ ។ សនាតថាចំ យោទមានចំ យលើ 𝑓 𝑥 ≠ 0 ។ ដូយចនេះមានចំ នន ួ 𝑎 ∈ ℝ តដល 𝑓 𝑎 = 𝑏 ≠ 0 ។ យ ើ ង ក្ 𝑦 = 𝑎 យ ើ ងទាញបាន 𝑓 𝑥𝑏 = 𝑥 𝑛 𝑓 𝑏 តាង 𝑡 = 𝑥𝑏 ។ យ ើ ងទាញបាន 𝑓 𝑡 = លឹម សុ វណ្ណវិចិត្រ | I. គណ្នា

𝑓 𝑏 𝑛 𝑡 = 𝐶𝑡 𝑛 𝑏𝑛 207


តដល 𝐶 ជាចំ នន ួ យេរ។ ជំនស ួ 𝑓 𝑥 = 𝐶𝑥 𝑛 ចូលក្នង ុ លក្ខខណ្ឌ យ ើ ងទាញបាន 𝐶 = 0 ដូយចនេះ 𝑓 𝑥 =0។

814.  លក្ខខណ្ឌ 𝑎𝑓 𝑥 2 + 𝑦𝑧 + 𝑏𝑓 𝑦 2 + 𝑧𝑥 + 𝑐𝑓 𝑧 2 + 𝑥𝑦 = 0 (1) មានលក្ខណ្ៈឆ្េះុ យ ៀប នឹង 𝑥, 𝑦, 𝑧 ។ ដូយចនេះយោ សារលក្ខណ្ៈយនេះ និងយោ សារ 𝑎, 𝑏, 𝑐 មិនសូ នយទាំងបីព្ពមគ្មន យ ើ ងអាច សនាតថា 𝑎 ≠ 0 ។ ក្ 𝑦 = 𝑧 = 0 យ ើ ងបាន 𝑎𝑓 𝑥 2 + 𝑏 + 𝑐 𝑓 0 = 0 ⟹ 𝑓 𝑥 2 = −

𝑏+𝑐 𝑓(0) 𝑎

ចំ យ េះព្គប់ 𝑥

។ ក្ 𝑥 = 0, 𝑧 = 1 យ ើ ងបាន 𝑎𝑓 𝑦 + 𝑏𝑓 𝑦 2 + 𝑐𝑓 0 = 0 ។ ដូយចនេះ 𝑏+𝑐 − 𝑎 𝑓 0 + 𝑐𝑓 0 𝑏𝑓 𝑦 2 + 𝑐𝑓 0 𝑏 2 + 𝑏𝑐 − 𝑎𝑐 𝑓 𝑦 =− =− = .𝑓 0 𝑎 𝑎 𝑎2 ចំ យ េះព្គប់ 𝑦 ∈ ℝ ដូយចនេះ 𝑓 ជាអនុគមន៍យេរយលើ ℝ ។ តាង 𝑓 𝑥 = 𝐾 ។ ជំនស ួ ចូលក្នង ុ (1) យ ើ ងទាញបាន 𝑎+𝑏+𝑐 𝐾 = 0 យបើ 𝑎 + 𝑏 + 𝑐 ≠ 0 យោេះ 𝐾 = 0 ដូយចនេះ 𝑓 𝑥 ≡ 0 ។ យបើ 𝑎 + 𝑏 + 𝑐 = 0 យោេះ 𝐾 ជាចំ នន ួ យេរណាមួ ក្៏បាន។

815. 

ក្ 𝑦 = 1 យ ើ ងទាញបាន 𝑓 𝑥𝑓 1

= 𝑥 𝑝 ។ 𝑓 1 មិនអាចយសាើ សូនយបានយទ យព្ េះយបើយសាើ

សូ នយ យោេះ 𝑓 0 = 𝑥 𝑝 ចំ យ េះព្គប់ 𝑥 មិនអាច។ តាង 𝑐 = 𝑓 1 ដូយចនេះ 𝑓 𝑥 = 𝑥 𝑝 /𝑐 𝑝 ។ ដូយចនេះ 2

𝑥𝑓 𝑦 𝑝 𝑥 𝑝 𝑦 𝑝 𝑥 𝑦 = 𝑓 𝑥𝑓 𝑦 = = 𝑝+𝑝 2 𝑐𝑝 𝑐 2 ព ិតចំ យ េះព្គប់ 𝑥, 𝑦 ∈ ℝ ។ ដូយចនេះ 𝑐 = 1 និង 𝑞 = 𝑝 ។ យបើ 𝑞 = 𝑝2 យោេះយ ើ ងទាញបាន 𝑝 𝑞

𝑓 𝑥𝑓 𝑦

= 𝑥𝑝 𝑓 𝑦

𝑝

2

= 𝑥𝑝 𝑦𝑝 = 𝑥𝑝 𝑦𝑞

ចំ យ េះព្គប់ 𝑥, 𝑦 ∈ ℝ+ ។

816.  ព ីលក្ខខណ្ឌទីមួ

ចំ យ េះព្គប់ 𝑚 ∈ ℤ និង 𝑘 ∈ ℕ យ ើ ងមាន

𝑓 𝑚 + 𝑘. 19 ≥ 𝑓 𝑚 + 𝑘. 19 𝑓 𝑚 − 𝑘. 19 ≤ 𝑓 𝑚 − 𝑘. 19 ព ីលក្ខខណ្ឌទីព ីរ ចំ យ េះព្គប់ 𝑚 ∈ ℤ និង 𝑘 ∈ ℕ យ ើ ងមាន 𝑓 𝑚 + 𝑘. 99 ≤ 𝑓 𝑚 + 𝑘. 99 𝑓 𝑚 − 𝑘. 99 ≥ 𝑓 𝑚 − 𝑘. 99 សមីការ 1 = 19𝑥 + 99𝑦 មានរ ឺស 𝑥 = −26 + 99𝑡; 𝑦 = 5 − 19𝑡, 𝑡 ∈ ℤ ។ ចំ យ េះ 𝑡 = 0; 1 យ ើ ង ទាញបាន 𝑥; 𝑦 = 208

−26; 5 ; (73; −14) ។

៥. សមីការអនុគមន៍ | លឹម សុ វណ្ណវិចិត្រ


ចំ យ េះព្គប់ 𝑚 ∈ ℤ យ ើ ងមាន 𝑓 𝑚 + 1 = 𝑓 𝑚 − 19.26 + 99.5 ≤ 𝑓 𝑚 + 99.5 − 19.26 ≤ 𝑓 𝑚 + 99.5 − 19.26 =𝑓 𝑚 +1 𝑓 𝑚 + 1 = 𝑓 𝑚 + 73.19 − 14.99 ≥ 𝑓 𝑚 − 14.99 + 19.73 ≥ 𝑓 𝑚 − 14.99 + 19.73 =𝑓 𝑚 +1 ដូយចនេះ 𝑓 𝑚 + 1 = 𝑓 𝑚 + 1 ចំ យ េះ 𝑚 ∈ ℤ ។ ដូយចនេះ 𝑓 𝑚 + 𝑛 = 𝑓 𝑚 + 𝑛 ចំ យ េះព្គប់ 𝑚, 𝑛 ∈ ℤ ។

ក្ 𝑚 = 0 យ ើ ងទាញបាន 𝑓 𝑛 = 𝑓 0 + 𝑛។ តាង 𝑎 = 𝑓 0 ។ ដូយចនេះ

𝑓 𝑛 = 𝑎 + 𝑛 ។ ជំនស ួ ចូលក្នង ុ លក្ខខណ្ឌយដើមយ ើ ងទាញបាន 𝑓 𝑛 = 𝑛 + 𝑎 យផទៀងផ្ទទត់ ចំយ េះព្គប់ 𝑎∈ℤ។

817.  យអា

𝑥 = 𝑦 = 0 យ ើ ងទាញបាន 𝑓 0 = 0 ។ យអា

𝑦 = 0 យ ើ ងទាញបាន

𝑥𝑓 𝑥 = 𝑓 2 𝑥 ⟹ 𝑓 𝑥 = 0 រ ឹ 𝑓 𝑥 = 𝑥 ។ យ ើ ងយឃើ ញថា 𝑓 𝑥 = 0 ចំ យ េះព្គប់ 𝑥 ∈ ℝ រ ឺ 𝑓 𝑥 = 𝑥 ចំ យ េះព្គប់ 𝑥 ∈ ℝ សុ ទតធ តយផទៀងផ្ទទត់ លក្ខខណ្ឌតដលយអា ។ បោទប់ មក្យទៀតយ ើ ងបង្ហាញ ថា បនសំននអនុគមន៍ទាំងព ីរមិនយផទៀងផ្ទទត់ យទ មានន័ ថា អនុគមន៍ 0; 𝑥 ∈ 𝐷1 𝑓 𝑥 = 𝑥; 𝑥 ∈ 𝐷2 តដល 𝐷1 ∪ 𝐷2 = ℝ មិនយផទៀងផ្ទទត់ លក្ខខណ្ឌយទ។ ឧបមាផទុ ព ីយនេះថា មាន 𝑥1 ∈ 𝐷1 ; 𝑥2 ∈ 𝐷2 តដល 𝑓 𝑥1 = 0 និង 𝑓 𝑥2 = 𝑥2 ។ ចំ យ េះ 𝑥1 ∈ 𝐷1 តដល 𝑥1 ≠ 0 យគអាចរក្បាន យមគុណ្ 𝑎 មួ តដល 𝑎𝑥1 = 𝑥2 គឺ 𝑎 = 𝑥2 /𝑥1 ។ ជំនស ួ 𝑦 យោ

𝑥 និង 𝑥 យោ

𝑦 យ ើ ងទាញបាន

𝑥𝑓 𝑥 + 𝑦 + 𝑦𝑓 𝑦 − 𝑥 = 𝑦𝑓 𝑥 + 𝑦 + 𝑥𝑓 𝑥 − 𝑦 𝑥−𝑦 𝑓 𝑥+𝑦 = 𝑥−𝑦 𝑓 𝑥−𝑦 ព ិតចំ យ េះព្គប់ 𝑥, 𝑦 ∈ ℝ ។ ដូយចនេះ 𝑓 𝑥+𝑦 =𝑓 𝑥−𝑦 តាង 𝑡 = 𝑥 − 𝑦; ⟹ 𝑥 = 𝑡 + 𝑦 ដូយចនេះ 𝑓 𝑡 + 2𝑦 = 𝑓 𝑡 ចំ យ េះព្គប់ 𝑡, 𝑦 ∈ ℝ ។

ក្ 𝑦 = 𝑏𝑡 ។

ដូយចនេះ 𝑓 1 + 2𝑏 𝑡 = 𝑓 𝑡 ។ តាង 𝐾 = 1 + 2𝑏 យោេះ 𝑓 𝐾𝑡 = 𝑓(𝑡) ចំ យ េះព្គប់ 𝐾, 𝑡 ∈ ℝ ។ ដូយចនេះ យោ

ក្ 𝐾 = 𝑎; 𝑓 𝑎𝑥1 = 𝑓 𝑥1 ⟹ 𝑎𝑥1 = 0 អាចតតក្រណ្ី 𝑎𝑥1 = 𝑥2 = 0 មួ

បុយណាណេះ។ ដូយចនេះ 𝑓 𝑥 = 0 ចំ យ េះព្គប់ 𝑥 ∈ ℝ រ ឺ 𝑓 𝑥 = 𝑥 ចំ យ េះព្គប់ 𝑥 ∈ ℝ

818.  យអា 𝑥 = 𝑦 យ ើ ងទាញបាន 𝑓 0 = 0 ។ យអា

𝑦 = 1 យ ើ ងទាញបាន

𝑓 𝑥+1 𝑓 𝑥 −1 =𝑓 𝑥−1 𝑓 𝑥 +1 លឹម សុ វណ្ណវិចិត្រ | I. គណ្នា

1 209


យអា

យោ

𝑥 = 2 យ ើ ងទាញបាន 𝑓 3 𝑓 2 − 1 = 𝑓 2 + 1 ។ យោ 𝑓 2 ≠ 1 យោេះ 𝑓 2 +1 2 𝑓 3 = =1+ 𝑓 2 −1 𝑓 2 −1 𝑓 2 , 𝑓(3) ជាចំ នន ួ គត់ យោេះ 𝑓 2 − 1 ព្តូវតតជាតួតចក្នន 2 ។ ដូយចនេះ

𝑓 2 − 1 = 2 យោេះ 𝑓 2 = 3; 𝑓 3 = 2 𝑓 2 − 1 = 1 យោេះ 𝑓 2 = 2; 𝑓 3 = 3 𝑓 2 − 1 = −1 យោេះ 𝑓 2 = 0; 𝑓 3 = −1 𝑓 2 − 1 = −2 យោេះ 𝑓 2 = −1; 𝑓 3 = 0 1) ក្រណ្ី 𝑓 2 = 3; 𝑓 3 = 2 5

ក្នង ុ (១) ជំនស ួ 𝑥 = 3 យ ើ ងទាញបាន 𝑓 4 = 9 ។ ជំនស ួ 𝑥 = 4 យ ើ ងទាញបាន 𝑓 5 = 2 មិន ក្។ 2) ក្រណ្ី 𝑓 2 = 2; 𝑓 3 = 3 ។ តាមវ ិចារយោ ក្ំយនើន យ ើ ងទាញបាន 𝑓 𝑛 = 𝑛, ∀𝑛 ∈ ℤ 3) ក្រណ្ី 𝑓 2 = 0; 𝑓 3 = −1 ។ តាមវ ិចារយោ ក្ំយនើន យ ើ ងទាញបាន 0, 𝑛 = 2𝑘 𝑓 𝑛 = 1, 𝑛 = 4𝑘 + 1 (𝑘 ∈ ℤ) −1, 𝑛 = 4𝑘 − 1 4) ក្រណ្ី 𝑓 2 = −1; 𝑓 3 = 0 ។ តាមវ ិចារយោ ក្ំយនើន យ ើ ងទាញបាន 0, 𝑛 = 3𝑘 𝑓 𝑛 = 1, 𝑛 = 3𝑘 + 1 (𝑘 ∈ ℤ) −1, 𝑛 = 3𝑘 − 1 819.  យ ើ ងមាន 𝑓 1 = 1; 𝑓 2 = 2 ។ សនាតថា 𝑓 𝑘 = 𝑘 ចំ យ េះ 𝑘 = 1,2, … , 𝑛 ។ យ ើ ងនឹង បង្ហាញថា 𝑓 𝑛 + 1 = 𝑛 + 1 ។ យបើ 𝑛 + 1 = 2𝑗 យោេះ 1 ≤ 𝑗 < 𝑛 យ ើ 𝑓 𝑛 + 1 = 𝑓 2𝑗 = 𝑓 2 𝑓 𝑗 = 2𝑗 = 𝑛 + 1 យបើ 𝑛 + 1 = 2𝑗 + 1 យោេះ 1 ≤ 𝑗 < 𝑛 យ ើ ដូយចនេះ

2𝑗 = 𝑓 2𝑗 < 𝑓 2𝑗 + 1 < 𝑓 2𝑗 + 2 = 𝑓 2 𝑓 𝑗 + 1 = 2𝑗 + 2 2𝑗 < 𝑓 2𝑗 + 1 < 2𝑗 + 2 ។ យោ 𝑓(2𝑗 + 1) ជាចំ នន ួ គត់ យោេះ 𝑓 2𝑗 + 1 =

2𝑗 + 1 = 𝑛 + 1 ។

820. 

ក្ 𝑥 = 𝑦 យ ើ ងទាញបាន 0 < 2𝑓 2 𝑥 2 ≤ 2𝑓 𝑥 𝑓(𝑥 3 ) ដូយចនេះ 𝑓 𝑥 និង 𝑓 𝑥 3 មាន

សញ្ញាដូចគ្មន ចំ យ េះព្គប់ 𝑥 ∈ ℝ ។ ក្ 𝑥 = 0 យ ើ ងទាញបាន 210

៥. សមីការអនុគមន៍ | លឹម សុ វណ្ណវិចិត្រ


0 < 2𝑓 2 0 ≤ 𝑓 0 𝑓 𝑦 3 + 𝑓 0 𝑓 𝑦 𝑓 0 2𝑓 0 − 𝑓 𝑦 − 𝑓 𝑦 3 ≤ 0 យ ើ ងទាញបាន ១) យបើ 𝑓 0 < 0 យោេះ 0 > 2𝑓 0 ≥ 𝑓 𝑦 + 𝑓 𝑦 3 យ ើ

យោ

𝑓 𝑦 និង 𝑓 𝑦 3 មានសញ្ញាដូច

គ្មនចំ យ េះព្គប់ 𝑥 ∈ ℝ យោេះ 𝑓 𝑦 < 0 ចំ យ េះព្គប់ 𝑦 ∈ ℝ ។ ផទុ ព ីសមា តក្ ិ មាតដល 𝑓 1999 > 0 ។ ២) យបើ 𝑓 0 > 0 យោេះ 0 < 2𝑓 0 ≤ 𝑓 𝑦 + 𝑓 𝑦 3 ; ∀ 𝑦 ∈ ℝ ។ ដូយចនេះ 𝑓 𝑦 > 0 ; ∀𝑦 ∈ ℝ ។ ដូយចនេះ 𝑓 2000 > 0 ។ ៣) យបើ 𝑓 0 = 0 យោេះ តាង 𝑓 𝑥 = 𝑥 𝑛 𝑔 𝑥 តដល 𝑔 0 ≠ 0 ។ យោ សញ្ញាដូចគ្មន យោេះ 𝑥 𝑛 𝑔 𝑥 និង 𝑥 𝑛 3 𝑔 𝑥 3 មានសញ្ញាដូចគ្មន។ យោ

𝑓 𝑥 និង 𝑓 𝑥 3 មាន 𝑥 𝑛 និង 𝑥 𝑛

3

មានសញ្ញាដូច

គ្មន យោេះ 𝑔 𝑥 និង 𝑔 𝑥 3 ក្៏ព្តូវតតមានសញ្ញាដូចគ្មនតដរ។ យ ើ ងមាន 0 < 2𝑥 4𝑛 𝑔2 𝑥 2 ≤ 2𝑥 𝑛 𝑔 𝑥 𝑥 3𝑛 𝑔 𝑥 3 = 2𝑥 4𝑛 𝑔 𝑥 𝑔 𝑥 3 ; ∀𝑥 ∈ ℝ ដូយចនេះ អាច 𝑔 𝑥 > 0; ∀𝑥 ∈ ℝ រ ឺ 𝑔 𝑥 < 0; ∀𝑥 ∈ ℝ។ តតយោ 𝑓 1999 > 0 យោេះ 𝑔 𝑥 > 0; ∀𝑥 ∈ ℝ ។ ដូយចនេះ 𝑓 2000 = 2000𝑛 𝑔 2000 > 0 ។

លឹម សុ វណ្ណវិចិត្រ | I. គណ្នា

211


212

៥. សមីការអនុគមន៍ | លឹម សុ វណ្ណវិចិត្រ


ឯកសារដ ម ើ

ស ៀវសៅស េះដកស្ ង់សេញពីស ៀវសៅខាងសរោមស េះ 1. Pierre Bornsztein, Inégalité, 2001 2. Hojoo Lee, Topic in Inequalities-Theorems and Techniques 3. A.I Prilepko, Problem Book in High-School Mathematics, MIR Moscow, 1985* 4. D.O. Shklarsky, N.N. Chentzov, I. M. Yaglom, The USSR Olympiad Problem Book, Dover Publications, INC. New York, 1993. 5. Dusan Djukic, Vladimir Jankovic, Ivan Matic, Nikola Petrovic, The IMO Compendium, Springer, 2006 6. GS. PHAN ÐỨC CHÍNH, 101 Bài Toán Chọn lọc, Nhà Xuất Bản Trẻ,1996 7. Tuyển Tập Ðề thi olympic 30-4,Môn Toán, Nhà Xuất Bản Giáo Dục, 1999.

* លំ ហាត់ គ្រឹះស្ទើ រតតទំងអ្់ គតូវបានដកស្​្ង់ សេញព ីស្ៀវសៅមួយកាលសនឹះ។

213


កម្មងលំហាត់គណិតវិទ្យា, កម្មិតវិទ្យាល័យ, ភាគ ២- ពីជគណិត វិភាគ ដោយ លឹម សុវណណវិចិម្ត

ដសៀវដៅកំណណលំហាត់ ណផែកពីជគណិត ណែលរួមមានសមភាព សមីការ វិសមភាព វិសមីការ ននអនុគមន៍ធមមតា អនុគមន៍ម្តីដកាណមាម្ត និង អុ ិចស្ប៉ូណង់ណសែល។ ដសៀវដៅដនេះម្បជុ​ុំ ដោយលំហាត់ ងាយ និងពិបាកចំនន ួ ម្បណែលជា៨០០លំហាត់ ែកម្សង់ដចញពី ការម្បឡង សិ សសពូណកដៅម្បដទ្យសនានាជុវុំ ញ ិ ពិភពដោក។ ដបើអែកចង់កា​ាយជាសិ សសពូណកមាែក់ ដសៀវដៅ មួយកាលដនេះម្បណែលជាដសៀវដៅណែលអែ កចង់បាន៕


Issuu converts static files into: digital portfolios, online yearbooks, online catalogs, digital photo albums and more. Sign up and create your flipbook.